Download as pdf or txt
Download as pdf or txt
You are on page 1of 267

Previous Years NEET Questions & Solutions

NEET
EXCLUSIVE SET OF
PREVIOUS QUESTIONS AND ANSWERS
2013-2020

Brilliant
STUDY CENTRE
PALA
Mutholy Campus, Ph: 04822 - 206100, 206800
Arunapuram Campus, Ph: 04822 - 212415, 210949, 216975
Ernakulam - Ph: 0484 - 2665080, 2665090

www.brilliantpala.org., email: brilliantstudycentre@gmail.com

1
Brilliant STUDY CENTRE

BLANK

2
Previous Years NEET Questions & Solutions

CONTENTS

1. NEET 2013 ----------------------------------------------------------------------------------------- 05

2. NEET 2016 Phase - I ------------------------------------------------------------------------- 36

3. NEET 2016 Phase - II ------------------------------------------------------------------------ 66

4. NEET 2017 ----------------------------------------------------------------------------------------- 95

5. NEET 2018 ---------------------------------------------------------------------------------------122

6. NEET 2019 ---------------------------------------------------------------------------------------153

7. NEET 2019 (Odisha) ------------------------------------------------------------------------178

8. NEET 2020 Phase - I -----------------------------------------------------------------------206

9. NEET 2020 Phase - II ----------------------------------------------------------------------237

3
Brilliant STUDY CENTRE

BLANK

4
Previous Years NEET Questions & Solutions

NEET QUESTION PAPER - 2013

PHYSICS 4. The upper half of an inclined plane of


1. Infinite number of bodies, each of mass inclination  is perfectly smooth while
2 Kg are situated on x-axis at distances lower half is rough. A block starting
1m, 2m, 4m, 8m, ......, respectively, from rest at the top of the plane will
from the origin. The resulting again come to rest at the bottom, if
gravitational potential due to this the coefficient of friction between the
system at the origin will be block and lower half of the plane is
given by
4
1)  G 2) -4G 1)   2tan  2)   tan 
3
1 2
8 3)   4)  
3) -G 4)  G tan  tan 
3
5. The wettability of a surface by a liquid
2. A plano convex lens fits exactly into a
depends primarily on
plano concave lens. Their plane
surfaces are parallel to each other. If 1) density
lenses are made of different materials
2) angle of contact between the surface
of refractive indices 1 and  2 and R and the liquid
is the radius of curvature of the curved
surface of the lenses, then the focal 3) viscosity
length of the combination is 4) surface tension
R 2R 6. A rod PQ of mass M and length L is
1)      2)      hinged at end P. The rod is ke pt
1 2 2 1
horizontal by a massless string tied to
point Q as shown in figure. When string
R R is cut, the initial angular acceleration
3) 2      4) 2     
1 2 1 2 of the rod is

3. In a common emitter (CE) amplifier


having a voltage gain G, the transistor
used has transconductance 0.03 mho
and current gain 25. If the above
transistor is replaced with another one
with transconductance 0.02 mho and
current gain 20, the voltage gain will
be

1 5 2g 2g
1) G 2) G 1) 2)
3 4 L 2L

2 3g g
3) G 4) 1.5 G 3) 4)
3 2L L

5
Brilliant STUDY CENTRE

7. Ratio of lo ngest wave lengths 11. The amount of heat energy required
corresponding to Lyman and Balmer to raise the te mperature of 1g of
series in hydrogen spectrum is Helium at NTP, from T1K to T2K is

7 9 5 3 3
1) 2) 3) 4) 1) Na K B  T2  T1 
29 31 27 23 4
8. The molar specific heats of an ideal
3  T2 
gas at constant pressure and volume 2) 4 Na K B  T 
 1
are denoted by Cp and C v respectively.
Cp 3
If   and R is the universal gas 3) NaKB  T2  T1 
Cv 8

constant, then C v is equal to 3


4) Na K B  T2  T1 
2
 1
1) 2) R
R 12. The resistances of the four arms P, Q,
R and S in a Wheatstone’s bridge are
1  R 10 ohm, 30 ohm, 30 ohm and 90 ohm,
3) 4)
1   1 respectively. The e.m.f. and internal
resistance of the cell are 7 volt and 5
9.  
A uniform force of 3iˆ  ˆj newton acts ohm respectively. If the galvanometer
resistance is 50 ohm, the current
on a particle of mass 2Kg. Hence the drawn from the cell will be
particle is displaced from position
1) 0.1 A 2) 2.0 A
 2iˆ  kˆ  meter to position  4iˆ  3ˆj  kˆ 
meter. The work done by the force on 3) 1.0 A 4) 0.2 A
the particle is 13. A certain mass of Hydrogen is changed
1) 13 J 2) 15 J 3) 9 J 4) 6 J to Helium by the process of fusion. The
mass de fect in fusion reaction is
10. A parallel beam of fast moving electron 0.02866 u. The energy liberated per u
is incident normally on a narrow slit. is (given 1 u=931 MeV)
A fluorescent screen is placed at a
larger distance from the slit. If the 1) 6.675 MeV 2) 13.35 MeV
speed of the electrons is increased,
which of the following statements is 3) 2.67 MeV 4) 26.7 MeV
correct?
14. The following four wires are made of
1) The angular width of the central the same material. Which of these will
maximum will decrease have the largest extension when the
same tension is applied?
2) The angular width of the central
maximum will be unaffected 1) length = 200 cm, diameter = 2 mm
3) Diffraction pattern is not observed 2) length = 300 cm, diameter = 3 mm
on the screen in the case of electrons
3) length = 50 cm, diameter = 0.5 mm
4) The angular width of the central
maximum of the diffraction pattern will 4) length = 100 cm, diameter = 1mm
increase.

6
Previous Years NEET Questions & Solutions

15. Three blocks with masses m, 2m and 18. In the given (V-T) diagram, What is the
3m are connected by strings, as shown relation between pressure P1 and P2?
in the figure. After an upward force F
is applied on block m, the masses move
upward at constant speed v. What is
the net force on the block of mass 2m?
(g is the acceleration due to gravity)

1) P2  P1

2) Cannot be predicted
1) 3 mg 2) 6 mg
3) P2  P1
3) zero 4) 2 mg
4) P2  P1
16. A coil of self-inductance L is connected
in series with a bulb B and an AC 19. A small object of uniform density rolls
source. Brightness of the bulb up a curved surface with an initial
decreases when velocity ‘v’. It reaches upto a maximum

1) A capacitance of reactance X C  X L 3v 2
height of with respect to the intial
is included in the same circuit 4g
position. The object is
2) an iron rod is inserted in the coil
1) hollow sphere
3) frequency o f the AC source is
decreased 2) disc

4) number o f turns in the coil is 3) ring


reduced
4) solid sphere
17. If we study the vibration of a pipe open
at both ends, then the following 20. A body of mass ‘m’ is taken from the
statement is not true. earth’s surface to the height equal to
twice the radius (R) of the earth. The
1) All harmonics of the fundamental change in potential energy of body will
frequency will be generated be
2) Pressure change will be maximum
at both ends 3
1) 3mgR 2) mgR
2
3) Open end will be antinode

4) Odd harmonics of the fundamental 2


3) mg2R 4) MgR
frequency will be generated 3
7
Brilliant STUDY CENTRE

21. The condition under which a 26. In Young’s double slit experiment, the
microwave oven heats up a food item slits are 2 mm apart and are
containing water molecules most illuminated by photons of two
efficiently is 0
wavelengths 1  12000 A and
1) Microwaves are heat waves, so
0
always produce heating  2  10000 A . At what minimum
2) Infra-red waves produce heating in distance from the common central
a microwave oven bright fringe on the screen 2 m from
3) The frequency of the microwave must the slit will a bright fringe from one
match the resonant frequency of the interference pattern coincide with a
water molecules bright fringe from the other?

4) The frequency of the microwaves has 1) 4 mm 2) 3 mm


no relation with natural frequency of
3) 8 mm 4) 6 mm
water molecules
22. The internal resistance of a 2.1 V cell 27. The output (X) of the logic circuit shown
which gives a current of 0.2 A through in figure will be
a resistance of 10  is

1) 0.8  2) 1.0  3) 0.2 4) 0.5 


23. In a n-type semiconductor, which of the 1) X  A .B 2) X = A+B
following statement is true
1) Holes are minority carriers and 3) X  A  B 4) X  A  B
pentavalent atoms are dopants
28. A wire loop is rotated in a magnetic
2) Holes are majority carriers and field. The frequency of change of
trivalent atoms are dopants direction of the induced e.m.f. is
3) Electrons are majority carriers and 1) four times per revolution
trivalent atoms are dopants
4) Electrons are minority carriers and 2) six times per revolution
pentavalent atoms are dopants 3) once per revolution
24. A wire of resistance 4  is stretched 4) twice per revolution
to twice its original length. The
resistance of stretched wire would be 29. A wave travelling in the +ve x-direction
having displacement along y-direction
1) 8  2) 16  3) 2  4) 4  as 1 m, wavelength 2 m and frequency
25. An explosion breaks a rock into three 1
parts in a horizontal plane. Two of them of Hz is represented by

go off at right angles to each other.
The first part of mass 1 Kg moves with 1) y  sin 10 x  20t 
a speed of 12 ms-1 and the second part
of mass 2Kg moves with 8 ms-1 speed.
If the third part flies off with 4 ms-1 2) y  sin  2 x  2t 
speed, then its mass is
3) y  sin  x  2t 
1) 7 Kg 2) 17 Kg
3) 3 Kg 4) 5 Kg 4) y  sin  2x  2 t 
8
Previous Years NEET Questions & Solutions

30. When a proton is released from rest in 33. A source of unknown frequency gives
a room, it starts with an initial 4 beats/s when sounded with a source
acceleration a 0 towards west. When it of known frequency 250 Hz. The second
harmonic of the source of unknown
is projected towards north with a speed
frequency gives five beats per second,
v 0 it moves with an initial acceleration when sounde d with a source of
3a 0 towards west. The electric and frequency 513 Hz. The unkno wn
magnetic fields in the room are frequency is
1) 240 Hz 2) 260 Hz
ma 0 ma0
1) east, v up 3) 254 Hz 4) 246 Hz
e 0

34. The velocity of a projectile at the initial

2)
ma 0
e
ma 0
east, v
0
down  
point A is 2ˆi  3ˆj m / s . It’s velocity (in
m/s) at point B is
ma 0 2ma 0
3) e0 west, ev 0 up

ma 0 2ma0
4) west, ev down
e 0

31. The wavelength  e of an electron and


 p of a photon of same energy E are
related by
1) 2ˆi  3ˆj 2) 2ˆi  3ˆj
1
1)  p  e 2)  p  e 3) 2ˆi  3ˆj 4) 2ˆi  3ˆj

35. A gas is taken through the cycle


3)  p  e2 4)  p  e A  B  C  A , as shown. What is the
net work done by the gas?
32. A, B and C are three points in a uniform
electric field. The electric potential is

1) maximum at C
2) same at all the three points A, B
and C
3) maximum at A 1) Zero 2) -2000 J

4) maximum at B 3) 2000 J 4) 1000 J

9
Brilliant STUDY CENTRE

36. A stone falls freely under gravity. it 39. Two pith balls carrying equal charges
converts distance h1, h2 and h3 in the are suspended from a common point
first 5 seconds, the next 5 seconds and by strings of equal length, the
the next 5 seconds respectively. The equilibrium separation between them
relation between h1, h2 and h3 is is r. Now the strings are rigidly
clamped at half the height. The
1) h 2  3h1 and h 3  3h 2 equilibrium separation between the
balls now become:
2) h1  h 2  h 3

3) h1  2h 2  3h 3

h2 h3
4) h1  
3 5

37. In an experiment four quantities


a,b,c and d are measured with
percentage error 1%, 2%, 3% and 4%  2r   2r 
1)   2)  
respectively. Quantity P is calculated  3  3
as follows
2
 1   r 
3)   4)  3 
3
a b 2
 2  2
P  % error in P is
cd 40. For photoelectric emission from certain
metal the cutoff frequency is  . If
1) 7%
radiation of frequency 2 impinges on
the metal plate, the maximum possible
2) 4%
velocity of the emitted electron will be
(m is the electron mass)
3) 14%
2h  h
4) 10% 1) 2) 2
m m
38. A current loop in a magnetic field
h h
1) can be in equilibrium in two 3)  2m  4)
m
orientations, both the equilibrium
states are unstable 41. A piece of iron is heated in a flame. It
first becomes dull red then becomes
2) can be in equilibrium in two reddish yellow and finally turns to
orientations, one stable while the order white hot. The correct explanation for
is unstable the above observation is possible by
3) experiences a torque whether the using
field is uniform or non uniform in all 1) Kirchhoff’s law
orientations
2) Newton’s law of cooling
4) can be in equilibrium on one 3) Stefan’s law
orientation
4) Wien’s displacement law
10
Previous Years NEET Questions & Solutions

42. A bar magnet of length ' ' and CHEMISTRY


magnetic dipole moment ‘M’ is bent in
46. The value of Planck’s constant is
the form of an arc as shown in figure.
6 .6 3 × 1 0 -34Js. The speed of light is
the new magnetic dipole moment will
3 × 1017 nms-1. Which value is closest
be
to the wavelength in nanometer of a
quantum of light with frequency of
6 × 1015 s-1?
1) 75 2) 10
3) 25 4) 50

47. The radical i s aromatic


because it has:
2 M 1) 6 p-orbitals and 7 unpaired electrons
1) M 2)
 2
2) 6 p-orbitals and 6 unpaired electrons
3 3) 7 p-orbitals and 6 unpaired electrons
3) M 4) M

4) 7 p-orbitals and 7 unpaired electrons
43. During an adiabatic process, the
pressure of a gas is found to be 48. Which of the following is electron-
proportional to the cube of its deficient?
Cp 1) PH3 2) (CH3)2
temperature. The ration of for the
C 3) (SiH3)2 4) (BH3)2
gas is
49. Which of the following statements
5 3 4 about the interstitial compounds is
1) 2) 3) 4) 2 incorrect?
3 2 3
1) They have higher melting points than
44. For a normal eye, the cornea of eye
the pure metal
provides a converging power of 40 D
and the least converging power of the 2) They retain metallic conductivity
eye lens behind the cornea is 20 D.
Using this information, the distance 3) They are chemically reactive
between the retina and the cornea - 4) They are much harder than the pure
eye lens can be estimated to be metal
1) 1.67 cm 2) 1.5 cm 50. How many grams of concentrated nitric
3) 5 cm 4) 2.5 cm acid solution should be used to prepare
250 mL of 2.0M HNO3?
45. The half life of a radioactive isotope ‘X’
is 20 years. It decays to another The concentrated acid is 70% HNO3.
element ‘Y’ were found to be in the 1) 54.0g conc. HNO3
ratio 1:7 in a sample of a given rock.
The age of rock is estimated to be 2) 45.0g conc. HNO3

1) 80 Years 2) 100 Years 3) 90.0g conc. HNO3

3) 40 Years 4) 60 Years 4) 70.0g conc. HNO3

11
Brilliant STUDY CENTRE

51. Which of the following lanthanide ions 55. The order of stability of the following
is diamagnetic? tautomeric compounds is:
(At nos. Ce = 58, Sm = 62. Eu = 63, Yb
= 70)

1) Yb 2 2) Ce 2

3) Sm 2 4) Eu 2

52. Which one of the following molecules 1) II > III > I 2) I > II > III
contains no  bond?
3) III > II > I 4) II > I > III
1) NO 2 2) CO2
56. Nylon is an example of:
3) H2O 4) SO2
1) Polythene 2) Polyester
53. Based on equation
3) Polysaccharide 4) Polyamide
57. XeF2 is isostructural with:
1) BaCl2 2) TeF2
certain conclusions are written. Which
3) ICI 2 4) SbCl3
of them is not correct?
1) For n = 1, the electron has a more 58. The basic structural unit of silicates
negative energy than it does for n = 6 is:
which means that the electron is more
loosely bound in the smallest allowed 1) SiO 24 2) SiO-
orbit
3) SiO 44 4) SiO 32
2) The negative sign in equation simply
means that the energy of electron 59. Which of the following structure is
bound to the nucleus is lower than it similar to graphite?
would be if the electrons were at the
infinite distance from the nucleus 1) B2H6 2) BN

3) Larger the value of n, the larger is 3) B 4) B4C


the orbit radius 60. The structure of isobutyl group in an
4) Equation can be used to calculate organic compound is:
the change in energy when the
electron changes orbit.
1)

54. In the reaction A

2)
is:

1) H  / H 2 O 2) HgSO 4 / H 2SO 4 3)

3) Cu 2 Cl 2 4) H 3 PO 2 and H2O 4)
12
Previous Years NEET Questions & Solutions

61. The number of carbon atoms per unit 67. Structure of the compound whose
cell of diamond unit cell is: IUPAC name is 3-Ethyl-2-hydroxyl-4-
methylhex-3-en-5-ynoic acid is:
1) 1 2) 4
3) 8 4) 6
62. An excess of AgNO3 is added to 100 mL 1) 2)
of a 0.01 M solution of
dichlorotetraaquachromium (III)
chloride. The number of moles of AgCl
precipitated would be:
3) 4)
1) 0.01 2) 0.001
3) 0.002 4) 0.003
68. Among the following ethers, which one
63. What is the maximum numbers of will produce me thyl alcohol on
electrons that can be associated with treatment with hot concentrated HI?
the following set of quantum numbers?
n = 3, l = 1 and m = -1 1)

1) 2 2) 10
3) 6 4) 4 2)
64. Which of these is not a monomer for a
high molecular mass silicone polymer? 3)

1) PhSiCl3 2) MeSiCl3
3) Me2SiCl2 4) Me3SiCl
65. A reaction having equal energies of
activation for forward and reverse 4)
reactions has:

1) H  G  S  0
69. Antiseptics and disinfectants either
2) S  0 kill or prevent grown of microganisms.
Identi fy which of the following
3) G  0 statements is not true:

4) H  0 1) Disinfe ctants harm the living


tissues
66. At 25oC molar conductance of 0.1 molar
aqueous solution of ammonium 2) A 0.2% solution of phenol is an
hydroxide is 9.54 ohm-1 cm2 mol-1 and antiseptic while 1% solution acts as a
at infinite dilution its molar disinfectant
conductance is 238 ohm-1 cm2 mol-1.
The degree of ionisation of ammonium 3) Chlorine and iodine are used as
hydroxide at the same concentration strong disinfectant
and temperature is:
4) Dilute solutions of boric acid and
1) 40.800% 2) 2.080% hydrogen peroxide are strong
antiseptics
3) 20.800% 4) 4.008%
13
Brilliant STUDY CENTRE

70. A magnetic moment of 1.73 BM will be 75. Dipole-induced dipole interaction are
shown by one among the following: present in which of the following pairs:
2 1) SiF4 and He atoms
1)  CoCl 6  2) Cu  NH 3  4 
4

2) H2O and alcohol


2 3) Cl2 and CCl4
3)  Ni  CN  4  4) TiCl4
4) HCl and He atoms
71. KMnO4 can be prepared from K2MnO4
76. Roasting of sulphides gives the gas X
as per the reaction:
as a by-product. This is a colourless
3MnO 24   2H 2 O  2MnO 4  MnO 2  4OH  gas with choking smell of burnt sulphur
and causes great damage to respiratory
The reaction can go to completion by organs as a result of acid rain. Its
removing OH- ions by adding: aqueous solution is acidic, acts as a
1) SO2 2) HCl reducing agent and its acid has never
been isolated. The gas X is:
3) KOH 4) CO2
1) SO3 2) H2S
72. Reaction by whi ch Benzaldehyde
cannot be prepared: 3) SO2 4) CO2
77. Some meta-directing substituents in
aromatic substitution are given. Which
1) one is most deactivating?
1) -NO2 2) C  N
. 2)
3) -SO3H 4) -COOH
78. Nitrobenzene on reaction with conc.
3) in presence of Pd- HNO3/H2SO4 at 80 - 100oC forms which
one of the following products?
BaSO4
1) 1,2,4-Trinitrobenzene
4) in presence of 2) 1,2-Dinitrobenzene
anhydrous AlCl3 3) 1,3-Dinitrobenzene
73. Which of the following does not give 4) 1,4-Dinitrobenzene
oxygen on heating?
79. A hydrogen gas electrode is made by
1)  NH 4 2 Cr2 O7 2) KClO3 dipping platinum wire in a solution of
HCl of pH=10 and by passing hydrogen
gas around the platinum wire at one
3) Zn  ClO3  2 4) K 2 Cr2 O 7
atm pressure. The oxidation potential
74. A metal has a fcc lattice. The edge of electrode would be?
length of the unit cell is 404 pm. The 1) 1.18 V 2) 0.059 V
density of the metal is 2.72 g cm-3. The
molar mass of the metal is: 3) 0.59 V 4) 0.118 V

(NA, Avogadro’s constant = 6.02 × 1023 80. Which of the following is a pol ar
mol-1) molecule?

1) 20 g mol-1 2) 10 g mol-1 1) XeF4 2) BF3

3) 30 g mol-1 4) 27 g mol-1 3) SF4 4) SiF4


81. A button ce ll used in watches
14
Previous Years NEET Questions & Solutions

functions as following 87. Which of the following is


paramagnetic?
Zn  s   Ag 2 O  s   H 2O  l   2Ag  s   Zn 2  aq   2OH   aq 
1) NO + 2) CO 3) O 2 4) CN-
If half cell potential are
88. Identify the correct order of solubility
Zn 2  aq   2e   Zn  s  ; E o  0.76 V in aqueous medium:

Ag2O  s   H2O  l   2e  2Ag  s   2OH  aq  , Eo  0.34V 1) Na 2S  ZnS  CuS

The cell potential will be: 2) CuS  ZnS  Na 2S


1) 1.34 V 2) 1.10 V 3) ZnS  Na 2S  CuS
3) 0.42 V 4) 0.84 V
4) Na 2S  CuS  ZnS
82. Which of these is least likely to act as
a Lewis base? 89. What is the activation energy for a
reaction if its rate doubles when the
1) PF3 2) CO
temperature is raised from 20 o C to
3) F- 4) BF3 35oC? (R = 8.314 J mol-1 K-1)
83. Which of the following compounds will 1) 15.1 kJ mol-1 2) 342 kJ mol-1
not undergo Friedal Craft’s reaction 3) 269 kJ mol-1 4) 34.7 kJ mol-1
easily: 90. Which is the strongest acid in the
1) Toluene 2) Cumene following?
1) H2SO3 2) H2SO4
3) Xylene 4) Nitrobenzene
3) HClO3 4) HClO4
84. Which is the monomer of Neoprene in
BIOLOGY
the following?
91. The eye of octopus and eye of cat show
1) different patterns of structure, yet they
perform similar function. This is an
2) example of:
1) Analogous organs that have evolved
3) due to divergent evolution
2) Homologous organs that have evolved
due to convergent evolution
4) 3) Homologous organs that have evolved
due to divergent evolution
4) Analogous organs that have evolved
85. 6.02 × 10 20 molecules of urea are due to convergent evolution
present in 100 mL of its solution. The 92. Select the correct statement with
concentration of solution if: respect to locomotion in humans:
1) 0.1 M 2) 0.02 M 1) The joint between adjacent vertebrae
3) 0.01 M 4) 0.001 M is a fibrous joint
2) A decreased level of progesterone
86. Maximum deviation from ideal gas is causes osteoporosis in old people
expected from:
3) Accumulation of uric acid crystals
1) NH3(g) 2) H2(g) in joints causes their inflammation
3) N2(g) 4) CH4(g) 4) The vertebral column has 10 thoracic
vertebrae

15
Brilliant STUDY CENTRE

93. A phosphoglyceride is always made up 100. Kyoto Protocol was endorsed at:
of: 1) CoP-4 2) CoP-3
1) a saturated or unsaturated fatty acid 3) CoP-5 4) CoP-6
esterified to a phosphate group which
101. Figure shows human urinary system
is also attached to a glycerol molecule
with structures labelled A to D. Select
2) only a saturated fatty acid esterified option which correctly identifies them
to a glycerol molecule to which a and gives their characteristics and/or
phosphate group is also attached functions
3) only a unsaturated fatty acid
esterified to a glycerol molecule to
which a phosphate gro up is also
attached
4) a saturated or unsaturated fatty acid
esterified to a glycerol molecule to
which a phosphate gro up is also
attached
94. Perisperm differs from endosperm in:
1) its formation by fusion of secondary
nucleus with several sperms
2) being a haploid tissue
3) having no reserve food
4) being a diploid tissue
95. A sedentary sea anemone gets
attached to the shell lining of hermit
crab. The association is:
1) Amensalism 2) Ectoparasitism 1) D-Cortex -outer part of kidney and
3) Symboisis 4) Commensalism do not contain any part of nephrons
96. The cell-mediated immunity inside the 2) A-Adrenal gland-located at the
human body is carried out by: anterior part of kidne y. Secre te
Catecholamines which stimul ate
1) Erythrocytes 2) T-lymphocytes
glycogen breakdown
3) B-lymphocytes 4) Thrombocytes
3) B-Pelvis-broad funnel shaped space
97. Which of the following are likely to be
inner to hilum, directly connected to
present in deep sea water?
loops of Henle
1) Saprophytic fungi
4) C-Medulla inner zone of kidney and
2) Archaebacteria
contains complete nephrons
3) Eubacteria
102. In china rose the flowers are:
4) Blue-breen algae
98. One of the representatives of Phylum 1) Zygomorphic,epigynous with twisted
Arthropoda is : aestivation
1) flying fish 2) cuttlefish 2) Actinomorphic, hypogynous with
3) silverfish 4) pufferfish twisted aestivation

99. Megasporangium is equivalent to: 3) Actinomorphic,epigynous with


valvate aestivation
1) Ovule 2) Embryo sac
4) Zygomorphic,hypogynous with
3) Fruit 4) Nucellus imbricate aestivation
16
Previous Years NEET Questions & Solutions

103. The Golgi complex plays a major role: 108. The colonies of recombinant bacteria
1) in post translational modification of appear whi te in contrast to blue
proteins and glycosidation of lipids colonies of non-recombinant bacteria
because of:
2) in trapping the lig ht and
transforming it into chemical energy 1) Inactivation of glycosidase enzyme
in recombinant bacteria
3) in digesting proteins and
2) Non-recombinant bacteria containing
carbohydrates
betagalactosidase
4) as energy transferring organelles
3) Insertional inactivation of alpha-
104. What external changes are visible galactosidase in non recombinant
after the last moult of a cockroach bacteria
nymph? 4) Insertional inactivation of alpha-
1) Labium develops galactosidase in recombinant bacteria
2) Mandibles become harder 109. Monoecious plant of Chara shows
3) Anal cerci develop occurrence of:
4) Both fore wings with hind wings 1)upper oogonium & lower antheridium
develop on the same plant
105. Isogamous condition with non- 2) antheridiophore & archegoniophore
flagallated gametes is found in: on the same plant
3) stamen and carpel on the same plant
1) Fucus
4) upper antheridium & lower oogonium
2) Chlamydomonas on the same plant
3) Spirogyra 110. Advantage of cleistogamy is:
4) Volvox 1) vivipary
106. Transition state structure of the 2) Higher genetic variability
substrate formed during an enzymatic 3) More vigorous offspring
reaction is:
4) No dependence on pollinators
1) permanent and stable 111. The H-zone in the skeletal muscle fibre
2) transient but stable is due to:
3) permanent but unstable 1) extension of myosin filaments in the
4) transient and unstable central portion of the A-band
107. Select the answer which correctly 2) the absence of myofibrils in the
matches the endocrine gland with the central portion of A-band
hormone it secretes and its function/ 3) the central gap between myosin
deficiency symptom: filaments in the A-band
4) the central gap between actin
Endocrine Function/deficiency filaments extending through myosin
gland Hormone symptoms filaments in the A-band
Corpus Stimulates 112. Artificial insemination means:
1 luteum Testosterone spermatogenesis 1) introduction of sperms of a healthy
Stimulates uterus donor directly into the ovary
Anterior contraction
2 pituitary Oxytocin during child birth
2) transfer of sperms of a healthy donor
Growth Oversecretion to a test tube containing ova
Posterior Hormons stimulates 3) transfer of sperms of husband to a
3 pituitary (GH) abnormal growh test tube containing ova
Thyroid Lack of lodine in 4) artificial introduction of sperms of a
4 gland Thyroxine diet results in goitre healthy donor into the vagina
17
Brilliant STUDY CENTRE

113. Which group of animals belong to the 119. Secondary productivity is rate of
same phylum? formation of new organic matter by:
1) Sponge, Sea anemone, Starfish 1) Decomposer 2) Producer
2) Malarial parasite, Amoeba, Mosquito
3) Parasite 4) Consumer
3) Earthworm, Pinworm,Tapeworm
120. Infection of Ascaris usually occurs by:
4) Prawn, Scorpion, Locusta
114. Seed coat is not thin, membranous in: 1) mosquito bite
1) Gram 2) Maize 2) drinking water containing eggs of
3) Coconut 4) Groundnut Ascaris
115. If two persons with ‘AB’ blood group 3) eating imperfectly cooked pork
marry and have sufficiently large 4) Tse-tse fly
number of children, these children
could be classified as ‘A’ blood group: 121. Figure shows schematic plan of blood
‘AB’ blood group ‘B’ blood group in 1:2:1 circulation in humans with labels A to
ratio. Modern technique of protein D, Identify the label and give its
electrophoresis reveals presence of function/s
both ‘A’ and ‘B’ type proteins in ‘AB’
blood group individuals. This is an
example of:
1) Complete dominance
2) Codominance
3) Incomplete dominance
4) Partial dominance
116. Which of the follo wing cannot be
detected in a developing foetus by
amniocentesis?
1) Jaundice
2) Klinefelter syndrome
1) D-Dorsal aorta-takes blood from
3) Sex of the foetus
heart to body parts, PO2 = 95 mm Hg
4) Down syndrome
2) A-Pulmonary vein-takes impure blood
117. The first stable product of fixation of
from body parts, PO2 = 60 mm Hg
atmospheric nitrogen in leguminous
plants is: 3) B-Pulmonary artery-takes blood from
heart to lungs, PO2 = 90 mm Hg
1) Glutamate 2) NO 2
4) C-Vena Cava-takes blood from body
 parts the right auricle, PCO2 = 45 mm
3) Ammonia 4) NO 3
Hg
118. A biologist studied the population of 122. The tendency of population to remain
rats in a barn. He found that the in genetic equilibrium may be disturbed
average natality was 250, average by:
mortality 240, immigration 20 and
1) lack of random mating
emigration 30. The net increase in
population is : 2) random mating
1) Zero 2) 10 3) lack of migration
3) 15 4) 05 4) lack of mutations
18
Previous Years NEET Questions & Solutions

123. A diagram showing axon terminal and 127. A stage in cell division is shown in the
synapse is given. Identify correctly at figure. Select the answer which gives
least two of A-D correct identification of the stage with
its characterstics.

1) C-Neurotransmitter, D-Ca++ Endoplasmic reticulum and


1 Telophase nucleolus not reformed yet

2) A-Receptor, C-Synaptic VESICLES Nuclear envelop reforms, golgi


2 Telophase complex reforms
3) B-Synaptic connection, D-K’ Chromosomes move away from
equatorial plate, golgi complex
4) A-Neurotransmitter, B-Synaptic cleft 3 Late anaphase not present
Cell plate formed mitochondria
distributed between two daughter
124. A good producer of citric acid is: 4 Cytokinesis cells

1) Saccharomyces
128. The figure shows a diagrammatic view
2) Aspergillus of human respiratory system with
labels A,B,C and D. Select the option
3) Pseudomonas which gives correct identification and
main function and/or characteristic:-
4) Clostridium
125. Age of a tree can be estimated by:
1) diameter of its heartwood
2) its height and girth
3) biomass
4) number of annual rings
126. The process by which organisms with
different evolutionary history evolve 1) D- Lower end of lungs - diaphragm
similar phenotypic adaptation in pulls it down during inspiration
response to a common environmental, 2) A - trachea- long tube supported by
CHALLENGE is called: comple te cartilaginous rings for
conducting inspired air
1) Adaptive radiation
3) B- pleural membrane-surround ribs
2) Natural selection
on both sides to provide cushion against
3) Convergent evolution rubbing
4) Non-random evolution 4) C-Alveoli-thin walled vascular bag
like structures for exchange of gases
19
Brilliant STUDY CENTRE

129. Interfascicular cambium develops from 137. Which one of the following processes
the cells of: during decomposition is correctly
1) Pericycle described?
2) Medullary rays 1) Leaching-Water soluble inorganic
nutrients rise to the top layers of soil
3) Xylem parenchyma
2) Fragmentation - Carried out by
4) Endodermis organisms such as earthworm
130. During seed germination its stored food 3) Humification - Leads to the
is mobilized by: accumulation of a dark coloured subs
1) Gibberellin 2) Ethylene tance humus which undergoes
3) Cytokinin 4) ABA 4) Catabolism-Last step in the
131. Meiosis takes place in: decomposition under fully anaerobic
condition
1) Megaspore 2) Meiocyte
138. If both parents are carriers, for
3) Conidia 4) Gemmule thalessemia, which is an autosomal
132. According to Darwi n, the organic re cessive disorder, what are the
evolution is due to: chances of pregnancy resulting in an
1) Reduced feeding efficiency in one affected child?
species due to the presence of 1) 100% 2) No chance
interfering species 3) 50% 4) 25%
2) Intraspecific competition 139. Which of the following statements is
3) Interspecific competition not true of two genes that show 50%
recombination frequency?
4) Competition within closely related
species 1) If the genes are present on the same
chromosome, they undergo more than
133. Which of the following criteria does
one cross overs in every meiosis
pertain to facilitated transport?
2) The genes may be on differe nt
1) Uphill transport chromosomes
2) Requirement of special membrane 3) The genes are tightly linked
proteins
4) The gene s show independe nt
3) High selectivity assortment
4) Transport saturation 140. One of the legal methods of birth
134. A major site for synthesis of lipids is: control is:
1) Nucleoplasm 2) RER 1) by a PREMATURE ejaculation during
3) SER 4) Symplast coitus
2) abortion by taking an appropriate
135. Natural reservoir of phosphorus is:
medicine
1) Fossils 2) Sea water
3) by abstaining from coitus from day
3) Animal bones 4) Rock 10 to 17 of the menstrual cycle
136. Which of the metabolites is common 4) by having coitus at the time of day
to respiration mediated breakdown of break
fats, carbohydrates and proteins? 141. Besides paddy fields, cyanobacteria
1) Acetyl CoA are also found inside vegetative part
2) Glucose-6-phosphate of:
3) Fructose1,6-bisphosphate 1) Psilotum 2) Pinus
4) Pyruvic acid 3) Cycas 4) Equisetum

20
Previous Years NEET Questions & Solutions

142. Which of the following are correctly 146. Which Mendelism idea is depicted by
matched with respect to their a cross in which the F 1 generation
taxonomic classification? resembles both the parents?
1) Spiny anteater, sea urchin, sea 1) co-dominance
cucumber - Echinodermata
2) incomplete dominance
2) Flying fish, cuttlefish,silverfish-
Pisces 3) law of dominance
3) Centipede,mill ipede, spider, 4) inheritance of one gene
scorpion- Insecta
147. The diagram given here is the standard
4) House fly, butterfly, tsetsefly, ECG of a normal person. The P-wave
silverfish-Insecta represents the:
143. Variation in gene frequencies within
populations can occur by chance rather
than by natural selection. This is
referred to as:
1) Genetic load 2) Genetic flow
3) Genetic drift 4) Random mating 1) End of systole

144. Select the correct match of the digested 2) Contraction of both the atria
products in humans given in column I 3) Initiation of the ventricular
with their absorption site and contraction
mechanism in column II.
4) Beginning of the systole
148. Which enzyme/s will be produced in a
Column I Column II cell in which there is a nonsense
Cholesterol, Large intestine, active mutation in the lac Y gene?
1 maltose absorption
1)Lactose perme ase and
Glycine, small intestine, active transacetylase
2 glucose absorption
small intestine, passive 2)  -galactosidase
3 Fructose, Na+ absorption
duodenum, move as
3) Lactose permease
4 Glycerol, fatty acids chilomicrons 4) Transacetylase
149. The most abundant intracellular cation
145. Select the wrong statement: is:
1) Chlamydomonas exhibits both 1) K+ 2) Na+
isogamy and anisogamy and Fucus 3) Ca++ 4) H+
shows oogamy
150. Which one of the following is not the
2) Isogametes are similar in structure, function of placenta ? It:
function and behaviour 1) Secretes oxytocin during parturition
3) Anisogametes differ either in 2) Facilitates supply of oxygen and
structure, function or behaviour nutrients to embyro

4) In Oomycetes female gamete is 3) Secretes estrogen


smaller and motile, while male gamete 4) Facilitates removal of carbon dioxide
is larger and non-motile and waste material from embryo
21
Brilliant STUDY CENTRE

151. In plant breeding programme, the 154. Which of the following represents
entire collection (of plants/seeds) maximum number of species among
having all the diverse alleles for all global biodiversity?
genes in a given crop is called:
1) Mosses and Ferns
1) germplasm collection
2) Algae
2) selection of superior recombinants
3) cross- hybridisation among the 3) Lichens
selected parents 4) Fungi
4) evaluation and selection of parents
155. Which of the following Bt crops is being
152. Which one of the following is not a grown in India by the farmers?
correct statements?
1) Soyabean
1) Key is taxonomic aid for
identification of specimens 2) Maize
2) Herbarium houses dried, pressed 3) Cotton
and preserved plant specimens
4) Brinjal
3) Botanical gardens have collection of
living plants for reference 156. Read the following statements (a-e)
and answer the question which follows
4) A museum has col lection of
them.
photographs of plants and animals
153. Which one of the following organelle a) In liverworts, mosses and ferns
in the figure correctly matches with gametophytes are free living
its function/ b) Gymnosperms and some ferns are
heterosporous
c) Sexual reproduction in Fucus, Volvox
and Albugo is oogamous
d) The sporophyte in liverworts is more
elaborate than that in mosses

e) Both, Pinus and Marchantia are


dioecious How many of the above
statements are correct?

1) Four 2) One

3) Two 4) Three

157. The essential chemical components of


1) Rough endoplasmic reticulum, many coenzymes are:
protein synthesis
1) Vitamins
2) Rough endoplasmic reticulum,
formation of glycoproteins 2) Proteins
3) Golgi apparatus, protein synthesis 3) Nucleic acids
4) Golgi apparatus, formation of
glycolipids 4) Carbohydrates

22
Previous Years NEET Questions & Solutions

158. Parts A,B,C and D of the human eye 161. Which one of the following statements
are shown in the diagram. Select the is correct?
option which gives correct identification
along with its functions/ 1) Tapetum nourishes the developing
characteristics: pollen
2) Hard outer layer of pollen is called
intine
3) Sporogenous tissue is haploid
4) Endothe cium produces the
microspores
162. The characteristics and an example of
a synovial joint in humans is:
1) lymph filled between two bones,
limited movement gliding joint between
carpals
1) D-Choroid - its anterior part forms
2) Fluid cartilage between two bones,
ciliary body
limited movements Knee joint
2) A-Retina - contains photo receptors-
rods and cones 3) fluid filled be tween two joints,
provides cushion skull bones
3) B-Blind spot - has only a few rods
and cones 4) fluid filled synovial cavity between
two bone joint between atlas and axis
4) C-Aqueous chamber-reflects the
light which does not pass through the 163. The Air Prevention and Control of
lens Pollution Act came into force in:
159. The three boxes in this diagram 1) 1990 2) 1975
represent the three major biosynthetic
3) 1981 4) 1985
pathways in aerobic respiration. Arrows
represent net reactants or products. 164. Product of sexual reproduction
generally generates:
1) Large biomass
2) Longer viability of seeds
3) Prolonged dormancy
Arrow numberd 4,8 and 12 can all be: 4) New genetic combination leading to
1) FAD+ or FADH2 2) NADH variation

3) ATP 4) H2O 165. Among bitter gourd, mustard, brinjal,


pumpkin china rose, Lupin, cucumber,
160. Pigment-containing membranous sunhemp,gram, guava,be an,chilli,
extensions in some cyanobacteria are: plum,Petunia,tomato,rose ,Withania,
1) Chromatophores potato, onion, Aloe and tulip how many
plants have hypogynous flower?
2) Heterocysts
1) Eighteen 2) Six
3) Basal bodies
3) Ten 4) Fifteen
4) Pneumatophores
23
Brilliant STUDY CENTRE

166. A pregnant female delivers baby who 172. The diagram shows an important
suffers from stunted growth, mental concept in the genetic implication of
retardation, low intelligence quotient DNA. Fill in the blanks A to C:
and abnormal skin.
This is the result of:
1) Over secretion of pars distalis 1) A-translation B-extensionC-Rosalind
2) Deficiency of iodine in diet Franklin
3) Low secretion of growth hormone 2) A-transciption B-replicationC-James
4) Cancer of the thyroid gland Watson
167. Which of the following is not correctly 3) A-translation B-transcriptionC-
matched for the organism and its cell Erevin Chargaff
wall degrading enzyme? 4) A-transcription B-translation C-
1) Fungi-Chitinase Francis Crick
2) Bacteria-Lysozyme 173. DNA fragments generated by the
3) Plant cells - Cellulase restriction endonucleases in a
chemical reaction can be separated by:
4) Algae-Methylase
1) Restriction mapping
168. Menstrual flow occurs due to lack of:
1) Vasopressin 2) Centrifugation
2) Progesterone 3) Polymerase chain reaction
3) FSH 4) Electrophoresis
4) Oxytocin 174. The comple x forme d by a pair of
synapsed homologous chromosomes is
169. Global warming can be controlled by:
called:
1) Increasing deforestation, reducing
efficiency of energy usage 1) Axoneme 2) Equatorial plate
2) Reducing deforestation cutting down 3) Kinetochore 4) Bivalent
use of fossil fuel 175. The incorrect statement with regard
3) Reducing reforestation, increasing to Haemophilia is:
the use of fossil fuel A) A single protein involved in the
4) Increasing deforestation slowing clotting of blood is affected
down the growth human population 2) It is a sex-linked disease
170. Which one of the following is not used 3) It is a recessive disease
for ex situ plant conservation? 4) It is a dominant disease
1) Botanical Gardens 176. Which of the following statements is
2) Field gene banks correct in relation to the endocrine
3) Seed banks system?
4) Shifting cultivation 1) Releasing and inhibitory hormones
171. During sewage treatment, biogases are are produced by the pituitary gland
produced which include: 2) Adenohypophysis is under direct
1) hydrogensulphide , ni trogen, neural regulation of the hypothalamus
methane 3) Organs in the body like
2) methane, hydrogensulphide, carbon gastrointestinal tract, heart, kidney
dioxide and liver do not produce any hormones
3) methane, oxygen, hydrogensulphide 4) Non-nutrient chemicals produced by
4) hydrogensulphide , me thane, the body in trace amount that act as
sulphur dioxide intercellular messenger are known as
hormones
24
Previous Years NEET Questions & Solutions

177. Lenticels are involved in: 179. What is the correct sequence of sperm
1) Photosynthesis formation?
2) Transpiration 1) Spermatogonia,
3) Gaseous exchange
spermatocyte,spermatid,spermatozoa
4) Food transport
178. Match the name of the animal (column 2)Spermatid,spermatocyte,
I), with one characteristics (column II), spermatogonia, spermatozoa
and the phylum/class (column III) to
3) Spe rmatogonia, spe rmatocyte,
which it belongs:
spermatozoa,spermatid
4) Spermatogonia,
Column I Column II Column III
radially spermatozoa,spermatocyte,spermatid
1 Adamsia Porifera
symmetrical 180. Macro molecule chitin is :
2 Petromyzone ectoparasite Cyclostomata
1) Simple polysaccharide
3 Ichthyophis terrestrial Reptilia 2) Nitrogen containing polysaccharide
Body covered 3)Phosphoruscontaining polysaccharide
4 Limulus by chitinous Pisces
exoskeleton 4) Sulphur containing polysaccharide

25
Brilliant STUDY CENTRE

KEY WITH HINTS

PHYSICS
3. 3
1. 2

4. 1

2. 1

26
Previous Years NEET Questions & Solutions

5. 2 8. 4

6. 3

9. 3

10. 1

7. 3

11. 3

27
Brilliant STUDY CENTRE

15. 3
12. 4

16. 2
13. 1

17. 2
18. 1

14. 3

28
Previous Years NEET Questions & Solutions

21. 3
19. 2

22. 4

23. 1

24. 2

20. 4
25. 4

29
Brilliant STUDY CENTRE

26. 4 31. 3

27. 1

28. 4
32. 4

33. 3

29. 3

30. 4
34. 1

30
Previous Years NEET Questions & Solutions

38. 2
35. 4

39. 4

36. 4

37. 3

31
Brilliant STUDY CENTRE

40. 1 43. 2

41. 4

42. 4

44. 1

45. 4

32
Previous Years NEET Questions & Solutions

CHEMISTRY
55. 3
46. 4

47. 2

56. 4 Nylon is a polyamide


57. 3

48. 4 58. 3

49. 3 59. 2
50. 2
60. 2

61. 3

62. 2

51. 1

63. 1

64. 4

52. 3
65. 1
53. 1

54. 4

66. 4

33
Brilliant STUDY CENTRE

76. 3
67. 3

77. 1

78. 3

79. 3

68. 4

69. 3

70. 2 80. 3

71. 4
81. 2

72. 1
73. 1

82. 4
83. 4
74. 4
84. 4

85. 3

75. 4 86. 1

34
Previous Years NEET Questions & Solutions

87. 3 89. 4

90. 4
88. 1

BIOLOGY

91. 4 106. 4 121. 4 136. 1 151. 1 166. 2


92. 3 107. 4 122. 1 137. 2 152. 4 167. 4
93. 4 108. 2 123. 2 138. 4 153. 1 168. 2
94. 4 109. 1 124. 2 139. 3 154. 4 169. 2
95. 4 110. 4 125. 4 140. 2 155. 3 170. 4
96. 2 111. 4 126. 3 141. 3 156. 4 171. 2
97. 2 112. 4 127. 2 142. 4 157. 1 172. 4
98. 3 113. 4 128. 4 143. 3 158. 2 173. 4
99. 1 114. 2 129. 2 144. 2 159. 3 174. 4
100. 2 115. 2 130. 1 145. 4 160. 1 175. 4
101. 2 116. 1 131. 2 146. 1 161. 1 176. 4
102. 2 117. 3 132. 3 147. 2 162. 4 177. 3
103. 1 118. 1 133. 1 148. 2 163. 3 178. 2
104. 4 119. 4 134. 3 149. 1 164. 4 179. 1
105. 3 120. 2 135. 4 150. 1 165. 4 180. 2

35
Brilliant STUDY CENTRE

NEET QUESTION PAPER - 2016 - PHASE - I


1. What is the minimum velocity with 6. Two ide ntical charged spheres
which a body of mass m must enter a suspended from a common point by two
vertical loop of radius R so that it can massless strings of lengths l, are
complete the loop? initially at a distance d(d<<l) apart
because of their mutual repulsion. The
1) 5 gR 2) gR charges begin to leak from both the
spheres at a constant rate. As a result,
3) 2gR 4) 3 gR the spheres approach each other with
a velocity v. Then  varies as a function
2. If the magnitude of sum of two vectors of the distance x between the spheres
is equal to the magnitude of difference as :
of the two vectors, the angle between 1
these vectors is : 1)   x 1 2)   x 2
1) 1800 2) 00 1
3)   x 4)   x2
3) 900 4) 450
3. At what height from the surface of
earth the gravitation potential and the
value of g are 5.4  107 J kg 2 and 7.
6.0 ms 2 respectively? Take the radius
of earth as 6400 km:
1) 2000 km
A capacitor of 2 F is charged as shown
2) 2600 km in the diagram. When the switch S is
3) 1600 km turned to position 2,the percentage of
its stored energy dissipated is :
4) 1400 km 1) 80% 2) 0%
4. A long solenoid has 1000 turns. When 3) 20% 4) 75%
a current of 4A flows through it, the 8. A particle moves so that its position
magnetic flux linked with each turn of vector is given by

the solenoid is 4  103 Wb .The self r  cos t xˆ  sin t yˆ .Where  is a
inductance of the solenoid is : constant. Which of the following is
1) 1 H 2) 4 H true?
1) Velocity is perpendicular to r and
3) 3 H 4) 2 H
acceleration is directed away from the
5. An inductor 20 mH, a capacitor origin.
50F and a resistor 40F are 2) Velocity and acceleration both are
connected in series across a source of parallel to r
emf V = 10 sin 340 t. The power loss in 3) Velocity and acceleration both are
A.C. circuit is : parallel to r
1) 0.89 W 2) 0.51W 4) Velocity is perpendicular to r and
acceleration is directed towards the
3) 0.67 W 4) 0.76 W origin.

36
Previous Years NEET Questions & Solutions

9. From a disc of radius R and mass M, a 13. To get output 1 for the following circuit,
circular hole of diameter R, whose rim the correct choice for the input is :
passes through the centre is cut. What
is the moment of inertia of the
remaining part of disc about a
perpendicular axis, passing through
the centre? 1) A  1,B  0,C  1
2) A  0,B  1,C  0
1) 9MR 2 /32
3) A  1,B  0,C  0
2) 15 MR 2 /32 4) A  1,B  1,C  0
14. In a diffraction pattern due to a single
3) 13MR 2 /32 slit of width ‘a’ the first minimum is
observed at an angle 300 when light of
4) 11MR 2 /32 0
wavelength 5000 A is incident on the
10. The ratio of escape velocity at
slit. The first secondary maximum is
earth  e  to the escape velocity at a observed at an angle of :

planet    whose
p radius and mean 1  3  1  1 
density are twice as that of earth is : 1) sin   2) sin  
4 4
1) 1: 2 2) 1 : 2
1  2  1  1 
3) sin   4) sin  
3) 1: 2 2 4) 1 : 4 3 2

11. A potentiometer wire is 100 cm long 15. When a metallic surface is illuminated
and a constant potential difference is with radiation of wavelength  ,the
maintained across it. Two cells are stopping potential is V. If the same
connected in series first to support one surface is illuminated with radiation
another and then in opposite direction. of wavelength 2 ,the stopping potential
The balance point are obtained at 50 V
cm and 10 cm form the positive end of is .The threshold wavelength for the
the wire in the two cases. The ratio of 4
emf’s is : metallic surface is :

1) 3 : 2 2) 5 : 1 1) 3 2) 4
3) 5 : 4 4) 3 : 4 5
3) 5 4) 
12. A siren emitting a sound of frequency 2
800 Hz moves away from an observer
16. When an   particle of mass ‘m’ moving
towards a cliff at a speed of 15 m s-1.
with velocity ‘v’ bombards on a heavy
Then, the frequency of sound that the
nucleus of charge ‘Ze’, its distance of
observer hears in the echo reflecte
closest approach from the nucleus
from the cliff is :
depends on m as:
(Take velocity of sound in air
1 1
 330 ms ) 1) m 2)
m
1) 885 Hz 2) 765 Hz
3) 800 Hz 4) 838 Hz 1 1
3) 4)
m m2
37
Brilliant STUDY CENTRE

17. Match the corresponding entries of 20. A disk and a sphere of same radius
column 1 with column 2. [Where m is but different masses roll off on two
the magnification produced by the inclined planes of the same altitude
mirror] and length.Whi ch one of the two
Column 1 Column 2 objects gets to the bottom of the plane
first?
A) m = –2 a) Convex mirror
1) Depends on their masses
1 2) Disk
B) m   b) Concave mirror
2 3) Sphere
C) m  2 c) Real image 4) Both reach at the same time
21. Coefficient of linear expansion of brass
1
D) m   d) Virtual image and steel rods are 1 and  2 .Lengths
2
of brass and steel rods are l1 and l2
1) A  c & d;B  b & d; respectively. If (l2 – l1 )is maintained
same at all temperatures, which one
C  b & c;D  a & d
of the following relations holds good?
A  b & c;B  b & c; 1) 1l1   2l 2 2) 1l 2   2l1
2)
C  b & d;D  a & d 3) 1l 22  2l12 4) 12l 2  22l 2
A  a & c;B  a & d; 22. A astronomical telescope has objective
3) and eyepiece of focal lengths 40 cm
C  a & b;D  c & d
and 4 cm respectively. To view an
A  a & d;B  b & c; object 200 cm away from the objective,
4) the lenses must be separated by a
C  b & d;D  b & c
distance:
18. A particle of mass 10 g moves along a 1) 54.0 cm 2) 37.3 cm
circle of radius 6.4 cm with a constant
3) 46.0 cm 4) 50.0 cm
tangential acceleration. What is the
magnitude of this acceleration if the 23. A uniform circular disc of radius 50
kinetic energy of the particle becomes cm at rest is free to turn about an
axis which is perpendicular to its plane
equal to 8  104 J by the end of the
and passes through its centre. It is
second revolution after the beginning subjected to a torque which produces
of the motion? a constant angular acceleration of 2.0
1) 0.2m /s2 2) 0.1m /s2 rad s-2. Its net acceleration in ms-2 at
the end of 2.0 s is approximately:
3) 0.15 m /s2 4) 0.18 m /s2 1) 3.0 2) 8.0
3) 7.0 4) 6.0
19. A small signal voltage V  t   V0 sin t is
applied across an ideal capacitor C: 24. A refrigerator works between 40C and
30 0 C. It is required to remove 600
1) Current I(t) leads voltage V(t) by 1800 calories of heat every second in order
2) Current I(t) lags voltage V(t) by 900 to keep the temperature of the
3) Over a full cycle the capacitor C does refrigerated space constant. The power
not cosume any energy form the voltage required is :
source. (Take 1 cal = 4.2 Joules)
4) Current I(t) is in phase with voltage 1) 2365 W 2) 2.365 W
V(t). 3) 23.65 W 4) 236.5 W
38
Previous Years NEET Questions & Solutions

25. A gas is compressed isothermally to 28. Consider the junction diode as ideal.
half its initial volume.The same gas is The value of current flowing through
compressed separately through an AB is :
adiabatic process until its volume is
again reduced to half. Then:
1) Which of the case (whether
compression through isothermal or
through adiabatic process) requires 1) 103 A 2) 0 A
more work will depe nd upon the 3) 102 A 4) 102 A
atomicity of the gas
29. A car is negotiating a curved road of
2) Compressing the gas isothermally
radius R. The road is banked at an
will require more work to be done.
3) Compressing the gas through angle  .The coefficient of friction
adiabatic process will require more between the tyres of the car and the
work to be done road is s .The maximum safe velocity
4) Compressing the gas isothermally on this road is :
or adiabatically will require the same
amount of work. g s  tan 
1) R 21  s tan 
26. The intensity at the maximum in a
You n g’s d ou b l e sl i t ex p er i m en t i s I 0.
Distance between two slits is 2 s  tan 

d  5 ,where  is the wavelength of 2) gR 1   tan 


s
light used in the experiment. What will
be the intensity in front of one of the s  tan 
slits on the screen placed at a distance 3) gR
1  s tan 
D  10 d ?
I0 g s  tan 
1) 2) I0 4) R 1  s tan 
2
I0 3 30. A long straight wire of radius a carries
3) 4) I0 a steady current I. The current is
4 4
27. Two non-mixing liquids of densities uniformly distributed over its cross-
section. The ratio of the magnetic fields
 and n  n  1 are put in a container.
a
The height of each liquid is h. A solid B and B ,at radial distances and 2a
2
cylinder of length L and density d is
put in this cntainer. The cylinder floats respectively, from the axis of the wire
with its axis vertical and length is :

pL  p  1 in the denser liquid. The 1 1


1) 4 2) 3) 4) 1
density d is equal to: 4 2

1) 1   n  1 p  31. Given the value of Rydberg constant is


107 m1 , the wave number of the last
2) 1   n  1 p  line of the Balmer series in hydrogen
spectrum will be:
3) 2   n  1 p  1
1) 2.5  10 m
7
2) 0.025  104 m1
4) 2   n  1 p  3) 0.5  107 m1 4) 0.25  107 m1
39
Brilliant STUDY CENTRE

32. If the velocity of a particle is 36. The magnetic susceptibility is


  At  Bt 2 ,where A and B are neagative for :
constant, then the distance travelled
by it between 1 s and 2 s is : 1) Paramagnetic and ferromagnetic
materials
A B 3
1)  2) A  4B 2) Diamagnetic material only
2 3 2
3) Paramagnetic material only
3 7
3) 3A  7B 4) A  B
2 3 4) Ferromagnetic material only
33. The angle of incidence for a ray of light
37. An electron of mass m and a photon
at a refracting surface of a prism is
have same energy E. The ratio of de-
450.The angle of prism is 600. If the
Broglie wavelengths associated with
ray suffers minimum deviation through
them is :
the prism, the angle of minimum
deviation and refractive index of the
1 1
material of the prism respectively, are:
1  2m  2 1  E 2
1)   2)  
0 1 0 1 c E  c  2m 
1) 30 , 2) 45 ,
2 2
1
 E 2 1
3) 300 , 2 4) 450 , 2 3)   4) c  2mE  2
 2m 
34. The molecules of a given mass of a gas
have r.m.s. velocity of 200 m s-1 at 270C 38. A body of mass 1 kg begins to move
and 1.0  105 Nm2 pressure. When the under the action of a time dependent

temperature and pressure of the gas  
force F  2t ˆi  3t 2ˆj N ,where î and ˆj
are respectively, 127 0 C and
are unit vectors along x and y axis.
0.05  105 Nm2 ,the r.m.s.velocity of its What power will be developed by the
molecules in ms-1 is : force at the time t?

1)  2t  3t  W 2)  2t  3t  W
3 5 2 3
100
1) 2) 100 2
3
3)  2t  4t  W 4)  2t  3t  W
2 4 3 4

400 100 2
3) 4) 39. The charge flowing through a
3 3
resistance R varies with time t as
35. An air column, closed at one end and Q  at  bt2 ,where a and b are positive
open at the other, resonate with a constants. The total heat produced in
tuning fork when the smallest length R is :
of the column is 50 cm. The next larger
length of the column resonating with a 3R a 3R
the same tuning fork is : 1) 2)
b 6b
1) 200 cm 2) 66.7 cm
a 3R a 3R
3) 4)
3) 100 cm 4) 150 cm 3b 2b

40
Previous Years NEET Questions & Solutions

40. A npn tansistor is connected in 43. A uniform rope of length L and mass
common emitter configuration in a m1 hangs vertically from a rigid support.
given amplifier. A load resistance of A block of mass m2 is attached to the
800  is connected in the collector free end of the rope. A transverse pulse
circuit and the voltage drop across it of wavelength 1 is produced at the
is 0.8 V. If the current amplification lower end of the rope. The wavelength
factor is 0.96 and the input resistance of the pulse when it reaches the top of
of the circuit is 192 ,the voltage gain the rop is  2 . The ratio  2 / 1 is :
and the power gain of the amplifier will
respectively be: m1  m2 m1
1) 2) m2
m1
1) 4,3.69 2) 4, 3.84

3) 3.69,3.84 4) 4,4 m1  m2 m2
3) 4)
m2 m1
41. A piece of ice falls from a height h so
that it melts completely. Only one- 44. A black body is at a temperature of 5760
quarter of the heat produced is K. The energy of radiation emitted by
absorbed by the ice and all energy of the body at wavelength 250 nm is U1,
ice gets converted into heat during its at wavelength 500 nm is U2 and that
fall. The value of h is : at 1000 nm is U 3 . Wien’s constant,
b  2.88  106 nmK .Which of the
[Latent heat of ice is 3.4  105 J / kg and following is corret
g  10N / kg ]
1) U2  U1 2) U1  0
1) 68 km 2) 34 km
3) U3  0 4) U1  U2
3) 544 km 4) 136 km 45. Out of the following options which one
can be used to produce a propagating
42. A square loop ABCD carrying a current
electromagnetic wave?
i, is placed near and coplanar with a
long straight conductor XY carrying a 1) An acceleration charge
current I, the net force on the loop will
2) A charge moving at constant velocit
be:
3) A stationary charges
4) A chargeless particle.
CHEMISTRY
46. Which one of the following
characteristics is associated with
adsorption?
1) G and S are negative but H is
positive
2) G is negative but H and S are
0IiL 20 li
1) 2) positive
2 3
3) G , H and S all are negative
0 li 20 liL 4) G and H are negative but S is
3) 4)
2 3 positive

41
Brilliant STUDY CENTRE

47. The presence of H2 required to make 51. The correct statement regarding the
the potential of H 2 electrode zero in basicity of arylamines is
pure water at 298 K is 1) Arylamines are generally more basic
1) 10–4 atm 2) 10–14 atm than alkylamine s, because the
3) 10–12 atm 4) 10–10 atm nitrogen atom in arylamines is sp-
hybridized.
48. The addition of a catalyst during a
chemical reaction alters which of the 2) Arylamines are generally less basic
following quantities? than alkylamines because the nitrogen
lone pair electrons are delocalized by
1) Activation energy
interaction with the aromatic ring  -
2) Entropy electrons system.
3) Internal energy 3) Arylamines are generally more basic
4) Enthalpy than alkylamines because the nitrogen
49. For the following reaction: lone pair electrons are not delocalized
by interaction with the aromatic ring
a) CH3CH2CH2Br + KOH   -electron system.
CH3CH=CH2+KBr + H2O 4) Arylamines are generally more basic
than alkylamines becau se of aryl
b) H3C CH3 + KOH  group.
52. Equal moles of hydrogen and oxygen
Br gases are placed in a container with a
pin-hole through which both can
H3C CH3 escape. What fraction of the oxygen
escapes in the time required for one-
+ KBr half of the hydrogen to escape?
OH 1) 1/2 2) 1/8
Br 3) 1/4 4) 3/8
53. The correct statement regarding the
c) + Br2  comparison of staggered and eclipsed
Br conformations of ethane, is

Which of the following statements is 1) The staggered conformation of


correct? ethane is more stable than eclipsed
1) (a) is substitution, (b) and (c) are conformation, because staggered
addition reactions conformation has no torsional strain
2) (a) and (b) are elimination reactions 2) The staggered conformation of
and (c) is addition reaction ethane is less stable than eclipsed
3) (a) is elimination, (b) is substitution conformation, because staggered
and (c) is addition reaction conformation has torsional strain
4) (a) is elimination, (b) and (c) are 3) The eclipsed conformation of ethane
substitution reaction is more stable than staggered
50. The product formed by the reaction of conformation, because e clipsed
an aldehyde with a primary amine is conformation has no torsional strain
1) Aromatic acid
4) The eclipsed conformation of ethane
2) Schiff base is more stable than staggered
3) Ketone conformation even though the eclipsed
4) Carboxylic acid conformation has torsional strain
42
Previous Years NEET Questions & Solutions

54. In which of the following options the 59. Match items of column-I with the items
order of arrangement does not agree of column-II and assign the correct
with the variation of property indicated code:
against it?
Column-I Column-II
1) Li < Na < K < Rb (increasing metallic a) Cyanide process i) Ultrapure Ge
radius) b) Froth flotation process ii) Dressin g of Zn S
c) Electrolytic reduction iii) Extraction of Al
2) Al 3+ < Mg2+ < Na+ < F- (increasing d) Zone refining iv ) Extraction of Au
ionic size) v ) Purification of Ni

3) B < C < N < O (increasing first 1) a  iii; b  iv; c  v; d  i


ionization enthalpy)
2) a  iv; b  ii; c  iii; d  i
4) I < Br < Cl < F (increasing electron
gain enthalpy) 3) a  ii; b  iii; c  i; d  v
55. The rate of a first-order reaction is 0.04 4) a  i; b  ii; c  iii; d  iv
mol l –1 s–1 at 10 seconds and 0.03 mol 60. Which one give n below is a non-
l –1 s–1 at 20 seconds after initiation of reducing sugar?
the reaction. The half-life period of the
reaction is 1) Sucrose 2) Maltose

1) 54.1 s 2) 24.1 s 3) Lactose 4) Glucose

3) 34.1 s 4) 44.1 s 61. The correct statement regarding RNA


and DNA, respectively is
56. When copper is heated with conc.HNO3
it produces 1) The sugar component in RNA is 2 -
deoxyribose and the sugar component
1) Cu(NO3)2 and N2O in DNA is arabinose.
2) Cu(NO3)2 and NO2 2) The sugar component in RNA is
arabinose and the sugar component in
3) Cu(NO3)2 and NO
DNA is 2 -deoxyribose.
4) Cu(NO3)2, NO and NO2
3) The sugar component in RNA is
57. In a protein molecule various amino ribose and the sugar component in DNA
acids are linked together by is 2 -deoxyribose.
1) dative bond 4) The sugar component in RNA is
2)  -glycosidic bond arabinose and the sugar component in
DNA is ribose.
3)  -glycosidic bond 62. The correct thermodynamic conditions
for the spontaneous reaction at all
4) Peptide bond
temperatures is
58. Fog is colloidal solution of
1) H  0 and S  0
1) Gas in gas
2) H  0 and S  0
2) Liquid in gas
3) Gas in liquid 3) H  0 and S  0

4) Solid in gas 4) H  0 and S  0


43
Brilliant STUDY CENTRE

63. Which is the correct statement for the 69. Predict the correct order among the
given acids? following:
1) Phosphinic acid is a diprotic acid 1) lone pair - bond pair > bond pair -
while phosphonic acid is a monoprotic bond pair > lone pair - lone pair
acid 2) lone pair-lone pair > long pair - bond
2) Phosphinic acid is a monoprotic acid pair > bond pair-bond pair
while phosphonic acid is a diprotic acid 3) lone -lone pair > bond pair - bond
3) Both are diprotic acids pair > lone pair-bond pair
4) bond pair-bond pair > lone pair-bond
4) Both are triprotic acids
pair > lone pair-long pair
64. M Y an d NY3, two nearly insoluble salts 70. Two electrons occupying the same
have the same Ksp value of 6.2  10 13 orbital are distinguished by
at room temperature, which 1) Spin quantum number
statements would be true in regard to 2) Principal quantum number
MY and NY3? 3) Magnetic quantum number
1) The addition of the salt of KY to 4) Azimuthal quantum number
solution of MY and NY3 will have no 71. The product obtained as a result of a
effect on their solubilities. reaction of nitrogen with CaC2 is
2) The molar solubilities of MY and NY3 1) Ca2CN 2) Ca(CN)2
in water are identical 3) CaCN 4) CaCN3
3) The molar solubility of MY in water 72. Natural rubber has
is less than that of NY3 1) Random cis-and trans-configuration
4) The salts MY and NY 3 are more 2) All cis-configuration
soluble in 0.5 M KY than in pure water 3) All trans-configuration
65. Which of the following is an analgesic? 4) Alternate cis- and trans-
1) Chloromycetin 2) Novalgin configuration
73. Which one of the following orders is
3) Penicillin 4) Streptomyciin
correct for the bond dissociation
66. The pai r of electron in the given enthalpy of halogen molecules?
carbanion, CH3C C is present in 1) F2 > Cl2 > Br2> I2
2) I2 > Br2 > Cl2 > F2
which of the following orbitals?
3) Cl2 > Br2 > F2 > I2
1) sp 2) 2p
4) Br2 > I2 > F2 > Cl2
3) sp 3
4) sp2 74. The reaction:
67. Among the following, the correct order
of acidity is NaH
OH O Na
1) HClO4 < HClO2 < HClO < HClO3
2) HClO3 < HClO4 < HClO2< HClO
3) HClO < HClO2 < HClO3 < HClO4 Me
4) HClO2 < HClO < HClO3 < HClO4 Me  I
 O
68. Which one of the following statements
can be classified as
is correct when SO2 is passed through
acidified K2Cr2O7 solution? 1) Williamson alcohol synthesis
reaction
1) Green Cr2(SO4)3 is formed
2) Williamson ether synthesis reaction
2) The solution turns blue 3) Alcohol formation reaction
3) The solution is decolourized 4) Dehydration reaction
4) SO2 is reduced
44
Previous Years NEET Questions & Solutions

75. Lithium has bcc structure. Its density 80. In the reaction
is 530 kg m–3 and its atomic mass is
6.94 g mol–1. Calculate the edge length H C CH
of a unit cell of Lithium metal
N A  6.02  1023 mol 1  
(1)NaNH2 /liq.NH3
(2) CH3CH2Br
 X 
(1)NaNH2 /liq.NH3
(2) CH3CH2Br
Y
1) 264 pm 2) 154 pm
X and Y are
3) 352 pm 4) 527 pm
76. The ionic radii of A+ and B– ions are 1) X = 1-Butyne; Y=2-Hexyne

0.98  1010 m and 1.81×10 -10 m. The 2) X = 1-Butyne; Y=3-Hexyne


coordination number of each ion in AB
is
3) X = 2-Butyne; Y=3-Hexyne
1) 2 2) 6
D) X = 2-Butyne; Y=2-Hexyne
3) 4 4)8
77. At 100°C the vapour pressure of a 81. Consider the following liquid-vapour
solution of 6.5 g of a solute in 100g equilibrium 
liquid  vapour .
water is 732 mm. If K b = 0.52, the
Which of the following relations is
boiling point of this solution will be
correct?
1) 103°C 2) 101°C
3) 100°C 4) 102°C d ln P Hy d ln G  H y
1)  2) 
78. The electronic configurations of Eu dT RT 2 dT 2 RT 2
(Atomic No. 63) Gd (Atomic No. 64) and
Tb (Atomic No.65) are
d ln P Hy d ln P Hy
1) [Xe]4f 6s , [Xe]4f 5d 6s and [Xe]4f
7 2 7 1 2 9 3)  4) 
dT 2 RT dT 2 T2
6s 2
2) [Xe]4f7 6s2, [Xe]4f8 6s2 and [Xe]4f8 82. Which of the following statements
5d1 6s2 about the composition of the vapour
3) [Xe]4f6 5d1 6s2, [Xe]4f7 5d16s 2 and over an ideal 1 : 1 molar mixture of
[Xe]4f9 5d1 6s2 benzene and tolue ne is correct?
Assume that the temperature is
4) [Xe]4f 6 5d 1 6s 2, [Xe]4f 7 5d 1 6s 2 and constant at 25°C. (Given, vapour
[Xe]4f8 5d1 6s2 pressure data at 25°C, Benzene=12.8
79. Which of the following statements kPa, toluene = 3.85 kPa)
about hydrogen is incorrecT? 1) Not enough information is given to
1) Dihydrogen does not act as a make a prediction
reducing agent
2) The vapour will contain a higher
2) Hydrogen has three isotopes of percentage of benzene
which tritium is the most common
3) The vapour will contain a higher
3) Hydrogen never acts as cation in percentage of toluene
ionic salts
4) Hydronium ion, H 3O+ exists freely 4) The vapour will contain equal amount
in solution of benzene and toluene.

45
Brilliant STUDY CENTRE

83. Which of the following biphenyls is 85. The correct statement regarding a
optically active? carbonyl compound with a hydrogen
atom on its alpha-carbon, is

CH3 1) A carbonyl compound with a


hydrogen atom on its alpha-carbon
rapidly equilibrate s with its
1) corresponding enol and this process is
CH3 known as keto-enol tautomerism

2) A carbonyl compound with a


hydrogen atom on its alpha carbon
O2N ne ver equilibrate s with its
corresponding enol

2) 3) A carbonyl compound with a


hydrogen atom on its alpha-carbon
I rapidly equilibrate s with its
corresponding enol and this process is
known as aldehyde-keto ne
Br Br equilibration

4) a carbonyl compound with a


hydrogen atom on its alpha-carbon
3)
rapidly equilibrate s with its
I I corresponding enol and this process is
known as carbonylation.

86. Con sider t h e m ol ecu les CH 4, NH3 and


I H2O. Which of the given statement is
false?

4) 1) The H–C–H bond angle in CH 4 is


larger than the H–N–H bond angle in
I
NH 3

84. Which of the following reagents would 2) The H–C–H bond angles is CH4 the
distinguish cis-cylopenta-1,2-diol from H–N–H bond angle in NH3 and the H–
the trans-isomer? O–H bond angle in H2O are all greater
than 90°
1) Aluminium isopropoxide 3) Then H–O–H bond angle in H2O is
larger than the H–C–H bond angle in
2) Acetone CH 4

3) Ozone 4) The H–O–H bond angle in H2O is


smaller than the H–N–H bond angle in
NH 3
4) MnO2

46
Previous Years NEET Questions & Solutions

87. Match the compound given in column- BIOLOGY


I with the hybridization and shape given 91. Gause’s principle of compe titive
in column II and mark the correct exclusion states that:
option 1) More abundant species will exclude
the less abundant species through
Column-I Column-II competition.
2) Competition for the same resources
a) XeF 6 i) distorted octahedral exclude s specie s having differe nt
food preferences.
b) XeO 3 ii) square planar 3) No two species can occupy the same
niche indefinitely for the same limiting
c) XeOF 4 iii) pyramidal
resources.
d) XeF 4 iv ) square pyramidal 4) Larger organism exclude smaller
ones through competition.
92. The two polypeptides of human insulin
1) a  iv; b  i; c  ii; d  iii are linked together by:
2) a  i; b  iii; c  iv; d  ii 1) Hydrogen bonds
2) Phosphodiester bond
3) a  i; b  ii; c  iv; d  iii
3) Covalent bond
4) a  iv; b  iii; c  i; d  ii 4) Disulphide bridges
88. Consider the nitration of benzene 93. The coconut water from tender coconut
using mixed conc. H2SO4 and HNO3. If represents:
a larger amount of KHSO4 is added to 1) Endocarp 2) Fleshy mesocarp
the mixture the rate of nitration will 3) Free nuclear proembryo
be
4) Free nuclear endosperm
1) doubled 2) faster 94. Which of the following statements is
3) slower 4) unchanged wrong for viroids?
89. Which of the following statement is 1) They lack a protein coat
false? 2) They are smaller than viruses
1) Mg2+ ions are important in the green 3) They cause infections
parts of plants 4) Their RNA is of high molecular
weight
2) Mg2+ ions form a complex with ATP
95. Which of the following features is not
3) Ca 2+ ions are important in blood present in the Phylum- Arthropoda?
clotting 1) Chitinous exoskeleton
4) Ca 2+ ions are not important in 2) Metameric segmentation
maintaining the regular beating of the 3) Parapodia
heart
4) Jointed appendages
90. Which of the following has longest 96. Which of the following most
C–O bond length? (Free C–O bond appropriately describes haemophilia?
length in CO is 1.128 A0) 1) Recessive gene disorder
 2) X-linked recessive gene disorder
1) Mn  CO 6  2) Ni(CO)4 3) Chromosomal disorder
4) Dominant gene disorder
 2
3) Co  CO 4  4) Fe  CO 4 
47
Brilliant STUDY CENTRE

97. Emerson’s enhancement effect and 103. In a chloroplast the highest number of
Red drop have been instrumental in protons are found in:
the discovery of: 1) Stroma
1) Photophosphorylation and non-cyclic 2) Lumen of thylakoids
electron transport. 3) Inter membrane complex
2) Two photosystems ope rating 4) Antennae complex
simultaneously.
104. Photosensitive compound in human eye
3) Photophosphorylation and cyclic is made up of:
electron transport
1) Guanosine and Retinol
4. Oxidative phosphorylation.
2) Opsin and Retinal
98. In which of the following, all three are
macronutrients? 3) Opsin and Retinol
1) Boron, zinc, manganese 4) Transducin and Retinene
2) Iron, copper, molybdenum 105. Spindle fibres attach on to:
3) Molybdenum, magnesium, 1) Telomere of chromosome
manganese 2) Kinetochore of the chromosome
4) Nitrogen, nickel, phosphorus 3) Centromere of the chromosome
99. Name the chronic respiratory disorder 4) Kinetosome of the chromosome
caused mainly by cigarette smoking: 106. Which is the National Aquatic Animal
1) Emphysema 2) Asthma of India?
3) Respiratory acidosis 1) Gangetic shark 2) River dolphin
4) Respiratory alkalosis 3) Blue whale 4) Sea-horse
100. A system of rotating crops with legume 107. Which of the following is required as
or grass pasture to improve soil inducer(s) for the expression of Lac
structure and fertility is called: operon?
1) Ley farming 2) Contour farming 1) Glucose 2) Galactose
3) Strip farming
3) Lactose
4) Shifting agriculture
4) Lactose and galactose
101. Mitochondria and chloroplast are:
(a) semi -autonomous organelles 108. Which of the foll owing pairs of
hormones are not antagonistic (having
(b) formed by division of pre-existing
opposite effects) to each other?
organelles and they contain DNA but
lack protein synthesizing machinery. 1) Parathormone – Calcitonin
Which one of the following options to 2) Insulin – Glucagon
correct? 3) Aldosterone – Atrial Natriuretic
1) Both (a) and (b) are correct Factor
2) (b) is true but (a) is false 4) Relaxin – Inhibin
109. Microtubules are the constituents of :
3) (a) is true but (b) is false
1) Cilia, Flagella and Peroxisomes
4) Both (a) and (b) are false 2) Spindle fibres, Centrioles and Cilia
102. In context of Amniocentesis, which of 3) Centrioles, Spindle fibres and
the following statement is incorrect? Chromatin
1) It is usually done when a woman is 4) Centrosome, Nucleosome and
between 14-16 weeks pregnant. Centrioles
2) It is used for prenatal sex 110. A complex of ribosomes attached to a
determination. single strand of RNA is known as:
3) It can be used for detection of Down 1) Polysome
syndrome. 2) Polymer
4) It can be used for detection of cleft 3) Polypeptide
palate. 4) Okazaki fragment
48
Previous Years NEET Questions & Solutions

111. Fertilization in humans is practically 118. In meiosis crossing over is initiated at:
feasible only if: 1) Pachytene 2) Leptotene
1) the sperms are transported into 3) Zygotene 4) Diplotene
vagina just after the release of ovum
in fallopian tube 119. A tall true breeding garden pea plant
is crossed with a dwarf true breeding
2) the ovum and sperms are transported garden pea plant. When the F1 plants
simultaneously to ampullary isthmic were selfed the resulting genotypes
junction of the fallopian tube were in the ratio of:
3) the ovum and sperms are transported
1) 1 : 2 : 1 : : Tall homozygous : Tall
simultaneously to ampullary -
heterozygous : Dwarf
isthmic junction of the cervix
4) the sperms are transported into 2) 1 : 2 : 1 : : Tall heterozygous : Tall
cervix within 48 hrs of release of ovum homozygous : Dwarf
in uterus 3) 3 : 1 : : Tall : Dwarf
112. Asthma may be attributed to: 4) 3 : 1 : : Dwarf : Tall
1) bacterial infection of the lungs. 120. Which of the following is the most
2) allergic reaction of the mast cells important cause of animals and plants
in the lungs. being driven to extinction?
3) inflammation of the trachea 1) Over - exploitation
4) accumulation of fluid in the lungs. 2) Alien species invasion
113. The Avena curvature i s used for
3) Habitat loss and fragmentation
bioassay of:
4) Co-extinctions
1) ABA 2) GA3
3) IAA 4) Ethylene 121. Which of the following is a
characteristic feature of cropl and
114. The standard petal of a papilionaceous
ecosystem?
corolla is also called:
1) Carina 2) Pappus 1) Absence of soil organisms
3) Vexillum 4) Corona 2) Least genetic diversity
115. Tricarpellay syncarpous gynoecium is 3) Absence of weeds
found in flowers of: 4) Ecological succession
1) Liliaceae 2) Solanaceae
122. Changes in GnRH pulse frequency in
3) Fabaceae 4) Poaceae
females is controlled by circulating
116. One of the major components of cell levels of :
wall of most fungi is:
1) estrogen and progesterone
1) Chitin 2) Peptidoglycan
3) Cellulose 4) Hemicellulose 2) estrogen and inhibin
117. Select the incorrect statement: 3) progesterone only
1) FSH stimulates the sertoli cells 4) progesterone and inhibin
which help in spermiogenesis 123. Which of the following is not a feature
2) LH triggers ovulation in ovary. of the plasmids?
3) LH and FSH decrease gradually 1) Independent replication
during the follicular phase. 2) Circular structure
4) LH triggers secretion of androgens 3) Transferable
from the Leydig cells.
4) Single- standed
49
Brilliant STUDY CENTRE

124. Which of the following features is not 128. Pick out the correct statements:
present in periplaneta americana? 1) Haemophilia is a sex-linked
recessive disease
1) Schizocoelom as body cavity
2) Down’s syndrome is due to
2) Indeterminate and radial cleavage aneuploidy
during embryonic development 3) Phenylketonuria is an autosomal
3) Exoskeleton composed of N- recessive gene disorder
acetylglucosamine 4) Sickle cell anaemia is a X-linked
recessive gene disorder
4) Metamerically segmented body
1) (a) and (d) are correct
125. In higher vertebrates, the immune 2) (b) and (d) are correct
system can distinguish self-cells and
3) (a), (c) and (d) are correct
non-self. If this property is lost due to
genetic abnormality and it attacks self- 4) (a), (b) and (c) are correct
cells, then it leads to: 129. Which one of the following statements
is wrong?
1) Allergic response 2) Graft rejection
1) Cyanobacteria are also called blue
3) Auto - immune disease green algae
4) Active immunity 2) Golden algae are called desmids
126. Match the terms in column -I with 3) Eubacteria are also called false
their description in column-II and bacteria
choose the correct option. 4) Phycomycetes are also called algal
fungi
Column -I Column II 130. Proximal end of the filament of stamen
Many genes govern is attached to the
(a) Dominance (i)
a single character 1) Anther 2) Connective
In a heterozygous 3) Placenta 4) Thalamus or
(b) Codominance (ii) organism only one allele petal
expresses itself
131. Which of the following approaches does
In a heterozygous not give the defi ned action of
(c)
Pleiotropy (iii) organism both alleles contraceptive?
express themselves fully
Polygenic A single gene influences Barrie r
(d) (iv) 1) Pre ve nt fe rtilization
inheritance many characters me thods
Incre ase phagocytosis of
Intra ute rine
2) spe rm mobility and
de vice
fe rtilizing capacity of
(a) (b) (c) (d) Pre ve nt /re tard e ntry of
Hormonal
3) spe rms, pre ve nt ovulation
1) (ii) (i) (iv) (iii) contrace ptive
and fe rtilization
2) (ii) (iii) (iv) (i) 4) Vasectomy Pre ve nts spe rmatoge ne sis
3) (iv) (i) (ii) (iii)
4) (iv) (iii) (i) (ii) 132. The taq polymerase enzyme is obtained
from:
127. Joint Forest Management Concept was
introduced in India during: 1) Thermus aquaticus
2) Thiobacillus ferroxidans
1) 1960s 2) 1970s
3) Bacillus subtilis
3) 1980s 4) 1990s
4) Pseudomonas putida
50
Previous Years NEET Questions & Solutions

133. Identify the correct statement on 141. Which of the following is not required
inhibin for any of the techniques of DNA
1) Inhibits the secretion of LH, FSH fingerprinting available at present?
and Prolactin. 1) Polymerase chain reaction
2) Is produced by granulose cells in 2) Zinc finger analysis
ovary and inhibits the secretion of FSH. 3) Restriction enzymes
3) Is produced by granulose cells in 4) DNA-DNA hybridization
ovary and inhibits the secretion of LH. 142. Which type of tissue matches with its
4) Is produced by nurse cells in testes location?
and inhibits the secretion of LH.
134. Which part of the tobacco plant is
Tissue Location
infected by Meloidogyne incognita?
1) Flower 2) Leaf 1) Smoth muscle Wall of intestine
3) Stem 4) Root 2) Areolar tissue Tendons
135. Antivenom injectio n contains Transitional
preformed antibodies while polio drops 3) Tip nose
epithelium
that are administered into the body
contain: Cuboidal
4) Lining of stomach
1) Activated pathogens epithelium
2) Harvested antibodies
3) Gamma globulin 143. A plant in your garden avoids
4) Attenuated pathogens photorespiratory, has improved water
136. Which one of the foll owing ce ll use efficiency shows high rates of
organelles to enclosed by a single photosynthesis at high temperatures
membrane? and has improved efficiency of nitrogen
1) Mitochondria 2) Chloroplasts utilisation. In which of the following
3) Lysosomes 4) Nuclei physiological groups would you assign
137. Lack of relaxation between successive this plant?
stimuli in sustained muscle 1) C3 2) C4
contraction is known as: 3) CAM 4) Nitrogen fixer
1) Spasm 2) Fatigue 144. Which of the following structures is
3) Tetanus 4) Tonus homologus to the wing of a bird?
138. Which of the following is not a stem 1) Dorsal fin of a Shark
modification?
2) Wing of a Moth
1) Pitcher of Nepenthes
3) Hind limb of Rabbit
2) Thorns of citrus
3) Tendrils of cucumber 4) Flipper of Whale
145. Which of the following characteristic
4) Flattened structures of Opuntia
features always holds true for the
139. Water soluble pigments found in plant
corresponding group of animals?
cells vacuoles are:
1) Xanthophylls 2) Chlorophylls
Cartilaginous Chondrich-
3) Carotenoids 4) Anthocyanins 1)
140. Select the correct statement: endoskeleton thyes
1) Gymnosperms are bot homosporous 2) Viviparous Mammalia
and heterosporous Possess a mouth with an
2) Salvinia, Ginkgo and Pinus all are 3) Chordata
upper and a lower jaw
gymnosperms
3) Sequoia is one of the tallest trees. 3-chambered heart wih
4) The leaves of gymnosperms are not 4) one incompletely divided Reptilia
well adapted to extremes of climate ventricle
51
Brilliant STUDY CENTRE

146. Which of the following statements is 152. The term ecosystem was coined by:
not true for cancer cells in relation to 1) E.P.Odum 2) A.G.Tansley
mutations?
3) E.Haeckel 4) E.Warming
1) Mutatio ns in proto-oncogenes
accelerate the cell cycle. 153. Which one of the following statements
is wrong?
2) Mutations destroy telomerase
inhibitor. 1) Sucrose is a disaccharide
3) Mutations inactive the cell control 2) Cellulose is a polysaccharide
4) Mutations inhibit production of 3) Uracil is a pyrimidine
telomerase 4) Glycine is a sulphur containing
147. The amino acid Tryptophan is the amino acid
precursor for the synthesis of: 154. In bryophytes and pteridophyte s,
1) Melatonin and Serotonin transport of male gametes requires:
2) Thyroxine and Triiodothyronine 1) Wind 2) Insects
3) Estrogen and Progesterone 3) Birds 4) Water
4) Cortisol and Cortisone
155. Whe n does the growth rate of a
148. Following are the two statements population following the logistic model
regarding the origin of life: equal zero? The logistic model is given
(a) The earliest organisms that as dN/dt = rN(1–N/K):
appeared on the earth were non- green
1) when N/K is exactly one.
and presumably anaerobes
2) when N nears the carrying capacity
(b) The first autotrophic organisms
of the habitat.
were the chemoautotrophs that never
released oxygen. 3) when N/K equals zero.
of the above statements which one of 4) when death rate is greater than
the following options is correct. birth rate.
1) (a) is correct but (b) is false 156. Which one of the following statements
2) (b) is correct but (a) is false is not true?
3) Both (a) and (b) are correct 1) Tapetum helps in the dehiscence of
4) Both (a) and (b) are false anther
149. Reduction in pH of blood will: 2) Exine of pollen grains is made up of
1) reduce the rate of heart beat. sporopollenin
2) reduce the blood supply to the brain 3) Pollen grains of many species cause
severe allergies
3) decrease the affinity of hemoglobin
with oxygen 4) Stored pollen in liquid nitrogen can
be use d in the crop breeding
4) release bicarbonate ions by the liver
programmes
150. Analogous structures are a result of:
157. Which of the following would appear
1) Divergent evolution as the pioneer organisms on bare
2) Convergent evolution rocks?
3) Shared ancestry 1) Lichens 2) Liverworts
4) Stabilizing selection 3) Mosses 4) Green algae
151. Which of the following is a restriction 158. Which one of the following is the
endonuclease? starting codon?
1) Hind II 2) Protease 1) AUG 2) UGA
3) DNAase I 4) RNAase 3) UAA 4) UAG

52
Previous Years NEET Questions & Solutions

159. Which o ne of the following 166. Water vapour comes out from the plant
characteristics is not shared by birds leaf through the stomatal opening.
and mammals? Through the same stomatal opening
1) Ossified endoskeleton carbon dioxide diffuses into the plant
during photosynthesis. Reason out the
2) Breathing using lungs
above statements using one of the
3) Viviparity following options:
4) Warm blooded nature
1) Both processes cannot happen
160. Nomenclature is governed by certain simultaneously
universal rules. Which one of the
following is contrary to the rules of 2) Both processes can happen together
nomenclature? because the diffusion coefficient of
1) Biological names can be written in water and CO2 is different.
any language 3) The above processes happen only
2) The first word in a biological name during night time.
represents the genus name and the 4) One process occurs during day time,
second is a specific epithet and the other at night
3) The names are written in Latin and
167. In mammals, which blood vessel would
are italicised
normally carry largest amount of urea?
4) When written by hand, the names
are to be underlined 1) Renal Vein
161. Blood pressure in the pulmonary artery 2) Dorsal Aorta
is: 3) Hepatic vein
1) same as that in the aorta.
4) Hepatic Portal Vein
2) more than that in the carotid
168. Seed formation without fertilization in
3) more than that in the pulmonary
flowering plants involves the process
vein
of:
4) less than that in the venae cavae
1) Sporulation
162. Cotyledon of maize grain is called:
1) plumule 2) coleorhiza 2) Budding
3) coleoptile 4) scutellum 3) Somatic hybridization
163. In the stomach, gastric acid is secreted 4) Apomixis
by the:
169. Which of the following is wrongly
1) gastrin secreting cells matched in the given table?
2) Parietal cells
3) peptic cells 4) acidic cells
Microbe Product Application
164. Depletion of which gas in the
Trichoderma immunosupp-
atmospheres can lead to an increased 1) Cyclosporion A
polysporum ressive drug
incidence of skin cancers:
lowering of
1) Nitrous oxide 2) Ozone Monascus
2) Statins blood
3) Ammonia 4) Methane purpureus
cholesterol
165. Chrysophytes, Euglenoids, removal of
Dinoflagellates and Slime moulds are 3) Streptococcus Streptokinase clot from
included in the kingdom: blood vessel
1) Monera 2) Protista Clostridium removal of
4) Lipase
3) Fungi 4) Animalia butylicum oil stains
53
Brilliant STUDY CENTRE

170. In a testcross involving F1 dihybrid flies, 174. Specialise d epide rmal cells
more parental-type offspring were surrounding the guard cells are called:
produced than the recombinant-type
offspring. This indicates: 1) Complementary cells
1) The two genes are located on two
2) Subsidiary cells
different chromosomes.
2) Chromosomes failed to separate 3) Bulliform cells 4) Lenticels
during meiosis.
3) The two genes are linked and present 175. Which of the following guards the
on the same chromosome. opening of hepatopancreatic duct into
4) Both of the characters are controlled the duodenum?
by more than one gene
1) Semilunar valve
171. It is much easier for a small animal to
run uphill than for a large animal, 2) Ileocaecal valve
because:
1) It is easier to carry a small body 3) Pyloric sphincter
weight.
4) Sphincter of Oddi
2) Smaller animals have a higher
metabolic rate. 176. Stem modified into flat green organs
3) Small animals have a lower O 2 performing the functions of leaves are
requirement known as:
4) The efficiency of muscles in large
animals is less than in the small 1) Caldodes 2) Phyllodes
animals
172. Which one of the following is not a 3) Phylloclades 4) Scales
characteristic feature during mitosis
in somatic cells? 177. The primitive prokaryotes responsible
1) Spindle fibres for the production of biogas from the
2) Disappearance of nucleolus dung of ruminant animals, include the:
3) Chromosome movement
4) Synapsis 1) Halophiles
173. Which of the following statements is 2) Thermoacidiophiles
not correct?
1) Pollen grains of many species can 3) Methanogens 4) Eubacteria
germinate on the stigma of a flower,
but only one pollen tube of the same 178. A river with an inflow of domestic
species grows into the style. sewage rich in organic waste may
2) Insects that consume pollen or result in:
nectar without bringing about
1) Drying of the river very soon due to
pollination are called pollen/nectar
algal bloom
robbers.
3) Pollen germination and pollen tube 2) Increased population of aquatic food
growth are regulated by chemical web organisms
components of pollen interacting with
those of the pistil. 3) An increased production of fish due
4) Some reptiles have also been to biodegradable nutrients
reported as pollinators in some plant
species. 4) Death of fish due to lack of oxygen

54
Previous Years NEET Questions & Solutions

179. A cell at telophase stage is observed 180. A typical fat molecule is made up of:
by a student in a plant brought from
1) Three glycerol molecules and one
the field. He tells his teacher that this
fatty acid molecules
cell is not like other cells at telophase
stage. There is no formation cell plate 2) One glycerol and three fatty acid
and thus the cell is containing more molecules
number of chromosomes as compared 3) One glycerol and one fatty acid
to other dividing cells. This would molecule
result in:
1) Aneuploidy 4) Three glycerol and three fatty acid
2) Polyploidy molecules.
3) Somaclonal variation
4) Polyteny

55
Brilliant STUDY CENTRE

KEY WITH HINTS

5. 2
1. 1

2. 3

3. 2 6. 4

4. 1

56
Previous Years NEET Questions & Solutions

7. 1 10. 3

11. 1

8. 4

12. 4

9. 3
13. 1

14. 1

57
Brilliant STUDY CENTRE

15. 1 20. 3

21. 1

16. 2

17. 2
22. 1

18. 2

23. 2

19. 3

58
Previous Years NEET Questions & Solutions

24. 4 27. 1

25. 3
28. 3

29. 3

26. 1

59
Brilliant STUDY CENTRE

30. 4 34. 3

31. 4

35. 4 (D) The first minimum resonating length for


closed organp pipe

= 3 × 50 = 150 cm
32. 4 36. 2

37. 2

38. 1
33. 3

60
Previous Years NEET Questions & Solutions

42. 2
39. 2

40. 2

43. 1

We know that,   T
  T (   )
41. 4

61
Brilliant STUDY CENTRE

44. 1 50. 2

51. 2

52. 2
45. 1 To produce an electromagnetic wave we need
46. 3

47. 2

53. 1

48. 1

49. 3

54. 3,4

62
Previous Years NEET Questions & Solutions

64. 3
55. 2

65. 2

66. 1
56. 2

67. 3
57. 4 In a protein molecule various
amino acids are linked together
68. 1
by peptide bonds.
58. 2 Fog is a colloidal solution of
liquid (water-dispersed phase) in
gas (air-dispersion medium).

59. 2 69. 2

70. 1
60. 1
71.
61. 3

62. 4
72. 2

73. 3

63. 2 74. 2

63
Brilliant STUDY CENTRE

78. 1

75. 3
79. 1, 2

80. 2

81. 1
76. 2

82. 2
77. 2

64
Previous Years NEET Questions & Solutions

87. 2

83. 3

84. 2
88. 3

85. 1

89. 4
86. 3

90. 2

an accelerating charge particle

BIOLOGY
91. 3 106. 2 121. 2 136. 3 151. 1 166. 2
92. 4 107. 3 122. 1 137. 3 152. 2 167. 3
93. 4 108. 4 123. 4 138. 1 153. 4 168. 4
94. 4 109. 2 124. 2 139. 4 154. 4 169. 4
95. 3 110. 1 125. 3 140. 3 155. 1 170. 3
96. 2 111. 2 126. 2 141. 2 156. 1 171. 2
97. 2 112. 2 127. 3 142. 1 157. 1 172. 4
98. No ans. 113. 3 128. 4 143. 2 158. 1 173. 1
99. 1 114. 3 129. 3 144. 4 159. 3 174. 2
100. 1 115. 1 130. 4 145. 1 160. 1 175. 4
101. 3 116. 1 131. 4 146. 4 161. 3 176. 3
102. 4 117. 3 132. 1 147. 1 162. 4 177. 3
103. 2 118. 1 133. 2 148. 3 163. 2 178. 4
104. 2 119. 1 134. 4 149. 3 164. 2 179. 2
105. 2 120. 3 135. 4 150. 2 165. 2 180. 2

65
Brilliant STUDY CENTRE

NEET QUESTION PAPER - 2016 - PHASE - II


1. A person can see clearly object only 5. If an electron in a hydrogen atom
when they l ie betwe en 50cm and jumps from the 3rd orbit to the 2nd orbit,
400cm from his eye s. In order to it emits a photon of wavelength  .
increase the maximum distance of When it jumps from the 4th orbit to the
distinct vision to infinity, the type and 3rd orbit, the corresponding wavelength
power of the correcting lens, the of the photon will be:
person has to use, will be:
1) convex, +0.15 diopter 20 16
1)  2) 
13 25
2) convex, +2.25 diopter
3) concave, –0.25 diopter 9 20
3)  4) 
4) concave, –0.2 diopter 16 7

2. A linear aperture whose width is 0.02cm 6. The half-life of a radioactive substance


is placed immediately in front of a lens is 30 minutes. The time (in minutes)
of focal length 60cm. The aperture is taken between 40% decay and 85%
illuminated normally by a parallel beam decay of the same radioactive
of wavelength 5 × 10–5cm. The distance substance is:
of the first dark band of the diffraction 1) 60 2) 15
pattern from the centre of the screen
is: 3) 30 4) 45
1) 0.15 cm 2) 0.10 cm 7. For CE transistor amplifier, the audio
signal voltage across the collector
3) 0.25 cm 4) 0.20 cm
resistance of 2K is 4V. If the current
3. Electrons of mass m with de-Broglie
amplification factor of the transistor is
wavelength  fall on the target in an
100 and the base resistance is 1K ,
X-ray tube. The cutoff wavelength   0  then the input signal voltage is:
of the emitted X-ray is:
1) 15 mV 2) 10 mV
2mc 2
3) 20 mV 4) 30 mV
1)  0   2)  0 
h
8. The given circuit has two ideal diodes
2h 2m2c 2 3 connected as shown in the figure
3) 0  4)  0  below. The current flowing through the
mc h2
resistance R1 will be:
4. Photons with energy 5eV are incident
on a cathode C in a photoelectric cell.
The maximum energy of e mitted
photoelectrons is 2eV. When photons
of energy 6eV are incident on C, no
photoelectrons will reach the anode A,
if the stopping potential of A relative to
C is:
1) –3 V 2) +3 V 1) 3.13 A 2) 2.5 A
3) 10.0 A 4) 1.43 A
3) +4 V 4) –1 V
66
Previous Years NEET Questions & Solutions

9. What is the output Y in the following 13. A rigid ball of mass m strikes a rigid
circuit, when all the three inputs A, wall at 60o and gets reflected without
B, C are first 0 and then 1? loss of speed as shown in the figure
below. The value of impulse imparted
by the wall in the ball will be

1) 1, 1 2) 0, 1
3) 0, 0 4) 1, 0
10. Planck’s constant (h), speed of light in
vacuum (c) and Newton’s gravitational
constant (G) are the three fundamental
constants. Which of the following
combinations of these has dimension
of length? mV
1) 2) mV
3
Gc hG
1) 2)
h3/2 c 3/2 mV
3) 2mV 4)
2
hG hc
3) 4) 14. A bullet of mass 10 g moving
c 5/2 G horizontally with a velocity of 400m–1
11. Two cars P and Q start from a point at strikes of wooden block of mass 2 kg
the same time in a straight line and which is suspended by a light
their positions are represented by inextensible string of length 5m. As a
result the centre of gravity of the block
x P (t)  at  bt 2 and x Q (t)  ft  t . At
2
is found to rise a vertical distance of
what time do the cars have the same 10 cm. The speed of the bullet after it
velocity? emerges out horizontally from the
f a block will be
af
1) 2 1  b  2) 1) 160 ms–1 2) 100 ms–1
1 b
3) 80 ms–1 4) 120 ms–1
af af 15. Two identical balls A and B having
3) 2  b  1 4) 2 1  b  velocities of 0.5 m/s and –0.3 m/s
respectively collide elastically in one
12. In the given figure, a = 15m/s 2 dimension. The velocities of B and A
represents the total acceleration of a after the collision respectively will be
particle moving in the clockwise
1) 0.3 m/s and 0.5 m/s
direction in a circle of radius R = 2.5m
at a given instant of time. The speed 2) –0.5 m/s and 0.3 m/s
of the particle is 3) 0.5 m/s and –0.3 m/s
4) –0.3 m/s and 0.5 m/s

16. A particle moves from a point 2iˆ  5ˆj  



to 4ˆj  3kˆ  
when a force of 4iˆ  3ˆj N
is applied. How much work has been
done by the force?
1) 6.2 m/s 2) 4.5 m/s 1) 2 J 2) 8 J
3) 5.0 m/s 4) 5.7 m/s 3) 11 J 4) 5 J
67
Brilliant STUDY CENTRE

17. Two rotating bodies A and B of masses 21. A satellite of mass m is orbitting the
m and 2m with moments of inertia IA earth (of radius R) at a height h from
and I B  IB  IA  have equal kinetic its surface. The total energy of the
satellite in terms of g 0, the value of
energy of rotation. If LA and LB be their acceleration due to gravity at the
angular momenta respectively, then earth’s surface, is
LB
1) LA > LB 2) L A  2mg 0R 2 mg 0R 2
2 1)  2) 2 R  h
Rh  
3) LA = 2LB 4) LB > LA
18. A solid sphere of mass m and radius R mg 0 R 2 Rmg 0R 2

3) 2 R  h 4)
is rotating about its diameter. A solid   Rh
cylinder of the same mass and same
radius is also rotating abo ut its 22. A rectangular film of liquid is extended
geometrical axis with an angular speed from (4 cm × 2 cm) to (5 cm × 4 cm). If
twice that of the sphere. The ratio of the work done is 1 × 10–4J, the value
their kinetic energies of rotation of the surface tension of the liquid is
(Esphere/Ecylinder) will be: 1) 8.0 Nm–1 2) 0.250 Nm–1
1) 3 : 1 2) 2 : 3 3) 0.125 Nm–1 4) 0.2 Nm–1
3) 1 : 5 4) 1 : 4 23. Three liquids of densities 1, 2 and 3
19. A light rod of length l has two masses
(with 1  2  3 ), having the same
m1 and m2 attached to its two ends.
The moment of inertia of the system vaule of surface tension T, rise to the
about an axis perpendicular to the rod same he ight in thre e identical
and passing through the centre of mass capillaries. The angles of contact 1, 2
is: and 3 obey
m1m2 2
1) m1m2 l 2 2) m  m l 
1 2 1)   1  2  3 
2
m1m2 2 
3) m  m l 4)  m1  m2  l 2 2)  1  2  3  0
1 2 2
20. Starting from the centre of the earth 
having radius r, the variation of g 3) 0  1  2  3 
2
(acceleration due to gravity) is shown
by 
4)  1  2  3  
2
24. Two identical bodies are made of a
material for which the heat capacity
1) 2) increases with temperature. One of
these is at 100oC, while the other one
is at 0oC. If the two bodies are brought
into contact, then assuming no heat
loss, the final common temperature is
1) 0oC
2) 50oC
3) 4)
3) more than 50oC
4) less than 50oC but greater than 0oC
68
Previous Years NEET Questions & Solutions

25. A body cools from a temperature 3T to 29. A body of mass m is attached to the
2T in 10 minute s. The room lower end of a spring whose upper end
temperature is T. A ssume that is fixed. The spring has negligible
Newton’s law of cooling is applicable. mass. When the mass m is slightly
The temperature of the body at the end pulled down and released, it oscillates
of next 10 minutes will be with a time period of 3s. When the mass
m is increased by 1 kg, the time period
7 of oscillations becomes 5s. The value
1) T 2) T of m in kg is:
4

3 4 9 3
3) T 4) T 1) 2)
2 3 16 4

26. One mole of an ideal monoatomic gas 4 16


undergoes a process described by the 3) 4)
3 9
equation PV 3 = constant. The heat
capacity of the gas during this process 30. The second overtone of an open organ
is: pipe has the same frequency as the
first overtone of a closed pipe L meter
3 long. The length of the open pipe will
1) R 2) R
2 be

1) 4L 2) L
5
3) R 4) 2R
2
L
3) 2L 4)
27. The temperature inside a refrigerator 2
is t 2oC and the room temperature is
t1oC. The amount of heat delivered to 31. Three sound waves of equal amplitudes
the room for each joule of electrical have frequencies (n – 1), n, (n + 1).
energy consumed ideally will be They superimpose to give beats. The
number of beats produced per second
will be
t1  t 2 t1
1) t  273 2) t  t
1 1 2 1) 2 2) 1

t1  273 t 2  273 3) 4 4) 3
3) t  t 4) t  t
1 2 1 2
32. An electric dipole is placed at an angle
of 30o with an electric field intensity 2
28. A given sample of an ideal gas occupies
× 10 5 N/C. It experiences a torque
a volume V at a pressure P and absolute
equal to 4 Nm. The charge on the
temperature T. The mass of each
dipole, if the dipole length is 2 cm, is
molecule of the gas is m. Which of the
following gives the density of the gas?
1) 7C 2) 8mC
1) mKT 2) P/(KT)
3) 2mC 4) 5mC
3) Pm/(KT) 4) P/(kTV)

69
Brilliant STUDY CENTRE

33. A parallel-plate capacitor of area A, 36. A long wire carrying a steady current
plate separation d and capacitance C is bent into a circular loop of one turn.
is filled with four dielectric materials The magnetic field at the centre of the
having dielectric constant k1, k2, k3 and loop is B. It is then bent into a circular
k4 as shown in the figure below. If a coil of n turns. The magnetic field at
single dielectric material is to be used the centre of this coil of n turns will
to have the same capacitance C in this be:
capacitor, then its dielectric constant
k is given by 1) 2n2B 2) nB

3) n2B 4) 2nB

37. A bar magnet is hung by a thin cotton


thread in a uni form horizontal
magnetic field and is in equilibrium
state. The energy required to rotate it
by 60o is W. Now the torque required
to keep the magnet in this new position
is:

1 1 1 1 3 2W W
1) k  k  k  k  2k 1)
3
2)
3
1 2 3 4

2) k = k1 + k2 + k3 + 3k4 3W
3) 3W 4)
2
2
3) k   k1  k 2  k 3   2k 4
3 38. An electron is moving in a circular path
under the influence of a transverse
2 3 1 magnetic field of 3.57 × 10–2 T. If the
4) k  k  k  k  k value of e/m is 1.76 × 1011 C/kg, the
1 2 3 4
frequency of revolution of the electron
34. The potential diffe rence  VA  VB  is:
between the points A and B in the 1) 6.82 MHz
given figure is:
2) 1 GHz
3) 100 MHz
4) 62.8 MHz

1) +9 V 2) –3 V 3) +3 V 4) +6 V 39. Which of the following combinations


should be selected for better tuning of
35. A filament bulb (500 W, 100 V) is to be an L-C-R circuit used for
used in a 230 V main supply. When a communication?
resistance R is connected in series, it
works perfectly and the bulb consumes 1) R  25 ,L  1.5 H,C  45F
500 W. The value of R is:
2) R  20 ,L  1.5 H,C  35F
1) 13  2) 230 
3) R  25 ,L  2.5 H,C  45F
3) 46  4) 26 
4) R  15 ,L  3.5 H,C  30F
70
Previous Years NEET Questions & Solutions

40. A uniform magnetic field is restricted


within a region of radius r. The
43. Two ide ntical glass  g  3 / 2
magnetic field changes with time at a equiconvex lenses of focal length f each
 are kept in contact. The space between
dB the two lenses is filled with water
rate . Loop 1 of radius R > r encloses
dt
the region r and loop 2 of radius R is
 w  4 / 3  . The focal length of the
outside the region of magnetic field as combination is
shown in the figure below. Then the 3f f
e.m.f. generated is: 1) 2)
4 3

4f
3) f 4)
3
44. An air bubble in a glass slab with
refractive inde x 1.5 (near normal
incidence) is 5 cm deep when viewed
from one surface and 3 cm deep when
viewed from the opposite face. The
thickness (in cm) of the slab is
 1) 16 2) 8
dB 2
1)   r in loop 1 and zero in loop 2 3) 10 4) 12
dt
45. The interference pattern is obtained
2) zero in loop 1 and zero in loop 2 with two coherent light sources of
intensity ratio n. In the interference
 
dB 2 dB 2 Imax  Imin
3)   r in loop 1 and   r in pattern, the ratio I will be
dt dt max  Imin

loop 2
2 n n
 1) 2)
 n  1
2
dB n 1
4)   R 2 in loop 1 and zero in loop 2
dt
2 n n
3) 4)
 n  1
2
41. The potential differences across the n 1
resistance, capacitance and
CHEMISTRY
inductance are 80V, 40V and 100V
46. A given nitrogen containing aromatic
respectively in an L-C-R circuit. The
compound A reacts with Sn/H Cl,
power factor of this circuit is:
followed by HNO2 to give an unstable
1) 1.0 2) 0.4 3) 0.5 4) 0.8 compound B. B, on treatment with
phenol, forms a beautiful coloured
42. A 100  resistance and a capacitor of compound C with the molecular
formula C 12H10N 2O. The structure of
100  reactance are co nnected in
compound A is:
series across a 220 V source. When
the capacitor is 50% charged, the peak
value of the displacement current is 1) 2)

1) 11 2 A 2) 2.2 A

3) 11 A 4) 4.4 A 3) 4)

71
Brilliant STUDY CENTRE

47. Consider the reaction: 51. The compound that will react most
CH3 CH2CH2Br  NaCN  CH3 CH2CH2 CN  NaBr readily with gaseous bromine has the
This reaction will be the fastest in: formula:
1) water 1) C2H4 2) C3H6
2) ethanol
3) C2H2 4) C4H10
3) methanol
4) N, N -dimethylformamide (DMF) 52. Which one of the following compounds
48. The correct structure of the product A shows the presence of intramolecular
formed in the reaction: hydrogen bond?
1) Concentrated acetic acid
2) H2O2
3) HCN
4) Cellulose
53. The molar conductivity of a 0.5 mol/
dm3 solution of AgNO3 with electrolytic
conductivity of 5.76 × 10–3 S cm–1 at
298 K is
1) 2) 1) 28.8 S cm2/mol
2) 2.88 S cm2/mol
3) 11.52 S cm2/mol
4) 0.086 S cm2/mol
3) 4) 54. The decomposition of phosphine (PH3)
on tungsten at low pressure is a first
order reaction. It is because the
49. Which among the given molecules can
exhibit tautomerism? 1) Rate of decomposition is very slow
2) Rate is proportional to the surface
coverage
3) Rate is inversely proportional to the
surface coverage
1) Both II and III 2) III only 4) Rate is independent of the surface
3) Both I and III 4) Both I and II coverage
50. The correct order of strengths of the 55. The coagulation values in millimoles
carboxylic acids is per litre of the electrolytes used for
the coagulation of As 2 S 3 are given
below:
I. (NaCl) = 52
II. (BaCl2) = 0.69
III. (MgSO4) = 0.22
The correct order of their coagulating
power is
1) III > I > II 2) I > II > III
1) II > I > III 2) I > II > III
3) II > III > I 4) III > II > I 3) II > I > III 4) III > II > I

72
Previous Years NEET Questions & Solutions

56. During the electrol ysis of molten 62. If the Eocell for a given reaction has a
sodium chloride, the time required to
negative value, which of the following
produce 0.10 mol of chlorine gas using
gives the correct relationships for the
a current of 3 amperes is
valu es of G o and Keq?
1) 330 minutes 2) 55 minutes
3) 110 minutes 4) 220 minutes 1) G o  0; K eq  1 2) G o  0; K eq  1
57. How many electrons can fit in the
3) G o  0; K eq  1 4) G o  0; K eq  1
orbital for which n = 3 and l = 1?
1) 14 2) 2 63. Which one of the following is incorrect
for ideal solution?
3) 6 4) 10
1) G mix  0
58. For a sample of perfect gas when its
pressure is changed isothermally from 2) Hmix  0
pi to pf, the entropy change is given by
3) Umix  0
 pi 
1) S  RTl n  p  4) P  Pobs  Pcalculated by Raoult 's law  0
 f
64. The solubility of AgCl(s) with solubility
 pf  product 1.6 × 10–10 is in 0.1 M NaCl
2) S  nRl n  p  solution would be
 i
1) Zero 2) 1.26 × 10–5 M
 pi  3) 1.6 × 10–9 M 4) 1.6 × 10–11 M
3) S  nRl n  p  65. Suppose the elements X and Y combine
 f to form two compounds XY2 and X3Y2.
When 0.1 mole of XY2 weighs 10g and
 pf  0.05 mole of X 3 Y 2 weighs 9 g, the
4) S  nRTl n  p 
 i atomic weights of X and Y are

59. The van’t Hoff factor (i) for a dilute 1) 30, 20 2) 40, 30
aqueous solution of the strong 3) 60, 40 4) 20, 30
electrolyte barium hydroxide is
66. The number of electrons delivered at
1) 3 2) 0 the cathode during electrolysis by a
3) 1 4) 2 current of 1 ampere in 60 seconds is
60. The percentage of pyridine (C5H5N) that (charge on electron = 1.60 × 10–19 C)
1) 7.48 × 1023
forms pyridinium ion  C H N H
5 5

in a
2) 6 × 1023
0.10 M aqueous pyridine solution (Kb
3) 6 × 1020
for C5H5N = 1.7 × 10–9) is
4) 3.75 × 1020
1) 1.6% 2) 0.0060%
67. Boric acid is an acid because its
3) 0.013% 4) 0.77% molecule
61. In calcium fluoride, having the fluorite 1) Combines with proton from water
structure, the coordination numbers for molecule
calcium ion (Ca2+) and fluoride ion (F–) 2) Contains replaceable H+ ion
are
3) Gives up a proton
1) 4 and 8 2) 4 and 2
4) Accepts OH – from water releasing
3) 6 and 6 4) 8 and 4
proton
73
Brilliant STUDY CENTRE

68. AlF3 is soluble in HF only in presence 75. Hot concentrated sulphuric acid is a
of KF. It is due to the formation of moderately strong oxidizing agent.
Which of the following reactions does
1) K  AlF3H 2) K 3  AlF3H3 
not show oxidizing behaviour?
3) K 3  AlF6  4) AlH3 1) CaF2  H2SO4  CaSO4  2HF
69. Zinc can be coated on iron to produce
2) Cu  2H2SO4  CuSO4  SO2  2H2O
galvanized iron but the reverse is not
possible. It is because 3) 2S  2H2SO4  2SO2  2H2O
1) Zinc has higher negative electrode
potential than iron 4) C  2H2SO4  CO2  2SO2  2H2O
2) Zinc is lighter than iron 76. Which of the following pairs of d-
3) Zinc has lower melting point than orbitals will have electron density
iron along the axis?
4) Zinc has lower negative electrode
1) d xy ,d x 2  y 2 2) d z2 ,d xz
potential than iron
70. The suspension of slaked lime in water 3) d xz ,d yz 4) d z 2 ,d x 2  y 2
is known as
1) Aqueous solution of slaked lime 77. The correct geometry and hybridization
2) Limewater for XeF4 are
3) Quicklime 1) square planar, sp3d2
4) Milk of lime 2) octahedral, sp3d2
71. The hybridizations of atomic orbitals of 3) trigonal bipyramidal, sp3d
nitrogen in NO2 ,NO3 and NH4 4) planar triangle, sp3d3
respectively are 78. Among the following, which one is a
1) sp 2, sp and sp3 wrong statement?
2) sp, sp3 and sp2 1) I3 has bent geometry
3) sp2, sp3 and sp
2) PH5 and BiCl5 do not exist
4) sp, sp2 and sp3
72. Which of the following fluoro- 3) p  d bonds are present in SO2
compounds is most likely to be have as 4) SeF4 and CH4 have same shape
a Lewis base? 79. The correct increasing order of trans-
1) SiF4 2) BF3 effect of the following species is
3) PF3 4) CF4
73. Which of the following pairs of ions is 1) CN  Br   C6H5  NH3
isoelectronic and isostructural? 2) NH3  CN  Br   C6H5
1) ClO3 ,SO32 2) CO23 ,NO3 3) CN  C6H5  Br   NH3
3) ClO3 ,CO32 4) SO23 ,CO23 4) Br   CN  NH3  C6H5
74. In context with beryllium, which one 80. Which one of the following statements
of the following statements is related to lanthanons is incorrect?
incorrect? 1) Ce(+4) solutions are widely used as
1) Its hydride is electron-deficient and oxidizing agent in volumetric analysis.
polymeric. 2) Europium shows +2 oxidations state.
2) It is rendered passive by nitric acid 3) The basicity decreases as the ionic
3) It forms Be2C radius decreases from Pr to Lu.
4) Its salts rarely hydrolyze 4) All the lanthanons are much more
reactive than aluminium.
74
Previous Years NEET Questions & Solutions

81. Jahn-Teller effect is not observed in 85. In pyrrole


high spin complexes of
1) d9 2) d7 3) d8 4) d4
82. Which of the following can be used as
the halide component for Friedel-Crafts
reaction?
1) Isopropyl chloride
2) Chlorobenzene
3) Bromobenzene
4) Chloroethene
83. In which of the following molecules, the electron density is maximum on
all atoms are coplanar? 1) 2 and 5 2) 2 and 3
3) 3 and 4 4) 2 and 4
86. Which of the following compounds shall
not produce propene by reaction with
1) 2) HBr followed by elimination or direct
only elimination reaction?

3) 4) 1) 2)

84. Which one of the following structures


represent nylon 6,6 polymer? 3) 4)
O
O H
87. Which one of the following nitro-
1) C (CH2)4 C N (CH2)6 NH compounds does not react wtih nitrous
n acid?

1) 2)
2)

3) 4)

3)
88. The central dogma of molecular
genetics state s that the genetic
information flows from
1) DNA  RNA  Carbohydrates
2) Amino acids  Proteins  DNA
4)
3) DNA  Carbohydrates  Proteins

4) DNA  RNA  Proteins


75
Brilliant STUDY CENTRE

89. The correct corresponding order of BIOLOGY


names of four aldoses with 91. Which one of the following generates
configuration given below new genetic combinations leading to
variation
1) Sexual reproduction
2) Nucellar polyembryony
3) Vegetative reproduction
4) Parthenogenesis
92. Match the column I with Column II and
select the correct option using the
codes given below

Column I Column II
a. Pistils fuse d
i. Game toge ne sis
toge the r
b. Formation of gametes ii. Pistillate
c. Hyphae of higher
respectively, is iii. Syncarpous
Ascomycetes
1) D-erythrose, D-threose, L-erythrose,
d. Unisexual female flower iv. Dikaryotic
L-threose
2) L-erythrose, L-threose, L-erythrose,
D-threose 1) a-i, b-ii, c-iv, d-iii
3) D-threose, D-erythrose, L-threose, 2) a-iii, b-i,c-iv, d-ii
L-erythrose 3) a-iv, b-iii, c-i, d-ii
4) L-erythrose, L-threose, D-erythrose, 4) a-ii, b-i, c-iv, d-iii
D-threose 93. In majority of angiosperms
90. In the given reaction 1) Reduction division occurs in the
megaspore mother cells
2) A small central cell is present in
the embryo sac
3) Egg has a filiform apparatus
the product P is 4) There are numerous antipodal cells
94. Pollination in water hyacinth and
water lily is brought about by the
agency of
1) Birds
1) 2)
2) Bats
3) Water
4) Insects or wind
95. The ovule of an angiosperm is
technically equivalent to
3) 4)
1) Megaspore mother cell
2) Megaspore
3) Megasporangium
4) Megasporophyll
76
Previous Years NEET Questions & Solutions

96. Taylor conducted the experiments to 103. Which of the following is not a
provide semiconservative mode of component of down stream processing
chromosome replication on 1) Preservation 2) Expression
1) Drosophila melanogaster 3) Separation 4) Purification
2) E. coli 104. Which of the following restriction
3) Vinca rosea enzymes produces blunt ends
4) Vicia faba 1) Xho I 2) Hind III
97. The mechanism that causes a gene to 3) Sal I 4) Eco RV
move from one linkage group to another 105. Which kind of therapy was given in
is called 1990 to a four-year-old girl with
1) Translocation adenosine deaminase (ADA) deficiency
2) Crossing-over 1) Immunotherapy
3) Inversion
2) Radiation therapy
4) Duplication
3) Gene therapy
98. The evuivalent of a structural gene is
4) Chemotherapy
1) Operon 2) Recon
106. How many hot spots of biodiversity in
3) Muton 4) Cistron
the world have been identified till date
99. A true breeding plant is
by Norman Mayer
1) Near homozygous and produces
1) 34 2) 43
offspring of its own kind
3) 17 4) 25
2) Always homozygous recessive in its
genetic constitution 107. The primary producers of the deep-sea
hydrothermal vent ecosystem are
3) One that is able to breed on its own
1) Blue-green algae
4) Produced due to cross-pollination
among unrelated plants 2) Coral reefs
100. Which of the following rRNAs acts as 3) Green algae
structural RNA as well as ribozyme in 4) Chemosynthetic bacteria
bacteria 108. Which of the following is correct for
1) 23 SrRNA 2) 5.8 S rRNA r-selected species?
3) 5 SrRNa 4) 18 SrRNA 1) Small numbers of progeny with small
101. Stirred-tank bioreactors have been size
designed for 2) Small numbers of progeny with large
1) Availability of oxygen throughout the size
process 3) Large numbers of progeny with small
2) Ensuring anaerobic conditions in the size
culture vessel 4) Large numbers of progeny with large
3) Purification of product size
4) Addition of preservatives to the 109. If ‘+’ sign is assigned to beneficial
product interaction, ‘–’ sign to detrimental and
‘0’ sign to neutral interaction, then the
102. A foreign DNA and plasmid cut by the
population interaction represented by
same restriction endonuclease can be
‘+’ ‘–’ refers to
joined to form a recombinant plasmid
using 1) Commensalism
2) Parasitism
1) Polymerase III 2) Ligase
3) Mutualism
3) Eco RI 4) Taq polymerase
4) Ammensalism
77
Brilliant STUDY CENTRE

110. Which of the following is correctly 116. Conifers are adapted to tolerate
matched extreme environmental conditions
1) Parthenium hysterophorus-Threat to because of
biodiversity 1) Thick cuticle
2) Stratification - Population 2) Presence of vessels
3) Aerenchyma-Opuntia 3) Broad hardy leaves
4) Superficial stomata
4) Age pyramid - Biome
117. Which one of the following statements
111. Red list contains data or information
is wrong
on
1) Agar-agar is obtained from Gelidium
1) Threatened species and Gracilaria
2) Marine vertebrates only 2) Laminaria and Sargassum are used
3) All economically important plants as food
4) Plants whose products are in 3) Algae increase the level of dissolved
international trade oxygen in the immediate environment
112. Which one of the following is wrong for 4) Algin is obtained from red algae, and
fungi? carrageenin from brown algae
1) They are heterotrophic 118. The term ‘polyadelphous’ is related to
1) Corolla
2) They are bo th unicellular and
multicellular 2) Calyx
3) They are eukaryotic 3) Gynoecium
4) Androecium
4) All fungi possess a purely cellulosic
cell wall 119. How many plants among Indigofera,
Se sbania, Salivia, Allium, Aloe,
113. Methanogens belong to mustard, groundnut, radish, gram and
1) Dinoflagellates turnip have stamens with different
2) Slime moulds lengths in their flowers
1) Five 2) Six
3) Eubacterta
3) Three 4) Four
4) Archaebacteria
120. Radial symmetry is found in the flowers
114. Select the wrong statement of
1) Diatoms are chief producers in the 1) Pisum
oceans 2) Cassia
2) Diatoms are microscopic and float 3) Brassica
passively in water 4) Trifolium
3) The walls of diatoms are easily 121. Free-central placentation is found in
destructible 1) Brassica
4) ‘Diatomaceous earth’ is formed by 2) Citrus
the cell walls of diatoms 3) Dianthus
115. The label of a herbarium sheet does 4) Argemone
not carry information on 122. Cortex is the region found between
1) Local names 1) Endodermis and pith
2) Height of the plant 2) Endodermis and vascular bundle
3) Date of collection 3) Epidermis and stele
4) Name of collector 4) Pericycle and endodermis
78
Previous Years NEET Questions & Solutions

123. The balloon-shaped structures called 130. A few drops of sap were collected by
tyloses cutting across a plant ste m by a
1) Are extensions of xylem parenchyma suitable method. The sap was tested
cells into vessels chemically. Which one of the following
test results indicates that it is phloem
2) Are linked to the ascent of sap sap
through xylem vessels
1) Low refractive index
3) Originate in the lumen of vessels 2) Absence of sugar
4) Characterize the sapwood 3) Acidic
124. A non-proteinaceous enzyme is 4) Alkaline
1) Ligase 131. You are given a tissue with its potential
2) Deoxyribonuclease for differentiation in an artificial
culture. Which of the following pairs
3) Lysozyme
of hormones would you add to the
4) Ribozyme medium to secure shoots as well as
125. Select the mismatch roots
1) Protists-Eukaryotes 1) Auxin and abscisic acid
2) Methanogens-Prokaryotes 2) Gibberellin and abscisic acid
3) IAA and gibberellin
3) Gas vacuoles-Green bacteria
4) Auxin and cytokinin
4) Large central vacuoles-Animal cells
132. Phytochrome is a
126. Select the wrong statement
1) Lipoprotein 2) Chromoprotein
1) Cyanobacteria lack flagellated cells 3) Flavoprotein 4) Glycoprotein
2) Mycoplasma is a wall-less 133. Which is essential for the growth of
microorganism root tip
3) Bacterial cell wall is made up of 1) Ca 2) Mn 3) Zn 4) Fe
peptidoglycan 134. The process which makes maj or
4) Pili and fimbriae are mainly involved differ en ce bet ween C 3 and C4 plants is
in motility of bacterial cells 1) Photorespiration
127. A cell organelle containing hydrolytic 2) Respiration
enzymes is 3) Glycolysis
1) Ribosome 2) Mesosome 4) Calvin cycle
3) Lysosome 4) Microsome 135. Which one of the following statements
128. During cell growth, DNA synthesis is not correct
takes place in 1) In potato, banana and ginger, the
1) G2 phase 2) M phase plantlets arise from the internodes
3) S phase 4) G1 phase present in the modified stem
129. Which of the following biomolecules is 2) Water hyacinth, growing in the
common to respiration-mediated standing water, drains oxygen from
breakdown of fats, carbohydrates and water that leads to the death of fishes
proteins 3) Offspring produced by the asexual
1) Pyruvic acid reproduction are called clone
2) Acetyl CoA 4) Microscopic, motile asexual
3) Glucose -6-phosphate reproductive structures are called
zoospores
4) Fructose 1, 6-bisphosphate
79
Brilliant STUDY CENTRE

136. The part of nephron involved in active 142. Se veral hormones like hCG, hPL,
reabsorption of sodium is estrogen, progesterone are produced by
1) Bowman’s capsule 1) Fallopian tube
2) Descending limb of Henle’s loop 2) Pituitary
3) Distal convoluted tubule 3) Ovary
4) Proximal convoluted tubule
4) Placenta
137. Which of the following is hormone-
143. If a colour-blind man marries a woman
releasing IUD
who is homozygous for normal colour
1) Lippes loop 2) Cu7 vision, the probability of their son being
3) LNG-20 4) Multiload 375 colour-blind is
138. Which of the following is incorrect 1) 0.75 2) 1
regarding vasectomy 3) 0 4) 0.3
1) Vasa deferentia is cut and tied 144. Genetic drift operates in
2) Irreversible sterility 1) Non-reproductive population
3) No sperm occurs in seminal fluid 2) Slow reproductive population
4) No sperm occurs in epididymis 3) Small isolated population
139. Embryo with more than 16 blastomeres 4) Large isolated population
formed due to in vitro fertilization is
145. In Hardy-Weinberg equ ation, the
transferred into
frequency of heterozygous individual is
1) Fimbriae 2) Cervix represented by
3) Uterus 4) Fallopian tube 1) pq 2) q2
140. Which of the following depicts the 3) p2 4) 2pq
correct pathway of transport of sperms
146. The chronological order of human
1) Rete testis  Vas defe rens 
evolution from early to the recent is
Efferent ductules  Epididymis
1) Ramapithecus  Homo habilis 
2) Efferent ductules  Rete testis 
Australopithecus  Homo erectus
Vas deferens  Epididymis
3) Rete testis  Efferent ductules 2) Australopithecus  Homo
habilis  Ramapithecus  Homo
 Epididymis  Vas deferens
erectus
4) Rete testis  Epididymis  Efferent
ductules  Vas deferens 3)Australopithecus  Ramapithecus
 Homo habilis  Homo erectus
141. Match Column -I with Column-II and
select the correct option using the 4) Ramapithecus  Australopithecus
codes given below  Homo habilis  Homo erectus
147. Which of the following is the correct
Column -I Column-II sequence of events in the origin of life
a. Mons pubis (i) Embryo formation I. Formation of protobionts
b. Antrum (ii) Sperm II. Synthesis of organic monomers
(iii) Female external III. synthesis of organic polymers
c.Trophectoderm
genitalia IV. Formation of DNA-based genetic
d. Nebenkern (iv) Graafian follicle systems
1) II, III, I, IV
Codes: 2) II, III, IV, I
1) a-iii,b-i,c-iv, d-ii 2) a-i,b-iv,c-iii,d-ii 3) I, II, III, IV
3) a-iii,b-iv,c-ii,d-ii 4) a-iii,b-iv,c-i,d-ii 4) I, III, II, IV
80
Previous Years NEET Questions & Solutions

148. A molecule that can act as a genetic 153. Match Column-I with Column-II and
material must fulfill the traits given select the correct given below using
below, except the codes given below
1) It should be unstable structurally
and chemically Column -I Column-II
2) It should provide the scope for slow a. Citric acid (i) Trichoderma
changes that are required for evolution b. Cyclosporin A (ii) Clostridium
3) It should be able to express itself in c.Statins (iii) Aspergillus
the form of Mendelian characters d. Butyric acid (iv) Monascus
4) It should be able to generate its
replica Codes
149. DNA-dependent RNA polymerase
1) a-i, b-iv, c-ii, d-iii
catalyzes transcription on one strand
of the DNA which is called the 2) a-iii,b-iv,c-i,d-ii
1) Alpha strand 3) a-iii,b-i,c-ii,d-iv
2) Antistrand
4) a-iii,b-i,c-iv,d-ii
3) Template strand
154. Biochemical Oxygen Demand (BOD)
4) Coding strand
may not be a good index for pollution
150. Interspecific hybridization is the for water bodies receiving effluents
mating of from
1) Superior males and females of
1) Petroleum industry
different breeds
2) More closely related individuals 2) Sugar industry
within same breed for 4-6 generations 3) Domestic sewage
3) Animals within same breed without
4) Dairy industry
having common ancestors
4) Two different related species 155. The principle of competitive exclusion
was stated by
151. Which of the following is corre ct
regarding AIDS causative agent HIV? 1) Mac Arthur
1) HIV is unenveloped retrovirus 2) Verhulst and Pearl
2) HIV does not escape but attacks the
3) C. Darwin
acquired immune response
3) HIV is enveloped virus containing 4) G.F. Gause
one molecule of single-stranded RNA 156. Which of the following National Parks
and one molecule of reverse is home to the famous musk deer or
transcriptase hangul
4) HIV is enveloped virus that contains
1) Eaglene st Wildl ife Sanctuary,
two identical molecules of single-
Arunachal Pradesh
stranded RNA and two molecules of
reverse transcriptase 2) Dachigam National Park, Jammu and
152. Among the following edible fishes, Kashmir
which one is a marine fish having rich 3) Keibul Lamjao National Park,
source of omega-3 fatty acids? Manipur
1) Mrigala 2) Mackerel
4) Bandhavgarh National Park, Madhya
3) Mystus 4) Magur Pradesh

81
Brilliant STUDY CENTRE

157. A lake which is rich in organic waste 162. Study the four statements (a-d) given
may result in below and select the two correct ones
1) Increased population of fish due to out of them
lots of nutrients a) Definition of biological species was
2) Mortality of fish due to lack of oxygen given by Ernst Mayr
3) Increased population of aquatic
b) Photoperiod does not affect
organisms due to minerals
reproduction in plants
4) Dieing of the lake due to algal bloom
c) Binomial nomenclature system was
158. The highest DDT concentration in
given by R.H. Whittaker
aquatic food chain shall occur in
1) Crab d) In unicellular organisms,
2) Eel re production is synonymous with
growth
3) Phytoplankton
4) Seagull The two correct statements are
159. Which of the following sets of diseases 1) a and d 2) a and b
is caused by bacteria
3) b and c 4) c and d
1) Tetanus and mumps
2) Herpes and influenza 163. In male cockroaches, sperms are stored
in which part of the reproductive
3) Cholera and tetanus
system
4) Typhoid and smallpox
160. Match Column-I with Column-II for 1) Testes
housefly classification and select the 2) Vas deferens
correct option using the codes given
below 3) Seminal vesicles
4) Mushroom glands
Column -I Column-II
a. Family (i)Diptera 164. Smooth muscles are
b. Order (ii) Arthropoda 1) Involuntary, cylindrical, striated
c.Class (iii) Muscidae
2) Voluntary, spindle-shape d,
d. Phylum (iv) Insecta uninucleate

Codes 3) Involuntary, fusiform, non-striated


1) a-iv,b-iii,c-ii,d-i 4) Voluntary, multinucleate,
2) a-iv, b-ii,c-i,d-iii cylindrical
3) a-iii,b-i, c-iv,d-ii 165. Oxidative phosphorylation is
4) a-iii,b-ii,c-iv,d-i
1) Addition of phosphate group to ATP
161. Choose the correct statement
1) All reptiles have a three-chambered 2) Formation of ATP by energy released
heart from electrons removed during
substrate oxidation
2) All Pisces have gills covered by an
operculum 3) Formation of ATP by transfer of
3) All mammals are viviparous phosphate group from a substrate to
ADP
4) All cyclostomes do not possess jaws
and paired fins 4) Oxidation of phosphate group in ATP

82
Previous Years NEET Questions & Solutions

166. Which of the following is the least 169. Match the stages of meiosis in
likely to be involved in stabilizing the Column-I to their characte ristic
three-dimensional folding of most features in Column-II and select the
proteins correct option using the codes given
below
1) Hydrophobic interaction
2) Ester bonds Column -I Column-II
(i)Pairing of
3) Hydrogen bonds
a.Pachytene homologous
4) Electrostatic interaction chromosomes
(ii) Terminalization
167. Which of the following describes the b. Metaphase I
of chiasmata
given graph correctly
(iii) crossing over takes
c.Diakinesis
place
(iv) Chromosomes
d. Zygotene align at equatorial
plate

Codes:
1) a-ii, b-iv, c-iii, d-i
2) a-iv, b-iii,c-ii, d-i
3) a-iii,b-iv, c-ii, d-i
4) a-i, b-iv, c-ii, d-iii
170. Which hormones do stimulate the
production of pancreatic juice and
1) Endothermic reaction with energy
bicarbonate
A in absence of enzyme and B in
1) Cholecystokinin and secretin
presence of enzyme
2) Insulin and glucagon
2) Exothermic reaction with energy A 3) Angiotensin and epinephrine
in absence of enzyme and B in 4) Gastrin and insulin
presence of enzyme 171. The partial pressure of oxygen in the
3) Endothermic reaction with energy alveoli of the lungs is
A in presence of enzyme and B in 1) Less than that in the blood
absence of enzyme 2) Less than that of carbon dioxide
3) Equal to that in the blood
4) Exothermic reaction with energy A 4) More than that in the blood
in presence of enzyme and B in 172. Choose the correct statement
absence of enzyme
1) Photoreceptors in the human eye are
168. When cell has stalled DNA replication depolarize d during darkne ss and
fork. Which checkpoint should be become hyperpolarized in response to
predominantly activated the light stimulus
2) Receptors do not produce graded
1) M potentials
2) Both G2/M and M 3) Nociceptors respond to changes in
pressure
3) G1/S 4) Meissner’s corpuscles arc thermo-
receptors
4) G2/M

83
Brilliant STUDY CENTRE

173. Graves’ disease is caused due to 177. Osteoporosis, an age-related disease


1) Hyposecretion of adrenal gland of skeletal system, may occur due to
1) Decreased level of estrogen
2) Hypersecretion of adrenal gland
3) Hyposecretion of thyroid gland 2) Accumulation of uric acid leading to
inflammation of joints
4) Hypersecretion of thyroid gland
3) Immune disorder affecting neuro
174. Name the ion responsible for muscular junction leading to fatigue
unmasking of active sites for myosin
4) High concentration of Ca++ and Na+
for cross-bridge activity during muscle
contraction 178. Serum differs from blood in
1) Sodium 1) Lacking clotting factors
2) Potassium 2) Lacking antibodies
3) Lacking globulins
3) Calcium
4) Lacking albumins
4) Magnesium
179. Lungs do not collapse between breaths
175. Name the blood cells, whose reduction some air always remains in the lungs
in number can cause clotting disorder, which can never be expelled because
leading to excessive loss of blood from
1) There is a positive intrapleural
the body
pressure
1) Neutrophils 2) Pressure in the lungs is higher than
2) Thrombocytes the atmospheric pressure
3) Erythrocytes 3) There is a negative pressure in the
4) Leucocytes lungs
4) There is a negative intrapleural
176. Name a peptide hormone which acts
pressure pulling at the lung walls
mainly on hepatocytes, adipocytes and
enhances cellular glucose uptake and 180. The posterior pituitary gland is not a
utilization ‘true’ endocrine gland because
1) Secretin 1) It is under the regulation of
hypothalamus
2) Gastrin
2) It secretes enzymes
3) Insulin 3) It is provided with a duct
4) Glucagon 4) It only stores and release hormones

84
Previous Years NEET Questions & Solutions

KEY WITH HINTS

1. 3 5. 4

2. 1

3. 2
6. 1

7. 3

4. 1

8. 2

85
Brilliant STUDY CENTRE

9. 4
13. 2

10. 2

14. 4

11. 1

12. 4

86
Previous Years NEET Questions & Solutions

15. 3 20. 3

16. 4

17. 4
.

21. 3

18. 3

22. 3

19. 2

23. 3

87
Brilliant STUDY CENTRE

28. 3
24. 3

29. 1

25. 3

30. 3

26. 1
31. 1

32. 4

27. 3

88
Previous Years NEET Questions & Solutions

37. 3
33. 4

38. 2

34. 1

39. 4

35. 4 40. 1

41. 4

36. 3

89
Brilliant STUDY CENTRE

42. 2
45. 3

43. 1

CHEMISTRY

46. 3

44. 4

47. 4

48. 3

90
Previous Years NEET Questions & Solutions

49. 2 59. 1

60. 3

50. 3

51. 4

61. 4

52. 4
62. 2

53. 3

63. 1
54. 4

55. 4 64. 3

56. 3

65. 2

57. 2

58. 3

91
Brilliant STUDY CENTRE

66. 4

76. 4

67. 4

68. 3
77. 2

69. 1

70. 4
78. 4

71. 4

79. 3

72. 3
80. 4

81. 3
73. 2

82. 1

74. 2

75. 1

92
Previous Years NEET Questions & Solutions

83. 2

89. 1

84. 1

90. 4
85. 4

86. 4

BIOLOGY

91. 1
87. 4 92. 2
93. 1
94. 4
95. 3
96. 4
97. 1
98. 4
99. 1
100. 1
101. 1
102. 2
103. 2
104. 4
105. 3
106. 1
88. 4
107. 4
108. 3
93
Brilliant STUDY CENTRE

109. 2 124. 4 139. 3 154. 1 169. 3


110. 1 125. 4 140. 3 155. 4 170. 1
111. 1 126. 4 141. 4 156. 2 171. 4
112. 4 127. 3 142. 4 157. 2 172. 3
113. 4 128. 3 143. 3 158. 4 173. 4
114. 3 129. 2 144. 3 159. 3 174. 3
115. 2 130. 4 145. 4 160. 3 175. 2
116. 1 131. 4 146. 4 161. 4 176. 3
117. 4 132. 2 147. 1 162. 1 177. 1
118. 4 133. 1 148. 1 163. 3 178. 1
119. 4 134. 1 149. 3 164. 3 179. 4
120. 3 135. 1 150. 4 165. 2 180. 4
121. 3 136. 4 151. 4 166. 2
122. 3 137. 3 152. 2 167. 4
123. 1 138. 4 153. 4 168. 4

94
Previous Years NEET Questions & Solutions

NEET QUESTION PAPER - 2017

PHYSICS 4. The resistance of a wire is ‘R’ ohm. If


1. Two blocks A and B of masses 3m and it is melted and stretched to ‘n’ times
m respectively are connected by a its original length, its new resistance
massless and inextensible string. The will be :
whole system is suspended by a
massless spring as shown in figure. R
1) nR 2)
The magnitudes of acceleration of A n
and B immediately after the string is
cut, are respectively. R
3) n2R 4)
n2
5. A capacitor charged by a battery. The
battery is removed and another
identical uncharge d capacitor is
connected in parallel. The total
electrostatic e nergy of resulting
system :
1) increases by a factor of 4
g g
1) g, 2) ,g 2) decreases by a factor of 2
3 3
3) remains the same
g g
3) g, g 4) , 4) increases by a factor of 2
3 3
2. The acceleration due to gravity at a 6. Two rods A and B of different materials
height 1 km above the earth is the are welded together as shown in figure.
same as at a depth d below the surface What is the effective thermal
of earth. Then : conductivity?
1
1) d  km 2) d = 1 km
2
3
3) d  4) d = 2 km
2
3. A particle executes linear simple
harmonic motion with an amplitude of
3 cm. When the particle is at 2 cm from K1  K 2 3  K1  K 2 
the mean position, the magnitude of 1) 2)
2 2
its velocity is equal to that of its
3) K1 + K2 4) 2(K1 + K2)
acceleration. Then its time period in
second is : 7. The two nearest harmonics of a tube
closed at one end and open at other
5 5 end are 220 Hz and 260 Hz. What is
1) 2) the fundamental frequency of the
 2
system ?
4 2 1) 10 Hz 2) 20 Hz
3) 4)
5 3 3) 30 Hz 4) 40 Hz
95
Brilliant STUDY CENTRE

8. The bulk modulus of a spherical object 11. One end of string of length  is
is ‘B’. If it is subjected to uniform connected to a particle of mass ‘m’ and
pressure ‘p’, the fractional decrease in the other end is connected to a small
radius is : peg on a smooth horizontal table. If the
particle moves in circle with speed ‘  ’
p B
1) 2) 3p the net force on the particle (directed
B towards centre) will be (T represented
the tension in the string)
3p p
3) 4)
B 3B m 2
1) T 2) T 
9. A physical quantity of the dimensions 
of length that can be formed out of c,
m 2
e2 3) T  4) zero
G and is [c is velocity of light, G 
4 0
12. The photoe lectric threshold
is universal constant of gravitation and
wavelength of silver is 3250 × 10-10 m.
e is charge] :
The velocity of the electron ejected
1/2 from a silver surface by ultraviolet light
1  e2 
1) 2 G  of wavelength 2536 × 10-10 m is :
c  40  (Given, h = 4.14 × 10 -15 e Vs and
1/2
c = 3 × 108 ms-1)
 e2 
2
2) c G  1)  6 × 105 ms-1
 4 0  2)  0.6 × 106 ms-1
1/2 3)  61 × 103 ms-1
1  e2 
3) 2   4)  0.3 × 106 ms-1
c  G4 0 
13. Radioactive material ‘A’ has decay
1 e 2
constant ‘ 8 ’ and material ‘B’ has decay
4) G
c 4 0 constant ‘  ’. Initially they have same
number of nuclei. After what time, the
10. Figure shows a circuit that contains
ratio of number of nuclei of material
three i dentical resistors with
resistance R  9.0  e ach., two 1
‘B’ to that ‘A’ will be ?
identical inductors with inductance e
L = 2.0 mH each and an ideal battery
1 1
with emf   18V . The curre nt ‘i’ 1) 2)
 7
through battery just after the switch
closed is. 1 1
3) 4)
8 9
14. A rope is wound around a hollow
cylinder of mass 3 kg and radius
40 cm. What is the angular acceleration
of the cylinder if the rope is pulled with
a force of 30 N ?
1) 2 mA 2) 0.2 A 1) 25 m/s2 2) 0.25 rad/s2
3) 2 A 4) 4 A 3) 25 rad/s2 4) 5 m/s2

96
Previous Years NEET Questions & Solutions

15. Two cars moving in opposite directions 19. Two astronauts are floating in
approach each other with speed of 22 gravitational free space after having
m/s and 16.5 m/s respectively. The lost contact with their spaceship. The
driver of the first car blows a horn two will :
having a frequency 400 Hz. The
frequency heard by the driver of the 1) keep floating at the same distance
second car is [velocity of sound between them
340 m/s] : 2) move towards each other
1) 350 Hz 2) 361 Hz
3) move away from each other
3) 411 Hz 4) 448 Hz
4) will become stationary
16. A 250 - Turn rectangular coil of length
2.1 cm and width 1.25 cm carries a 20. The ratio of wavelengths of the last line
current of 85A are subjected to a of Balmer series and the last line of
magnetic field of strength 0.85 T. Work Lyman series is :
done for rotating the coil by 180 0
1) 2 2) 1
against the torque :
3) 4 4) 0.5
1) 9.1J 2) 4.55 J
21. The de -Broglie wavele ngth of a
3) 2.3 J 4) 1.15 J neutron in thermal equilibrium with
17. A long solenoid of diameter 0.1 m has heavy water at a temperature T(Kelvin)
2 × 104 turn per meter. At the centre and mass m, is :
of the solenoid, a coil of 100 turns and
radius 0.01 m is placed with its axis h
1)
coinciding with the solenoid axis. The mKT
current in the solenoid reduces at a
constant rate to 0 A from 4 A in 0.05 s. h
If the resistance of the coil is 10   , 2 2)
3mKT
the total charge flowing through the
coil during this time is : 2h
3)
1) 32 C 2) 16 C 3mKT

3) 32 C 4) 16 C 2h
4)
18. Suppose the charge of a proton and mKT
an electron differ slightly. One of them
22. A thin prism having refracting angle
is -e, the other is  e  e  . If the net of 100 is made of glass of refractive index
electrostatic force and gravitational 1.42. This prism is combined with
force between two hydrogen atoms another thin prism of glass of refractive
placed at a distance d(much greater index 1.7. This combination produces
than atomic size) apart is zero, then dispersion without deviation. The
e is of the order of [Given mass of refracting angle of second prism should
hydrogen mh = 1.67 × 10-27 kg] be :

1) 10-20 C 2) 10-23 C 1) 40 2) 60
3) 10-37 C 4) 10-47 C 3) 80 4) 100
97
Brilliant STUDY CENTRE

23. Thermodynamic pro cesses are 25. A spring of force constant k is cut into
indicated in the following diagram. lengths of ratio 1 : 2 : 3. They are
connected in series and the new force
constant is k ' . Then they are
connected in parallel and force
constant is k '' . Then k ' : k '' is :

1) 1 : 6 2) 1 : 9

3) 1 : 11 4) 1 : 14

26. Which of following statements are


Match the following : correct ?

Column-I Column-II a) Centre of mass of a body always


coincides with the centre of gravity of
P) Process I a) Adiabatic
the body.
Q) Process II b) Isobaric
b) Centre of mass of body is the point
R) Process III c) Isochoric at which the total gravitational torque
S) Process IV d) Isothermal on the body is zero.

1) P  a, Q  c, R  d,S  b c) A couple on a body produce both


transitional and rotational motion in a
2) P  c,Q  a, R  d,S  b body.

3) P  c,Q  d, R  b,S  a d) Mechanical advantage greater that


one means that small effort can be
4) P  d, Q  b, R  a,S  c used to lift a large load.
24. A U-tube with both ends open to the 1) b and d 2) a and b
atmosphere, is partially filled with
water, oil, which is immiscible with 3) b and c 4) c and d
water, is poured into one side untill it
stands at a distance of 10 mm above 27. A beam of light from a source L is
the water level on the other side. incident normally on a plane mirror
Meanwhile the water rises by 65 mm fixed at a certain distance x from the
from its original level (see diagram). source. The beam is reflected back as
The density of the oil is : a spot on a scale placed just above the
source L. When the mirror is rotated
through a small angle  , the spot of
the light is found to move through a
distance y on the scale. The angle 
is given by :

y y
1) 2)
2x x

1) 650 kg m-3 2) 425 kg m-3 x x


3) 2y 4) y
3) 800 kg m-3 4) 928 kg m-3
98
Previous Years NEET Questions & Solutions

28. A gas mixture consists of 2 moles of O2 32. The x and y coordinates of the particle
and 4 moles of Ar at temperature T. at any time are x = 5t - 2t 2 and
Neglecting all vibrational modes, the y = 410t respectively, where x and y
total internal energy of the system is : are in meters and t in seconds. The
1) 4 RT 2) 15 RT acceleration of the particle at t = 2s
is:
3) 9 RT 4) 11 RT
1) 0 2) 5 m/s2
29. Consider a drop of rain water having
mass 1 g falling from a height of 1 km. 3) -4 m/s2 4) -8 m/s2
It hits the ground with a speed of 33. The ratio of resolving powers of an
50 m/s. Take ‘g’ constant with a value optical microscope for two wavelengths
10 m/s 2 . The work done by the (i) 0 0
gravitational force and the (ii) resistive 1  4000 A and  2  6000 A is :
of air is : 1) 8 : 27 2) 9 : 4
1) (i) - 10J (ii) -8.25J 3) 3 : 2 4) 16 : 81
2) (i) 1.25 J (ii) -8.25 J 34. Preeti reached the metro station and
3) (i) 100 J (ii) 8.75 J found that the escalator was not
4) (i) 10 J (ii) -8.75 J working. The walked up the stationary
in time t 1 . On other days, if she
30. A carnot engine having an efficiency
remains stationary on the moving
1 escalator, then the escalator takes her
of as heat engine, is used as a up in time t2. The time taken by her to
10
refrigerator. If the work done on the walk up on the moving escalator will
system is 10 J, the amount of energy be :
absorbed from the reservoir at lower
t1  t 2 t1 t 2
temperature is : 1) 2) t  t
2 2 1
1) 1 J 2) 90 J
3) 99 J 4) 100 J t1 t 2
31. An arrangement of thre e parallel 3) t  t 4) t1  t 2
2 1
straight wires placed perpendicular to
plane of paper carrying same current 35. A spherical black body with a radius of
‘I’ along the same direction is shown 12 cm radiates 450 watt power at
in figure. Magnitude of force per unit 500 K. If the radius were halved and
length on the middle wire ‘B’ is given the temperature doubled, the power
by : radiated in watt would be :
1) 225 2) 450
3) 1000 4) 1800
36. A potentiometer is an accurate and
versatile device to make electrical
measurements of E.M.F. because the
method involves :
1) cells
0i 2 2 0 i 2 2) potential gradients
1) 2)
2d d 3) a condition of no current flow
through the galvanometer
2 0i 2 0 i 2
3) 4) 4) a combination of cells, galvanometer
d 2d
and resistance

99
Brilliant STUDY CENTRE

37. The given electrical network is 41. Which one of the following represents
equivalent to : forward bias diode ?

1)

1) AND gate 2) OR gate


2)
3) NOR gate 4) NOT gate
38. In a common emitter transistor
amplifier the audio signal voltage 3)
across the collector is 3V. The
resistance o f collector is 3k . If
4)
current gain is 100 and the base
resistance is 2 k , the voltage and 42. Two polaroids P1 and P2 are placed with
power gain of the amplifier is : their axis perpendicular to each other.
Unpolarised light I0 is incident on P1.
1) 200 and 1000 A third polaroid P3 is kept in between
2) 15 and 200 P1 and P2 such that its axis makes an
angle 450 with that of P1. The intensity
3) 150 and 15000 of transmitted light through P2 is :
4) 20 and 2000 I0 I0
1) 2)
39. Two discs of same moment of inertia 2 4
rotating about their regular axis
passing through centre and I0 I0
3) 4)
perpendicular to the plane of disc with 8 16
angular velocities 1 and 2 . They are
43. In an electromagnetic wave in free
brought into contact face to face space the root mean square value of
coinciding the axis of rotation. The the electric field is Erms = 6 V/m. The
expression for loss of energy during this peak value of the magnetic field is :
process is :
1) 1.41 × 10-8 T 2) 2.83 × 10-8 T
1 1
 1  2   1  2 
2 2
1) 2) 3) 0.70 × 10-8 T 4) 4.23 × 10-8 T
2 4
44. If 1 and 2 be the apparent angle of dip
1 observed in two vertical planes at right
 1  2 
2
3) I  1  2 
2
4)
8 angles to each other, then the true
angle of dip  is given by :
40. Young’s double slit experiment is first
performed in air and then in a medium 1) cot 2   cot 2 1  cot 2 2
other than air. It is found that 8th bright
fringe in the medium lies where 5th
2) tan 2   tan 2 1  tan 2 2
dark fringe lies in air. The refractive
index of the medium is nearly.
3) cot 2   cot 2 1  cot 2 2
1) 1.25 2) 1.59
4) tan 2   tan 2 1  tan 2 2
3) 1.69 4) 1.78

100
Previous Years NEET Questions & Solutions

45. The diagrams below show regions of 47. An example of a sigma bonded
equipotentials. organometallic compound is :

1) Ruthenocene

2) Grignard’s reagent

3) Ferrocene
a) b)
4) Cabaltocene

48. Which one is the wrong statement?

1) de-Broglie’s wavelength is given by


h
 where m = mass of the particle,
mv
 = group velocity of the particle

2) The uncertainty principle is


c) d)
h
E  t 
4

3) Half filled and fully filled orbitals


have greater stability due to greater
exchange energy, greater symmetry
Positive charge is moved from A to B in
and more balanced arrangement
each diagram
1) Maximum work is required to move 4) The energy of 2s - orbital is less than
q in figure (c) the energy of 2p-orbital in case of
hydrogen like atoms.
2) In all the cases the work done is
the same 49. Mixture of chloroxylenol and terpineol
acts as :
3) Minimum work is required to move
q in figure (a) 1) Analgesic
4) Maximum work is required to move 2) Antiseptic
q in figure (b)
3) Antipyretic
CHEMISTRY
4) Antibiotic
46. The reason for greate r range of
oxidation states in actinoids is 50. The element Z = 114 has be en
attributed to : discovered recently. It will belong to
which of the following family/group and
1) The radioactive nature of actinoids
electronic configuration?
2) Actinoid contraction
1) Halogen family, [Rn]5f14 6d10 7s2 7p5
3) 5f, 6d and 7s levels having
comparable energies 2) Carbon family, [Rn] 5f14 6d10 7s2 7p2
3) Oxygen family, [Rn]5f146d107s27p4
4) 4f and 5d levels being close in
energies 4) Nitrogen family, [Rn]5f146d107s27p6

101
Brilliant STUDY CENTRE

51. A 20 litre container at 400 K contains 55. The species, having bond angles of 120o
CO 2 (g) at pressure 0.4 atm and an is :
excess of SrO (neglect the volume of 1) PH3 2) ClF3
solid SrO). The volume of the container
is now decreased by moving the movable 2) NCl3 4) BCl3
piston fitted in the container. The 56. The correct order of the
maximum volume of the container, stoichiometries of AgCl formed when
when pressure of CO 2 attains its AgNO 3 in excess is treated with the
maximum value, will be : complexes CoCl 3 .6NH 3 , CoCl 3 .5NH 3 ,
(Given that : CoCl3.4NH3 respectively is :
SrCO3 (s)  SrO(s)  CO2 (g),K p  1.6atm) 1) 1AgCl, 3AgCl, 2AgCl
1) 5 litre 2) 10 litre 2) 3AgCl, 1AgCl, 2AgCl
3) 4 litre 4) 2 litre 3) 3AgCl, 2AgCl, 1AgCl
52. Predict the correct intermediate and
4) 2AgCl, 3AgCl, 1AgCl
product in the following reaction :
H3C  C  CH 
H2O,H2 SO 4
HgSO 4
 int ermediate  product 57. For a given reaction H  35.5kJ mol1
( X) ( Y)

and S  83.6 JK 1 mol1. The reaction


1) X : H3C C CH2 Y : H3C C CH3
is spontaneous at :
SO4 O (Assume that H and S do not vary
with temperature)
2) X : H3C C CH2 Y : CH3 C CH2
1) T < 425K
OH SO4 2) T > 425 K
3) X : CH3 C CH3 Y : H3C C CH 3) All temperatures
4) T > 298 K
O
58. Match the interhalogen compounds of
4) X : H3C C CH2 Y : H3C C CH3 Column I with the geometry in column
II and assign the correct code :
OH O Column I Column II
53. Which of the following is a sink for CO? a) XX ' i) T-shape
1) Hemoglobin b) XX'3 ii) Pentagonal bipyramidal
2) Micro-organisms present in the soil
c) XX'5 iii) Linear
3) Oceans
4) Plants d) XX'7 iv) Square-pyramidal
54. Which of the following reactions is v) Tetrahedral
appropriate for converting acetamide to
Code :
methanamine?
(a) (b) (c) (d)
1) Carbylamine reaction
1) (iii) (iv) (i) (ii)
2) Hoffmann hypobromamide reaction
2) (iii) (i) (iv) (ii)
3) Stephens reaction 3) (v) (iv) (iii) (ii)
4) Gabriels phthalimide synthesis 4) (iv) (iii) (ii) (i)

102
Previous Years NEET Questions & Solutions

59. Identify P and predict the type of 61. Name the gas that can readily
reaction : decolorise acidified KMnO4 solution :
1) CO2 2) SO2 3) NO2 4) P2O5
OCH3
62. The correct increasing order of basic
strength for the following compounds
is :

NaNH2

P
Br NH2 NH2 NH2

OCH3

1) and substitution
NO2 CH3
NH2
(I) (II) (III)
reaction
1) II < III < I 2) III < I < II
OCH3
3) III < II < I 4) II < I < III
NH2 63. If molality of the dilute solution is
2) and e limination doubled, the value of molal depression
constant (Kf) will be :
1) doubled 2) halved
addition reaction 3) tripled 4) unchanged
OCH3 64. Of the following, which is the product
formed when cyclohexanone undergoes
Br aldol condensation followed by heating?
3) and cine substitution
O

reaction
OCH3 1)

OH
4) and cine substi tution

reaction 2)
60. Which one of the following statements O
is not correct?
1) Catalyst does not initiate any
reaction
2) The value of equilibrium constant is 3)
changed in the presence of a catalyst OH
in the reaction at equilibrium
3) Enzymes catalyse mainly bio-
chemical reactions
4) Coenzymes increase the catalytic 4)
activity of enzyme. O O
103
Brilliant STUDY CENTRE

65. The equilibrium constants of the 69. Which of the following is incorrect
following are : statement?
1) FeO0.98 has non-stoichiometric metal
N2  3H2  2NH3 K1 deficiency defect.
N2  O 2  2NO K2 2) Density decreases in case of crystals
H2  1 O 2  H2O K3 with Schottky’s defect
2
3) NaCl(s) is insulator, silicon is
The equilibrium constant (K) of the semiconductor, silver is conductor,
reaction : quartz is piezo electric crystal
5 4) Frenkel defect is favoured in those
2NH3  
K
O2 
 2NO  3H2O, will be : ionic compounds in which sizes of
2
cation and anion are almost equal.
1) K1K 33 / K 2 2) K 2K 33 / K1 70. The heating of phenyl-methyl ethers
with HI produces :
3) K 2K 3 / K1 4) K 32K 3 / K1 1) ethyl chlorides 2) iodobenzene
66. The correct statement regarding 3) phenol 4) benzene
electrophile is : 71. Correct increasing order for the
1) Electrophile is a negatively charged wavelength of absorption in the visible
species and can form a bond by region for the complexes of Co3+ is :
accepting a pair of electrons from a
nucleophile.
1) Co(en)3  , Co(NH3 ) 6  ,Co(H2O)6 
3 3 3
2) Electrophile is a negatively charged
species and can form a bond by
2) Co(H2O)6  ,Co(en)3  ,Co(NH3 )6 
3 3 3
accepting a pair of electrons from
another electrophile.
3) Co(H2O)6  ,Co(NH3 )6  , Co(en)3 
3 3 3
3) Electrophiles are generally neutral
species and can form a bond by
4) Co(NH3 )6  ,Co(en)3  ,Co(H2O)6 
3 3 3
accepting a pair of electrons from a
nucleophile.
72. Pick out the correct statement with
4) Electrophile can be either neutral respect to [Mn(CN)6]3- :
or positively charged species and can
1) It is sp3d2 hybridised and octahedral
form a bond by accepting a pair of
electrons from a nucleophile. 2) It is sp3d2 hybridised and tetrahedral
67. A gas is allowed to expand in a well 3) It is d2sp3 hybridised and octahedral
insulated container against a constant 4) It is dsp 2 hybridised and square
external pressure of 2.5atm from an planar
initial volume of 2.50 L to a final volume 73. With respect to the conformers of
of 4.50 L. The change in internal ethane, whi ch of the following
energy U of the gas in joules will be : statements is true?
1) 1136.26 J 2) -500 J 1) Bond angle remains same but bond
3) -505 J 4) +505 J length changes
68. Which of the foll owing pairs of 2) Bond angle changes but bond length
compounds is isoele ctronic and remains same
isostructural? 3) Both bond angle and bond length
1) BeCl2, XeF2 2) TeI2, XeF2 change
3) IBr2 , XeF2
– 4) IF3, XeF2 4) Both bond angles and bond length
remain same
104
Previous Years NEET Questions & Solutions

74. Which of the following is dependent on 79. Mechanism of a hypothetical reaction


temperature?
X2  Y2  2XY is given below :
1) Molality
2) Molarity i) X2  X  X(fast)
3) Mole fraction
ii) X  Y2  XY Y(slow)
4) Weight percentage
75. Which of the following statement is not iii) X  Y  XY(fast)
correct? The overall order of the reaction will
1) Insulin maintains sugar level in the be :
blood of a human body 1) 1 2) 2
2) Ovalbumin is a simple food reserve 3) 0 4) 1.5
in egg white
80. Concentration of the Ag + ions in a
3) Blood proteins thrombin and saturated solution of Ag 2 C 2 O 4 is
fibrinogen are involved in blood clotting 2.2 × 10–4 mol L–1. Solubility product of
4) Denaturation makes the proteins Ag2C2O4 is :
more active
1) 2.42 × 10–8
76. The IUPAC name of the compound is :
2) 2.66 × 10–12
O O
3) 4.5 × 10–11

H C 4) 5.3 × 10–12
81. Extraction of gold and silver involves
leaching with CN– ion. Silver is later
1) 3-keto-2-methylhex-4-enal recovered by :
2) 5-formylhex-2-en-3-one 1) liquation
3) 5-methyl-4-oxohex-2-en-5-al 2) distillation
4) 4-keto-2-methylhex-5-enal
3) zone refining
77. HgCl2 and I2 both when dissolved in
water containing I – ions the pair of 4) displacement with Zn
species formed is : 82. Which one is the correct order of
1) HgI2, I3– 2) HgI2, I– acidity?
3) HgI24 ,I3 4) Hg2I2 ,I 1) CH2  CH2  CH3  CH  CH2  CH3  C  CH  CH  CH

78. It is because of inability of ns2 electrons 2) CH  CH  CH3  C  CH  CH2  CH2  CH3  CH3
of the valence shell to participate in 3) CH  CH  CH2  CH2  CH3  C  CH  CH3  CH3
bonding that :
4) CH3  CH3  CH2  CH2  CH3  C  CH  CH  CH
1) Sn 2+ is reducing while Pb 4+ is
oxidising 83. Ionic mobility of which of the following
alkali metal ions is lowest when
2) Sn 2+ is oxidising while Pb 4+ is
aqueous solution of their salts are put
reducing
under an electric field?
3) Sn2+ and Pb2+ are both oxidising and
reducing 1) Na 2) K
4) Sn 4+ is reducing while Pb 4+ is 3) Rb 4) Li
oxidising
105
Brilliant STUDY CENTRE

84. Consider the reaction : 87. In the electrochemical cell :


Zn | ZnSO4 (0.01M) || CuSO4 (1.0M) | Cu, the
emf of this Daniel cell is E 1. When the
concentration of ZnSO4 is changed to 1.0 M
and that of CuSO4 changed to 0.01 M, the
emf changes to E2. From the followings which
one is the relationship between E1 and E2?
RT
Identify P, X, Y and Z. (Given,   0.059)
F
1)P-Methoxymethane, X-Ethanoic acid, 1) E1 = E2 2) E1 < E2
Y- Acetate ion, Z-Hydrazine 3) E1 > E2 4) E2 = 0  E1
2)P-Methoxymethane, X-Ethanol, 88. A first order reaction has a specific reaction
Y- Ethanoic acid, Z-Semicarbazide rate of 10–2. How much time will it take for 20
g of the reaction to reduce to 5g?
3)P-Ethanal, X-Acetaldehyde,
1) 238.6 sec 2) 138.6 sec
Y-But-2-enal, Z-Semicarbazone 3) 346.5 sec 4) 693.0 sec
4)P-Ethanol, X - Acetaldehyde, 89. The most suitable method of separation of 1
Y - Butanone, Z - Hydrazone : 1 mixture of ortho and para-nitrophenols is :
1) Sublimation 2) Chromatography
85. In which pair of ions both the species
contain S-S bond? 3) Crystallisation 4) Steam distillation
90. Which one of the following pairs of species
1) S 2 O72 ,S 2O32 have the same bond order?
1) CO, NO 2) O2, NO+
2) S 4O62  ,S 2O32 
3) CN–, CO 4) N2, O2–
3) S 2O72 ,S 2O82 BIOLOGY
91. Which of the following are found in
4) S 4O62  ,S 2O72  extreme saline conditions?
86. Which one is the most acidic compound? 1) Eubacteria 2) Cyanobacteria
3) Mycobacteria 4) Archaebacteria
OH 92. Which of the following components
OH provides sticky character to the
bacterial cell
1) 2) 1) Nuclear membrane
2) Plasma membrane
CH3
3) Glycocalyx
4) Cell wall
OH
OH 93. Viroids differ from viruses in having
O2 N NO2 1) DNA molecules without protein coat
2) RNA molecules with protein coat
3) 4)
3) RNA molecules without protein coat
NO2 NO2
4) DNA molecules with protein coat
106
Previous Years NEET Questions & Solutions

94. Which among of the following are the 102. In Bougainvill ea, thorns are the
smallest living cells, known without a modifications of
definite cell wall, pathogenic to plants 1) Adventitious root 2) Stem
as well as animals and can survive
3) Leaf 4) Stipules
without oxygen
103. Root hairs develop from the region of
1) Pseudomonas
1) Elongation
2) Mycoplasma
2) Root cap
3) Nostoc 3) Meristematic activity
4) Bacillus 4) Maturation
95. An example of colonial alga is 104. The morphological nature of the edible
1) Volvox 2) Ulothrix part of coconut is
3) Spirogyra 4) Chlorella 1) Cotyledon 2) Endosperm
96. Zygotic meiosis is characteristic of 3) Pericarp 4) Perisperm
105. The vascular cambium normally gives
1) Fucus 2) Funaria
rise to
3) Chlamydomonas 4) Marchantia
1) Primary phloem 2) Seconday xylem
97. Life cycle of Ecotocarpus and Fucus 3) Periderm 4) Phelloderm
respectively are
106. Which of the following is made up of
1) Diplontic,Haplodiplontic dead cells
2) Haplodiplontic, Diplontic 1) Collenchyma
3) Haplodiplontic, Haplontic 2) Phellem
4) Haplontic, Diplontic 3) Phloem
98. An important characteristi c that 4) Xylem parenchyma
Hemichordate share with Chordates is 107. Identify the wrong statement in context
1) Ventral tubular nerve cord of heartwood
2) Pharynx with gills slits 1) It is highly durable
3) Pharynx without gill slits 2) It conducts water and minerals
4) Absence of notochord efficiently
99. Which of the following represents order 3) It comprises dead elements with
of Horse highly lignified walls
4) Organic compounds are deposited in
1) Perissodactyla it.
2) Caballus 108. Select the correct route for the passage
3) Ferus of sperms in male frogs
4) Equidae 1) Teste s  Vasa effere ntia 
Kidney  Semi nal Vesicle
100. In case of poriferans, the spongocoel
is lined with flagellated cells called Urinogenital duct  Cloaca
2) Teste s  Vasa effere ntia 
1) Oscula
Bidder’s canal  Ureter  Cloaca
2) Choanocytes
3) Teste s  Vasa effere ntia 
3) Mesenchymal cells Kidney  Bidder’s canal 
4) Ostia Urinogenital duct  Cloaca
101. Coconut fruit is a 4) Testes  Bidder’s canal  Kidney
1) Berry 2) Nut  Vasa efferentia  Urinogential
3) Capsule 4) Drupe duct  Cloaca

107
Brilliant STUDY CENTRE

109. Adult human RBCs are enucleate. 114. Anaphase Promoting Complex (APC) is
Which of the following statement(s) is/ a protein degradation machine ry
are most appropriate explanation for necessary for proper mitosis of animal
this feature cells. If APC is defective in a human
a) They do not need to reproduce cell,
b) They are somatic cells which of the following is expected to
occur
c) They do not metabolize
1) Chromosomes will be fragmented
d) All their internal space is available 2) Chromosomes will not segregate
for oxygen transport
3) Recombination of chromosome arms
Options: will occur
1) Only a 2) a,c and d 4) Chromosomes will not condense
3) b and c 4) Only d 115. Which of the following facilitates
110. Which of the following cell organelles opening of stomatal aperture
is responsible for extracting energy 1) Decrease in turgidity of guard cells
from carbohydrates to form ATP 2) Radial orientation of ce llul ose
1) Ribosome 2) Chloroplast microfibrils in the cell wall of guard
3) Mitochondrion 4) Lysosome cells
3) Longitudinal orientation of cellulose
111. Which one of the following statements
microfibrils in the cell wall of guard
is correct, with reference to enzymes?
cells
1) Holoenzyme = Apoenzyme +
4) Contraction of outer wall of guard
Coenzyme
cells
2) Coenzyme = A poenzyme + 116. The water potential of pure water is
Holoenzyme
1) Less than zero
3) Holoenzyme = Coenzyme +Co-factor 2) More than zero but less than one
4) Apoenzyme = Coenzyme +Co-factor 3) More than one
+Coenzyme 4) Zero
112. Which of the following are not polymeric 117. Select the mismatch
1) Proteins 2) Polysaccharides 1) Rhodospirillum - Mycorrhiza
3) Lipids 4) Nucleic acids 2) Anabaena - Nitrogen fixer
113. Which of the following options gives the 3) Rhizobium - Alfalfa
correct sequence of events during 4) Frankia - Alnus
mitosis 118. With reference to factors affecting the
1) Condensation  nuclear membrane rate of photosynthesis, which of the
disassembly  arrangement at following statements is not correct
equator  centromere division 1) Increasing atmospheric CO 2
segregation  telophase concentration up to 0.05 % can
2) Condensation  crossing over enhance CO2 fixation
 nuclear membrane disassembly  2) C 3 plants respond to higher
segregation  telophase temperature s with e nhanced
photosynthesis while C 4 plants have
3) Condensation  arrangement at
much lower temperature optimum
equator  centromere division 
3) Tomato is a greenhouse crop which
segregation  telophase
can be grown in CO 2 -e nriched
4) Condensation  nuclear membrane atmosphere for higher yield
disassembly  crossing over  4) Light saturation for CO 2 fixation
segregation  telophase occurs at 10% of full sunlight
108
Previous Years NEET Questions & Solutions

119. Phosphoenol pyruvate (PEP) is the 125. Lungs are made up of air-filled sacs,
primary CO2 acceptor in the alveoli. They do not collapse even
1) C4 plants after forceful expiration, because of
2) C2 plants 1) Inspiratory Reserve Volume
3) C3 and C4 plants 2) Tidal Volume
4) C3 plants 3) Expiratory Reserve Volume
120. Which statement is wrong for Kreb’s 4) Residual Volume
cycle 126. The hepatic portal vein drains blood
1) There is one point in the cycle where to liver from
FAD+ is reduced to FADH2 1) Stomach 2) Kidneys
2) During conversion of succinyl CoA 3) Intestine 4) Heart
to succinic acid, a molecule of GTP is 127. Frog’s heart when taken out of the body
synthesised continues to beat for sometime
3) The cycle starts with condensation Select the best option from the following
of acetyl group (acetyl CoA) with pyruvic statements
acid to yield citric acid 1) Frog is poikilotherm
4) There are three points in the cycle 2) Frog does not have any coronary
where NAD+ is reduced to NADH+H+ circulation
121. Fruit and leaf drop at early stages can 3) Heart is ‘myogenic” in nature
be prevented by the application of
4) Heart is autoexcitable
1) Ethylene 2) Auxins
Options:
3) Gibberellic acid 4) Cytokinins
1) Only d 2) a and b
122. Which cells of ‘Crypts of Lieberkuhn’
secrete antibacterial lysozyme? 3) c and d 4) Only c
1) Paneth cells 128. A decrease in blood pressure/volume
will not cause the release of
2) Zymogen cells
1) Atrial Natriuretic factor
3) Kupffer cells 2) Aldosterone
4) Argentaffin cells. 3) ADH
123. Which of the following options best 4) Renin
represents the enzyme composition of 129. Which of the following statements is
pancreatic juice correct
1) Amylase, pepsin, trypsinogen, 1) The descending limb of loop of Henle
maltase is impermeable to water
2) Peptidase, amylase, pepsin, renin 2) The ascending limb of loop of Henle
is permeable to water
3) Lipase, amylase, trypsinogen,
procarboxypeptidase 3) The descending limb of loop of Henle
is permeable to electrolytes
4) Amylase, peptidase, trypsinogen,
4) The ascending limb of loop of Henle
renin
is impermeable to water
124. A baby boy aged two years is admitted
130. The pivot joint between atlas and axis
to play school and passes through a
is a type of
dental check-up. The dentist observed
that the boy had twenty teeth. Which 1 Cartilaginous joint
teeth were absent 2) Synovial joint
1) Canines 2) Pre-molars 3) Saddle joint
3) Molars 4) incisors 4) Fibrous joint
109
Brilliant STUDY CENTRE

131. Out of ‘X’ pairs of ribs in humans only 135. Hypersecretion of Growth Hormone in
‘Y’ pairs are true ribs. Select the option adults does not cause further increase
that correctly represents values of X in height, because
and Y and provides their explanation 1) Epiphyseal plates close after
adolescence
2) Bones loose their sensitivity to
True ribs are attached
Growth Hormone in adults
dorsally to vertebral
1 X = 12, Y = 5 3) Muscle fibres do not grow in size
column and sternum on two
ends after birth
True ribs are dorsally 4) Growth Hormone becomes inactive
attached to vertebral in adults
2 X = 24, Y = 7
column but are free 136. Select the mismatch
on ventral side
1) Cycas - Dioecious
True ribs are dorsally
2) Salvinia - Heterosoporous
attached to vertebral
3 X = 24, Y = 12 3) Equisetum - Homosporous
column but are free
on ventral side 4) Pinus - Dioecious
True ribs are attached 137. Functional megaspore in an
4 X = 12, Y = 7
dorsally to vertebral angiosperm develops into
column & ventrally to the
1) Endosperm 2) Embryo sac
sternum
3) Embryo 4) Ovule
138. Attractants and rewards are required
132. Receptor sites for neurotransmitters for
are present on 1) Entomophily 2) Hydrophily
1) Pre-synaptic membrane 3) Cleistogamy 4) Anemophily
2) Tips of axons 139. Plants which produce characteristic
3) Post-synaptic membrane pneumatophore and show vivipary
4) Membranes of synaptic vesicles belong to
133. Good vision depends on adequate 1) Halophytes 2) Psammophytes
intake of carotene rich food 3) Hydrophytes 4) Mesophytes
Select the best option from the following
140. Flowers which have single ovule in the
statements
ovary and are packe d into
a) Vitamin A derivates are formed from inflorescence are usually pollinated by
carotene
1) Bee 2) Wind
b) The photopigments are embedded in
3) Bat 4) Water
the membrane discs of the inner
segment 141. A dioecious flowering plant prevents
c) Retinal is a derivative of vitamin A both
d) Retinal is a light absorbing part of 1) Autogamy and geitonogamy
all the visual photopigments 2) Geitonogamy and xenogamy
1) a, c and d 2) a and c 3) Cleistogamy and xenogamy
3) b, c and d 4) a and b 4) Autogamy and xenogamy
134. Myelin sheath is produced by 142. Capacitation occurs in
1) Astrocytes and Schwann Cells 1) Epididymis
2) Oligodendrocytes and Osteoclasts 2) Vas deferens
3) Osteoclasts and Astrocytes
3) Female reproductive tract
4) Schwann Cells and Oligodendrocytes
4) Rete testis
110
Previous Years NEET Questions & Solutions

143. A temporary endocrine gland in the 148. In case of couple where the male is
human body is having a very low sperm count, which
1) Corpus cardiacum technique will be suitable for
fertilisation?
2) Corpus luteum
1) Gamete intracytoplasmic fallopian
3) Corpus allatum
transfer
4) Pineal gland
2) Artificial Insemination
144. GnRH, a hypothalamic ho rmone,
3) Intracytoplasmic sperm injection
needed in reproduction, acts on
4) Intrauterine transfer
1) Anterior pitui tary gland and
stimulates secretion of LH and FSH 149. Thalassemia and sickle cell anemia are
caused due to a problem in globin
2) Posterior pituitary gland and
molecule synthesis. Select the correct
stimulates secretion of oxytocin and
statement
FSH
1) Both are due to quantitative defect
3) Posterior pituitary gland and
in globin chain synthesis
stimulates secretion of LH and relaxin
2) Thalassemia is due to less synthesis
4) Anterior pitui tary gland and
of globin molecules
stimulates secretion of LH and oxytocin
3) Sickle cell anemia is due to a
145. Double fertilization is exhibited by
quantitative proble m of globin
1) Algae 2) Fungi molecules
3) Angiosperms 4) Gymnosperms 4) Both are due to a qualitative defect
146. Match the following se xually in globin chain synthesis
transmitted disease Column-I with
their causative agent Column-II and 150. The genotypes of a Husband and Wife
select the correct option are IAIB and IAi. Among the blood types
of their children, how many different
genotypes and phenotypes are possible
Column I Column II
1) 3 genotypes; 4 phenotypes
a Gonorrhea i HIV 2) 4 genotypes ; 3 phenotypes
3) 4 genotypes ; 4 phenotypes
b Syphilis ii Neisseria
4) 3 genotypes ; 3 phenotypes
c Genital Warts iii Treponema 151. A disease caused by an autosomal
primary non-disjunction is
d AIDS iv Human Papilloma-Virus 1) Klinefelter’s sydrome
2) Turner’s sydrome
1) a-iii,b-iv,c-i,d-ii 2) a-iv,b-ii,c-iii,d-i 3) Sickle Cell Anemia
3) a-iv,b-iii,c-ii,d-i 4) a-ii,b-iii,c-iv,d-i 4) Down’s sydrome
147. The function of copper ions in copper 152. Among the following characters, which
releasing IUD’s is one was not considered by Mendel in
1) They inhibit gametogenesis his experiments on pea
1) Tri chomes- Glandular or no n-
2) They make uterus unsuitable for
glandular
implantation
2) Seed-Green and yellow
3) They inhibit ovulation
3) Pod-Inflated or Constricted
4) They suppress sperm motility and
fertilising capacity of sperms 4) Stem-Tall or Dwarf

111
Brilliant STUDY CENTRE

153. Which one from those given below is 161. The association of histone H1 with a
the period for Mendel’s hybridization nucleosome indicates
experiments 1) DNA replication is occuring
1) 1840-1850 2) 1857-1869 2) The DNA is conde nsed into a
3) 1870-1877 4) 1856-1863 Chromatin Fibre
154. The final proof for DNA as the genetic
3) The DNA double helix is exposed
material can from the experiments of
1) Hershey and Chase 4) Transcription is occuring
2) Avery, Mcleod and McCarty 162. Artificial selection to obtain cows
3) Hargobind Khorana yielding higher milk output represents
4) Griffith 1) Directional as it pushes the mean
155. DNA fragments are of the character in one direction
1) Negatively charged 2) Disruptive as it splits the population
2) Neutral into two, one yielding higher output an
3) Either positively or negatively the other lower output
charged depending on their size 3) Stabilizing followeed by disruptive as
4) Positively charged it stabilizes the population to produce
156. If there are 999 bases in an RNA that higher yeilding cows
code for a protein with 333 amino acids, 4) Stabilizing selection as it stabilizes
and the base at position 901 is deleted this character in the population
such that the length of the RNA 163. Which among these is the correct
becomes 998 bases, how many codons combination of aquatic mammals
will be altered? 1) Dolphins, Seals, Trygon
1) 11 2) 33 2) Whales, Dolphins, Seals
3) 333 4) 1
3) Trygon, Whales, Seals
157. During DNA replication, Okazaki
4) Seals, Dolphins, Sharks
fragments are used to elongate
1) The lagging strand towards 164. Transplantation of tissues/organs fails
replication fork often due to non acceptance by the
patients body. Which type of
2) The leading strand away from
immuneresponse is responsible for
replication fork
such rejections
3) The lagging strand away from the
replication fork 1) Cell-mediated immune response
4) The leading strand towards 2) Hormonal immune response
replication fork 3) Physiological immune response
158. Which of the following RNAs should be 4) Auto immune response
most abundant in animal cell 165. MALT constitutes about ........... percent
1) t-RNA 2) m-RNA of the lymphoid tissue in human body
3) mi-RNA 4) r-RNA 1) 20% 2) 70%
159. DNA replication in bacteria occurs 3) 10% 4) 50%
1) Within nucleolus 166. Which of the following is correctly
2) Prior to fission matched for the product produced by
3) Just before transcription them
4) During S phase 1) Methanobacterium : Lactic acid
160. Spliceosomes are not found in cells of 2) Penicillium notatum : Acetic acid
1) Fungi 2) Animals 3) Sacchromyces cerevisiae : Ethanol
3) Bacteria 4) Plants 4) Acetobacter aceti : Antibiotics

112
Previous Years NEET Questions & Solutions

167. Homozygous purelines in cattle can be 173. Asymptote in a logistic growth curve is
obtained by: obtained when
1) Mating of unrelated individuals of 1) K = N
same breed 2) K > N
2) Mating of individuals of different 3) K < N
breed 4) The value of ‘r’ approaches zero
3) Mating of individuals of different 174. Which ecosystem has the maximum
breed biomass
4) Mating of related individuals of same 1) Grassland ecosystem
breed 2) Pond ecosystem
168. Which of the following in sewage 3) Lake ecosystem
treatment remove suspended solids 4) Forest ecosystem
175. Mycorrhizae are the example
1) Secondary treatment
1) Amensalism 2) Antibiosis
2) Primary treatment
3) Mutualism 4) Fungistasis
3) Sludge treatment 176. Presence of plants arranged into well
4) Tertiary treatment defined vertical layers depending on
169. The DNA fragments separated on an their height can be seen best in
agarose gel can be visualised after 1) Tropical rain forest
staining with 2) Grassland
1) Acetocarmine 3) Temperate forest
2) Aniline blue 4) Tropical Savannah
3) Ethidium bromide 177. Which one of the following is related
4) Bromophenol blue to Ex-situ conservation of threatened
170. The process of separation and animals and plants
purification of expressed protein before 1) Biodiversity hot spots
marketing is called 2) Amazon rainforest
3) Himalayan region
1) Downstream processing
4) Wildlife Safari parks
2) Bioprocessing
178. Alexander Von Humbolt described for
3) Postproduction processing the first time
4) Upstream processing 1) Laws of limiting factor
171. A gene whose expression helps to 2) Species area relationships
identify transformed cell is known as 3) Population growth equation
1) Vector 4) Ecological biodiversity
2) Plasmid 179. The region of Biosphere Reserve which
3) Structural gene is legal protected and where no human
4) Selectable marker activity is allowed is known as
172. What is the criterion for DNA 1) Buffer zone 2) Transition zone
fragments movement on agarose gel 3) Restoration zone 4) Core zone
during gel electrophoresis 180. Which one of the following statements
1) The smaller the fragment size, the is not valid for aerosols
farther it moves 1) They alter rainfall and monsoon
2) Positively charged fragments move patterns
to farther end 2) They cause increased agricultural
3) Negatively charged fragments do not productivity
move 3) They have negative impact on
4) The larger the fragment size, the agricultural land
farther it moves 4) They are harmful to human health
113
Brilliant STUDY CENTRE

KEY WITH HINTS

PHYSICS
4. 3
1. 2

5. 2

6. 1

7. 2
2. 4

3. 3

8. 4

114
Previous Years NEET Questions & Solutions

13 2
9. 1

14. 3

10. 4
15. 4

16. 1 W  MB  cos 1  cos 2 

11. 1 1  00 , 2  180 0

12. 1, 2

17. 3

115
Brilliant STUDY CENTRE

18. 3 23. 2

24. 4

25. 3

26. 1 Centre of gravity coincides with centre of


19. 2 mass for bodies when gravity field is
uniform (usually for bodies of small height)
since gravitational force can be assumed
to be centred at the centre of gravity, so
torque of gravitational force about the
20. 3 centre of gravity must be zero. In the
mechanics, generally we deal with the
objects of small height for which torque of
gravitational for about centre of mass will
also be zero

27. 1

21. 2

22. 2 28. 4

116
Previous Years NEET Questions & Solutions

29. 4 34. 3

30. 2

35. 4

31. 4
36. 3

37. 3

32. 3 38. 3

33. 3

117
Brilliant STUDY CENTRE

CHEMISTRY
39. 2
46. 3

47. 2
48. 4

49. 2
50. 2

51. 1

40. 4

52. 4
41. 1

42. 3
53. 2

54. 2

55. 4 BCl 3 is sp2 hybridised and central atom


43. 2 does not have any lone pair of electrons,
hence all bond angles are 120o
56. 3

57. 2
44. 1
45. 2

118
Previous Years NEET Questions & Solutions

58. 2 65. 2

59. 1

66. 4

67. 3

68. 3

69. 1, 4
60. 2

61. 2

70. 3

71. 1

62. 4

72. 3

73. 4

74. 2
63. 2 75. 4

64. 2
76. 1

77. 3

119
Brilliant STUDY CENTRE

78. 1 83. 4

79. 4
84. 3

85. 2

86. 4
80. 4

87. 3

88. 2

81. 4

89. 4

82. 2

90. 3

120
Previous Years NEET Questions & Solutions

BIOLOGY

91. 4 106. 2 21. 2 136. 4 151. 4 166. 3


92. 3 107. 2 122. 1 137. 2 152. 1 167. 4
93. 3 108. 3 123. 3 138. 1 153. 4 168. 2
94. 2 109. 4 124. 2 139. 1 154. 1 169. 3
95. 1 110. 3 125. 4 140. 2 155. 1 170. 1
96. 3 111. 2 126. 3 141. 1 156. 2 171. 4
97. 2 112. 3 127. 3 142. 3 157. 3 172. 1
98. 2 113. 1 128. 1 143. 2 158. 4 173. 1
99. 1 114. 2 129. 4 144. 1 159. 2 174. 4
100. 2 115. 1 130. 2 145. 3 160. 3 175. 3
101. 4 116. 4 131. 4 146. 4 161. 2 176. 1
102. 2 117. 1 132. 3 147. 4 162. 1 177. 4
103. 4 118. 2 133. 1 148. 2 163. 2 178. 2
104. 2 119. 1 134. 4 149. 2 164. 1 179. 4
105. 2 120. 3 135. 1 150. 2 165. 4 180. 2

121
Brilliant STUDY CENTRE

NEET QUESTION PAPER - 2018


PHYSICS 5. An electrons falls from rest through a
vertical distance h in a uniform and
1. The power radiated by a black body is
vertically upward directed electric field
P and it radiates maximum energy at
E. The direction of electric field is now
wavelength,  0 . If the temperature of reversed, keeping its magnitude the
the black body is now changed so that same. A proton is allowed to fall from
it radiates max imum energy at rest in it through the same vertical
distance h. The time of fall of the
3
wavelength  0 , the power radiated electron, in comparison to the time of
4 fall of the proton is
by it becomes nP. The value of n is
1) Smaller 2) Equal
3 81 3) 10 times greater4) 5 time greater
1) 2)
4 256 6. A pendulum is hung from the roof of
sufficiently high building and is moving
256 4 freely to and fro like a simple harmonic
3) 4) oscillator. The acceleration of the bob
81 3
of the pe ndulum is 20 m/s 2 at a
2. Two wires are made of the same distance of 5m from the mean position.
material and have the same volume. The time period of oscillation is
The first wire has cross-sectional area
A and the second wire has cross- 1) 2s 2) 1 s 3) 2 s 4) s
sectional area 3A. If the length of the 7. The electrostatic force between the
first wire is increased by l on metal pates of an isolated parallel plate
applying a force F, how much force is capacitor C having a charge Q and
needed to stretch the second wire by area A, is
the same amount? 1) Independe nt of the distance
1) 9F 2) F 3) 4F 4) 6 F between the plates

3. A sample of 0.1g of water at 100oC and 2) Inve rsely proportional to the


normal pressure (1.013 × 10 5 Nm -2 ) distance between the plates
requi res 54 cal of heat energy to 3) Proportional to the square root of
convert to steam at 100 o C. If the the distance between the plates
volume of the steam produced is 167.1 4) Linearly proportional to the distance
cc, the change in internal energy of between the plates
the sample, is
8. A tuning fork is use d to produce
1) 104.3 J 2) 84.5 J resonance in a glass tube. The length
3) 42.2 J 4) 208.7 J of the air column is this tube can be
adjusted by a variable piston. At room
4. A small sphere of radius ‘r’ falls from temperature of 27 oC two successive
rest in a viscous force. As a result, resonances are produced at 20 cm and
heat is produced due to viscous force. 73 cm of column length. If the
The rate of production of heat when frequency of the tuning fork is 320 Hz,
the sphere attains its terminal velocity, the velocity of sound in air at 27oC is
is proportional to
1) 330 m/s 2) 300 m/s
1) r3 2) r4 3) r5 4) r2
3) 350 m/s 4) 339 m/s
122
Previous Years NEET Questions & Solutions

9. The ratio of kinetic energy to the total 14. A metallic rod of mass per unit length
energy of an electron in a Bohr orbit 0.5 kg m-1 is lying horizontally on a
of the hydrogen atom, smooth inclined plane which makes an
angle of 30o with the horizontal. The
1) 1 : 1 2) 1 : -2
rod is not allowed to slide down by
3) 2 : -1 4) 1 : -1 flowing a current through it when a
10. When the light of frequency 2v0 (where magnetic field of induction 0.25 T is
v0 is threshold frequency), is incident acting on it in the vertical direction.
on a metal plate, the maximum velocity The current flowing in the rod to keep
of electrons emitted is v 1. When the it stationary is
frequency of the incident radiation is 1) 7.14 A 2) 11.32 A
increased to 5v 0 , the maximum
velocity of electrons emitted from the 3) 14.76 A 4) 5.98 A
same plate is v2. The ratio of v1 to v2 is
15. A thin diamagnetic rod is place d
1) 1 : 2 2) 2 : 1 vertically between the poles of an
3) 4 : 1 4) 1 : 4 electromagnet. When the current in
the electromagnet is switched on,
11. For a radioactive material, half-life is then the diamagnetic rod is pushed up,
10 minutes. If initially there are 600 out of the horizontal magnetic field.
number of nuclei, the time taken (in Hence the rod gains gravitational
minutes) for the disintegration of 450 potential energy. The work required to
nuclei is do this comes from
1) 20 2) 15 3) 30 4) 10
1) The current source
12. An electron of mass m with an initial
 2) The induced electric field due to the
ve locity V  V0 ˆi  V0  0  e nters an changing magnetic field

electric field E  E 0ˆi (E0 = constant > 3) The lattice structure of the material
of the rod
0) at t = 0. If  0 is its de-Broglie
wavelength initially, the n its de- 4) The magnetic field
Broglie wavelength at time t is 16. Current sensitivity of a moving coil
galvanometer is 5 div/mA and its
0 voltage sensitivity (angular deflection
 eE 0  per unit voltage applied) is 20 div/V.
1) 1  t 2)  0
mV0 The resistance of the galvanometer is
 
1) 40  2) 500 
 eE 0  3) 250  4) 25 
3)  0 t 4)  0  1  t
 mV0  17. A solid sphere is in rolling motion. In
13. An inductor 20 mH, a capacitor 100 F rolli ng motion a body possesse s
translational kinetic energy (K t ) as
and a resistor 50  are connected in well as rotational kinetic energy (Kr)
series across a source of emf, V = 10 simultaneously. The ratio Kt : (Kt +Kr)
sin 314 t. The power loss in the circuit for the sphere is
is
1) 7 : 10 2) 2 : 5
1) 0.79 W 2) 1.13 W
3) 10 : 7 4) 5 : 7
3) 2.74 W 4) 0.43 W
123
Brilliant STUDY CENTRE

18. The kinetic energies of a planet in an 21. Unpolarised light incident from air on
e llipti cal orbit about the Su n, at a plane surface of a mate rial of
positions A, B and C are KA, KB and KC refractive index '  ' . At a particular
respectively. AC is the major axis and
angle of incidence ‘i’, it is found that
SB is perpendicular to AC at the
the reflected rays are perpendicular
position of the sun S as shown in the
to each other. Which of the following
figure. Then
options is correct for this situation?
1) Reflected light is polarised with its
electric vector parallel to the plane of
incidence

1
1
2) i  tan  


1) K A  K B  K C 1
1
3) i  sin  

2) K B  K A  K C
4) Reflected light is polarised with its
3) K B  K A  K C electric vector perpendicular to the
plane of incidence.
4) K A  K B  K C 22. In Young’s double slit experiment the
separation d between the slits is 2 mm,
19. If the mass of the sun were ten times the wavelength  of the light used is
smaller and the universal gravitational
o
constant were ten times larger in 5896 A and distance D between the
magnitude, which of the following is screen and slits is 100 cm. It is found
not correct? that the angular width of the fringes
is 0.20 o . To increases the fringe
1) Raindrops will fall faster
angular width to 0.21o (with same 
2) ‘g’ on the Earth will not change and D) the separation between the
slits needs to be changed to
3) Time period of a simple pendulum
on the Earth would decrease 1) 1.8 mm 2) 1.7 mm
3) 2.1 mm 4) 1.9 mm
4) None of these
23. An astronomical refracting telescope
20. A solid sphere is rotating freely about will have large angular magnification
the symmetry axis in free space. The and high angular resolution, when it
radius of the sphere is increased has an objective lens of
keeping its mass same. Which of the
1) Smal l focal length and large
following physical quantities would
diameter
remain constant for the sphere?
2) Small focal length and small
1) Angular velocity diameter
2) Angular momentum 3) Large focal l ength and large
diameter
3) Rotational kinetic energy
4) Large focal length and small
4) Moment of inertia diameter
124
Previous Years NEET Questions & Solutions

27. A body initially at rest and sliding along


24. A carbon resistor of  47  4.7  k is to a frictionless track from a height h (as
be marked with rings of different shown in the figure) just completes a
colours for is identification. The colour vertical circle of diameter AB = D. The
code sequence will be height h is equal to

1) Violet - Yellow - Orange - Silver

2) Green - Orange - Violet - Gold

3) Yellow - Green - Violet - Gold

4) Yellow - Violet - Orange - Silver

25. A set of ‘n’ equal resistor, of value ‘R’


each, are connected in series to a 3 5
battery of emf ‘E’ and internal 1) D 2) D
2 4
resistance ‘R’. The current drawn is I.
Now, the ‘n’ resistors are connected
in parallel to the same battery. Then 7
3) D 4) D
the current drawn from battery 5
becomes 10 I. The value of ‘n’ is
28. Three objects, A : (a solid sphere), B :
1) 10 2) 9 (a thin circular disk) and C : (a circular
ring), each have the same mass M and
3) 20 4) 11 radius R. They all spin with the same
angular speed  about their own
26. A battery consists of a variable number symmetry axes. The amounts of work
‘n’ of identical cells (having internal (W) required to bring them to rest,
resistance ‘r’ each) which are would satisfy the relation
connected in series. The terminals of
the battery are short-circuited and the
1) WC  WB  WA 2) WA  WC  WB
current I is measured. Which of the
graphs shows the correct relationship
between I and n? 3) WB  WA  WC 4) WA  WB  WC

29. Which one of the following statements


is incorrect?

1) Rolling friction is smaller than


1) 2) sliding friction

2) Coefficient of sliding friction has


dimensions of length

3) Frictional force opposes the relative


motion

4) Limiting value of static friction is


directly proportional to normal
3) 4) reaction

125
Brilliant STUDY CENTRE

30. A moving block having mass m, collides 35. In the circuit shown in the figure, the
with another stationary block having input voltage Vi is 20V, VBE = 0 and VCE
mass 4m. The lighter block comes to = 0. The values of IB, IC and  are given
rest after collision. When the initial
by
velocity of the lighter block is v, then
value of coefficient of restitution (e)
will be
1) 0.5 2) 0.4 3) 0.8 4) 0.25
31. An em wave is propagating in medium

with a vel ocity V  V ˆi .The
instantaneous oscillating electric field
of this em wave is along +y axis. Then
the direction of oscillating magnetic
field of the em wave will be along
1) -z direction 2) -x direction
3) -y direction 4) +z direction
32. The refractive index of the material of 1) I B  40A, IC  10 mA,   250
a prism is 2 and the angle of the
prism is 30o. One of the two refracting 2) I B  40A, IC  5mA,   125
surfaces of the prism is made a mirror
inwards, by silver coating. A beam of 3) I B  20A, I C  5 mA,   250
monochro matic light entering the
prism from the other face will retrace 4) I B  25A, IC  5 mA,   200
its path (after reflection from the
silvered surface) if its angle of 36. In a p-n junction diode, change in
incidence on the prism is temperature due to beating
1) 60o 2) Zero 1) Affects only reverse resistance
3) 30o 4) 45o 2) Affects the overall V-I characteristics
33. The magnetic potential energy stored of p-n junction
in a certain inductor is 25 mJ, when 3) Does not affect resistance of p-n
the current in the inductor is 60mA. junction
This inductor is of inductance
4) Affects only forward resistance
1) 0.138 H 2) 13.89 H
3) 1.389 H 4) 138.88 H 37. In the combination of the following
gates the output Y can be written in
34. An object is placed at a distance of 40
terms of inputs A and B as
cm from a concave mirror of focal
length 15 cm. If the object is displaced
through a distance of 20 cm towards
the mirror, the displacement of the
image will be
1) 30 cm away from the mirror
2) 36 cm towards the mirror
3) 30 cm towards the mirror 1) A B 2) A  B
4) 36 cm away from the mirror
3) A  B  A  B 4) A  B  A  B
126
Previous Years NEET Questions & Solutions

38. A toy car with charge q moves on a 41. The moment of the force ,
frictionless horizontal plane surface 
F  4iˆ  5jˆ  6kˆ at (2, 0, -3), about the
under the influence of a uniform
  point (2, -2, -2) is given by
electric field, E . Due to the force q E ,
its velocity increases from 0 to 6 m/s 1) 8iˆ  4ˆj  7kˆ
in one second duration. At that instant
the direction of the field is reversed. 2) 7iˆ  4ˆj  8kˆ
The car continues to move for two
more seconds under the influence of 3) 7iˆ  8jˆ  4kˆ
this field. The average velocity and the
average speed of the toy car between
4) 4iˆ  ˆj  8kˆ
0 to 3 seconds are respectively
1) 2 m/s, 4 m/s 2) 1.5 m/s, 3 m/s 42. The volume (V) of a monoatomic gas
varies with its temperature (T), as
3) 1 m/s, 3.5 m/s 4) 1 m/s, 3 m/s shown in the graph. The ratio of work
39. A block of mass m is placed on a done by the gas, to the heat absorbed
smooth incl ined wedge ABC of by it, when it undergoes a change from
inclination  as shown in the figure. state A to state B, is
The wedge is given an acceleration ‘a’
towards the right. The relation
between a and  for the block to
remain stationary on the wedge is

2 2
1) 2)
5 7
1 2
3) 4)
g 3 3
1) a  2) a  g tan 
cos ec  43. The fundamental frequency in an open
organ pipe i s equal to the third
g harmonic of a closed organ pipe. If the
3) a  g cos  4) a  length of the closed organ pipe is 20
sin 
cm, the length of the open organ pipe
40. A student measured the diameter of a is
small steel ball using a screw gauge
1) 13.2 cm 2) 16 cm
of least count 0.001 cm. The main scale
reading is 5mm and zero of circular 3) 12.5 cm 4) 8 cm
scale division coincide s with 25 44. The efficiency of an ideal heat engine
divisions above the reference level. If working between the freezing point and
screw gauge has a zero error of -0.004 boiling point of water, is
cm, the correct diameter of the ball is
1) 26.8% 2) 12.5%
1) 0.521 cm 2) 0.529 cm
3) 0.053 cm 4) 0.525 cm 3) 6.25% 4) 20%

127
Brilliant STUDY CENTRE

45. At what temperature will the rms 51. In the structure of ClF3, the number
speed of oxygen molecules become just of lone pairs of electrons of central
sufficient for escaping from the Earth’s atom ‘Cl’ is
atmosphere?
1) Four 2) Two
(Given: Mass of oxygen molecule (m) =
3) One 4) Three
2.76 × 1026 kg
52. Identify the major products P, Q and R
Boltzmann’s constant kB = 1.38 × 10-23
in the following sequence of reactions:
JK-1)
1) 2.508 × 104K 2) 1.254 × 104K
3) 5.016 × 104K 4) 8.360 × 104K + CH3CH2CH2Cl 
Anhydrous AlCl
 3

CHEMISTRY
46. The correct order of N-compounds in P  (i )O2
(ii )H O /
Q  R
3

its decreasing order of oxidation states


is
1) HNO3, NH4Cl, NO, N2
1)
2) HNO3, NO, NH4Cl, N2
3) HNO3, NO, N2, NH4Cl
4) NH4Cl, N2, NO, HNO3 2)
47. Which one of the following elements
is unable to form MF63 ion?
1) B 2) Al 3) Ga 4) In 3)
48. Considering Ellingham diagram, which
of the following metals can be used to
reduce alumina?
1) Mg 2) Zn 3) Fe D) Cu 4)
49. The correct order of atomic radii in
group 13 elements is 53. Which of the following compounds can
form a zwitter ion?
1) B<Ga<Al<Tl<In
1) Benzoic acid 2) Acetanilide
2) B<Al<Ga<In<Tl
3) Aniline 4) Glycine
3) B<Al<In<Ga<Tl
54. Regarding cross-linked or network
4) B<Ga<Al<In<Tl polyme rs, which of the following
50. Which of the following statements is statements is incorrect?
not true for halogens? 1) Examples are bakelite and melamine
1) All but fluorine show positive 2) They are formed from bi-and tri-
oxidation states functional monomers
2) All are oxidizing agents 3) They contain covalent bonds between
3) All form monobasic oxyacids various linear polymer chains
4) Chlorine has the highest electron- 4) They contain strong covalent bonds
gain enthalpy in their polymer chains

128
Previous Years NEET Questions & Solutions

55. Nitration of aniline in strong acidic 60. The compound X on treatment with Na
medium also gives m-nitroanili ne gives Y, and with PCl5 gives Z. Y and X
because react together to give diethyl ether. X,
Y and Z are in the order
1) In absence of substitutents intro
group always goes to m position 1) C2H5Cl, C2H6, C2H5OH

2) In electrophilic su bstitution 2) C2H5OH, C2H5Cl, C2H5ONa


reactions amino gro up is meta 3) C2H5OH, C2H6, C2H5Cl
directive
4) C2H5OH, C2H5ONa, C2H5Cl
3) Inspite of substituents nitro group 61. The compound C 7 H 8 undergoes the
always goes to only m-position following reactions:
4) In acidic (strong) medium aniline is 3Cl2/
C7H8   X 
Br2 /Fe
 Y 
Zn/HCl
Z
present as anilinium ion
1) 3-bromo-2, 4, 6-trichlorotoluene
56. The difference between amylose and
amylopectin is 2) o-bromotoluene

1) Amylopectin has 1  4  -linkage and 3) m-bromotoluene


1  6  -linkage 4) p-Bromotoluene

2) Amylose has 1  4  -linkage and 62. Hydrocarbon (A) reacts with bromine
by substitution to form an alkyl bromide
1  6  -linkage
which by Wurtz reaction is converted
3) Amylopectin has 1  4  -linkage and to gaseous hydrocarbon containing less
1  6  -linkage than four carbon atoms. (A) is
1) CH3–CH3 2) CH2=CH2
4) Amylose is made up of glucose and
galactose 3) CH  CH 4) CH4

57. A mixture of 2.3g formic acid and 4.5g 63. Which of the following molecules
oxalic acid is treated with conc. H2SO4. represents the order of hybridisation
The evolved gaseous mixture is passed sp2, sp2, sp, sp from left to right atoms?
through KOH pellets. Weight (in g) of 1) CH2=CH–CH=CH2
the remaining product at STP will be
2) CH2=CH–C  CH
1) 2.8 2) 3.0
3) HC  C–C  CH
3) 1.4 4) 4.4 4) CH3–CH=CH–CH3
58. Which of the following oxides is most 64. Which of the following carbocations is
acidic in nature? expected to be most stable?
1) BaO 2) BeO
3) MgO 4) CaO
59. Which oxide of nitrogen is not a 1) 2)
common pollutant introduced into the
atmosphere both due to natural and
human activity?
1) N2O 2) NO2
3) 4)
3) N2O5 4) NO.

129
Brilliant STUDY CENTRE

65. Which of the following is correct with 69. Match the metal ions given in column
respect to –I effect of the substitutents? I with the spin magnetic moments of
(R=alkyl) the ions given in Column II and assign
1)   –NH2>–OR>–F 2) –NR2<–OR<–F the correct code:
3) –NH2<–OR<–F 4) –NR2>–OR>–F
66. In the reaction

The electrophile involved is


 

1) Dichloromethyl anion  C HCl2  (a) (b) (c) (d)
 
1) iv i ii iii
  2) i ii iii iv
2) Formyl cation  C HO 
  3) iv v ii i
 
 4) iii v i ii
3) Dichloromethyl cation  C HCl2 
 
70. Which one of the following ions exhibits
4) Dichlorocarbene (:CCl2) d-d transition and paramagnetism as
67. Carboxylic acids have higher boiling well?
points than aldehydes, ketones and
even alcohols of comparable molecular 1) MnO4 2) Cr2O27
mass. It is due to their
1) More extensive association of 3) CrO24 4) MnO24
carboxylic acid via van der Waals force 71. Iron carbonyl, Fe(CO)5 is
of attraction
1) Trinuclear 2) Mononuclear
2) Formation of carboxylate ion
3) Formation of intramolecular H- 3) Tetranuclear 4) Dinuclear
bonding 72. The type of isomerism shown by the
4) Formation of intermolecular H- complex [CoCl2(en)2] is
bonding
1) Ionization isomerism
68. Compound X, C8H10O, is found to react
with NaOI(produced by reacting Y with 2) Coordination isomerism
NaOH) and yields a yellow precipitate 3) Geometrical isomerism
with characteristic small.
4) Linkage isomerism
X and Y are respectively
73. The geometry and magnetic behaviour
of the complex [Ni(CO)4] are
1)
1) Square planar ge ometry and
paramagnetic
2) 2) Tetrahe dral ge ometry and
diamagnetic
3) 3) Square planar ge ometry and
diamagnetic
4) Tetrahe dral ge ometry and
4) paramagnetic
130
Previous Years NEET Questions & Solutions

74. Following solutions were prepared by 78. In which case is number of molecules
mixing different volumes of NaOH and of water maximum?
HCl of different concentrations 1) 0.00224 L of water vapour at 1 atm
and 273 K
a)
2) 0.18 g of water
b) 3) 18 mL of water
4) 10–3 mol water
c) 79. The correct difference between first
and second order reactions is that
d) 1) A first order reaction can be
catalyzed, a second order reaction
pH of which one of them will be equal cannot be catalyzed
to 1 2) The half-life of a first order reaction
1) (d) 2) (a) does not depend on [A]0; the half-life
of a second order reaction does depend
3) (b) 4) (c) on [A]0
75. On which of the following properties 3) The rate of a first order reaction does
does the coagulating power of an ion not depend on reactant concentrations;
depend? the rate of a second order reaction does
depend on reactant concentrations
1) Both magnitude and sign of the
charge on the ion 4) The rate of a first order reaction does
depend on reactant concentrations; the
2) Size of the ion alone rate of a second order reaction does
not depend on reactant concentrations
3) The magnitude of the charge on the
ion alone 80. Among CaH2, BeH2, BaH2 the order of
ionic character is
4) The sign of charge on the ion alone
1) BeH2<BaH2<CaH2
76. Given van der Waals constant for NH3, 2) CaH2<BeH2<BaH2
H2, O2 and CO2 are respectively 4.17,
0.244, 1.36 and 3.59, which one of the 3) BeH2<CaH2<BaH2
following gases is most easily liquefied? 4) BaH2<BeH2<CaH2
1) O2 2) H2 81. Consider the change in oxidation state
of bromine corresponding to different
3) NH3 4) CO2 emf values as shown in the diagram
77. The solubility of BaSO 4 in water is below:
2.42×10–3 gL–1 at 298 K. The value of
its solubility product (Ksp) will be
(Given molar mass of BaSO4 = 233 g
mol–1)
1) 1.08×10–14mol2L–2 Then the species unde rgoing
disproportion is
2) 1.08×10–12mol2L–2
1) Br2 2) BrO4
3) 1.08×10–10mol2L–2
4) 1.08×10–8mol2L–2 3) BrO3 4) HBrO

131
Brilliant STUDY CENTRE

82. For the redox reaction 87. Consider the following species:
CN+, CN–, NO and CN
Which one of these will have the
The correct coefficients of the
highest bond order?
reactants for the balanced equation
are 1) CN+ 2) CN–
3) NO 4) CN
MnO4 C2O24 H
88. Magnesium reacts with an element (X)
1) 2 16 5
to form an ionic compound. If the
2) 2 5 16 ground state electronic configuration
3) 16 5 2 of (X) is 1s22s22p3, the simplest formula
4) 5 16 2 for this compound is
1) Mg2X
83. Which one of the following conditions
will favour maximum formation of the 2) MgX2
product in the reaction 3) Mg2X3
4) Mg3X2
1) High temperature and high pressure 89. Iron exhibits bcc structure at room
2) Low temperature and low pressure te mperature. Above 900 o C, it
transforms to fcc structure. The ratio
3) Low temperature and high pressure of density of iron at room temeprature
4) High temperature and low pressure to that at 900oC (assuming molar mass
and atomic radii of iron remai ns
84. When initial co ncentration of the
constant with temperature) is
reactant is doubled, the half life period
of a zero order reaction
3 3 4 2
1) is tripled 2) is doubled 1) 2)
4 2 3 2
3) is halved
4) remains unchanged 3 1
3) 4)
2 2
85. The bond dissociation energies of X2,
Y2 and XY are in the ratio of 1:0.5:1. 90. Which one is a wrong statement?
H for the formation of XY is –200 kJ
mol–1. The bond dissociation energy of 1) The electronic configuration of N
X2 will be atom is
1) 800 kJ mol–1 2) 100 kJ mol–1
3) 200 kJ mol–1 4) 400 kJ mol–1
86. The correction factor ‘a’ to the ideal
gas equation corresponds to 2) An orbital is designated by three
1) Electric field present between the quantum numbers while an electron
gas molecules in an atom is designated by four
quantum numbers
2) Volume of the gas molecules
3) Total orbital angular momentum of
3) Density of the gas molecules electron in ‘s’ orbital is equal to zero
4) Forces of attraction between the gas
4) The value of m for dz2 is zero
molecules
132
Previous Years NEET Questions & Solutions

BIOLOGY 94. After karyogamy followed by meiosis,


91. Match the items given in Column I spores are produced exogenously in
with those in Column II and select the 1) Neurospora 2) Alternaria
correct option given below 3) Agaricus 4) Saccharomyces
95. Oxygen is not produced during
photosynthesis by
Column I Column II 1) Green sulphur bacteria
i) It is a place having a 2) Nostoc
a) Herbarium collection of preserved 3) Cycas
plants and animals
4) Chara
ii) A list that
96. Which of the following organisms are
enumerates
known as chief producers in the
methodically all the
b) Key species found in an
oceans?
area with brief 1) Dinoflagellates
description 2) Diatoms
aiding identification
3) Cyanobacteria
iii) Is a place where 4) Euglenoids
dried and 97. Ciliates differ from all other protozoans
c) Museum pressed plant specimen in
mounted on sheets
1) using flagella for locomotion
are kept
2) having a contractile vacuole for
removing excess water
iv) A booklet containing
3) using pseudopodia for capturing prey
a list
of characters and their 4) having two types of nuclei
d) Catalogue
alternates which are 98. Which of the following statements is
helpful in identification correct?
of various taxa 1) Ovules are not enclosed by ovary
wall in gymnosperms
2) Selaginella is heterosporous, while
Salvinia is homosporous
1) a - i, b - iv, c - iii, d - ii
2) a - iii, b - ii, c - i, d - iv 3) Horsetails are gymnosperms
3) a - ii, b - iv, c - iii, d - i 4) Stems are usually unbranched in
4) a - iii, b - iv, c - i, d - ii both Cycas and Cedrus
92. Which among the following is not a 99. Which one is wrongly matched?
prokaryote?
1) Saccharomyces 2) Mycobacterium
1) Uniflagellate game te s Polysiphonia
3) Nostoc 4) Oscillatoria
93. Select the wrong statement 2) Biflagellate zoospore s Brown algae
1) Cell wall is present in members of 3) Ge mma cups Marchantia
Fungi and Plantae
2) Mushrooms belong to Basidiomycetes 4) Unicellular organism Chlorella
3) Pseudopodia are locomotory and
feeding structures in Sporozoans 100. Winged pollen grains are present in
4) Mitochondria are the powerhouse of 1) Mustard 2) Cycas
the cell in all kingdom except Monera 3) Mango 4) Pinus
133
Brilliant STUDY CENTRE

101. Identify the vertebrate group of animals 110. Which of the following features is used
characterized by crop and gizzard in to identify a male cockroach from a
its digestive system female cockroach?
1) Amphibia 2) Reptilia 1) Presence of a boat shaped sternum
3) Aves 4) Osteichthyes on the 9th abdominal segment
102. Which one of the these animals is not 2) Presence of caudal styles
a homeotherm? 3) Forewings with darker tegmina
1) Macropus 2) Chelone 4) Presence of anal cerci
3) Camelus 4) Psittacula 111. Which of the following is true for
103. Pneumatophores occur in nucleolus?
1) Halophytes 1) Larger nucle oli are present in
2) Free- floating hydrophytes dividing cells
3) Carnivorous plants 2) It is a membrane bound structure
4) Submerged hydrophytes 3) It takes part in spindle formation
104. Sweet potato is a modified 4) It is a site for active ribosomal RNA
synthesis
1) Stem
112. The Golgi complex participates in
2) Adventitious root
1) Fatty acid breakdown
3) Tap root
2) Formation of secretory vesicles
4) Rhizome
3) Respiration in bacteria
105. Stomata in grass leaf are
4) Activation of amino acid
1) Dumb-bell shaped
113. Which of the following events do not
2) Kidney shaped occur in rough endoplasmic reticulum?
3) Rectangular 1) Protein folding
4) Barrel shaped 2) Protein glycosylation
106. Secondary xylem and phloem in dicot 3) Cleavage of signal peptide
stem are produced by
4) Phospholipid synthesis
1) Apical meristems
114. Many ribosomes may associate with a
2) Vascular cambium single mRNA to form multiple copies of
3) Phellogen a polypeptide simultaneously. Such
4) Axillary meristem strings of ribosomes are termed as
107. Casparian strips occur in 1) Polysome
1) Epidermis 2) Pericycle 2) Polyhedral bodies
3) Cortex 4) Endodermis 3) Plastidome
108. Plants having little or no secondary 4) Nucleosome
growth are 115. Select the incorrect match
1) Grasses
2) Deciduous angiosperms 1) Lamp brush
Diplote ne bivale nts
chromosomes
3) Conifers
2) Allosome s Sex chromosomes
4) Cycads 3) Submetacentric L-shape d
109. Which of the following animals does chromosomes chromosomes
not undergo metamorphosis? 4) Polyte ne Oocytes of
1) Earthworm 2) Tunicate chromosomes amphibians

3) Moth 4) Starfish
134
Previous Years NEET Questions & Solutions

116. The two functional groups 123. Which of these stateme nts is
characteristic of sugars are incorrect?
1) hydroxyl and methyl 1) Enzymes of TCA cycle are present
in mitochondrial matrix
2) carbonyl and methyl
2) Glycolysis occurs in cytosol
3) carbonyl and phosphate
3) Glycolysis operates as long as it is
4) carbonyl and hydroxyl supplied with NAD+ that can pick up
117. The stage during which separation of hydrogen atoms
the paired homologous chromosomes 4) Oxidative phosphorylation takes
begins in place in outer mitochondrial
1) Pachytene 2) Diplotene membrane
3) Diakinesis 4) Zygotene 124. Which of the following terms describe
118. Stomatal movement is not affected by human dentition?
1) Thecodont, Diphyodont, Homodont
1) Temperature
2) Thecodont, Diphyodont, Heterodont
2) Light 3) Pleurodont,Monophyodont,Homodont
3) O2 concentration 4) Pleurodont, Diphyodont, Heterodont
4) CO2 concentration 125. Which of the following gastric cells
119. In which of the following forms is iron indirectly help in erythropoiesis?
absorbed by plants? 1) Chief cells 2) Mucous cells
1) Ferric 3) Goblet cells 4) Parietal cells
2) Ferrous 126. Which of the following options correctly
represents the lung conditions in
3) Free element
asthma and emphysema, respectively?
4) Both ferric and ferrous 1) Inflammation of bronchiole s,
120. Which of the following elements is Decreased respiratory surface
responsible for maintaining turgor in 2) Increased number of bronchioles,
cells? Increased respiratory surface
1) Magnesium 3) Increase d re spiratory surface,
2) Sodium Inflammation of bronchioles
3) Potassium 4) Decrease d re spiratory surface,
Inflammation of bronchioles
4) Calcium
127. Match the items in Column I with
121. Which of the following is not a product
those in Column II and select the
of light reaction of photosynthesis?
correct option given below
1) ATP
2) NADH Column I Column II
3) NADPH a) Tidal volume i) 2500 -3000 mL
4) Oxygen b) Inspiratory
ii) 1100 - 1200 mL
Reserve volume
122. What is the role of NAD + in cellular
respiration? c) Expiratory
iii) 500 - 550 mL
Reserve volume
1) It functions as an enzyme
2) It functions as an electron carrier d) Residual volume iv) 1000 - 1100 mL
3) It is a nucleotide source for ATP
1) a - iii, b - ii, c- i, d - iv
synthesis
2) a - iii, b - i, c - iv, d - ii
4) It is the final electron acceptor for 3) a - i, b - iv, c - ii, d - iii
anaerobic respiration
4) a - iv, b - iii, c - ii, d - i
135
Brilliant STUDY CENTRE

128. Which of the follo wing is an 131. Match the items given in column I with
occupational respiratory disorder? those in column II and select the
correct option given below
1) Anthracis
2) Silicosis
Column I Column II
3) Botulism
i) Accumulation of
a) Glycosuria
4) Emphysema uric acid in joints
129. Match the items given in Column I ii) Mass of
with those in Column II and select the crystalised
b) Gout
correct option given below salts within the
kidney
iii) Inflammation
Column I Column II c) renal calculi
in glome ruli
i) Between left
a) Tricuspid valve atrium d) Glomerular iv) Presence of
and left ventricle nephritis glucose in urine
ii) Between right
ventricle
b) Bicuspid valve 1) a - ii, b - ii, c - iv, d - i
and pulmonary
artery 2) a - i, b - ii, c - iii, d - iv
iii) Between right 3) a - ii, b - iii, c - i, c - iv
c) Semilunar valve atrium and right
ventrcile 4) a - iv, b - i, c - ii, d - iii
132. Match the items given in column I with
those in column II and select the
1) a-iii, b - i, c - ii correct option given below
2) a - i, b - iii, c - ii
3) a - i, b - ii, c - iii Column I Column II

4) a - ii, b - i, c- iii (Part of


(Function)
Excretory System)
130. Match the items given in column I with a) Ultrafiltration i) Henle's loop
those in column II and select the b) Concentration
correct option given below ii) Ureter
of urine
c) Transportation
iii) Urinary bladder
Column I Column II of urine
iv) Malpighian
a) Fibrinogen i) Osmotic balance d) Storage of urine
corpuscle
b) Globulin ii) Blood clotting v) Proximal
c) Albumin iii) Defence mechanism convoluted tubule

1) a - iii, b - ii, c - i 1) a - iv, b - v, c - ii, d - iii


2) a - i, b - ii, c - iiii 2) a - iv, b - i, c - ii, d - iii
3) a - i, b - iii, c- ii 3) a - v, b - ov, c - o, d - ii
4) a - ii, b - iii, c - i 4) a - v, b - iv, c - i, d - iii

136
Previous Years NEET Questions & Solutions

133. Which of the following hormones can 137. The transparent lens in the human eye
play a significant role in osteoporosis is held in its place by
1) Aldosterone and Prolactin 1) ligaments attached to the ciliary
2) Progesterone and Aldosterone body
3) Estrogen and Parathyroid hormone 2) ligaments attached to the iris
4) Parathyroid hormone and Prolactin 3) smooth muscles attached to the iris
134. Calcium is important in skeletal 4) smooth muscles attached to the
muscle contraction because it ciliary body
1) binds in troponin to remove the
138. Which of the following is an amino acid
masking of active sites on actin for
derived hormone?
myosin
1) Epinephrine
2) activates the myosin ATPase by
binding to it 2) Ecdysone
3) detaches the myosin head from the 3) Estradiol
actin filament 4) Estriol
4) prevents the formation of bonds 139. Offsets are produced by
between the myosin cross bridges and
1) Meiotic divisions
the actin filament
2) Mitotic divisions
135. Nissl bodies are mainly composed of
1) Proteins and lipids 3) Parthenocarpy
2) DNA and RNA 4) Parthenogenesis
3) Nucleic acid and SER 140. Which of the following flowers only once
4) Free ribosome and RER in its life time?
136. Which of the following statements or 1) Bamboo species
regions is incorrectly paired with its 2) Jackfruit
function? 3) Mango
4) Papaya
141. Which of the following has proved
controls respiration helpful in preserving pollen as fossils?
1) Medula
and cardiovascular 1) Pollenkitt
oblongata
reflexes 2) Cellulosic intine
consists of fibre 3) Oil content
tracts that 4) Sporopollenin
2) Limbic interconnect 142. Pollen grains can be stored for several
system different regions
years in liquid nitroge n having a
of brain, controls
temperature of
movement
1) 1200C 2) -800C
production of
releasing 3) -196 C
0 4) -1600C
3) Hypo hormones and 143. Double fertilisation is
thalamus regulation 1) Fusion of two male gametes of a
of temperature, pollen tube with two different eggs
hunger and thirst 2) Fusion of one male gamete with two
polar nuclei
band of fibers
4) Corpus connecting left 3) Fusion of two male gametes with
callosum and right cerebral one egg
hemispheres 4) Syngamy and triple fusion

137
Brilliant STUDY CENTRE

144. Which one of the following plants 148. Match the items column I with those
shows a very close relationship with a in column II and select the correct
species of moth, where none of the two option given below
can complete its life cycle without the
other? Column I Ciolumn II
1) Hydrilla a) Proliferative i) Breakdown of
Phase endometrial lining
2) Yucca
b) Secretory
3) Banana ii) Follicular Phase
Phase
4) Viola c) Menstruation iii) Luteal Phase
145. Hormones secreted by the placenta to
1) a - iii, b - i, c - i
maintain pregnancy are
2) a - i, b - iii, c - ii
1) hCG, hPL, progestogens, prolactin 3) a - ii, b - iii, c - i
2) hCG, hPL, estrogens, re laxin, 4) a - iii, b - i, c - ii
oxytocin 149. The contraceptive SAHELI?
3) hCG, hPL, progestogens, estrogens 1) blocks estrogen receptors in the
uterus, preventing eggs from getting
4) hCG, progestoge ns, estrogens,
implanted
glucocorticoids
2) increase s the concentration of
146. The difference between spermiogenesis estrogen and prevents ovulation in
and spermiation is female
1) In spermiogenesis, spermatids are 3) is an IUD
formed while in spermiation, 4) is a post - coital contraceptive
spermatozoa are formed 150. Select the correct statement
2) In spermiogenesis, spermatozoa are 1) Franklin Stahl coined the term
formed while in spermiation, “linkage”
spermatids are formed’ 2) Punnett square was developed by
British scientist
3) In spermiogenesis, spermatozoa
from Sertoli cells are released into the 3) Spliceosome s take part in
cavity of seminiferous tubules, while translation
in spermiation, spermatozoa are formed 4) Transduction was discovered by S.
Altman
4) In spermiogenesis, spermatozoa are
formed while in spermiation, 151. Which of the following pairs is wrongly
spermatozoa are released from sertoli matched?
cells into the cavity of seminiferous
tubules 1) Starch synthe sis
Multiple alle le s
in pea
147. The amnion of mammalian embryo is
derived from 2) ABO blood
Co-dominance
grouping
1) ectoderm and mesoderm
3) XO type
Grasshoppe r
2) endoderm and mesoderm se x de te rmination
3) mesoderm and trophoblast 4) T. H. Morgan Linkage
4) ectoderm and endoderm

138
Previous Years NEET Questions & Solutions

152. Select the correct match 157. AGGTATCCCAT is a sequence from the
coding strand of a gene. What will be
1) Ribozyme Nucle ic acid the corresponding sequence of the
transcribed mRNA?
2) F2 x Re ce ssive
Dihybrid cross 1) AGGUAUCCCAU
pare nt
2) UGGTUTCGCAT
3) T.H. Morgan Transduction
3) ACCUAUGCGAU
4) G. Me nde l Transformation
4) UCCAUAGCGUA
153. Which of the following characteristics 158. All of the following are part of an operon
represent ‘Inheritance of blood groups’ except
in humans? 1) an -operator
a) Dominance 2) structural genes
b) Co-dominance 3) an enhancer
c) Multiple allele 4) a promoter
d) Incomplete dominance 159. The similarity of bone structure in the
e) Polygenic inheritance forelimbs of many vertebrates in an
1) b, c and e 2) a,b an c example of
3) b, d and e 4) a, c and e 1) Homology
154. A woman has an X-linked condition on 2) Analogy
one of her X chromosome s. This 3) Convergent evolution
chromosome can be inherited to 4) Adaptive radiation
1) Only daughters 160. Among the following sets of examples
2) Only sons for divergent evolution, select the
3) Only grandchildren incorrect option
4) Both sons and daughters 1) Forelimbs of man, bat and cheetah
155. The experimental proof for 2) Heart of bat, man and cheetah
semiconservative replication of DNA 3) Brain of bat, man and cheetah
was first shown in a 4) Eye of octopus, bat and man
1) Fungus 161. According to Hugo de Vrie s, the
2) Bacterium mechanism of evolution is
3) Plant 1) Multiple step mutation
4) Virus 2) Saltation
156. Select the correct match 3) Phenotypic variations
4) Minor mutations
Streptococcus 162. Which part of poppy plant is used to
1) Alec Jeffreys
pneumoniae obtain the drug “Smack”?
2) Alfred He rshey 1) Flowers 2) Latex
TMV
and Martha Chase 3) Roots 4) Leaves
3) Mathe w Me se lson 163. Which of the following is not an
Pisum sativum autoimmune disease?
and F. Stahl
1) Psoriasis
4) Francois Jacob
Lac operon 2) Rheumatoid arthritis
and Jacques Monod
3) Alzheimer’s disease
4) Vitiligo
139
Brilliant STUDY CENTRE

164. In which disease does mosquito 170. A new variety of rice was patented by
transmitted of pathogen cause chronic a foreign company, though such
inflammation of lymphatic vessels? varieties have been present in India
1) Elephantiasis for a long time, This is related to
2) Ascariasis 1) Co- 667
3) Ringworm disease 2) Sharbati Sonora
4) Amoebiasis 3) Lerma Rojo
165. Conversion of milk to curd improves its 4) Basmati
nutritional value by increasing the 171. Niche is
amount of 1) all the biological factors in the
1) Vitamin D organism’s environment
2) Vitamin A 2) the physical space where an
3) Vitamin B12 organism lives
4) Vitamin E 3) the range of temperature that the
166. The correct order of steps in organism needs to lives
Polymerase Chain Reaction (PCR) is
4) the functional role played by the
1) Extension, Denaturation, Annealing
organism where it lives
2) Annealing, Extension, Denaturation
172. Natality refers to
3) Denaturation, Extension, Annealing
4) Denaturation, Annealing, Extension 1) Death rate
167. Which of the following is commonly 2) Birth rate
used as a vector for introducing a DNA 3) Number of individual leaving the
fragment in human lymphocytes habitat
1) Retrovirus 2) T1 plasmid
4) Number of individuals entering a
3) Phage 4) PBR 322 habitat
168. In India, the organisation responsible
173. Which one of the following population
for assessing the safety of introducing
interaction is widely used in medical
genetically modified organisms for
science for the production of
public use is
antibiotics?
1) Indian Council of Medical Research
(ICMR) 1) Commensalism
2) Council for Scientific and Industrial 2) Mutualism
Research (CSIR) 3) Parasitism
3) Research Committee on Genetic
Manipulation (RCGM) 4) Amensalism
4) Genetic Engineering Appraisal 174. In a growing population of a country
Committee((GEAC) 1) pre-reproductive individuals are
169. Use of bioresources by multinational more than the reproductive individuals
companies and organisations without
2) reproductive individuals are less
authorisation from the concerned
than the post reproductive individuals
country and its people is called
1) Bio- infringement 3) reproductive and pre-reproductive
individuals are equal in number
2) Biopiracy
3) Biodegradation 4) pre-reproductive individuals are less
than the reproductive individuals
4) Bioexploitation
140
Previous Years NEET Questions & Solutions

175. What type of ecological pyramid would 178. Which of the following is a secondary
be obtained with the following data? pollutant?
Secondary consumer - 120 g 1) CO 2) CO2
Primary consumer - 60 g 3) SO2 4) O3
Primary producer - 10 g 179. World Ozone Day is celebrated on
1) Inverted pyramid of biomass 1) 5th June
2) Pyramid of energy 2) 21st April
3) Upright pyramid of numbers 3) 16th September
4) Upright pyramid of biomass 4) 22nd April
176. All of the following are included in 180. Match the items given in Column I
‘Ex - situ conservation’ except with those in Column II and select the
1) Wildlife safari parks correct option given below
2) Sacred groves Column I Column II
3) Botanical gardens a) Eutrophication i) UV-B radiation
4) Seed banks b) Sanitary landfill ii) Deforestation
c) Snow blindness iii) Nutrient enrichment
177. In stratosphere, which of the following
d) Jhum cultivation iv) Waste disposal
elements acts as a catalyst in
degradation of ozone and release of 1) a - ii, b - i, c - iii, d - iv
molecular oxygen 2) a - i, b - iii, c - iv, d - ii
1) Carbon 2) Cl 3) a - iii, b - iv, c - i, d - ii
3) Fe 4) Oxygen 4) a - i, b - ii, c - iv, d - iii

141
Brilliant STUDY CENTRE

KEY WITH HINTS


PHYSICS
4. 3

1. 3

5. 1

6. 4
2. 1

7. 1

8. 4

9. 4
3. 4
 54  4.18  U  1.013  105 167.1 106  0.110 6 

U  208.7J

142
Previous Years NEET Questions & Solutions

10. 1
12. 1

11. 1

13. 1

143
Brilliant STUDY CENTRE

20. 2
14. 2

21. 4

15. 1

16. 3

22. 4

17. 4

23. 3

18. 4

24. 3

19. 2

144
Previous Years NEET Questions & Solutions

27. 2
25. 1

28. 1

29. 2

30. 4

26. 1 Equivalent emf of n cells connected in series =


nE
Equivalent resistance of n equal resistance =nr

31. 4

145
Brilliant STUDY CENTRE

32. 4 35. 2

33. 2

36. 2 Due to heating, number of electron hole pairs


34. 4 will increase, so overall resistance of diode
will change. Due to which forward and
reverse biasing both are changed
37. 4

38. 4

146
Previous Years NEET Questions & Solutions

42. 1

39. 2

43. 1

40. 2

41. 2

44. 1

147
Brilliant STUDY CENTRE

45. 4
52. 4

In the first step i.e. F.C., alkyla-


tion the propyl carbocation first
formed undergoes rearrangement
due to hydride shift ferming a
m or e st able 2 o carbocation
CHEMISTRY
53. 4
46. 3

47. 1 B has no vacant d-orbitals in its


va lence shell, hence it is inca-
pable of extending its covalency
beyond 4 i.e., B cannot form the
ion such as
54. 4 Cross linked or network polymers
48. 1 The metal that is more reactive are formed from bi-functional and
than ‘Al’ may reduce alumina i.e, tri-functional monomers and con-
Mg is then the correct answer tain strong covalent bonds be-
49. 4 tween various line ar polymer
chains, e.g., bakelite, malamine
etc.

Hence option (4) is not related to


50. 1 cross linking.  option (4) is not
51. 2 The structure of ClF3 is as given the correct answer.
below

55. 4

There are two lone


*
 N H3 is m-directing hence besides

pairs of electrons on the central para (51%) and ortho (2%), meta
Cl product (47%) is also formed in
significant amount.

148
Previous Years NEET Questions & Solutions

56. 3 Amylose and amylopectin are poly-


mers of   D glucose, so   link is 61. 3
not possible. Amylose is linear
with 1  4  -linkage where amy-
lopectin is branched and has both
1  4 and 1  6  -linkage

57. 1

62. 4

When gaseous mixture formed CO


an d CO 2 is passed through KOH,
only CO2 is absorbed. Hence re-
sidual gas is CO. 63. 2

Weight of the remaining gaseous 64. 1 –NO2 group exhibits -I effect and
2 it decreases with increase in dis-
CO  mol  28g mol 1  2.8g tance. In option 1 positive charge
20
is present on C-atom at maximum
Option (1) is correct answer. distance, so -I effect reaching to
it is minimum and stability is
maximum
58. 2 ; Basic
character increases 65. 3 –I effect increases on increasing
electronegativity of atom. So, cor-
The most acidic should be BeO. rect order of –I effect is –NH 2<–
In fact BeO is amphoteric oxide OR<–F
while other given oxides are ba-
sic 66. 4

59. 3

60. 4

67. 4

149
Brilliant STUDY CENTRE

68. 1
72. 3

73. 2

69. 3

74. 4

70. 4

75. 1

76. 3

71. 2
77. 3

150
Previous Years NEET Questions & Solutions

84. 2
78. 3

85. 1

79. 2

80. 3
86. 4

81. 4

87. 2

82. 2

83. 3 88. 4

151
Brilliant STUDY CENTRE

89. 1

90. 1

BIOLOGY

91. 4 106. 2 121. 2 136. 2 151. 1 166. 4


92. 1 107. 4 122. 2 137. 1 152. 1 167. 1
93. 3 108. 1 123. 4 138. 1 153. 2 168. 4
94. 3 109. 1 124. 2 139. 2 154. 4 169. 2
95. 1 110. 2 125. 4 140. 1 155. 2 170. 4
96. 2 111 4 126. 1 141. 4 156. 4 171. 4
97. 4 112. 2 127. 2 142. 3 157. 1 172. 2
98. 1 113. 4 128. 2 143. 4 158. 3 173. 4
99. 1 114. 1 129. 1 144. 2 159. 1 174. 1
100. 4 115. 4 130. 4 145. 3 160. 4 175. 1
101. 3 116. 4 131. 4 146. 4 161. 2 176. 2
102. 2 117. 2 132. 2 147. 1 162. 2 177. 2
103. 1 118. 3 133. 3 148. 3 163. 3 178. 4
104. 2 119. 1 134. 1 149. 2 164. 1 179. 3
105. 1 120. 3 135. 4 150. 2 165. 3 180. 3

152
Previous Years NEET Questions & Solutions

NEET QUESTION PAPER - 2019


1. In which of the following processes, 6. Body A of mass 4m moving with speed
heat is neither absorbed nor released u collides with another body B of mass
by a system? 2m, at rest. The collision is head on
1) isothermal 2) adiabatic and elastic in nature. After the collision
3) isobaric 4) isochoric the fraction of energy lost by the
colliding body A is:
2. Increase in temperature of a gas filled
in a container would lead to:
1 8
1) increase in its mass 1) 2)
9 9
2) increase in its kinetic energy
3) decrease in its pressure 4 5
3) 4)
4) decrease in intermolecular distance 9 9
3. The total energy of an electron in an 7. The speed of a swimmer in still water
atom in an orbit is –3.4 eV. Its kinetic is 20 m/s. The speed of river water is
and potential energie s are, 10 m/s and is flowing due east. If he
respectively:
is standing on the south bank and
1) –3.4 eV, –3.4 eV 2) –3.4 eV, –6.8 eV wishes to cross the river along the
3) +3.4 eV, –6.8 eV 4) 3.4 eV, 3.4 eV shortest path, the angle at which he
should make his strokes w.r.t. north
is given by:
1) 30o west 2) 0o
3) 60o west 4) 45o west
8. A mass m is attached to a thin wire
4.
and whirled in a vertical circle. The
wire is most likely to break when:
The correct Boolean ope ration 1) the mass is at the highest point
represented by the circuit diagram
2) the wire is horizontal
drawn is:
1) AND 2) OR 3) the mass is at the lowest point
3) NAND 4) NOR 4) inclined at an angle of 60 o from
5. A block of mass 10 kg is in contact vertical
against the inner wall of a hollow 9. The displacement of a particle
cylindrical drum of radius 1 m. The
executing simple harmonic motion is
coefficient of friction between the block
given by
and the inner wall of the cylinder is
0.1. The minimum angular velocity y = A0 + Asin  t + Bcos  t
needed for the cylinder to keep the
block stationary when the cylinder is Then the amplitude of its oscillation is
vertical and rotating about its axis, will given by:
be : (g = 10 m/s2)
1) A 0  A 2  B2 2) A 2  B2
10
1) 10 rad / s 2) rad/s
2
A 02   A  B
2
3) 4) A + B
3) 10 rad/s 4) 10  rad/s
153
Brilliant STUDY CENTRE

10. A 800 turn coil of effective area 0.05 17. In an experiment, the percentage of
m2 is kept perpendicular to a magnetic error occurred in the measurement of
field 5 × 10–5 T. When the plane of the physical quantities A, B, C and D are
coil is rotated by 90o around any of its 1%, 2%, 3% and 4% respectively. Then
coplanar axis in 0.1 s, the emf induced the maximum percentage of error in
in the coil will be:
1) 2 V 2) 0.2 V A 2 B1/ 2
the measurement X, where X = 1/3 3 ,
3) 2 × 10 V
–3 4) 0.02 V C D
11. Average velocity of a particle executing will be
SHM in one complete vibration is: 3
A 1)  % 2) 16%
1) 2) A  13 
2 3) –10% 4) 10%
A2 18. A body weighs 200 N on the surface of
3) 4) Zero the earth. How much will weigh half
2
12. A soap bubble having radius of 1 mm, way down to the centre of the earth?
is blown from a detergent solution 1) 150 N 2) 200 N
having a surface tension of 2.5 × 10–2 3) 250 N 4) 100 N
N/m. The pressure inside the bubble 19. Which colour of the light has the
equals at a point Z 0 below the free longest wavelength?
surface of water in a container. Taking 1) red 2) blue
g = 10 m/s2 density of water = 103kg/
m3, the value of Z0 is: 3) green 4) violet
1) 100 cm 2) 10 cm 20. A solid cylinder of mass 2 kg and radius
4 cm is rotating about its axis at the
3) 1 cm 4) 0.5 cm
rate of 3 rpm. The torque required to
13. A copper rod o f 88 cm and an stop after 2  revolutions is:
aluminium rod of unknown length have
their increase in length independent 1) 2 × 10–6 N m 2) 2 × 10–3 N m
of increase in temperature. The length 3) 12 × 10–4 N m 4) 2 × 106 N m
of aluminium rod is: 21. The radius of circle the period of
5 1 5 1
( Cu  1.7  10 K and  Al  2.2  10 K ) revolution initial position and sense of
revolution are indicated in the fig.
1) 6.8 cm 2) 113.9 cm
3) 88 cm 4) 68 cm
14. The unit of thermal conductivity is:
1) J m K–1 2) J m–1K–1
3) W m K–1 4) W m–1K–1
15. When a block of mass M is suspended
by a long wire of length L, the length
of the wire become (L + l). The elastic
potential energy stored in the extended
wire is: y-projection of the radius vector of
1) Mgl 2) MgL rotating particle P is:
1) y(t) = –3cos2  t, where y in m
1 1
3) Mgl 4) MgL  t 
2 2
2) y(t) = 4 sin   where y in m
16. A disc of radius 2m and mass 100 kg 2
rolls on a horizontal floor. Its centre of
mass has speed of 20 cm/s. How much  3t 
3) y(t) = 3 cos   where y in m
work is needed to stop it?  2 
1) 3 J 2) 30kJ
 t 
3) 2 J 4) 1 J 4) y(t) = 3 cos   where y in m
2
154
Previous Years NEET Questions & Solutions

22. A hollow metal sphere of radius R is 26. A force F = 20 + 10y acts on a particle
uniformly charged. The electric field in y-direction where F is in newton and
due to the sphere at a distance r from y in meter. Work done by this force to
the centre: move the particle from y = 0 to y = 1 m
1) increases as r increases for r < R is:
and for r > R
1) 30 J 2) 5 J
2) zero as r increases for r < R,
decreases as r increases for r > R. 3) 25 J 4) 20 J
3) zero as r increases for r < R, 27. Pick the wrong answer in the context
increases as r increases for r > R with rainbow.
4) decreases as r increases for r < R
1) When the light rays undergo two
and for r > R.
internal reflections in a water drop, a
23. In which of the following devices, the secondary rainbow is formed.
eddy current effect is not used?
1) induction furnace 2) The order of colours is reversed in
the secondary rainbow.
2) magnetic braking in train
3) electromagnet 4) electric heater 3) An observer can see a rainbow when
24. Six similar bulbs are connected as his front is towards the sun.
shown in the figure with a DC source 4) Rainbow is a combined effect of
of emf E, and zero internal resistance. dispersion refraction and reflection
The ratio of power consumption by the sunlight.
bulbs when
28. A cylindrical conductor of radius R is
(i) all are glowing and (ii) in the carrying a constant current. The plot
situation when two from section A and of the magnitude of the magnetic field,
one from section B are glowing, will B with the distance d, from the centre
be:
of the conductor, is correctly
represented by the figure:

1) 2)

1) 4 : 9 2) 9 : 4
3) 1 : 2 4) 2 : 1
25. At a point A on the earth’s surface the
angle of dip,  = +25o. At a point B on 3) 4)
the earth’ surface the angle of dip,  =
–25o. We can interpret that:
1) A and B are both located in the 29. Two particles A and B are moving in
northern hemisphere uniform circular motion in concentric
2) A is located in the southern circles of radius rA and rB with speed
hemisphere and B is located in the A and B respectively. The time period
northern hemisphere of rotation is the same. The ratio of
3) A is located in the northern angular speed of A to that of B will be:
hemisphere and B is located in the
southern hemisphere 1) rA : rB 2) A : B
4) A and B are both located in the 3) rB : rA 4) 1 : 1
southern hemisphere
155
Brilliant STUDY CENTRE

30. Two similar thin equi-convex lenses, 35. A parallel plate capacitor of capacitance
of focal length f each, are kept coaxially 20  F is being charged by a voltage
in contact with each other such that source whose potential is changing at
the local length of the combination is the rate of 3V/s. The conduction
F1. When the space between the two current through the connecting wires
lenses is filled with glycerin (which and the displacement current through
has the same refractive index (  = 1.5 the plates of the capacitor, would be
as that of glass) then the equivalent respectively:
focal length is F2. The ratio F1 : F2 will 1) zero, 60  A 2) 60  A, 60  A
be:
3) 60  A, zero 4) zero, zero
1) 2 : 1 2) 1 : 2
36. In the circuits shown below, the
3) 2 : 3 4) 3 : 4 readings of the voltmeters and the
32
31. In total internal reflection when the ammeters will be:
angle of incidence is equal to the
critical angle for the pair of media in
contact, what will be angle of
refraction?
1) 180o 2) 0o
3) equal to angle of incidence
4) 90o
32. Two parallel infinite line charges with
1) V2 > V1 and i1 = i2
linear charge densities +  C/m and –
2) V2 = V1 and i1 > i2
 C/m are placed at a distance of 2R
3) V1 = V2 and i1 = i2
in free space. What is the electric field
mid-way between the two line charges? 4) V2 > V1 and i1 > i2
37.  -particle consists of:
2 1) 2 protons and 2 neutrons only
1) zero 2)  R N / C
0 2) 2 e lectrons, 2 protons and 2
neutrons
  3) 2 electrons and 4 protons only
3)  R N / C 4) 2  R N / C
0 0 4) 2 protons only
33. For a p-type semiconductor which of 38. An electron is accelerated through a
the following statement is true? potential difference of 10,000 V. Its de
Broglie wavelength is, (nearly):
1) Electrons are the majority carriers
and trivalent atoms are the dopants. (me = 9 × 10–31 kg)
1) 12.2 × 10–13 m 2) 12.2 × 10–12 m
2) Holes are the majority carriers and
trivalent atoms are the dopants. 3) 12.2 × 10–14 m 4) 12.2 nm
3) Holes are the majority carriers and 39. When an object is shot from the bottom
pentavalent atoms are the dopants. of a long smooth inclined plane kept at
4) Electrons are the majority carriers an angle 60 o with horizontal, it can
and pentavalent atoms are the travel a distance x 1 along the plane.
dopants. But when the inclination is decreased
to 30o and the same object the shot
34. Which of the following acts as a circuit with the same velocity, it can travel x2
protection device? distance. Then x1 : x2 will be
1) conductor 2) inductor 1) 1: 2 2) 2 :1
3) switch 4) fuse
3) 1 : 3 4) 1: 2 3
156
Previous Years NEET Questions & Solutions

40. A small hole of area of cross-section 2 45. In double slit experiment, when light
mm2 is present near the bottom of a of wavelength 400 nm was used, the
fully filled open tank of height 2m. angular width of the first minima
Taking g = 10 m/s2, the rate of flow of formed on a screen placed 1m away,
water through the open hole would be was found to be 0.2o. What will be the
nearly: angular width of the first minima, If
1) 12.6 × 10–6m3/s 2) 8.9 × 10–6m3/s the entire experimental apparatus is
3) 2.23 × 10–6m3/s 4) 6.4 × 10–6m3/s immersed in water (  water = 4/3)
41. Two point charges A and B, having 1) 0.266o 2) 0.15o
charges +Q and –Q respectively, are
placed at certain distance apart and 3) 0.05o 4) 0.1o
force acting between them is F. If 25% 46. Under isothermal condition, a gas at
charge of A is transferred to B, then 300 K expands from 0.1L to 0.25L
force between the charges becomes: against a constant external pressure
9F of 2 bar. The work done by the gas is:
1) F 2) [Given that 1L bar = 100 J]
16
16F 4F 1) –30 J 2) 5 kJ
3) 4)
9 3 3) 25 J 4) 30 J
42. Ionized hydrogen atoms and  - 47. A compound is formed by cation C and
particles with same momenta enters anion A. The anions form hexagonal
perpendicular to a constant magnetic close packed (hcp) lattice and the
field B. The ratio of their radii of their cations occupy 75% of octahedral voids.
paths rH : r  will be The formula of the compound is:
1) 2 : 1 2) 1 : 2
1) C2A3 2) C3A2
3) 4 : 1 4) 1 : 4
 3) C3A4 4) C4A3
43. A particle moving with velocity V is
acted by three forces shown by the 48. pH of a saturated solution of Ca(OH)2
vector triangle PQR. The velocity of the is 9. The solubility product (K sp) of
particle will: Ca(OH)2 is.
1) 0.5 × 10–15 2) 0.25 × 10–10
3) 0.125 × 10–15 4) 0.5 × 10–10
49. The number of moles of hydrogen
molecules required to produce 20
moles of ammonia through Haber’s
process is:
1) increase 2) decrease
1) 10 2) 20
3) remain constant
4) change according to the smallest 3) 30 4) 40
 50. For an ideal solution, the correct option
force QR
is:
44. The work done to raise a mass m from
the surface of the earth to a height h, 1)  mix S = 0 at constant T and P
which is equal to the radius of the 2)  mix V  0 at constant T and P
earth, is:
1) mgR 2) 2mgR 3)  mix H = 0 at constant T and P
1 3 4)  mix G = 0 at constant T and P
3) mgR 4) mgR
2 2
157
Brilliant STUDY CENTRE

51. For a cell involving one electron 58. Which mixture of the solutions will
lead to the formation of negatively
E cell = 0.59V at 298 K, the equilibrium
charged colloidal (AgI)I– sol.?
constant for the cell reaction is: 1) 50mL of 1M AgNO3+ 50 mL of 1.5 M
2.303RT KI
[Given that = 0.059 V at 2) 50mL of 1M AgNO3 + 60mL of 2 M KI
F
T = 298K] 3) 50mL of 2M AgNO3+50mL of 1.5 M KI
1) 1.0 × 102 2) 1.0 × 105 4) 50mL of 0.1M AgNO3+50mL of 0.1MKI
59. Conjugate base for Bronsted acids H2O
3) 1.0 × 1010 4) 1.0 × 1030
and HF-are:
52. Among the following, the one that is
1) OH– and H2F+ respectively
not a green house gas is:
2) H3O+ and F–, respectively
1) nitrous oxide 2) methane
3) OH– and F–, respectively
3) ozone 4) sulphur dioxide
4) H3O+ and H2F+, respectively
53. The number of sigma    and pi    60. Which will make basic buffer?
bonds in pent-2-en-4-yne is: 1)50 mL of 0.1 M NaOH+25 ML of 0.1 M
1) 10  bonds and 3  bonds CH3COOH
2) 8  bonds and 5  bonds 2) 100 mL of 0.1 M CH 3 COOH +
3) 11  bonds and 2  bonds 100 mL of 0.1 M NaOH
4) 13  bonds and no  bonds 3) 100 mL of 0.1 M HCl + 200 mL of
0.1M NH4OH
54. Which of the following diatomic
molecular species has only  bonds 4) 100 mL of 0.1 M HCl + 100 mL of
according to Molecular Orbital Theory? 0.1 M NaOH
1) O2 2) N2 61. The compound that is most difficult to
protonate is:
3) C2 4) Be2
55. Which of the following reactions are
disproportionation reaction? 1) 2)
(a) 2cu+  Cu2+ + Cu0
(b)3Mn O24 +4H+  2Mn O 4 +MnO2+ 2H2O 3) 4)

(c) 2KMnO4   K2MnO4 + MnO2 + O2 62. The most suitable reagent for the
 following conversion is:
(d)2Mn O +3Mn2++2H2O  5MnO2+ 4H+
4

Select the correct option from the


following:
1) (a) and (b) only 2) (a), (b) and (c)
3) (a), (c) and (d) 4) (a) and (d) only 1) Na/liquid NH3
56. Among the following, the narrow 2) H2, Pd/C, quinoline
spectrum antibiotic is:
3) zn/HCl 4) Hg2+ /H+, H2O
1) pencillin G 2) ampicillin
63. Which of the following species is not
3) amoxycillin 4) chloramphenicol
stable?
57. The correct order of the basic strength
1) [SiF6]2– 2) [GeCl6]2–
of methyl substituted amines in
aqueous solution is: 3) [Sn(OH)6]2– 4) [SiCl6]2–
1) (CH3)2NH > CH3NH2 > (CH3)3N 64. Which of the following is an amphoteric
hydroxide?
2) (CH3)3N > CH3NH2 > (CH3)2NH
1) Sr(OH)2 2) Ca(OH)2
3) (CH3)3N > (CH3)2NH > CH3NH2
3) Mg(OH)2 4) Be(OH)2
4) CH3NH2 > (CH3)2NH > CH3)3N
158
Previous Years NEET Questions & Solutions

65. The structure of intermediate A in the 69. Ide ntify the incorrect stateme nt
following reaction is: related to PCl5 from the following:
1) Three equatorial P-Cl bonds make
an angle of 120o with each other.
2) Two axial P-Cl bonds make an angle
of 180o with each other
3) Axial P-Cl bonds are longer than
equatorial P-Cl bonds
4) PCl5 molecules is non-reactive
70. 4d, 5p, 5f and 6p orbitals are arranged
in the order of decreasing energy. The
correct option is:
1) 2)
1) 5f > 6p > 5p > 4d
2) 6p > 5f > 5p > 4d
3) 6p > 5f > 4d > 5p
4) 5f > 6p > 4d > 5p
71. The biodegradable polymer is:
1) nylon – 6,6 2) nylon 2-nylon 6
3) 4) 3) nylon –6 4) Buna-S
72. Match the Xenon compounds in
Column-I with its structure in
66. The mangnate and permanganate ions Column-II and assign the correct code:
are tetrahedral, due to Column-I Column-II
1) The  -bonding involves overlap of p-
(a) XeF4 (i) pyramidal
orbital of oxygen with d-orbitals of
manganese (b) XeF6 (ii) square planar
2) There is no  - bonding (c) XeOF4 (iii) distorted
3) The  - bonding involves overlap of - octahedral
p orbitals of oxygen with p-orbital of (d) XeO3 (iv) square
manganese
pyramidal
4) The  -bonding involves overlap of d
orbitals of oxygen with d - orbital of Code:
manganese (a) (b) (c) (d)
67. For the second period elements the 1) (i) (ii) (iii) (iv)
correct increasing order of first
ionisations enthalpy is: 2) (ii) (iii) (iv) (i)
1) Li < Be < B < C < N < O < F < Ne 3) (ii) (iii) (i) (iv)
2) Li < B < Be < C < O < N < F < Ne 4) (iii) (iv) (i) (ii)
3) Li < B < Be < C < N < O < F < Ne 73. Which is the correct thermal stability
4) Li < Be < B < C < O < N < F < Ne order for H2E
68. If the rate constant for a first order (E = 0, S, Se, Te and Po)?
reaction is k, the time (t) required for 1) H2S < H2O < H2Se < H2Te < H2Po
the completion of 99% of the reaction
2) H2O < H2S < H2Se < H2Te < H2Po
is given by:
1) t = 0.693 /k 2) t = 6.909/k 3) H2Po < H2Te < H2Se < H2S < H2O
3) t = 4.606 /k 4) t = 2.303 /k 4) H2Se < H2Te < H2Po < H2O < H2S
159
Brilliant STUDY CENTRE

74. The correct structure of 81. In which case change in entropy is


tribromooctaoxide is: negative?
1) Evaporation of water
2) Expansion of a gas at constant
1) 2)
temperature
3) Sublimation of solid to gas
4) 2H(g)  H2(g)
3) 4) 82. Match the following:
(a) Pure nitrogen (i) Chlorine
75. An alkene “A” on reaction with O3 and
Zn-H2O gives propanone and ethanal (b) Haber process (ii) Sulphuric acid
in equimolar ratio. Addition of HCl to (c) Contact process (iv) Ammonia
alkene “A” gives “B” as the major (d) Deacon’s process (v) Sodium azide
product. The structure of product “B” or Barium azide
is:
Which of the following is the correct
option?
1) 2) (a) (b) (c) (d)
1) (i) (ii) (iii) (iv)
2) (ii) (iv) (i) (iii)
3) 4) 3) (iii) (iv) (ii) (i)
4) (iv) (iii) (ii) (i)
76. Enzymes that utilize ATP in phosphate
83. Which of the following is incorrect
transfer require an alkaline earth
statement?
metal (M) as the cofactor. M is:
1) Be 2) Mg 1) PbF4 is covalent in nature
3) Ca 4) Sr 2) SiCl4 is easily hydrolysed
77. Which one is malachite from the 3) GeX4(X = F, Cl, Br, I) is more stable
following? than GeX2
1) CuFeS2 2) Cu(OH)2 4) SnF4 is ionic in nature
3) Fe3O4 4) CuCO3.Cu(OH)2
78. Which of the following series of 84. The non -essential amino acid among
transitions in the spectrum of hydrogen the following is:
atom falls in visible region? 1) valine 2) leucine
1) Lyman series 2) Balmer series 3) alanine 4) lysine
3) Paschen series 4) Bracket series 85. A gas at 350 K and 15 bar has molar
79. The mixture that forms maximum volume 20 percent smaller than that
boiling azeotrope is: of an ideal gas under the same
1) Water + Nitric acid conditions. The correct option about
2) Ethanol + Water the gas and its compressibility factor
3) Acetone + Carbon disulphide (Z) is:
4) Heptane + Octane 1) Z > 1 and attractive forces are
80. For the cell reaction dominant
2Fe3+ (aq) + 2I– (aq)  2Fe2+ (aq) + I2(aq) 2) Z > 1 and repulsive forces are
E cell = 0.24V at 298 K. Te standard Gibbs dominant
3) Z < 1 and attractive forces are
energy  0
r G   of the cell reaction is:
dominant
1) –46.32 kJ mol–1 3) –23.16 kJ mol–1 4) Z < 1 and repulsive forces are
1) 46.32 kJ mol–1 4) 23.16 kJ mol–1 dominant
160
Previous Years NEET Questions & Solutions

86. Among the following, the reaction that 90. The method used to remove temporary
proceeds through an electrophilic hardness of water is:
substitution is: 1) Calgon’s method 2) Clark’s method
3) Ion-exchange method
1) 4) Synthetic resins method
91. Which of the following statements is
incorrect?
2)
1) Viroids lack a protein coat
2) Viruses are obligate parasites
3) Infective constituent in viruses is
3) the protein coat
4) Prions consist of abnormally folded
proteins
4) 92. Purines found both in DNA and RNA
87. The major product of the following are:
reaction is: 1) Adenine and thymine
2) Adenine and guanine
3) Guanine and cytosine
4) Cytosine and thymine
93. Which of the following glucose
transporters is insulin-dependent?
1) GLUT I 2) GLUT II
3) GLUT III 4) GLUT IV
1) 94. Identify the cells whose secretion
protects the lining of gastro-intestinal
tract from various enzymes:
1) Chief cells 2) Goblet Cells
3) Oxyntic Cells 4) Duodenal cells
3)
95. Which one of the following equipments
is essentially required for growing
88. For the chemical reaction microbes on a large scale, for industrial
N2(g) + 3H2(g)  2NH3(g) production of enzymes?
the correct option is: 1) BOD incubator 2) Sludge digester
3) Industrial oven 4) Bioreactor
1) 2) 96. Which of the following is true for
Golden rice?
1) It is Vitamin A enriched, with a gene
3) 4)
from daffodil
89. What is the correct e lectronic 2) It is pest resistant, with a gene from
configuration of the central atom in Bacillus thuringiesis
[K 4 F e (CN) 6 ] based on crystal field 3) It is drought tolerant, developed
theory? using Agrobacterium vector
1) t 42g e 2g 2) t 62g e g0 4) It has yellow grains, because of a
gene introduce d from a primitive
3) e3 t 32 4) e4 t 22 variety of rice

161
Brilliant STUDY CENTRE

97. Which one of the following is not a 101. What map unit ( Centimorgan) is
method of in situ conservation of adopted in the construction of genetic
biodiversity? maps?
1) Biosphere Reserve 1) A unit of distance between two
expressed genes, representing 10%
2) Wildlife Sanctuary cross over
3) Botanical Garden 2) A unit of distance between two
expressed genes, representing 100%
4) Sacred Grove
cross over
98. Under which of the following conditions 3) A unit of distance between genes
will there be no change in the reading on chromosomes, representing 1%
frame of following mRNA? cross over
4) A unit of distance between genes
5 AACAGCGGUGCUAUU 3 on chromosomes, representing 50%
1) Insertion of G at 5th position cross over.
102. Select the hormone-releasing Intra-
2) Deletion of G from 5th position
Uterine Devices:
3) Insertion of A and G in 4th and 5th 1) Vaults, LNG-20
position respectively 2) Multiload 375, Progestasert
4) Deletion of GGU from 7th, 8th and 3) Progestasert, LNG-20
9th position 4) Lippes Loop, Multiload 375
99. Which of these following methods is 103. Which of the following can be used as
the most suitable for disposal of a biocontrol agent in the treatment of
nuclear waste? plant disease?
1) Shoot the waste into space 1) Trichoderma 2) Chlorella
3) Anabaena 4) Lactobacillus
2) Bury the waste under Antarctic ice
104. Expressed Sequence Tags (ESTs) refers
-cover
to:
3) Dump the waste within rocks under 1) Genes expressed as RNA
deep ocean 2) Polypeptide expression
4) Burry the waste within rocks deep 3) DNA polymorphism
below the Earth’s surface 4) Novel DNA sequences
100. Match the following organisms with the 105. Colostrum, the ye llowish fluid,
products they produce: secreted by mother during the initial
days of lactation is very essential to
(a ) L actob acillus (i) C h e e se
impart immunity to the newborn infants
(b) S accharomyces
(ii) C ur d because it contains:
cerevisiae
1) Natural killer cells
(c ) As pe r gillu s n ige r (iii) C itr ic Acid
2) Monocytes 3) Macrophages
(d) Acetoacter aceti (iv) B re a d
(v) A ce tic Acid 4) Immunoglobulin A
106. Select the incorrect statement:
Select the correct option. 1) Inbreeding increases homozygosity
(a) (b) (c) (d) 2) Inbreeding is essential to evolve
purelines in any animal
1) (ii) (iv) (v) (iii)
3)Inbreeding selects harmful recessive
2) (ii) (iv) (iii) (v) ge nes that reduce fertility and
3) (iii) (iv) (v) (i) productivity
4) Inbreeding helps in accumulation of
4) (ii) (i) (iii) (v) superior genes and elimination of
undesirable genes
162
Previous Years NEET Questions & Solutions

107. Select the correct sequence of 111. What is the site of perception of
transport of sperm cells in male photoperiod necessary for induction of
reproductive system? flowering in plants?
1) Testis  Epididymis  Vasa 1) Lateral buds 2) Pulvinus
efferentia  Rete testis  Inguinal
3) Shoot apex 4) Leaves
canal  Urethra
2) Seminiferous tubules  Rete testis 112. Which part of the brain is responsible
for thermoregulation?
 Vasa efferentia  Epididymis 
Vas deferens  Ejaculatory duct  1) Cerebrum 2) Hypothalamus
Urethra  Urethral meatus 3) Corpus callosum
3) Seminiferous tubule s  Vasa 4) Medulla oblongata
efferentia  Epididymis  Inguinal
113. Which of the following pair of
canal  Urethra
organelles does not contain DNA:
4) Testis  Epididymis  Vasa
efferentia  Vas defe rens  1) Mitochondria and Lysosomes
Ejaculatory duct  Inguinal canal  2) Chloroplast and Vacuoles
Urethra  Urethral meatus 3) Lysosomes and Vacuoles
108. A gene locus has two alleles A, a. If 4) Nuclear envelope and Mitochondria
the frequency of dominant allele A is
0.4 then what will be the frequency of 114. What is the genetic disorder in which
homozygous dominant, heterozygous an individual has an overall masculine
and homozygous recessive individuals development, gynaecomastia, and is
in the population? sterile?
1) 0.36 (AA); 0.48 (Aa); 0.16 (aa) 1) Turner’s syndrome
2) 0.16 (AA); 0.24 (Aa); 0.36 (aa) 2) Klinefelter’s syndrome
3) 0.16 (AA); 0.48 (Aa); 0.36 (aa) 3) Edward syndrome
4) 0.16 (AA); 0.36 (Aa); 0.48 (aa) 4) Down’s syndrome
109. Match the following organisms with
115. Xylem translocates:
their respective characteristics:
1) Water only
(a) Pila (i) Flame cells
2) Water and mineral salts only
(b) Bombyx (ii) Comb plates
3) Water, mineral salts and some
(c) Pleurobrachia (iii) Radula organic nitrogen only
(iv) Malpighian 4) Water, mineral salts, some organic
(d) Taenia
tubules nitrogen and hormones
Select the correct option from the 116. Which of the following pairs of gases
following: is mainly responsible for green house
(a) (b) (c) (d) effect?
1) (iii) (ii) (i) (iv) 1) Ozone and Ammonia
2) (iii) (iv) (ii) (i) 2) Oxygen and Nitrogen
3) (ii) (iv) (iii) (i) 3) Nitrogen and Sulphur dioxide
4) (iii) (ii) (iv) (i) 4) Carbon dioxide and Methane
110. The shorter and larger arms of a
117. Which of the following protocols did aim
submetacentric chro mosome are
for reduci ng e mission of chloro-
referred to as:
flurocarbons into the atmosphere?
1) s-arm and l-arm respectively
2) p-arm and q-arm respectively 1) Montreal Protocol 2) Kyoto Protocol
3) q-arm and p-arm respectively 3) Gothenburg Protocol
4) m-arm and n-arm respectively 4) Geneva Protocol
163
Brilliant STUDY CENTRE

118. Is some plants, the female gamete 124. The concept of “Omnis cellula-e
develops into embryo without cellula” regarding cell division was first
fertilization. This phe nomenon is proposed by:
known as: 1) Rudolf Virchow
1) Autogamy 2) Parthenocarpy 2) Theodore Schwann
3) Syngamy 4) Parthenogenesis 3) Schleiden 4) Aristotle
119. Which of the following se xually 125. Use of an artificial kidney during
transmitted diseases is not completely hemodialysis may result in:
curable? (a) Nitrogenous waste build-up in the
1) Gonorrhoea 2) Genital warts body
3) Genital herpes 4) Chlamydiasis (b) Non-elimination of excess potassium
120. Which of the fo llowing immune ions
responses is responsible for rejection (c) Reduced absorption of calcium ions
of kidney graft? from gastro- intestinal tract
1) Auto-immune responses (d) Reduced RBC production
2) Humoral immune response Which of the following options is the
3) Inflammatory immune response most appropriate?
4) Cell-mediated immune response 1) (a) and (b) are correct
121. Which of the following factors is 2) (b) and (c) are correct
responsible for the fo rmation of 3) (c) and (d) are correct
concentrated urine? 4) (a) and (d) are correct
126. What is the direction of movement of
1) Low levels of antidiuretic hormone
sugars in phloem?
2) Maintaining hyperosmolarity towards
1) Non-multidirectional
inner medullary interstitium in the
2) Upward 3) Downward
kidneys
4) Bi-directional
3) Secretion of erythropoietin by
juxtaglomerular complex 127. Which of the following muscul ar
disorders is inherited?
4) Hydrostatic pressure during
1) Tetany
glomerular filtration
2) Muscular dystrophy
122. Which of the following features of 3) Myasthenia gravis 4) Botulism
genetic code does allow bacteria to
128. Consider following features:
produce human insulin by recombinant
(a) Organ system level of organisation
DNA technology?
(b) Bilateral symmetry
1) Genetic code is not ambiguous
(c) True coelomates with segmentation
2) Genetic code is redundant of body
3) Genetic code is nearly universal Select the correct option of animal
4) Genetic code is specific groups which possess all the above
characteristics.
123. Which of the following statement is not
correct. 1) Annelida, Arthropoda and Chordata
2) Annelida, Arthropoda and Mollusca
1) Lysosomes have numerous
3) Antropoda, Mollusca and Chordata
hydrolytic enzymes
4) Annelida, Mollusca and Chordata
2) The hydrolytic enzymes of lysosomes 129. The frequency of re combination
are active under acidic pH. between gene pairs on the same
3) Lysosomes are membrane bound chromosome as a measure of the
structures distance between genes was explained
4) Lysosomes are formed by the process by:
of packaging in the endoplasmic 1) T.H.Morgan 2)Gregor J. Mendel
reticulum. 3) Alfred Sturtevant 4) Sutton Boveri
164
Previous Years NEET Questions & Solutions

130. Following statements describes the 136. In Antirrhinum (Snapdragon), a red


characteristics of the enzyme flower was crossed with a white flower
Restriction endonuclease. Identify the and in F1 generation, pink flowers were
incorrect statement. obtained. When pink flowers were
1) The enzyme cuts DNA molecule at selfed, the F 2 generation showed
identified position within the DNA white, red and pink flowers. Choose
the incorrect statement from the
2) The enzyme binds DNA at specific
sites and cuts only one of the two following:
strands 1) This experiment does not follow the
3) The enzyme cuts the sugar- Principle of Dominance
phosphate backbone at specific sites 2) Pink colour in F1 is due to incomplete
on each strand. dominance.
4) the enzyme recognize a specific
1 2 1
palindromic nucleotide sequence in 3) Ratio of F2 is (Red): (Pink) :
the DNA 4 4 4
(White)
131. Which of the following statements is
incorrect 4) Law of Segregation does not apply
1) Morels and truffles are edible in this experiment
delicacies 137. Select the incorrect statement
2) Claviceps is a source of many 1) Male fruit fly is heterogametic
alkaloids and LSD 2) In male grasshoppers, 50% of sperms
3) Conidia are produced exogenously have no sex-chromosome
and ascospores endogenously 3) In domesticated fowls sex of progeny
4) Yeasts have filamentous bodies with depends on the type of sperm rather
long thread like hyphae than egg.
132. Placentation, in which ovules develop 4) Human males have one of their sex-
on the inner wall of the ovary or in chromosome much shorter than the
peripheral part, is: other.
1) Basal 2) Axile 138. The correct sequence of phases of cell
3) Parietal 4) Free central cycle is:
133. Which of the following is the most 1) M  G1  G2  S
important causes for animals and 2) G1  G2  S  M
plants being driven to extinction.
3) S  G1  G2  M
1) Habitat loss and fragmentation
4) G1  S  G2  M
2) Drought and floods
139. Thiobacillus is a group of bacteria
3) Economic exploitation
helpful in carrying out:
4) Alien species invasion
1) Nitrogen fixation
134. Variations caused by mutation, as
proposed by Hugo de Vries, are 2) Chemoautotrophic fixation
1) random and directional 3) Nitrification 4) Denitrification
2) random and directionless 140. Polyblend, a fine powder of recycled
3) small and directional modified plastic, has proved to be a
good material for:
4) small and directionless
1) making plastic sacks
135. Respiratory Quotient (RQ) value of
tripalmittin is: 2) use as a fertilizer
1) 0.9 2) 0.7 3) Construction of roads
3) 0.07 4) 0.09 4) making tubes and pipes
165
Brilliant STUDY CENTRE

141. From evolutionary point of view, 146. The earth Summit held in Rio de
retention of the female gametophyte Janeiro in 1992 was called:
with developing young embryo on the 1) to reduce CO2 emissions and global
parent sporophyte for some time, is warming
first observed in: 2) for conservation of biodiversity and
1) Liver worts 2) Mosses sustainable utilization of its benefits.
3) to assess threat posed to native
3) Pteridophytes 4) Gymnosperms species by invasive weed species.
142. Select the correct option. 4) for immediate steps to discontinue
use of CFCS that were damaging the
1) 8 th , 9 th and 10 th pairs of ribs
ozone layer.
articulate directly with the sternum.
147. DNA percipitation out of a mixture of
2) 11 th and 12 th pairs of ribs are biomolecules can be achie ved by
connected to the sternum with the treatment with:
help of hyaline cartiage.
1) Isopropanol
3) Each rib is a flat thin bone and all
2) Chilled ethanol
the ribs are connceted dorsally to the
thoracic vertebrae and ventrally to the 3) Methanol at room temperature
sternum. 4) Chilled chloroform
4) There are seve n pairs of 148. Grass level curl inwards during very
vertebrosternal, thre e pairs of dry we athe r. Select the most
vertebrochondral and two pairs of appropriate reason from the following:
vertebral ribs. 1) Closure of stomata
143. Concanavalin A is: 2) Flaccidity of bulliform cells
1) an alkaloid 3) Shrinkage of air spaces in spongy
mesophil
2) an essential oil
4) Tyloses in vessels
3) a lectin
149. Match the following structures with
4) a pigment their respective location in organs:
144. Extrusion of second polar body from egg
nucleus occurs: (a) Crypts of
(i) Pancre as
lie be rkuhn
1) after entry of sperm but before
fertilization (b) Glissons’s
(ii) Duode num
Capsule
2) after fertilization
(c) Isle ts of
3) before entry of sperm into ovum (iii) Smallinte stine
Lange rhans
4) simultaneously with first cleavage (d) Brunne r’s
(iv) Live r
145. Pinus seed cannot germinate and Glands
establish without fungal association.
This is because Select the correct option from the
1) its embryo is immature following
2) it has obligate association with (a) (b) (c) (d)
mycorrhizae 1) (iii) (i) (ii) (iv)
3) is has very hard seed coat 2) (ii) (iv) (i) (iii)
4) its seeds contain inhibitors that 3) (iii) (iv) (i) (ii)
prevent germination 4) (iii) (ii) (i) (iv)

166
Previous Years NEET Questions & Solutions

150. Match the following hormones with the 154. Conversion of glucose to glucose 6-
respective disease: phosphate, the first irreve rsible
reaction of glycolysis, is catalyzed by:
(a) Insulin (i)Addison’s disease 1) Aldolase 2) Hexokinase
(b) Thyroxin (ii)Diabetes insipidus 3) Enolase
4) Phophofructokinase
(c) Corticods (iii) Acromegaly
155. Which of the following statement is
(d) Growth Hormone (iv) Goitre correct?
(v) Diabetes mellitus 1) Cornea is an external, transparent
and protective proteinacious covering
of the eye-ball.
Select the correct option
2) Cornea consists of dense connective
(a) (b) (c) (d) tissue of elastin and can repair itself.
1) (v) (i) (ii) (iii) 3) Cornea is convex, transparent layer
which is highly vascularised.
2) (ii) (iv) (iii) (i)
4) Cornea consists of dense matrix of
3) (v) (iv) (i) (iii) collagen and is the most sensitive
4) (ii) (iv) (i) (iii) portion of the eye.
156. Which of the following ecological
151. Which of the following contraceptive pyramids is generally inverted?
methods to involve a role of hormone?
1) Pyramid of numbers in grassland
1) Lactational amenorrhea, Pills, 2) Pyramid of energy
Emergency contraceptives 3) Pyramid of biomass in a forest
2) Barrier method, Lactational 4) Pyramid of biomass in a sea
amenorrhea, Pills 157. Consider the following statements.
3) CuT, Pills, Emergency contraceptive (A) Coenzyme or metal ion that is
tightly bound to enzyme protein is
4) Pills, Emergency contraceptives, called prosthetic group.
Barrier methods.
(B) A complete catalytic active enzyme
152. Drug called ‘Heroin’ is synthesized by: with its bound prosthetic group is
called apoenzyme.
1) methylation of morphine
Select the correct option.
2) acetylation of morphine 1) Both (A) and (B) are true.
3) glycosylation of morphine 2) (A) is true and (B) is false
4) nitration of morphine 3) Both (A) and (B) are false
4) (A) is false and (B) is true
153. In a species, the weigt of newborn
158. Due to increasing air-borne allergens
ranges from 2 to 5 kg. 97% of the
and pollutants, many people in urban
newborn with an ave rage weight
between 3 to 3.3 kg survive whereas areas are suffering from respiratory
99% of the infants born with weight disorder causing wheezing due to:
from 2 to 2.5 kg or 4.5 to 5 kg die. Which 1) benign growth on mucous lining of
type of selection process is taking nasal cavity
place? 2) inflammation of bronchi and
bronchioles.
1) Directional Selection
3) Proliferation of fibrous tissues and
2) Stabilizing Selection damage of the alveolar walls.
3) Disruptive Selection 4) reduction in the se cretion of
surfactants by pneumocytes.
4) Cyclical Selection

167
Brilliant STUDY CENTRE

159. Which one of the following statements 164. Match Column -I with Column -II
re garding post-fertilization
development in flowering plants is Column I Column II
incorrect. (i)Symbiotic association
(a) Saprophyte
of fungi with plant roots
1) Ovary develops into fruit
(ii) Decomposition of
2) Zygote develops into embryo (b) Parasite
dead organic materials
3) Central cell develops into endosperm (iii) Living on living
(c) Lichens
plants or animals
4) Ovules develop into embryo sac
(iv) Symbiotic association
(d) Mycorrhiza
160. Phloem in gymnosperms lacks: of algae and fungi

1) Albuminous cells and sieve cells


Choose the correct answer from the
2) Sieve tubes only options given below:
(a) (b) (c) (d)
3) Companion cells only
1) (i) (ii) (iii) (iv)
4) Both sieve tubes and companion 2) (iii) (ii) (i) (iv)
cells 3) (ii) (i) (iii) (iv)
161. It takes very long time for pineapple 4) (ii) (iii) (iv) (i)
plants to produce flowers. Whi ch 165. What would be the heart rate of a
combination of hormones can be applied person if the cardiac output is 5L, blood
to artificially induce flowering in volume in the ventricles at the end of
pineapple plants throughout the year diastole is 100 mL and at the end of
to increase yield? ventricula systole is 50 mL?
1) Auxin and Ethylene 1) 50 beats per minute
2) 75 beats per minute
2) Gibberellin and Cytokinin
3) 100 beats per minute
3) Gibberellin and Abscisic acid 4) 125 beats per minute
4) ytokinin and Abscisic acid 166. What triggers activation of protoxin to
active Bt toxin of Bacillus thuringiensis
162. Persistent nucellus in the seed is in boll worm?
known as:
1) Body temperature
1) Chalaza 2) Moist suyrface of Midgut
2) Perisperm 3) Alkaline pH of gut
4) Acidic pH of stomach
3) Hilum
167. The ciliate d epithe lial cells are
4) Tegmen required to move particles or mucus
in a specific direction. In humans,
163. Cell is G0 phase. these cells are mainly present in:
1) exit the cell cycle 1) Bile duct and Bronchioles
2) enter the cell cycle 2) Fallopian tubes and Pancreatic duct
3) Eustachian tube and Salivary duct
3) suspend the cell cycle
4) Bronchioles and Fallopian tubes
4) terminate the cell cycle

168
Previous Years NEET Questions & Solutions

168. Which of the statements given below 172. Match the hominids with their correct
is not true about formation of Annual brain size:
Rings in trees?
1) Annual ring is a combination of (a) Homo habilis (i) 900 cc
spring wood and autumn wood (b) Homo
produced in a year. (ii) 1350 cc
neanderthalensis
2) Differential activity of cambium (c) Homo erectus (iii) 650-800 cc
causes light and dark bands of tissue
(d) Homo sapiens (iv) 1400 cc
- early and late wood respectively
3) Activity of cambium depends upon Select the correct option.
variation in climate. (a) (b) (c) (d)
4) Annual rings are not prominent in
1) (iii) (i) (iv) (ii)
trees of temperate region.
2) (iii) (ii) (i) (iv)
169. What is the fate of the male gametes
discharged in the synergid? 3) (iii) (iv) (i) (ii)
1) One fuses with the egg, other(s) 4) (iv) (iii) (i) (ii)
degenerate(s) in the synergid. 173. Identify the correct pair representing
2) All fuse with the egg. the causative agent of typhoid fever and
3) One fuses with the egg, other(s) the confirmatory test for typhoid.
fuse(s) with synergid nucleus. 1) Plasmodim vivax/UTI test.
4) One fuses with the egg and other 2) Streptococcus pneumoniae/Widal
fuses with central cell nuclei. test
170. Match the following genes of the Lac 3) Salmonella typhi/Anthrone test
operon with their respective products: 4) Salmonella typhi/Widal test
(a ) i ge n e (i) ga la c tos id a s e 174. How does steroid hormone influence
( b) z g e n e (ii) Pe rm e a se the cellular activities?
(c) a ge n e ( i i i ) R e pr e s s o r 1) Changing the permeability of the cell
(d) y ge n e ( i v) T r a n s a c e t y l a s e membrane.
2) Binding to DNA and forming a gene-
Select the correct option. hormone complex.
(a) (b) (c) (d) 3) Activating cyclic AMP located on the
1) (i) (iii) (ii) (iv) cell membrane.
2) (iii) (i) (ii) (iv) 4) Using aquaporin channels as second
3) (iii) (i) (iv) (ii) messenger.
4) (iii) (iv) (i) (ii) 175. Tidal Volume and Expiratory Reserve
Volume of an athlete is 500 mL and
171. Select the correct sequence of organs
1000 mL respectively. What will be his
in the alimentary canal of cockroach
Expiratory capacity if the Residual
starting from mouth:
Volume is 1200 mL?
1) Pharynx  Oesophagus  Crop 
Gizzard  Ileum  Colon  Rectum 1) 1500 mL 2) 1700 mL
2) Pharynx  Oesophagus  Gizzard 3) 2200 mL 4) 2700 mL
 Crop  Ileum  Colon  Rectum 176. Which of the following is a commerial
3) Pharynx  Oesophagus  Gizzard blood cholesterol lowering agent?
 Ileum  Crop  Colon  Rectum
4) Pharynx  Oesophagus  Ileum 1) Cyclosporin A 2) statin
 Crop  Gizzard  Colon  Rectum 3) Streptokinase 4) Lipases
169
Brilliant STUDY CENTRE

177. Which of the following statement 179. Select the correct group of biocontrol
regarding mitochondria is incorrect? agents.
1) Outer membrane is permeable to
monomers of carbohydrates fats and 1) Bacillus thuringiensis, Tobacco
proteins. mosaic virus, Aphids
2) Enzymes of electron transport are
embedded in outer membrane.
2) Trichoderma, Baculovirus, Bacillus
3) Inner membrane is convoluted with
thuringiensis
infoldings.
4) Mitochondrial matrix contains single
circular DNA molecule and ribosomes. 3) Oscillatoria, Rhizobium,
178. Match the Column-I and Column -II. Trichoderma

Column I Column II 4) Nostoc, Azospirillium,


Nucleopolyhedrovirus
(i) Depolarisation
(a) P-wave
of ventricles
180. Select correctly written scientific
(ii) Repolarisation name of Mango which was first
(b) QRS complex
of ventricles described by Carlous Linnaeus:
(iii) Coronary
(c) T-wave 1) Mangifera Indica Car. Linn.
ischemia
(d) Reduction in
(iv) Depolarisation 2) Mangifera indica Linn.
the size of T-
of atria
wave
3) Mangifera Indica
(v) Repolarisation
of atria
4) Mangifera Indica
Select the correct option.
(a) (b) (c) (d)
1) (iv) (i) (ii) (iii)
2) (iv) (i) (iii) (ii)
3) (ii) (i) (iv) (iii)
4) (ii) (iii) (v) (iv)

170
Previous Years NEET Questions & Solutions

KEY WITH HINTS


1. 2 Adiabatic processs
7. 1 v = 20 m/s; u = 10 m/s
Q = 0
2. 2 KE  Temperature
As temperature increases KE also
increases
3. 3 TE = –3.4eV
KE = –T.E PE = 2T.E
 KE = + 3.4 eV  PE = –6.8 eV
mv 2
8. 3 T – mg cos  
R
4. 3  It is a NAND Gate

5. 3 FL = N  mr2
Fs = mg
As fs  fL T will be maximum when   0o
When mass is at lowest point.
9. 2 y = A0 + A sin t   B cos t
y = A0 + A 2  B2 sin  t   

A0 is mean position, and A 2  B2


is amplitude
10. 4 Given
N = 800, A = 0.05 m2, B = 5 × 10-5T
 t = 0.15 s
 mg  mr2

g
  r 11. 4 Displacement = zero in one com-
plete oscillation
 min  10 rad / s

6. 2 12. 3 P = P0 +  gZ0 ---- (i)
4T
Also, P = P0 + ---- (ii)
R
From (i) and (ii)
4T
pgZ0 =
R


= 10–2m = 1 cm

171
Brilliant STUDY CENTRE

13. 4 At any temperature


 l Cu   l Al
l11T  l2  2 T
21. 4
88 × 1.7 × 10–5 = l2 × 2.2. × 10–5
l2 = 68 cm

14. 4

For y-projection
= A cos t

15. 3
22. 2 For metal sphere Ein = 0
 Kq
and E out  2 rˆ
16. 1 r

A 2 B1/ 2
17. 2 x
C1/3 B3 23. 4 Eddy current effect is not used in
electric heater.

18. 4
24. 2

19. 1 Longest wavelength is of red colour


20. 1

25. 3 In northern hemisphere dip is +ve


and in southern hemisphere dip is
–ve.

26. 3

172
Previous Years NEET Questions & Solutions

27. 3 An observer can see a rainbow 34. 4 Fuse is used for protection.
when his back is towards the sun.
35. 2

28. 3

Also, conduction current in wires


is equal to displacement current
between the plates of capacitor.

29. 4 TA = TB
36. 3
2 2 
  A  B
A B
10  is in series with ideal volt-
30. 2 meter. Therefore it will not affect
the circuit (Circuit -2)
10 10
i1   1A i2   1A
31. 4 At critical angle 10 10
V1= 10V V2 = 10V
37. 1   2 He 4 = Helium Nuclei
2 protens and 2 neutrons

150 0
38. 2  A
V

angle of refraction = 90o 150 0


 A = 12.27 × 10–12m
104
32. 3
39. 3 v2 = u2 – 2as

40. 1 Velocity of efflux v = 2gh

Volume flow rate = Av = A 2gh


= (2 × 10–6) (2 × 10 × 2)1/2
= 4 10 × 10–6 m3/s
12.6 × 10-6 m3/s
33. 2 For P type
173
Brilliant STUDY CENTRE

47. 3 Anion A in HCP


41. 2 No of ions of A in Unit cell = 6
75% is occupied by cations C
25% charge from A is transferred
to B 75
No of cations C = 6 ×
100
3 9
=6× =
4 2
C9/2A6
C9A12 Simple ratio C3 A4
48. 1 Ca(OH)2|(s)  Ca2(aq) + 2OH–(aq)
S 2S
pH = 9; pOH = 5; [OH–] = 10–5 = 2S
42. 1

Ksp = [Ca2+] [OH–]2


Ksp = S × (2S)2
1
For same momenta, r  q Ksp = 4S3

Ksp = 4

Ksp = 0.5 × 10–15


49. 3 N2(g) + 3H2(g)  2NH3(g)
43. 3
2 mole NH3(g) requires 3 mole H2(g)
20 mole NH3(g) requires

= =30 mole

44. 3 50. 3 For an ideal solution,  H mix  0

2.303 RT
Ecell = E cell 
0
51. 3 log10 Q
nF
at equilibrium Ecell = 0, Q = Keq.
45. 2

46. 1 W = –Pext (V2 – V1) E 0cell   0.0591 log10 K eq


Pext = 2 bar
0.59 = + 0.0591 log10 Keq
V1 = 0.1 L
V2 = 0.25 L +10 = log10Keq
W = –2 bar[0.25 - 0.1] L Keq = 10+10
W = –2 × 0.15 bar L 52. 4 Beside s carbon dioxide other
W = –0.30 bar L greenhouse gases are methane,
W = (–0.30) × 100 = –30 J water vapour, nitrous oxide, CFCs
and ozone.
174
Previous Years NEET Questions & Solutions

60. 3 Basic buffer is mixture of weak


base and salt of weak base with
53. 1 strong acid.
milli mole of HCl = 100 × 0.1
Number of sigma bonds = 10 = 10 milli mole
Numbe of  bonds = 3 m illi m ole of NH 4OH = 200 × 0.1
54. 3 According to M.O.T. Electronic con- = 20 milli mole
figuration if C2 molecule is –
HCl + NH4OH  NH4Cl + H2O
 1S2 <  *1s2 <  2S2 <  *2S2 <  2 Px2 10 20 – –
=  2 Py2 – 10 10
61. 4 In case of phenol lone pair of oxy-
S0, C2 molecule contain only ‘  ’ gen is delocalized in ring.
bond.
55. 1 (a) 2Cu+  Cu+2 + Cu

(b) MnO 24   MnO 4 (oxidation


+6 +7
MnO 24   MnO2 (Reduction) 62. 2
+6 +4
The above two reaction are dispro- 63. 4 SiCl 26  does not exist since
portionation.
(i) size of Cl– is large so it cannot
56. 1 The antibiotics which effective
accommodate around Si+4 due to
mainly against Gram-positive or
Gram-negative bacteria are narrow limitation of size.
spectrum antibiotics, Penicillin G (ii) Interaction between lone pair
has a narrow spectrum. of chloride ion and Si+4 is not very
ampicillin, amoxycillin, chloram- strong.
phenicol are broad spectrum anti-
biotics. 64. 4 Be(OH)2 is an amphoteric hydrox-
ide rest all are basic hydroxides.
57. 1 The order of basic strength in case
of methyl substituted amines and 65. 2 Phenol is manufactured from the
ethyl substituted amines in aquous hydrocarbon cumene. Cumene
solution is as follows: (isopropylbenzene) is oxidised in
(C2H5)2 NH > (C2H5)3 N > C2H5NH2 > the presence of air to cumene hy-
NH3 droperoxide. It is converted to phe-
(CH3)2NH > CH3NH2 > (CH3)3N > NH3 nol and acetone by treating it with
58. 2 In negatively charged colloid [Agl] dilute acid. Acetone, a by-product
1–, 1– is preferentially adsorbed of this reaction is also obtained in
AgNO3 + KI  Agl + KNO3 large quantities by this method.
When KI is in excess, 1–1 will be
adsorbed on the surface of Agl and
(Agl)1– is formed.
59. 3 Conjugate base of H2O is OH–
Conjugate base of HF is F–

175
Brilliant STUDY CENTRE

66. 1 MnO 4 2 (Mangnate ion) and MnO 4 72. 2 (a) XeF4 – sp3d2, lp = 2,
(Permangnate ion) both are tetra- square planar
hedral. (b) XeF6 – sp3d3, lp = 1,
Distorted octahedral
(c) XeOF4 – sp3d2, lp = 1,
square pyramidal
(d) XeO3 – sp3, lp = 1, Pyramidal
73. 3 H2O H2S H2Se H2Te H2Po
Since '  ' bond is formed between
size of ce ntral atom
p-orbital of oxygen and d-orbital of
Managnese. increases, the rmal stability
decreases.
67. 2 For same shell
74. 1 The correct structure is:
[s1 < p1 < s2 < p2 < p4 < p3 < p5 < p6]
Li < B < Be < C < O < N < F < Ne
68. 3 For first order reaction

other options are anionic

75. 3

76. 2 All enzymes that utilize ATP in


phosphate transfer required mag-
nesium as the cofactor.
77. 4 malachite  CuCO3.Cu(OH)2
78. 2 In spectrum of hydrogen atom,
spectral lines of Balmer series lies
69. 4 in visioble region.
79. 1 Maximum boiling azeotrope are
formed by solutions which show
PCl5 is reactive molecule. negative de viation from ide al
70. 1 According to (n + l) rule, correct or behavious.
der of energy is 5f > 6p > 5p > 4d Water + Nitric acid shows negaive
For same value of (n + l), higher is deviation
the value of n, higher will be the 80. 1 2Fe3+ (aq) + 2l– (aq)  2Fe2+ (aq) +
energy. I2(aq)
71. 2 Nylon 2-nylon 6 n=2
It is an alternating polyamide co-  Go = -nFeo = –2 × 96500 × (0.24)
polymer of glycine (H 2N – CH 2 – = –46320 J = –46.32 kJ mol–1
COOH) and amino caproic acid 81. 4 2H(g)  H2(g)
[H 2 N(CH 2 ) 5 COOH] and is biode- Due to bond formation, entropy
gradable. decreases
176
Previous Years NEET Questions & Solutions

82. 4 (a) Pure nitrogen  Thermal


88. 3
decomposition of sodiumazide or
Bariumazide
(2 NaN3 
 2Na + 3N2) 89. 2 In K4[Fe(CN)6]
(Ba(N3)2 
 Ba + 3N2) Fe(26) = 3d6 4s2
Fe+2 = 3d6
(b) Haber process  Formation of 6 0
Ammonia in presence of SFL 3d6  t 2g e g
(N2 + 3H2  2NH3) 90. 2 Clark’s method used to remove
temporary hardness of water.
(c) Contact process  manufac- Ca(HCO3)2 + Ca(OH)2 
ture of H2SO4 2CaCO3 + 2H2O
(d) Deacon\s process  Formation
of Cl2 gas 91. 3 92. 2 93 4
(HCl +O2(Atmosphere) 
CuCl2
 H2O+ Cl2) 94. 2 95. 4 96. 1
83. 1 pbF4 is an ionic compound due to 97. 3 98. 4 99. 4
large size of cation and small size 100. 2 101. 3 102. 3
of anion. Rest all are correct op-
tions. 103. 1 104. 1 105. 4
84. 3 non-essential amino acid - alanine 106. 3 107. 2 108. 3
109. 2 110. 2 111. 4
Essential amino acid - valine, leu-
cine, lysine. 112. 2 113. 3 114. 2
85. 3 115. 4 116. 4 117. 1
118. 4 119. 3 120. 4
121. 2 122. 3 123. 4
124. 1 125. 3 126. 4
Z < 1 and attractive forces are
dominant. 127. 2 128. 1 129. 3
130. 2 131. 4 132. 3
86. 2 Halogenation (Electrophilic substi-
tution reactions) : Arenes react 133. 1 134. 2 135. 2
with halogens in the presence of a 136. 4 137. 3 138. 4
Lewis acid like anhydrous AlCl3
139. 4 140. 3 141. 3
142. 4 143. 3 144. 1
145. 2 146. 2 147. 2
148. 2 149. 3 150. 3
151. 1 152. 2 153. 2
87. 2
154. 2 155. 4 156. 4
157. 2 158. 2 159. 4
160. 4 161. 1 162. 2
163. 3 164. 4 165. 3
166. 3 167. 4 168. 4
169. 4 170. 3 171. 1
172. 3 173. 4 174. 2
175. 1 176. 2 177. 2
178. 1 179. 2 180. 2

177
Brilliant STUDY CENTRE

NEET QUESTION PAPER - 2019[ODISHA]


1. Two metal sphere, one of radius R and 5. An object flying in air with velocity
the other of radius 2R respectively have
the same surface charge density .
 20iˆ  25jˆ  12kˆ  suddenly breaks in two
The are bro ught in contact and pieces whose masses are in the ratio
separated. What will be the new 1 : 5. The smaller mass flies off with a
surface charge densities on them?
 
velocity 100iˆ  35jˆ  8kˆ . The velocity of
5 5 the larger piece will be:-
1) 1  , 2  
6 2
1) 4iˆ  23jˆ  16kˆ 2) 100iˆ  35jˆ  8kˆ
5 5
2) 1  , 2   3) 20iˆ  15ˆj  80kˆ 4) 20iˆ  15jˆ  80kˆ
2 6
6. An object kept in a large room having
5 5 air temperature of 25 o C takes 12
3) 1  , 2  
2 3 minutes to cool from 80oC to 70oC.
The time taken to cool for the same
5 5
4) 1  , 2   object from 70 o C to 60 o C would be
3 6 nearly:-
2. The distance covered by a particle 1) 10 min 2) 12 min
undergoing SHM in one time period is 3) 20 min 4) 15 min
(amplitude = A): -
7. Two small spherical metal balls, having
1) zero 2) A equal masses, are made from
3) 2A 4) 4A materials of densitie s 1 and
3. A mass falls from a height ‘h’ and its 2 (1  8 2 ) and have radii of 1mm and
time of fall ‘t’ is recorded in terms of 2mm, respectively. They are made to
time period T of a simple pendulum. fall vertically (from rest) in a viscous
On the surface of earth it is found that medium whose coefficient of viscosity
t = 2T. The entire set up is taken on
equals  and whose density is 0.1 2 .
the surface of another planet whose
mass is half of that of earth and radius The ratio of their terminal velocities
the same. Same experiment is would be:-
repeated and corresponding times 79 19
noted as t ' and T ' , then 1) 2)
72 36
1) t   2 T 2) t   2T  39 79
3) 4)
72 36
3) t   2T  4) t   2T 
8. A particle starting from rest, moves in
4. A tuning fork with frequency 800 Hz a circle of radius ‘r’. It attains a velocity
produces resonance in a resonance of V0 m/s in the nth round. Its angular
column tube with upper end open and acceleration will be:-
lower end closed by water surface.
Successive resonance are observed at V0 V02
1) rad / s 2 2) rad / s 2
length 9.75 cm, 31.25 cm and 52.75 n 2nr 2
cm. The speed of sound in air is:-
1) 500 m/s 2) 156 m/s V02 V02
3) rad / s 2 4) rad / s 2
4nr 2 4nr
3) 344 m/s 4) 172 m/s
178
Previous Years NEET Questions & Solutions

9. A person standing on the floor of an 11. In a U-tube as shown in figure, water


elevator drops a coin. The coin reaches and oil are in the left side and right
the floor in time t1 if the elevator is at side of the tube respectively. The
rest and in time t 2 if the elevator is heights from the bottom for water and
moving uniformly. Then:- oil columns are 15 cm and 20 cm
respectively. The density of the oil is:

1) t 1 < t 2 of t 1 > t 2 depending upon


 take  water  1000 kg / m3 
whether the lift is going up or down.

2) t1 < t2

3) t1 > t2

4) t1 = t2

10. A truck is stationary and has a bob


1) 1200 kg/m3 2) 750 kg/m3
suspended by a light string, in a frame
attached to the truck. The truck,
3) 1000 kg/m3 4) 1333 kg/m3
suddenly moves to the right with an
acceleration of a. The pendulum will
tilt:- 12. A deep rectangular pond of surface area
A, containing water (density =  ,
specific heat capacity = s), is located
1) to the left and angle of inclination in a region where the outside air
of the pendulum with the vertical is temperature is at a steady value of –
26oC. The thickness of the frozen ice
g
sin 1   layer in this pond, at a certain instant
a is x.

Taking the thermal conductivity of ice


2) to the left and angle of inclination as K, and its specific latent heat of
of the pendulum with the vertical is fusion as L, the rate of increase of the
a thickness of ice layer, at this instant
tan 1   would be given by:-
g
1) 26K/ ρ r(L–4s) 2) 26K/( ρ x2–L)
3) to the left and angle of inclination
of the pendulum with the vertical is 3) 26K/( ρ xL) 4) 26K/ ρ r(L+4s)
a
sin 1   13. An LED is constructed from a p-n
g juction diode using GaAsP. The energy
gap is 1.9 eV. The wavelength of the
light emitted will be equal to:-
4) to the left and angle of inclination
of the pendulum with the vertical 1) 10.4 × 10–26 m 2) 654 nm
1  g 
is tan  
a 3) 654A
o
4) 654 × 10-11 m

179
Brilliant STUDY CENTRE

14. The circuit diagram shown here 18. A double convex lens has focal length
corresponds to the logic gate, 25 cm. The radius of curvature of one
of the surfaces is double of the other.
Find the radii if the refractive index of
the material of the lens is 1.5:-
1) 100 cm, 50 cm 2) 25 cm, 50 cm
3) 18.75 cm, 37.5 cm
4) 50 cm, 100 cm
19. Two bullets are fired horizontally and
simultaneously towards each other
from roof of two buildings 100 m apart
and of same height of 200 m with the
same velocity of 25 m/s. When and
where will the two bullets collide.
1) NOR 2) AND 3) OR 4) NAND (g = 10 m/s2)

 Cp  1) after 2s at a height 180 m


15. The value of    C  , for hydrogen,
 v 2) after 2s at a height 20 m
helium and another ideal diatomic gas
X (whose molecules are not rigid but 3) after 4s at a height 120 m
have an additional vibrational mode) 4) they will not collide
are respectively equal to:-
20. The stress-strain curves are drawn for
7 5 9 5 7 9 two different materials X and Y. It is
1) , , 2) , ,
5 3 7 3 5 7 observed that the ultimate strength
point and the fracture point are close
5 7 7 7 5 7 to each other for material X but are
, ,
3) 4) , ,
3 5 5 5 3 5 far apart for material Y. We can say
16. An equiconvex lens has power P. It is that materials X and Y are likely to be
cut into two symmetrical halves by a (respectively).
plane containing the principal axis. The
power of one part will be:- 1) ductile and brittle
2) brittle and ductile
P
1) 0 2) 3) brittle and plastic
2
4) plastic and ductile
P
3) 4) P
4 21. A body of mass m is kept on a rough
17. In a Young’s double slit experiment if horizontal surface (coefficient of friction
there is no initial phase difference = ) A horizontal force is applied on the
body, but it does not move. The
between the light from the two slits, a
resultant of normal reaction and the
point on the screen correspondng to
frictional force acting on the object is
the fifth minimum has path difference.
given by F, where F is:
   
1) 5 2) 10 1) F  mg  mg 2) F  mg
2 2
   
3) 9 4) 11 3) F  mg 1   2 4) F  mg
2 2
180
Previous Years NEET Questions & Solutions

22. A particle of mass 5 m at rest suddenly 26. A straight conductor carrying current
breaks on its own into three fragments. i splits into two parts as shown in the
Two fragmentsof mass m each move figure. The radius of the circular loop
along mutually perpendicular direction is R. The total magnetic field at the
with speed v each. The energy released centre P of the loop is:
during the process is:
3 2 5 2
1) mv 2) mv
5 3
3 2 4 2
3) mv 4) mv
2 3
23. An object of mass 500g, initially at rest
acted upon by a variable force, whose
X component varies with x in the 1) zero 2) 30i/32R, outward
manner shown. The velocities of the 3) 30i/32R, inward
object at point X = 8 m and X = 12 m,
would be the respective values of 2) 0i/2R, inward
(nearly) 27. The variation of EMF with time for four
types of generators are shown in the
figures. Which amonged them can be
called AC?

1) 18 m/s and 24.4 m/s


2) 23 m/s and 24.4 m/s
3) 23 m/s and 20.6 m/s
4) 18 m/s and 20.6 m/s
24. A solid cylinder of mass 2 kg and radius
50 cm rolls up an inclined plane of 1) (a) and (d) 2) (a), (b), (c) and (d)
angle inclination 30 0. The centre of
mass of cylinder has speed of 4 m/s. 3) (a) and (b) 4) only (a)
The distance travelled by the cylinder 28. The radius of the first permitted Bohr
on the incline surface will be: orbit for the electron, in a hydrogen
(Take g = 10 m/s2) 0
atom equals 0.51A and its ground state
1) 2.2 m 2) 1.6 m energy equals –13.6 eV. If the electron
3) 1.2 m 4) 2.4 m in the hydrogen atom is replaced by
25. Two toroids 1 and 2 have total number m u on (–)
of turns 200 and 100 respectively with [charge same as electron and mass
average raddi 40 cm and 20 cm 207 m e ), the first Bohr radius and
re spectively. If they carry same ground state energy will be:
current i, then the ratio of the 1) 0.53 × 10–13 m, –3.6 eV
magnetic fields along the two is;
2) 25.6 × 10–13 m, –2.8 eV
1) 1 : 1 2) 4 : 1 3) 2.56 × 10–13 m, –2.8 eV
3) 2 : 1 4) 1 : 2 4) 2.56 × 10–13 m, –13.6 eV
181
Brilliant STUDY CENTRE

29. The reading of an ideal voltmer in the 32. The rate of radioactive disintegration
circuit shown is: at an instant for a radioactive sample
of half life 2.2 × 109 s is 1010 s–1. The
number of radioactive atoms in that
sample at that instant is,
1) 3.17 × 1020 2) 3.17 × 1017
3) 3.17 × 1018 4) 3.17 × 1019
33. The time period of a geostationary
satellite is 24 h, at a height 6RE (RE is
radius of earth) from surface of earth.
The time period of another satellite
whose height is 2.5 RE from surface
1) 0.6 V 2) 0 V will be,
3) 0.5 V 4) 0.4 V 1) 6 2 h 2) 12 2 h
30. The metre bridge shown is in balanced
24 12
P l1 3) h 4) h
position with Q  l . If we now 2.5 2.5
2
34. A circuit when connected to an AC
interchange the position of
source of 12V gives a current of 0.2 A.
galvanometer and cell, will the bridge
The same circuit when connected to a
work? If yes what will be balance
DC source of 12 V, gives a current of
condition?
0.4 A. The circuit is
1) series LR 2) series RC
3) series LC 4) series LCR
35. A cycle wheel of radius o.5 m is rotated
with constant angular velocity of 10
rad/s in a region of magnetic field of
0.1 T which is perpendicular to the
plane of the wheel. The EMF generated
between its centre and the rim is,
1) 0.25 V 2) 0.125 V
P l2 - l1
1) yes, Q  l  l 2) no, no null point 3) 0.5 V 4) zero
2 1
36. For a transparent medium relative
P l2 P l1 permeability and permittivity, r and  r
3) yes, Q  l 4) yes, Q  l are 1.0 and 1.44 respectively The
1 2
velocity of light in this medium would
31. The relation amongest the three be:
elements earth’s magne tic fiel d, 1) 2.5 × 108 m/s 2) 3 × 108 m/s
namely horizontal component H,
3) 2.08 × 108 m/s 4) 4.32 × 108 m/s
vertical component V and dip  are,
(B E = total magnetic field) 37. A sphere encloses an electric dipole
with charge  3 × 10–6 C. What is the
1) V = BE tan  , H = BE total electric flux across the sphere?
2) V = BE sin  , H = BE cos  1) –3 × 10–6 Nm2/C
3) V = BE cos  , H = BE sin  2) zero
3) 3 × 10–6 Nm2/C
4) V = BE , H = BE tan 
4) 6 × 10–6 Nm2/C
182
Previous Years NEET Questions & Solutions

38. Two identical capactitors C1 and C2 of 42. 1g of wat er of volu m e 1 cm 3 at 100o C,


equal capacitance are connected as is converted i nto ste am at same
shown in the circuit. Terminals a and temperature under normal
b of the key k are connected to charge atmospheric pressure (~ 1 × 105 Pa).
capacitor C1 using battery of emf V volt. The volume of steam formed equals
Now disconnecting a and b the 1671 cm3. If the specific latent heat of
terminals b and c are connected. Due vaporisation of water is 2256 J/g, then
to this, what will be the percentage the change in internal energy is,
loss of energy? 1) 2423 J 2) 2089 J
3) 167 J 4) 2256 J
43. Angular width of the central maxima
in the Fraunhofer diffraction for
o
  6000 A is 0. When the same slit is
illuminated by another monochromatic
light, the angular width decreases by
30%. The wavelength of this light is,
1) 75% 2) 0% 3) 50% 4) 25% o o
39. The main scale of a vermier calliper 1) 1800 A 2) 4200 A
has n divisions/cm. n divisions of the o o
vermier scale coincide with (n – 1) 3) 6000A 4) 420A
divisions of main scale. The least count 44. The work function of a photosensitive
of the vermier calliper is, material is 4.0 e V. The longe st
1 wavelength of light that can cause
cm 1
1) 2) cm photon emission from the substance is
   n 1
n  1 n (approximately)
1 1 1) 3100 nm 2) 966 nm
3) 2
cm 4) n  n  1 cm 3) 31 nm 4) 310 nm
n
45. A proton and an -particle are
40. A person travelling in a straight line
accelerated from rest to the same
moves with a constant velocity V1 for
certain distance ‘x’ and with a constant energy. The de Broglie wavelength  p
velocity v2 for next equal distance. The and   are in the ratio.
average velocity v is given by the
relation. 1) 2 : 1 2) 1 : 1

1 1 1 2 1 1 3) 2 : 1 4) 4 : 1
1) v  v  v 2) v  v  v 46. The reaction that does not give benzoic
1 2 1 2
acid as the major product is:-
v v1  v 2
3)  4) v  v1v 2
2 2 1)
41. Assumi ng that the gravitational
potential energy of an object at infinity
is zero, the change in potential energy
(final – initial) of an object of mass m, 2)
when taken to a height h from the
surface of earth (of radius R), is given
by,
3)
GMm GMmh
1)  2) R  R  h 
Rh
GMm 4)
3) mgh 4)
Rh
183
Brilliant STUDY CENTRE

47. The amine that reacts will Hinsberg’s 52. The hydrolysis reaction that takes
reagent to give an alkali insoluble place at the slowest rate, among the
product is:- following is:-
1) CH3–CH–NH–CH–CH3
| | 1)
CH3 CH3
CH2CH3
| 2) H3C–CH2–Cl 
aq.NaOH
 H3C–CH2–OH
2) CH3–CH2–N-CH2CH3 3)H2C=CH–CH2Cl 
aq.NaOH
 H 2C
NH2
=CH–CH2OH
|
3) CH3–C–CH2CH2CH3
4)
|
CH3 53. When vapours of a secondary alcohol
CH3 is passed over heated copper at 573K,
the product formed is:
|
1) a carboxylic acid
4) CH3– C – CH–NH2
2) an aldehyde
| |
3) a ketone 4) an alkene
CH3 CH3 54. The major product C and D formed in
48. Which structure(s) of proteins the following reactions respectively
remains(s) intact during denaturation are:
process?
H3C–CH2–CH2–O–C(CH3)3 
excess Hl

C + D
1) Both secondary and tertiary
structures 1) H3C–CH2–CH2–I and I–C(CH3)3
2) Primary structure only 2) H3C–CH2–CH2–OH and I–C(CH3)3
3) Secondary structure only 3) H3C–CH2 – CH2 - I and HO – C (CH3)3
4) Tertiary structure only 4) H3C–CH2–CH2–OH and HO–C(CH3)3
49. The polymer that is use d as a 55. Match the oxide given in column A with
substitute for wool in making its property given in column B:
commercial fibres is:-
Column A Column B
1) Melamine 2) nylon-6, 6
(i) Na2O (a) Neutral
3) polyacrylonitrile 4) Buna-N
(ii) Al2O3 (b) Basic
50. The artificial sweetner stable at
cooking temperature and does not (iii) N2O (c) Acidic
provide calories is:- (iv) Cl2O7 (d) Amphoteric
1) Saccharin 2) Aspartame Which of the following options has all
3) Sucralose 4) Alitame correct pairs.
51. The liquified gas that is used in dry 1) (i)-(b), (ii)-(a), (iii)-(d), (iv)-(c)
cleaning along with a suitable 2) (i)-(c), (ii)-(b), (iii)-(a), (iv)-(d)
detergent is:-
3) (i)-(a), (ii)-(d), (iii)-(b), (iv)-(c)
1) Water gas 2) Petroleum gas
3) NO2 4) CO2 4) (i)-(b), (ii)-(d), (iii)-(a), (iv)-(c)

184
Previous Years NEET Questions & Solutions

56. Match the catalyst with the process:- 60. Aluminium chloride in aci dified
aqueous solution forms a complex ‘A’,
Catalyst Process
in which hybridisation state of Al is
(i) V2O5 (a) The oxidation of ‘B’. What are ‘A’ and ‘B’, respectively.
ethyne to ethanal 1) [Al(H 2O)6]3+, sp3d2 2) [Al(H2O)4]3+sp3
(ii)TiCl4+Al(CH3) 3 (b) Polymerisation 3) [Al(H2O)4]3+dsp2 4) [Al(H2O)6]3+d2sp2
of alkynes 61. Which of the follwing compounds is
used on cosmetic surgery?
(iii) PdCl2 (c) Oxidation of SO2
1) Silica 2) Silicates
in the manufacture
3) Silicones 4) Zeolites
of H2SO4
62. Identify the incorrect statement.
(iv) Nickel complexes (d) Polymerisation
of ethylene 1) The scientific and technological
process used for Isolation of the metal
Which of the following is the correct from its ore is known as metallurgy.
option?
2) Minerals are naturally occurring
1) (i)-(c), (ii)-(d), (iii)-(a), (iv)-(b) chemical substances in the earth’s
2) (i)-(a), (ii)-(b), (iii)-(c), (iv)-(d) crust

3) (i)-(a), (ii)-(c), (iii)-(b), (iv)-(d) 3) Ores are minerals that may contain
a metal
4) (i)-(c), (ii)-(a), (iii)-(d), (iv)-(b)
4) Gangue is an ore contaminated with
57. The most stable carbocation, among the undesired materials.
following is:-
63. A compound ‘X’ upon reaction with H2O
 produces a colorless gas ‘Y’ with rotten
1)  CH 3 3 C  C H  CH 3 fish smell. Gas ‘Y’ is absorbed in a
solution of CuSO4 to give Cu3P2 as one

2) CH 3  CH 2  C H  CH 2  CH 3 of the products. Predict the compound
‘X’.

3) CH 3  C H  CH 2  CH 2  CH 3 1) Ca3P2 2) NH4Cl
3) As2O3 4) Ca3(PO4)2

4) CH 3  CH 2  C H 2 64. Which of the following oxoacids of
phosphorus has strongest reducing
58. The alkane that gives only one mono- property?
chloro product on chlorination with Cl2
in presence of diffused sunlight is:- 1) H4P2O7 2) H3PO3

1) 2,2-dimethylbutane 3) H3PO2 4) H3PO4


65. Identify the correct formula of oleum
2) neopentane
from he following
3) n-pentane 4) Isopentane
1) H2S2O7 2) H2SO3
59. In the following reaction, 3) H2SO4 5) H2S2O8
red hot 66. When neutral or faintly alkaline
iron tube KMnO 4 is treated with potassium
CH3 C CH A iodide, iodide ion is converted into ‘X’.
873 K ‘X’ is:-
the number of sigma() bonds present
1) I2 2) IO4
in the product A is:-
1) 21 2) 9 3) 24 4) 18 3) IO3 4) IO-
185
Brilliant STUDY CENTRE

67. The Crystal Field Stabilisation Energy 73. Which of the following is the correct
(CFSE) for [CoCl 6]4- is 18000 cm-1. The order of dipole moment?
CFSE for [CoCl4]2- will be- 1) NH3 < BF3 < NF3 < H2O
1) 6000 cm–1 2) 16000 cm-1 2) BF3 < NF3 < NH3 < H2O
3) 18000 cm-1 4) 8000 cm-1 3) BF3 < NH3 < NF3 < H2O
68. Following limiting molar conductivities 4) H2O < NF3 < NH3 < BF3
are given as 74. Crude sodium chloride obtained by
1 crystallisation of brine solution does not
 o
m H 2SO4 
 xScm mol
2
contain
 om K2SO4   yScm 2mol1 1) MgSO4 2) Na2SO4
3) MgCl2 4) CaSO4
 om CH3COOK   zScm 2 mol1 75. Which of the alkali metal chloride (MCl)
forms its dihydrate salt (MCI.2H 2O)
 0m  in S cm 2 mol1  for CH3COOH will be easily?
1) LiCl 2) CsCI
1) x – y + 2 z 2) x + y – z 3) RbCl 4) KCl
 x  y  z 76. The pH of 0.01 M NaOH (aq) solution
3) x – y + z 4) will be
2
1) 7.01 2) 2
69. A first order reaction has a rate
3) 12 4) 9
constant of 2.303 × 10–3s-1. The time
required for 40g of this reactant to 77. Which of the following cannot act both
reduce to 10 g will be- as Bronsted acid and as Bronsted base?
[Given that log10 2 = 0.3010] 1) HCO3 2) NH3
1) 230.3 s 2) 301 s 3) HCl 4) HSO4
3) 2000 s 4) 602 s 78. The molar solubility of CaF2 (Ksp = 5.3
70. For a reaction, activation energy × 10–11) in 0.1 M solution of NaF will be
Ea = 0 and the rate constant at 200 K 1) 5.3 × 10–11 mol L–1
is 1.6 × 106s–1. The rate constant at 2) 5.3 × 10–8 mol L–1
400 K will be-
3) 5.3 × 10–9 mol L–1
[Given that gas constant] 4) 5.3 × 10–10 mol L–1
R = 8.314 J K–1 mol–1 79. The oxidation state of Cr in CrO6 is:
1) 3.2 × 104s–1 2) 1.6 × 106s–1 1) –6 2) +12
3) 1.6 × 103s–1 4) 3.2 × 106s–1 3) +6 4) +4
71. The correct option representing a 80. The number of hydrogen bonded water
Freundich adsorption isothem is molecule(s) associate d with
CuSO4.5H2O is :-
x x 1) 3 2) 1
1)  KP0.3 2) KP2.5
m m 3) 2 4) 5
x x 1
81. Formula of nickel oxide with metal
3)KP0.5 4) KP defici ency defect in its crystal is
m m
Ni0.98O. The crystal contains Ni2+ and
72. Which of the following is Ni3+ ions the fraction of Nickel existing
paramagnetic? as Ni2+ ions in the crystal is
1) N2 2) H2 1) 0.96 2) 0.04
3) Li2 4) O2 3) 0.50 4) 0.31
186
Previous Years NEET Questions & Solutions

82. Which of the following statements is 88. The volume occupied by 1.8 g of water
correct regarding a solution of two vapour at 374oC and 1 bar pressure will
compounds A and B exhibiting positive be:
deviation from ideal behaviour?
[Use R = 0.083 bar L K–1 mol–1)
1) Intermolecular attractive forces
between A-A and B-B are stronger than 1) 96.66 L 2) 55.87 L
those between A-B. 3) 3.10 L 4) 5.37 L
2)  mix H = 0 at constant T and P 89. An ideal gas expands isothermally from
10-3 m3 to 10–2 m3 at 300 K against a
3)  mix V = 0 at constant T and P constant pressure of 10 5 Nm -2 . The
4) Intermolecular attractive forces work done on the gas is:
between A-A and B-B are equal to 1) + 270 kJ 2) –900 J
those between A-B.
3) + 900 kJ 4) –900 kJ
83. In water saturated air the mole fraction
of water vapour is 0.02. If the total 90. Reversibe expansion of an ideal gas
pressure of the saturated air is 1.2 under isothermal and adiabatic
atm, the partial pressure of dry air is: conditions are as shown in the figure.
1) 1.18 atm 2) 1.76 atm
3) 1.176 atm 4) 0.98 atm
84. The standard electrode potential (E–)
values of Al3+ / Al, Ag+/Ag, K+/K and
Cr3+/Cr are –1.66 V, 0.80V, –2.93 V
and -0.74 V, respectively. The correct
decreasing order of reducing power of
the metal is:
1) Ag > Cr > Al > K
2) K > Al > Cr > Ag AB  Isothermal expansion
3 K > Al > Ag > Cr AC  Adiabatic expansion
4) Al > K > Ag > Cr Which of the following options is not
85. The density of 2 M aqueous solution of correct?
NaOH is 1.28 g/cm3. The molality of
the solution is [Given that molecular 1) Sisothermal  Sadiabatic 2) TA = TB
mass of NaOH = 40 g mol–1] 3) Wisothermal > Wadiabatic 4) TC > TA
1) 1.20 m 2) 1.56 m 91. Which of the following cell organelles
3) 1.67 m 4) 1.32 m is present in the highest number in
86. Orbital having 3 angular nodes and 3 secretory cells?
total nodes is: 1) Mitochondria 2) Golgi complex
1) 5 p 2) 3 d 3) Endoplasmic reticulum
3) 4 f 5) 6 d
4) Lysosomes
87. In hydrogen atom, the de Broglie
wavelength of an e lectron in the 92. Non-me mbranous nucleoplasmic
second Bohr orbit is: structures in nucleus are the site for
active synthesis of :
[Given that Bohr Radius, a0 = 52.9 pm]
1) Protein synthesis 2) mRNA
1) 211.6 pm 2) 211.6  pm
3) 52.9  pm 4) 105.8 pm 3) rRNA 4) tRNA
187
Brilliant STUDY CENTRE

93. Which of the following nucleic acids is 97. Match the items given in column I with
present in an organism having 70S those in column II and choose the
ribosomes only? correct option.
1) Single standard DNA with protein Column I Column II
coat (a) Rennin (i) Vitamin B12
2) Double standard circular naked DNA (ii) Facilitated
(b) Enterokinase
3) Double standard DNA enclosed in transport
nuclear membrane
(c) Oxyntic cells (iii) Milk proteins
4) Double standard circular DNA with
histone proteins (d) Fructose (iv) Trypsinogen
94. After meiosis-I, the resultant daughter
cells have: 1) a-iii, b-iv, c-ii, d-i
2) a-iv, b-iii, c-i, d-ii
1) Same amount of DNA as in the
3) a-iv, b-iii, c-ii, d-i
parent cell in S phase.
4) a-iii, b-iv, c-i, d-ii
2) Twice the amount of DNA in 98. Kwashiorkor disease is due to:
comparison to haploid gamete.
1) Simultaneous deficiency of proteins
3) Same amount of DNA in comparison and fats
to haploid gamete. 2) Simultaneous deficiency of proteins
and calories
4) Four times the amount of DNA in
comparison to haploid gamete. 3) Deficiency of carbohydrate
4) Protein deficiency not accompained
95. Which of the following organic by calorie deficiency
compounds is the main constituent of
Lecithin? 99. Select the correct sequence of events:
1) Gametogenesis  Gamete transfer
1) Arachidonic acid 2) Phospholipid  Syngamy  Zygote  Ce ll
division (Cle avage)  Ce ll
3) Cholesterol 4) Phosphoprotein
differentiation  Organogenesis
96. The main difference between active and 2) Gametogenesis  Gamete transfer
passive transport across cell membrane  Syngamy  Zygote  Ce ll
is: division (Cleavage)  Organogenesis
 Cell differentiation
1) Passive transport is non-selective
whereas active transport is selective 3) Gametogenesis  Syngamy 
Gamete transfer  Zygote  Cell
2)Passive transport requires a division (Cle avage)  Ce ll
concentration gradient across a differentiation  Organogenesis
biological membrane whereas active 4) Gametogenesis  Gamete transfer
transport requires energy to move
 Syngamy  Zy gote  Ce ll
solutes.
diffe rentiation  Cell division
3)Passive transport is confined to (Cleavage)  Organogenesis
anionic carrier proteins whereas 100. Which of the following hormones is
active tansport is confined to cationic responsible for both the milk ejection
channel proteins. reflex and the foetal ejection reflex?
4) Active transport occurs more rapidly 1) Estrogen 2) Prolactin
than passive transport 3) Oxytocin 4) Relaxin
188
Previous Years NEET Questions & Solutions

101. No new follicles develop in the luteal 105. Select the incorrect stateme nt
phase of the menstrual cycle because regarding inbreeding
1) Follicles do not remain in the ovary 1) Inbreeding helps in elimination of
after ovulation deleterious alleles from the population.
2) FSH levels are high in the luteal
phase 2) Inbreeding is necessary to evolve a
3) LH levels are high in the luteal pureline in any animal
phase 3) Continu ed inbreeding reduces
4) Both FSH and LH levels are low in fertility and leads to inbreeding
the luteal phase depression
102. In Australia, marsupials and placental
mammals have evolved to share many 4) Inbreeding depression can not be
similar characteristics. This type of overcome by out-crossing
evolution may be referred to as: 106. A biocontrol agent to be a part of an
1) Adaptive Radiation integrated pest management should be
2) Divergent Evolution
1) Species-specific and symbiotic
3) Cyclical Evolution
4) Convergent Evolution 2) Free living and broad spectrum
103. Match the items of column I with 3) Narrow spectrum and symbiotic
column II.
4) Species-specific and inactive on non-
(a) XX-XO method (i) Turner’s target organisms
of sex determination syndrome 107. Match the following enzymes with their
functions:
(b) XX-XY method of (ii) Female
sex determination heterogametic Column I Column II
(a)Restriction
(iii) (i) Joins the DNA fragme nts
(c) Karyotype-45 endonuclease
Grasshopper (b) Restriction (ii)Exte nds primers
(d) ZW-ZZ method of (iv) Female exonuclease on genomic DNA te mplate
sex determination homogametic (iii) Cuts DNA at specific
(c) DNA ligase
position
Select the correct option from the (d) Taq (iv) Remove s nucleotide
following : polymerase from the e nds of DNA
1) a-ii, b-iv, c-i, d-iii
2) a-i, b-iv, c-ii, d-iii Select the correct option from the
3) a-iii, b-iv, c-i, d-ii following:
4) a-iv, b-ii, c-i, d-iii
1) a-iii, b-i, c-iv, d-ii
104. What will be the sequence of mRNA
produced by the following stretch of 2) a-iii, b-iv, c-i, d-ii
DNA? 3) a-iv, b-iii, c-i, d-ii
3 ATGCATGCATGCATGS 5 TEMPLATE 4) a-ii, b-iv, c-i, d-iii
STRAND
108. The two antibiotic resistance genes on
5 TACGTACGTACGTAC 3 CODING vector pBR322 are:
STRAND
1) Ampicillin and Tetracycline
1) 3 AUGCAUGCAUGCAUG 5
2) Ampicillin and Chloramphenicol
2) 5 UACGUACGUACGUAC 3
3) Chloramphenicol and Tetracycline
3) 3 UACGUACGUACGUAC 5
4) 5 AUGCAUGCAUGCAUG 3 4) Tetracycline and Kanamycin

189
Brilliant STUDY CENTRE

109. Exploitation of bioresources of a nation 114. A population of species invades a new


by multinational companies without area. Which of the following condition
authorization from the concerned will lead to Adaptive Radiation?
country is referred to as:
1) Area with large number of habitats
1) Bioweapon 2) Biopiracy having very low food supply
3) Bioethics 4) Biowar 2) Area with a single type of vacant
110. Carnivorous animals - lions and habitat
leopards, occupy the same niche but 3) Area with many types of vacant
lions predate mostly larger animals habitats
and leopards take smaller ones. This
4) Area with many habitats occupied
mechanism of competition is referred
by a large number of species
to as:
115. Identify A, B and C in the diagramatic
1) Character displacement
representation of the mechanism of
2) Altruism hormone action.
3) Resource partitioning
4) Competitive exclusion
111. Decline in the population of Indian
native fishes due to introduction of
Clarias gariepinus in river Yamuna can
be categorised as:
1) Co-extinction
2) Habitat fragmentation
Select the correct option from the
3) Over exploitation following:
4) Alien species invasion 1) A-Steroid Hormone; B-Hormone
112. Match the follwing RNA polymerase receptor Complex, C-Protein
with their transcribed products: 2) A-Protein Hormone; B-Receptor;
(a) RNA polymerase I (i) tRNA C Cyclic AMP
(b) RNA polymerase II (ii) rRNA 3) A-steroid Hormone; B-Receptor;
(c) RNA polymerase III (iii) hnRNA C-Second Messenger
4) A-Protein Hormone; B-Cyclic AMP;
Select the correct option from the C-Hormone-receptor Complex
following:
116. Human have acquired immune system
1) a-i, b-iii, c-ii 2) a-i, b-ii, c-iii that produces antibodies to neutralise
3) a-ii, b-iii, c-i 4) a-ii, b-ii, c-i pathogens. Still innate immune system
is present at the time of birth because
113. In a marriage between male with blood it
group A and female with blood group
B, the progeny had either blood group 1) is very specific and uses different
AB or B. What could be the possible macrophages
genotype of parents? 2) produces memory cells for mounting
1) IA i (Male) : IBIB(Female) fast secondary response
2) IAIA (Male) : IBIB (Female) 3) has natural killer cells which can
phagocytose and destroy microbes.
3) IAIA (Male) : IBi (Female)
4) provides passive immunity
4) IAi (Male) : IBi (Female)
190
Previous Years NEET Questions & Solutions

117. Which of the following statements is 122. Which of the following is a correct
not correct? statement?
1) An action potential in an axon does
1) IUDs once inserted need not be
not move backward because the
replaced.
segment behind is in a refractory
phase. 2) IUDs are generally inserted by the
2) Depolarisation of hair cells of user herself
cochlea results in the opening of the
3) IUDs increase phagocytosis of
mechanically gated potassium-ion
channels. sperms in the uterus.
3) Rods are very sensitive and 4) IUDs suppress gametogenesis
contribute to daylight vision
123. Which of the follwing sexually
4) In the knee-jerk reflex, stimulus is
transmitted diseases do not specifically
the stretching of muscle and response
affect reproductive organs?
is its contraction.
118. Match the following joints with the 1) Genital warts and Hepatitis - B
bones involved.
2) Syphilis and Genital herpes
Column I Column II 3) AIDS and Hepatitis B
(a) Gliding (i) Between carpal and 4) Chlamydiasis and AIDS
joint metacarpal of thumb
(b) Hinge 124. Match the following genera with their
(ii) Between Atlas and Axis respective phylum:
joint
(c) Pivot (a) Ophura (i) Mollusca
(iii) Between the carpals
joint
(b) Physalia (ii) Platyhelminthes
(d) Saddle (iv)Between
joint Humerus and Ulna (c) Pinctada (iii) Echinodermata
(d) Planaria (iv) Coelenterata
1) (a)-(iii), (b)-(iv), (c)-(ii), (d) - (i)
2) (a)-(iv), (b)-(i), (c)-(ii), (d) - (iii) Select the correct option
3) (a)-(iv), (b)-(ii), (c)-(iii), (d) - (i) 1) (a) - (iv), (b) - (i), (c) - (iii), (d) - (ii)
4) (a)-(i), (b)-(iii), (c)-(ii), (d) - (iv)
2) (a) - (iii), (b) - (iv), (c) - (i), (d) - (ii)
119. Which of the following diseases is an
autoimmune disorder? 3) (a) - (i), (b) - (iii), (c) - (iv), (d) - (ii)
1) Myasthenia gravis 2) Arthritis 4) (a) - (iii), (b) - (iv), (c) - (ii), (d) - (i)
3) Osteporosis 4) Gout
120. Artifical light, extended work-time and 125. Which of the following animals are true
reduced sleep-time disrupt the activity coelomates with bilateral symmetry?
of 1) Adult Echinoderms
1) Thymus gland 2) Pineal gland
2) Aschelminthes
3) Adrenal gland
4) Posterior pituitary gland 3) Platyhelminthes 4) Annelids
121. Which of the following condition will 126. The contrasting characteristics
stimulate parathyroid gland to release generally in a pair used for
parathyroid hormone? identification of animals in Taxonomic
1) Fall in active Vitamin D levels Key are referred to as:
2) Fall in blood Ca+2 levels 1) Lead 2) Couplet
3) Fall in bone Ca+2 levels
4) Rise in blood Ca+2 levels 3) Doublet 4) Alternate
191
Brilliant STUDY CENTRE

127. Match the following cell structure with 130. The maximum volume of air a person
its characteristic feature: breathe in after a forced expiration is
known as:
C o lu m n I C o lu m n II 1) Expiratory Capacity
( a ) Ti g h t ( i ) C e m e n t n e igh b o u rin g ce l l s 2) Vital Capacity
ju n cti o n s to ge th e r to f o rm s h e e t 3) Inspiratory Capacity
( b ) A d h e ri n g ( i i )Tran s m it in f o rm ati o n th ro u g h 4) Total lung capacity
ju n cti o n s ch e m i ca l to an o th e r ce l ls 131. All the components of the nodal tissue
( i i i) Esta b l is h a b arrie r to p re v e n t are autoexcitable. Why does the SA
( c) G a p
ju n ctio n s
l e ak ag e o f flu i d acro s s e p ith e lia l node act as the normal pacemakar?
ce ll s 1) SA node has the lowest rate of
( i v ) C y to p l asm i c ch an n e ls to depolarisation.
( d ) S y n a p tic
f a ci li tate co m m u n ica tio n 2) SA node is the only component to
ju n ctio n s
b e tw e e n ad jace n t ce l ls generate the threshold potential.
3) Only SA node can convey the action
potential to the other components.
Select correct option from the following 4) SA node has the highest rate of
1) (a) - (ii), (b) - (iv), (c) - (i), (d) - (iii) depolarisation.
132.A specialised nodal tissue embedded in
2) (a) - (iv), (b) - (ii), (c) - (i), (d) - (iii) the lower corner of the right atrium,
close to Atrio-ventricular septum,
3) (a) - (iii), (b) - (i), (c) - (iv), (d) - (ii) delays the spreading of impulses to
4) (a) - (iv), (b) - (iii), (c) - (i), (d) - (ii) heart apex for about 0.1 sec. The delay
allows.
128. Which of the following statements in 1) blood to enter aorta
INCORRECT? 2) the ventricles to empty completely
3) blood to enter pulmonary arteries
1) Cockroaches exhibit mosaic vision 4) the atria to empty completely
with less sensitivity and more 133. Match the following parts of a nephron
resolution. with their function.
2) A mushroom - shaped gland is
present in the 6 th -7 th abdominal Column I Column II
segments of male cockroach. (a) Descending (i) Reabsorption of salts
limb Henle's tubule at Henle’s loop only
3) A pair of spermatheca is present in
the 6th segment of female cockroach. (b) Proximal (ii) Reabsorption of water
convoluted tubule only
4) Female cockroach possesses sixteen
ovarioles in the ovaries. (iii) Conditional
(c) Ascending limb of
reabsorption of sodium
129. Select the correct statement. enle’s loop
ions and water
1) Expiration occurs due to external (d) Distal convoluted Reabsorption of ion,
intercoastal muscles tubules water and organic
2) Intrapulmonary pressure is lower nutrients
than the atmospheric pressure during
inspiration Select the correct option from the
following.
3) Inspiration occurs when atmospheric
pressure is less than intrapulmonary 1) (a) - (i), (b) - (iii), (c) - (ii), (d) - (iv)
pressure. 2) (a) - (ii), (b) - (iv), (c) - (i), (d) - (iii)
4) Expiration is initiated due to 3) (a) - (i), (b) - (iv), (c) - (ii), (d) - (iii)
contraction of diaphragm. 4) (a) - (iv), (b) - (i), (c) - (iii), (d) - (ii)
192
Previous Years NEET Questions & Solutions

134. Match the items in Column I with 139. Match the organisms in column-I with
those in Column II. habitats in column-II.

Column I Column II Column I Column II


(a) Podocytes (i) Crystallised oxalates (a) Halophile s (i) Hot springs
(b) Protonephridia (ii) Annelids
(ii) Aquatic
(c) Nephridia (iii) Amphioxus (b) The rmoacidophile s
e nvironme nt
(d) Renal calculi (iv) Filtration slits (iii) Guts of
(c) Me thanoge ns
ruminants
Select the correct option from the
following: (d) Cyanobacte ria (iv) Salty are a
1) (a) - (iii), (b) - (iv), (c) - (ii), (d) - (i)
2) (a) - (iii), (b) - (ii), (c) - (iv), (d) - (i) Select the correct answer from the
3) (a) - (iv), (b) - (iii), (c) - (ii), (d) - (i) option given below:
4) (a) - (iv), (b) - (ii), (c) - (iii), (d) - (i) 1) (a) - (iv), (b) - (i), (c) - (iii), (d) - (ii)
135. Which of the following receptors are
specifical ly responsible for 2) (a) - (i), (b) - (ii), (c) - (iii), (d) - (iv)
maintenance of balance of body and 3) (a) - (iii), (b) - (iv), (c) - (ii), (d) - (i)
posture/ 4) (a) - (ii), (b) - (iv), (c) - (iii), (d) - (i)
1) Basilar membrane and otoliths
140. In the dicot root the vascular cambium
2) Hair cells and organ of corti
originates from:
3) Tectorial membrane and macula
4) Crista ampullaris and macula 1) Tissue located below the pholem
136. Which of the following is against the bundles and a portion of pericycle
rules of ICBN? tissue above protoxylem.
1) Hand written scientific names 2) Cortical region
should be underlined. 3) Parenchyma between endodermis
2) Every species should have a generic and pericycle.
name and a specific epithet.
4) Intrafascicular and interfascicular
3) Scientific names are in Latin and
tissue in a ring.
should be italized
4) Generic and specific names should 141. Which of the following shows whorled
be written starting with small letters. phyllotaxy?
137. Mad cow disease in cattle is caused 1) Mustard 2) China rose
by an organism which has:
3) Alstonia 4) Calotropis
1) inert crystalline structure
2) abnormally folded protein 142. Regeneration of damaged growing
grass following grazing is largely due
3) free RNA without protein coat
to:
4) free DNA without protein coat
138. Which of the following statement is 1) Lateral meristem
correct? 2) Apical meristem
1) Lichens do not grow in polluted 3) Intercalary meristem
areas.
2) Algal component of lichens is called 4) Secondary meristem
mycobiont. 143. Bicarpellary ovary with obliquely placed
3) Fungal component of lichens is septum is seen in:
called phycobiont. 1) Brassica 2) Aloe
4) Lichens are not good pollution
indicators. 3) Solanum 4) Sesbania
193
Brilliant STUDY CENTRE

144. Which is the most common type of 149. What initiation and termination factors
embryo sac in angiospems? are involved in transcription in
1) Tetrasporic with one mitiotic stage Eukaryotes?
of divisions 1)  and  , respectively
2) Monosporic with three sequential 2)  and   respectively
divisions
3)   and   respectively
3) Monosporic with two sequential
mitotic divisions. 4)  and   respectively
150. Which of the following statements is
4) Bisporic with two sequential mitotic
divisions. correct about the origin and evolution
of men?
145. From the following, identify the correct
1) Agriculture came around 50,000
combination of salient features of
years back.
Genetic Code:
2) The Dryopithecus and Ramapithecus
1)Universal, Non-ambiguous primates existing 15 million years ago,
overlapping walked like men.
2) Degenerate, overlapping, Commaless 3) Homo habilis probably ate meat.
4) Neanderthal men lived in Asia
3) Universal, Ambiguous, Degenerate
between 1,00,000 and 40,000 years
4) Degenerate, Non-overlapping, Non- back.
ambiguous
151. The production of gametes by the
146. Which scientist experimentally proved parents, the formation of zygotes, the
that DNA is the sole genetic material F 1 and F 2 plants can be understood
in bacteriophage? using:
1) Beadle and Tautum 1) Pie diagram 2) A pyramid diagram
2) Messelson and Stahl 3) Punnet square 4) Wenn diagram
3) Hershey and Chase 152. Match the column-I with column-II.

4) Jacob and Monod


Column I Column II
147. In the process of transcription in (a) Golgi (i) Synthe sis of
Eukaryotes, the RNA polymerase I
apparatus prote in
transcribes:
(ii) Trap waste and
1) mRNA with additional processing, (b) Lysosome s
capping and tailing. e xcre tory products
(iii) Formation of
2) tRNA, 5 S rRNA and snRNAs
(c) Vacuole s glycoprote ins
3) rRNAs.28 S, 18 S and 5.8 S and glycolipids
4) Precursor of mRNA, hnRNA (iv) Dige sting
(d) Ribosome s
148. In which genetic condition, each cell biomole cule s
in the affected person has three sex
chromosomes XXY? Choose the right match from the options
1) Thalassemia given below.
2) Kleinfelter’s Syndrome 1) (a) - (iii), (b) - (iv), (c) - (ii), (d) - (i)
2) (a) - (iv), (b) - (iii), (c) - (i), (d) - (ii)
3) Phenylketonuria
3) (a) - (iii), (b) - (ii), (c) - (iv), (d) - (i)
4) Turner’s syndrome
4) (a) - (i), (b) - (ii), (c) - (iv), (d) - (iii)
194
Previous Years NEET Questions & Solutions

153. Prosthetic groups differ from 159. Where is respiratory electron transport
co-enzymes in that: system (ETS) located in plants?
1) they require metal ions for their 1) Mitochondrial matrix
activity.
2) they (prosthetic groups) are tightly 2) Outer mitochondrial membrane
bound to apoenzymes 3) Inner mitochondrial membrane
3) their association with apoenzymes
is transient 4) Intermembrane space
4) they can serve as co-factors in a 160. In Hatch and Slack pathway, the
number of enzyme-catalyzed reactions. primary CO2 acceptor is:
154. Crossing over takes place between 1) Oxaloacetic acid
which chromatids and in which stage
of the cell cycle? 2) Phosphoglyceric acid
1) Non-sister chro matids of non-
3) Phosphoenol pyruvate
homologous chromosomes at zygotene
stage of prophase I. 4) RuBisCO
2) Non-sister chromatids of homologous
161. Removal of shoot tips is a very useful
chromosomes at Pachytene stage of
technique to boost the production of
prophase I.
tea-leaves. This is because:
3) Non-sister chromatids of homologous
chromosomes at zygotene stage of 1) Gibberellins prevent bolting and are
prophase I inactivated
4) Non-sister chro matids of non- 2) Auxins prevent leaf drop at early
homologous chromosomes at Pachytene stages.
stage of prophase I.
155. “Ramachandran plot” is used to confirm 3) Effect of auxins is removed and
the structure of: growth of lateral buds is enhanced.
1) RNA 2) Proteins 4) Gibberellins delay senescence of
3) Triacylglycerides 4) DNA leaves.
156. Which of the following is not a feature 162. One scientist cultured Cladophora in
of active transport of solutes in plants? a suspension of A zotobacter and
1) Occurs against concentration illiuminated the culture by splitting
gradient light through a prism. He observed that
2) Non-selective bacteria accumulated mainly in the
3) Occurs through membranes region of:
4) Requires ATP 1) Violet and green light
157. Which of the following bacteria reduce
nitrate in soil into nitrogen? 2) Indigo and green light
1) Nitrobacter 2) Nitrococcus 3) Orange and yellow light
3) Thiobacillus 4) Nitrosomonas
4) Blue and red light
158. What will be the direction of flow of
water when a plant cell is placed in a 163. In order to increase the yield of
hypotonic solution? sugarcane crop, which of the following
1) Water will flow in both directions plant growth regulators should be
sprayed?
2) Water will flow out of the cell
3) Water will flow into the cell 1) Ethylene 2) Auxins
4) No flow of water in any direction 3) Gibberellins 4) Cytokinins
195
Brilliant STUDY CENTRE

164. What type of pollination takes place in 168. Western Ghats have a large number
Vallisneria? of plant and animal species that are
not found anywhere else. Which of the
1) Pollination occurs in submerged
following terms will you use to notify
condition by water.
such species?
2) Flowers emerge above surface of 1) Endemic 2) Vulnerable
water, and pollination occurs by
insects. 3) Threatened 4) Keystone
3) Flowers emerge above water surface, 169. Which of the following statements
and pollen is carried by wind. about ozone is correct?
4) Male flowers are carried by water 1) Tropospheric ozone protects us from
currents to female flowers at surface UV radiations.
of water.
2) Stratospheric ozone is ‘bad’.
165. In which one of the following, both 3) Tropospheric ozone is ‘good’.
autogamy and geitonogamy are
prevented? 4) Stratospheric ozone protects us from
UV radiations.
1) Wheat 2) Papaya
170. Exploration of molecular, genetic and
3) Castor 4) Maize species level diversity for novel
166. Match the placental types (column-I) products of economic importance is
with their examples (column-II) known as:
1) Biopiracy 2) Bioenergetics
Column I Column II 3) Bioremediation 4) Bioprospecting
(a) Basal (i) Mustard
171. Which of the following is an innovative
(b) Axile (ii) China rose remedy for plastic waste?
(c) Parietal (iii) Dianthus
1) Burning in the absence of oxygen
(d) Free central (iv) Sunflower
2) Burrying 500 m deep below soil
Choose the correct answer from the surface
following options. 3) Polyblend
1) (a) - (ii), (b) - (iii), (c) - (iv), (d) - (i) 4) Electrostatic precipitator

2) (a) - (i), (b) - (ii), (c) - (iii), (d) - (iv) 172. Between which among th following, the
relationship is not an example of
3) (a) - (iv), (b) - (ii), (c) - (i), (d) - (iii)
commensalism?
4) (a) - (iii), (b) - (iv), (c) - (i), (d) - (ii)
1) Orchid and the tree on which it
167. A selectable marker is used to: grows
1) help in eliminating the non- 2) Cattle Egret and grazing cattle
transformants, so that the 3) Sea Anemone and Clown fish
transformants can be regenerated.
2) Identify the gene for a desired trait 4) Female wasp and fig species
in an alien organism. 173. If an agricultural field is liberally
3) select a suitable vector for irrigated for a prolonged period of time,
transformation in a specific crop. it is likely to face problem of:

4) mark a gene on a chromosome for 1) Metal toxicity 2) Alkalinity


isolation using restriction enzyme. 3) Acidity 4) Salinity
196
Previous Years NEET Questions & Solutions

174. Which of the following statements 178. Given below are four statements
about methanogens is not correct? pertai ning to separation of DNA
1) They can be used to produce biogas. fragments using gel electrophoresis.
Identify the incorrect statements.
2) They are found in the rumen of
cattle and their excreta (a) DNA is negatively charged molecule
and so it is loaded on gel towards
3) they grow aerobically and breakdown the Anode terminal.
cellulose- rich food.
(b) DNA fragments travel along the
4) They produce methane gas. surface of the gel whose concentration
175. In mung bean, resistance to yellow does not affect movemnet of DNA.
mosaic virus and powdery mildew were (c) Smaller the size of DNA fragment
brought about by: larger is the distance it travels
1) Mutation breeding through it.
2) Biofortification 3) Tissue culture (d) Pure DNA can be visualized directly
4) Hybridization and selection by exposing UV radiation.
176. Coca alkaloid or cocaine is obtained Choose correct answer from the options
from: given below.
1) Papaver somniferum 1) (a), (c) and (d) 2) (a), (b) and (c)
2) Atropha belladona 3) (b), (c) and (d) 4) (a), (b) and (d)
3) Erythroxylum coca 4) Datura 179. An enzyme catalysing the removal of
177. Among the following pairs of microbes, nucleotides from ends of DNA is:
which pair has both the microbes that 1) DNA ligase 2) Endonuclease
can be used as biofertilizers? 3) Exonuclease 4) Protease
1) Aspergillus and Rhizopus
180. In RNAi, the genes are silenced using:
2) Rhizobium and Rhizopus 1) ds-RNA 2) ss-DNA
3) Cyanobacteria and Rhizobium 3) ss-RNA 4)ds-DNA
4) Aspergillus and Cyanobacteria

197
Brilliant STUDY CENTRE

KEY WITH HINTS

T1  T2  T  T2 
6. 4  K 1  Ts 
t  2 
1. 4
80  70  80  70 
 K  25  .... (1)
Total charge = 12  2 
  4R 2    4  2R   20R 2
2
70  60  70  60 
 K  25  .... (2)
QA 1 t  2 

QB 2 on solving : t = 15 min

20 40
QA = R 2 and Q B  R 2 7. 4
3 3
Q 20 R 2 5
A   
A 3 4R 2 3 2r 2      g
vT 
40R 2 5 9
B   
4  2R 
2
6
 1   
2
v1  r1 
2. 4 Distance in one oscillation = 4A  
V2  r2  2   
2h 1
3. 4 Time of flight =  2
v1  1   82  0.12  79
g g = V   2     0.1   36
2    2 2 
Time period of pendulum
8.  = (2n), 0 = 0,  = V0/r
1 1
= 2 g  2  02  V0 / r   0
2
V2
g    02
2 2 2n  4nr
Ratio of time of flight and time pe-
riod of pendulum is independent
2h
of g. Hence t = 2 ‘ T1’ 9. 4 t= a rel
4. 3 Here  = 2(l2 – l1)
Where l1 = 9.75 cm, l2 = 31.25 cm In both cases arel = g – 0 = g.
so v = 2n(l2 – l1) = 2 × 800 Hence t1 = t2
(31.25 – 9.75) = 344 m/s
10. 2

5. 1

Conservation of linear momentum,


 m 5m  Fpseudo a
mv 0  v1  v2 tan    towards left
6 6 mg g


 m 20iˆ  25jˆ  12k  11.  oil hoil = water hwater
1000 15 
5m   oil   750kgm 3
=
m
6

100iˆ  35jˆ  8kˆ 
6

v2 20
[0  (  26 )] dx 26K
 12. 3 KA d t  A (d x )  L  
 v 2  4iˆ  23jˆ  16kˆ x dt Lx

198
Previous Years NEET Questions & Solutions

1240nm
13. 2   652.6nm  654nm
1.9
14. 1 A B Y 22. 4
0 0 1
0 1 0 
3mv  mviˆ  mvjˆ  0
1 0 0
 v  v   v
1 1 0  v  ˆi   ˆj  v  2
3  3  3
15. 1 Hydrogen  7 / 5(diatomic),
Energy released
Helium  5 /3 (monoatomic),
X  9/7
16. 4 Focal length do not change 
Power do not change 23. 3 W =  KE
17. 3 Path difference for nth minima =
1 1 2

At x = 8 : 130  v
2  2
 2n  1
2
 v  2 130  22.8 ms 1
9
For fifth minima (n = 5) = For x = 12 : v
2
= 20.6 ms–1
1   2  1   1 1 
18. 3    
f  1   R1 R 2  24. 4

1  1.5  1   1 1 
     
25  1   R  2R  

2 3 75
   R   18.75
25 2R 4

d 100
19. 1 t   2s
Vrel 50

1 1
sy   gt 2    10  4   20
2 2
 Height = 180 m
20. 2
21. 3 N = mg, F = f 25. 1
Resultant =

 mg 
2
N2  f 2   f 2  mg 1   2

199
Brilliant STUDY CENTRE

29. 4
26. 1

30 20 10 2
(V1 – V2) =     0.4 V
25 25 25 5
30. 4 Interchanging cell and galvanom-
eter do not effect balance condi-
tion.
 1l1  2 l 2
31. 2 V = BE sin 
H = BE cos 

B1 and B2 are in opposite direc-


tions, hence resultant field at cen-
tre is zero.
27. 2 Changing polarity is termed as AC.

28. 3 m   207me , q   q e , M nucleus  1836 me 32. 4 T1/2 = 2.2. × 109 s, R = N 

Reduced mass R R
N  T1/2
 0.693
mM 207me  1836 me
  1010  2.2  109
M  m 207m e  1836me =186 me  = 3.17 × 1019
0.693
n 2h 2 o 33. 1 Kepler’s Third Law
 r1   0.51A
42 mkze 2
(Given in question)
Radius of first orbit of new atom

me r1 me o
r1    0.51A
 186me

 2.56  1013 m

 186 me
E1'  E1   13.6eV  34. 1
m me

  2.8 keV

200
Previous Years NEET Questions & Solutions

41. 2 U =
35. 2
 1 1 GMmh
= –GMm  R  h  R   R R  h
v
1

1

3  108    
36. 1   r r 0 0 1.44
42. 2 U  W  mL  U  P(V2  V1 )

 1 2256   U  1  105 1670  106 


30
  108  2.5  108 ms 1
12
U = 2089 J
37. 2 
43. 2 Angular width 
d
1
38. 3 Uinitial = CV 2
2

1
= CV 2
4

44. 4

39. 3

45. 1

46. 3 1)

2)

3)
Total dis tance
40. 2 Average velocity 
Total time

4)

47. 1

v=

201
Brilliant STUDY CENTRE

48. 2 During denaturation 2 o and 3 0 63. 1 Ca3P2 + H2O  Ca(OH)2 + PH3


structures are destroyed but 1 o
(x) colourless
structure remains intact.
49. 3 Polyacrylonitrile is used as substi- gas (y)
tute for wool in making commer-
cial fibres as Orlon or Acrilan. CuSO4 + PH3  Cu3P2 + H2SO4
50. 3 Sucralose is stable at cooking tem- (Y)
perature and does not provide calo-
ries. hence X is Ca3P2
51. 4 (Calcium phosphide)
52. 1 Aryl halides don’t show NSR eas-
64. 3 Reducing property  no. of P-H
ily at room temperature.
bond
H 3 PO 2 contains maximum P-H
53. 3 bond.

54. 1

65. 1 Oleum  pyrosulphuric acid


(H2S2O7)
55. 4 Na2O  Basic 66. 3 KMnO4 + I– + OH  MnO2+ lO3 + H2)
Al2O3  Amphoteric
N2O  Neutral (x)
Cl2O7  Acidic 4
56. 1 67. 4  tetrahedral    octahedral
9
57. 3 Due to +H effect of alkyl group
(more  H) 4
  18000 = 8000 cm–1
58. 2 9
59. 1 68. 4 CH3COOH  CH3COO– + H+ ...(1)
H2SO4  2H+ + SO42 ... (2)

K2SO4  2K+ + SO42 ... (3)


CH3COOK  CH3COO– + K+ ...(4)
According to Kohlrausch’s law
 0CH3COOH   0CH COO   0H
3
60. 1 AlCl3 in acidified aqueous solution
form a (2) (3)
eq. (1) = eq. (4) + eq.  eq.
[Al(H 2O)63+] 2 2
Al+3 = [Ne]
x y
sp3d2  0CH3COOH  z  
61. 3 2 2
62. 4 Contaminated undesired material (x-y)
present in an ore is called gangue.  λ CH
0
3COOH
= +z(S × cm 2mol -1 )
2
202
Previous Years NEET Questions & Solutions

69. 4 For a first order reaction; 75. 1 Li + having maximum hydration ten-
0.693 dency.
t 1/2 = K 76. 3 NaOH(aq) is strong base solution

0.693 So, [OH–) = N = 10–2N


t1/2   301s
2.303  103 pOH = –log[OH–) = –log10-2 = 2
pH = 14 – pOH = 14 – 2
The time required for 40 g of reac-
tant to reduce to 10g pH = 12
t75% = 2 × t1/2 77. 3 HCl cannot act both as Bronsted
acid and Bronsted base because
t75% = 2 × 301 = 602 s HCl can only donate proton.
78. 3 CaF2(s)  Ca2+ (aq) + 2F– (aq)
70. 2 (a – s) s' 2s'
NaF(aq)  Na+(aq) + F–(aq)
Ea = 0
C 0 0
0 C C
In solution [F–) = (2 s ' + C)
[F–) = C (due to common ion effect)
K sp(CaF2 ) = s ' .C2

5.3 1011
s' 
(101 ) 2
s' = 5.3 × 10–9 mol L–1
71. 1 Freundlich adsorption isothem is
79. 3
x
 K(P)1/n 80. 2 In CuSO 4 .5H 2 O, only one water
m molecule take part in hydrogen
It it, value of 1/n lies in between 0 bonding
to 1. So, correct option is 81. 1 Let, Ni+2 in the crystal Ni0.98O be
x.
 Ni+3 in the crystal Ni0.98 will be
(0.98 – x)
72. 4 According to MOT
Ni x2 Ni (0.98
3
 x )O

By law of conservation of charge


73. 2 2x + 3 × (0.98 – x) – 2 = 0
x = 0.94
So, the fraction of Ni+2 ions in the
0.94
crystal = = 0.96
0.98
82. 1 If, intermolecular attractive force
74. 1 Crude NaCl obtained by between A - A and B - B are stron-
crystalisation of brine solution con- ger than those A - B then it show
tains Na 2 SO 4 , CaCl 2 , MgCl 2 & positive de viation from ide al
CaSO4 behaviour.
203
Brilliant STUDY CENTRE

X H2O  0.02 88. 4 pV = nRT


83. 3
n = 1.8/18 = 0.1 mole
 X gas  0.98 P = 1 bar
Ptotal = 1.2 atm T = 374 + 273 = 647K
partial pressure of dry-air = PT × nRT 0.1  0.083  647
mole fraction of dry-air V=  = 5.37 L
P 1
Partial pressure of dry-air = 1.2
atm × 0.98 = 1.176 atm. 89. 2 wpv = –P(Vf – VI)
= –105Nm–2(10–2 m3 – 10–3 m3)
1
84. 2 Reducing power of metal  = –105 Nm–2 × 10–3 [10 – 1]m3
SRP
K > Al > Cr > Ag. = –900 J
85. 3 2M solution of NaOH means 2 mole 90. 4 In adiabatic expansion cooling ef-
NaOH is present in 1L solution: fect will take place. TC will be less
than TA . In adiabatic expansion
density = v1.28 g/ml q=0
mass of solution = volume of
solution × density U  w
wpv < 0
= 1200 × 1.28 = 1280 g
mass of solvent = mass of solution U < 0
– mass of solute nCvm  T < 0
= 1280 – 80 = 1200 g T < 0
2 TC – TA < 0
molality = 1000
1200 TC < TA

20 10 5 91. 2 92. 2 93. 2


   1.67m 94. 2 95. 2 96. 2
12 6 3
86. 3 Orbital having angular node (l) = 3 97. 4 98. 4 99. 1
Total node = Radial node + angu- 100. 3 101. 4 102. 4
lar node 103. 3 104. 2 105. 4
=n–l–1+l 106. 4 107. 2 108. 1
3=n–1 109. 2 110. 3 111. 4
n=4 112. 3 113. 1 114. 3
Subshell “nl” = 4f 115. 2 116. 3 117. 3
87. 2   2 r 118. 1 119. 1 120. 2
n2 121. 2 122. 3 123. 3
n  2  a0
z 124. 2 125. 4 126. 2

n2 127. 3 128. 1 129. 2


n  2    52.9 pm
z 130. 2 131. 4 132. 4
133. 2 134. 3 135. 4
n  2  52.9  2pm
136. 4 137. 2 138. 1
= 211.6  pm

204
Previous Years NEET Questions & Solutions

139. 1 150. 4 151. 3 152. 1


140. 1 153. 2 154. 2 155. 2
In official answer key released by 156. 2 157. 3 158. 3
NTA, answer this question was
given (i) in reference Prokaryotes 159. 3 160. 3 161. 3
by defaults instead of Eukaryotes.
162. 4 163. 3 164. 4
141. 3 142. 3 143. 3
165. 2 166. 3 167. 1
144. 2 145. 4 146. 3
168. 1 169. 4 170. 4
147. 3 148. 2
171. 3 172. 4 173. 4
149. 1
According to the question, none of 174. 3 175. 1 176. 3
the options is correct. So question 177. 3 178. 4 179. 3
is borus. In reference to prokary-
ote option (i) will be correct. 180. 1

205
Brilliant STUDY CENTRE

NEET QUESTION PAPER - 2020 [PHASE - I]


BIOLOGY
5. In which of the following techniques,
1. Which of the following is not an the embryos are transferred to assist
attribute of a population? those females who cannot conceive?
1) Sex ratio 1) ZIFT and IUT
2) Natality
2) GIFT and ZIFT
3) Mortality
3) ICSI and ZIFT
4) Species interaction
4) GIFT and ICSI
2. The process of growth is maximum
during: 6. Identify the wrong statement with
reference to the gene ‘I’ that controls
1) Log phase ABO blood groups.
2) Lag phase 1) The gene(1) has three alleles
3) Senescence 2) A person will have only two of the
4) Dormancy three alleles

3. The roots that originate from the base 3) When IA and IB are present together,
of the stem are: they express same type of sugar

1) Fibrous roots 4) Allele ‘i’ does not produce any sugar

2) Primary roots 7. Choose the correct pair from the


following:
3) Prop roots
1) Ligases - Join the two DNA
4) Lateral roots molecules
4. Match the following diseases with the 2) Polymerases - Break the DNA into
causative organisms and select the fragments
correct option.
3) Nucleases - separate the two strands
Column - I Column - II of DNA
a) Typhoid i) Wuchereria
4)Exonucleases - Make cuts at specific
b) Pneumonia ii) Plasmodium positions within DNA
c) Filariasis iii) Salmonella 8. Select the correct match
d) Malaria iv) Haemophilus 1) Haemophilia - Y linked
a) b) c) d) 2) Phenylke tonu ria - Autosomal
dominant trait
1) i iii ii iv
2) iii iv i ii 3) Sickle cell anaemia - Autosomal
recessive trait, chromosome - 11
3) ii i iii iv
4) Thalassemia - X linked
4) iv i ii iii

206
Previous Years NEET Questions & Solutions

9. Match the following columns and 13. The product(s) of reaction catalyzed by
select the correct option nitroge nase in root nodules of
leguminous plants is/ are:
1) Ammonia alone
COLUMN - I COLUMN - II
2) Nitrate alone
a) Gregarious, polyphagous(i) pest i) Asterias 3) Ammonia and oxygen
Adult with radial symmetry and
b)
larva with bilateral symmetry
ii) Scorpion 4) Ammonia and hydrogen
c) Book lungs iii) Ctenoplana 14. Identify the correct statement with
regard to G1 phase (Gap 1) of interphase
d) Bioluminescence iv) Locusta
1) DNA synthesis or replication takes
place.
(a) (b) (c) (d)
2) Reorganisation of al l ce ll
1) i iii ii iv components takes place.
2) iv i ii iii 3) Cell is metabolically active, grows
3) iii ii i iv but does not replicate its DNA.

4) ii i iii iv 4) Nuclear Division takes place


10. The infectious stage Plasmodium that 15. Cuboidal epithelium with brush border
enters the human body is: of microvilli is found in:
1) Trophozoites 1) lining of intestine
2) Sporozoites 2) ducts of salivary glands
3) Female gametocytes 3) Proximal convoluted tubule of
4) Male gametocytes nephron

11. Ide ntify the substances having 4) eustachian tube


glycosidic bond and peptide bond, 16. Which of the following statements
respectively in their structure: about inclusion bodies is incorrect?
1) Chitin, cholesterol 1) The y are not bound by any
2) Glycerol, trypsin membrane.
3) Cellulose, lecithin 2) These are involved in ingestion of
food particles.
4) Inulin, insulin
3) They lie free in the cytoplasm
12. The plant parts which consist of two
generations one within the other 4) These represent reserve material in
cytoplasm.
a) Pollen grains inside the anther
17. Which is the important site of formation
b) Germinated pollen grain with two of glycoproteins and glycolipids in
male gametes eukaryotic cells?
c) Seed inside the fruit
1) Endoplasmic reticulum
d) Embryo sac inside the ovule
2) Peroxisomes
1) a only 2) a, b and c
3) Golgi bodies
3) c and d 4) a and d
4) Polysomes
207
Brilliant STUDY CENTRE

18. In gel electrophoresis, separated DNA 22. Identify the wrong statement with
fragments can be visualized with the regard to Restriction Enzymes.
help of:
1) Each restriction enzyme functions
1) Acetocarmine in bright blue light by inspecting the length of a DNA
sequence.
2) Ethidium bromide in UV radiation
2) They cut the strand of DNA at
3) Acetocarmine in UV radiation palindromic sites.
4) Ethidium bromide i n infrared 3) They are use ful in genetic
radiation. engineering.
19. Identify the wrong statement with 4) Sticky ends can be joined by using
reference to transport of oxygen. DNA ligases.

1) Binding of oxygen with haemoglobin 23. Which of the following is put into
is mainly related to partial pressure of Anaerobic sludge digester for further
O 2. sewage treatement?
1) Primary sludge
2) Partial pressure of CO2 can interfere
with O2 binding with haemoglobin 2) Floating debris
3) Effluents of primary treatment
3) Higher H+ conc. in alveoli favours
the formation of oxyhaemoglobin. 4) Activated sludge.

4) Low pCO 2 in alveoli favours the 24. Select the correct events that occur
formation of oxyhaemoglobin during inspiration.
a) Contraction of diaphragm
20. Ray florets have:
b) Contraction of external inter - costal
1) inferior ovary muscles
2) superior ovary c) Pulmonary volume decreases

3) Hypogynous ovary d) Intra pulmonary pressure increases


1) a and b 2) c and d
4) Half inferior ovary
3) a, b and d 4) only d
21. The specific plaindromic sequence
25. If the head of cockroach is removed, it
which is recognized by EcoRI is :
may live for few days because :
1) 5' GAATTC  3' 1) the supra- oesophageal ganglia of
3' CTTAAG  5' the cockroach are situated in ventral
part of abdomen.

2) 5' GGAACC  3' 2) the cockroach does not have nervous


3' CCTTGG  5' system.
3) the head holds a small proportion of
3) 5' CTTAAG  3' a nervous system while the rest is
3' GAATTC  5' situated along the ventral part of its
body.
4) 5' GGATCC  3' 4) the head holds a 1/3rd of a nervous
3' CCTAGG  5' system while the rest is situated along
the dorsal part of its body.

208
Previous Years NEET Questions & Solutions

26. Which of the following statements are 28. Match the following concerning
true for the phylum - Chordata? essential elements and their functions
in plants:
a) In Urochordata notochord extends
from head to tail and it is present a) Iron i) Photolysis of water
throughout their life. b) Zinc ii) Pollen germination
b) In Vertebrata notochord is present c) Boron iii) Required for
during the embryonic period only. chlorophyll
biosynthesis
c) Central nervous system is dorsal and d) Manganese iv) IAA biosynthesis
hollow.
Select the correct option.
d) Chordata is divided into 3 subphyla: a) b) c) d)
Hemichordata, Tunicata and
Cephalochordata. 1) ii i iv iii
2) iv iii ii i
27. Match the organism with its use in
biotechnology 3) iii iv ii i
4) iv i ii iii
a) Bacillus i) cloning vector
29. Identify the incorrect statement
thuringiensis
1) Heart wood does not conduct water
b) Thermus but gives mechanical support.
2) Sapwood is involved in conduction
aquaticus ii) Construction of of water and minerals from root to leaf.
first rDNA 3) Sapwood is the innermost secondary
moleucle xylem and is lighter in colour

c) Agrobacterium 4) Due to deposition of tannins, resins,


oils etc., heart wood is dark in colour.
tumefaciens iii)DNA polymerase 30. Match the following:
d) Salmonella a) Inhibitor of catalytic
activity i) Ricin
typhimurium iv) Cry proteins
b) Possess peptide bonds ii) Malonate
c) Cell wall material
Select the correct option from the in fungi iii) Chitin
following:
d) Secondary metabolite iv) Collagen
a) b) c) d) Choose the correct option from the
following
1) ii iv iii i
a) b) c) d)
2) iv iii i ii
1) ii iv iii i
3) iii ii iv i 2) iii i iv ii
4) iii iv i ii 3) iii iv i ii
4) ii iii i iv

209
Brilliant STUDY CENTRE

31. Meiotic division of the secondary oocyte 36. The transverse section of a plant shows
is completed: following anatomical features:
1) Prior to ovulation a) Large number of scattered vascular
bundles surrounded by bundle sheath.
2) At the time of copulation b) Large conspicuous parenchymatous
3) After zygote formation ground tissue.

4) At the time of fusion of a sperm with c) Vascular bundle s conjoint and


an ovum closed.

32. According to Robert May, the global d) Phloem parenchyma absent.


species diversity is about: Identify the category of plant and its
part:
1) 1.5 million
1) Monocotyledonous stem
2) 20 million
2) Monocotyledonous root
3) 50 million
3) Dicotyledonous stem
4) 7 million
4) Dicotyledonous root
33. The first phase of translation is:
37. Match the following columns and
1) Binding of mRNA to ribosome select the correct option
2) Recognition of DNA molecule
3) Aminoacylation of tRNA COLUMN - I COLUMN - II

4) Recognition of an anti- codon. Trygon


a) 6-15 pairs of gill slits i)

34. Which of the following regions of the


b) Heterocercal caudal fin ii) Cyclostomes
globe exhibits highest species diversity?
c) Air Bladder iii) Chondrichthyes
1) Western Ghats of India
d) Poison sting iv) Osteichthyes
2) Madagascar
3) Himalayas\
a) b) c) d)
4) Amazon forests
1) ii iii iv i
35. Which of the following statements is 2) iii iv i ii
not correct?
3) iv ii iii i
1) In man insulin is synthesised as a
proinsulin. 4) i iv iii ii
38. From his experiments, S.L. Miller
2) The proinsulin has an extra peptide
produced amino acids by mixing the
called C- peptide
following in a closed flask:
3) The functional insulin has A and B 1) CH4, H2, NH3 and water vapour at
chains linked together by hydrogen 800oC
bonds
2) CH3 , H 2 , NH 4 and water vapor at 800o C
4) Genetically engineered insulin is
produced in E- Coli. 3) CH 4 , H 2 , NH 3 and water vapor at 600o C

4) CH 3 , H 2 , NH3 and water vapor at 600o C


210
Previous Years NEET Questions & Solutions

39. Embryological support for evolution was 44. Which of the following refer to correct
disapproved by : example(s) of organisms which have
evolved due to changes in environment
1) Karl Ernst von Baer
brought about by anthropogenic action?
2) Alfred Wallace
a) Darwin’s Finche s of Galapagos
3) Charles Darwin islands

4) Oparin b) Herbicide resistant weeds

40. The process responsible for facilitating c) Drug resistant eukaryotes


loss of water in liquid form the tip of
d) Man - created breeds of domesticated
grass blades at night and in early
animals like dogs.
morning is :
1) only (a)
1) Transpiration
2) a and c
2) Root pressure
3) b, c and d
3) Imbibition
4) only d
4) Plasmolysis
41. Secondary metabolites such as 45. Identify the wrong statement with
nicotine, strychnine and caffeine are reference to immunity.
produced by plants for their:
1) When exposed to antigen (living or
1) Nutritive value dead) antibodies are produced in the
2) Growth response host’s body. It is called “Active
immunity”.
3) Defence action
2) When ready - made antibodies are
4) Effect on reproduction
directly given, it is called “ Passive
42. The oxygenation activity of RuBisCo immunity”.
enzyme in photorespiration leads to the
formation of : 3) Active immunity is quick and gives
full response.
1) 2 molecules of 3 - C compound
2) 1 molecule of 3 - C compound 4) Foetus receives some antibodies
from mother, it is an example for
3) 1 molecule of 6 - C compound passive immunity.
4) 1 molecule of 4 - C compound and 1
molecules of 2 - C compound 46. By which method was a new breed
‘Hisardale’ of sheep formed by using
43. Bt cotton variety that was developed Bikaneri ewes and Marino rams?
by the introduction of toxin gene of
Bacillus thuringiensis (Bt) is resistant to: 1) Out crossing
1) Insect pests 2) Mutational breeding
2) Fungal diseases 3) cross breeding
3) Plant nematodes 4) Inbreeding
4) Insect predators

211
Brilliant STUDY CENTRE

47. Identify the correct statement with 50. Floridean starch has structure similar
reference to human digestive system. to:
1) Ileum opens into small intestine 1) Starch and cellulose
2) Serosa is the innermost layer of the 2) Amylopectin and glycogen
alimentary canal
3) Mannitol and algin
3) Ileum is a highly coiled part
4) Laminarin and cellulose
4) Vermiform appendix arises from
51. Select the option including all sexually
duodenum.
transmitted diseases.
48. Match the following columns and
select the correct option. 1) Gonorrhoea, Syphilis, Genital herpes

Column - I Column - II 2) Gonorrhoea, Malaria, Genital herpes

a) Clostridium i) Cyclosporin - A 3) AIDS, Malaria, Filaria


butylicum 4) Cancer, AIDS, Syphilis
b) Trichoderma ii) Butyric Acid 52. Match the following with respect to
Polysporum meiosis:

c) Monascus iii) Citric Acid a) zygotene i) Terminalization

Purpureus b) Pachytene ii) Chiasmata


d) Aspergillus c) Diplotene iii) Crossing over
niger iv)Blood d) Diakinesis iv) Synapsis
cholesterol
Select the correct option from the
lowering agent following:

a) b) c) d) a) b) c) d)
1) iii iv ii i 1) iii iv i ii
2) ii i iv iii 2) iv iii ii i
3) i ii iv iii
3) i ii iv iii
4) iv iii ii i
4) ii iv iii i
49. Presence of which of the following
conditions in urine are indicative of 53. Which of the following pairs is of
Diabetes Mellitus? unicellular algae?

1) Uremia and Ketonuria 1) Laminaria and sargassum

2) Uremia and Renal Calculi 2) Gelidium and Gracilaria

3) Ketonuria and Glycosuria 3) Anabaena and Volvox


4) Renal calculi and Hyperglycaemia 4) Chlorella and Spirulina

212
Previous Years NEET Questions & Solutions

54. Which of the following hormone levels 58. The ovary is half inferior in:
will cause release of ovum (ovulation)
1) Brinjal
from the graffian follicle
2) Mustard
1) High concentration of Estrogen
3) Sunflower
2) High concentration of Progesterone
4) Plum
3) Low concentration of LH
59. The enzyme enterokinase helps in
4) Low concentration of FSH conversion of :
55. Match the following columns and 1) Protein into polypeptides
select the correct option.
2) trypsinogen into trypsin
COLUMN - I COLUMN - II
3) caseinogen into casein
a) Bt cotton i) Gene therapy
4) Pepsinogen into pepsin
b) Adenosine deaminase deficiency ii) Cellular defence

c) RNAi iii) Detection of HIV infection 60. Match the trophic levels with their
correct species examples in grassland
d) PCR iv) Bacillus thuringiensis
ecosystem.

a) b) c) d)
COLUMN - I COLUMN - II
1) iv i ii iii
a) Fourth trophic level i) Crow
2) iii ii i iv
b) Second trophic level ii) Vulture
3) ii iii iv i
c) First trophic level iii) Rabbit
4) i ii iii iv
d) Third trophic level iv) Grass
56. Montreal protocol was signed in 1987
for control of :
1) Transport of Genetically modified a) b) c) d)
organisms from one country to another.
1) ii iii iv i
2) Emission of ozone depleting
substances. 2) iii ii i iv
3) Release of Green House gases 3) iv iii ii i
4) Disposal of e- wastes
4) i ii iii iv
57. Which of the following is correct about
viroids? 61. How many true breeding pea plant
varieties did Mendel select as pairs.
1) They have RNA with protein coat. which were similar except in o ne
2) They have free RNA without protein character with contrasting traits?
coat.
1) 4
3) They have DNA with protein coat.
4) They have free DNA without protein 2) 2
coat. 3) 14
4) 8

213
Brilliant STUDY CENTRE

62. Match the following columns and 66. Which of the following is not an
select the correct option. inhibitory substance governing seed
dormancy?

COLUMN - I COLUMN - II 1) Gibberellic acid

a) Organ of Corti i)
Connects middle ear 2) Abscisic acid
and pharynx

b) Cochlea ii)
Coiled part of the 3) Phenolic acid
labyrinth
c) Eustachian tube iii)
Attached to the oval 4) Para - ascorbic acid
window
d) Stapes iv)
Located on the basilar 67. Name the enzyme that facil itates
membrane
opening of DNA he lix during
transcription.
a) b) c) d)
1) DNA ligase
1) ii iii i iv
2) DNA helicase
2) iii i iv ii
3) DNA polymerase
3) iv ii i iii
4) RNA polymerase
4) i ii iv iii
68. Which of the following would help in
63. In water hyacinth and water lily, prevention of diuresis?
pollination takes place by :
1) More water reabosorption due to
1) insects or wind undersecretion of ADH.
2) water currents only 2) Reabsorption of Na+ and water from
3) wind and water renal tubules due to aldosterone.

4) insects and water 3) Atrial natriuretic factor causes


vasoconstriction.
64. Name the plant growth regulator which
upon spraying on sugarcane crop, 4) Decrease in secretion of renin by
increases the length of stem, thus JG cells.
increasing the yield of sugarcane crop. 69. In relation to Gross primary productivity
1) Cytokinin and Net primary productivity of an
ecosystem, which one of the following
2) Gibberellin statements is correct?
3) Ethylene 1) Gross primary productivity is always
less than net primary productivity.
4) Abscisic acid
2) Gross primary productivity is always
65. In light reaction, plastoquinone
more than net primary productivity
facilitates the transfer of electrons
from: 3) Gross primary productivity and Net
primary productivity are one and same.
1) PS - II t o Cyt b 6f complex
4) There is no relationship between
2) Cytb6f complex to PS - I
Gross primary productivity and Net
3) PS - I to NADP + primary productivity.

4) PS - I to ATP synthase

214
Previous Years NEET Questions & Solutions

70. Match the following columns and 74. If the distance between two consecutive
select the correct option. base pairs is 0.34nm and the total
number of base pairs of a DNA double
helix in a typical mammalian cell is
COLUMN - I COLUMN - II
6.6 ×109 bp, then the length of the DNA
a) Placenta i) Androgens is approximately.
Human Chorionic
b) Zona pellucida ii)
Gonadotropin (hCG) 1) 2.0 meters
c) Bulbo- urethral glands iii) Layer of the ovum

Lubrication of the 2) 2.5 meters


d) Leydig cells iv)
Penis
3) 2.2 meters
a) b) c) d)
4) 2.7 meters
1) iv iii i ii
75. The QRS complex in a standard ECG
2) i iv ii iii represents
3) iii ii iv i 1) Repolarisation of auricles
4) ii iii iv i
2) Depolarisation of auricles
71. Strobili or cones are found in :
3) Depolarisation of ventricles
1) Salvinia
4) Repolarisation of ventricles
2) Pteris

3) Marchantia 76. Match the following columns and


select the correct option
4) Equisetum

72. Some dividing cells exit the cell cycle


and enter vegetative inactive stage. COLUMN - I COLUMN - II
This is called quiescent state (G0). This
a) Eosinophils i) Immune response
process occurs at the end of :
b) Basophils ii) Phagocytosis
1) M phase
Release histaminase,
c) Neutrophils iii)
2) G1 phase destructive enzymes
Release granules
d) Lymphocytes iv)
3) S phase containing histamine

4) G2 phase
a) b) c) d)
73. Flippers of Penguins and Dolphins are
examples of : 1) iii iv ii i
1) Adaptive radiation
2) iv i ii iii
2) Convergent evolution
3) i ii iv iii
3) Industrial melanism

4) Natural selection 4) ii i iii iv

215
Brilliant STUDY CENTRE

77. Which of the following statements is 81. Select the correct statement
correct?
1) Glucocorticoids stimulate
1) Adenine pairs with thymine through gluconeogenesis.
two H- bonds
2) Glucagon is associated with
2) Adenine pairs with thymine through hypoglycemia.
one H - bond
3) Insulin acts on pancreatic cells and
3) Adenine pairs with thymine through adipocytes.
three H- bonds
4) Insulin is associate d with
4) Adenine does not pair with thymine. hyperglycemia.
78. The sequence that controls the copy 82. Which one of the following is the most
number of the linked DNA in the abundant protein in the animals?
vector, is termed:
1) Haemoglobin
1) Selectable marker
2) Collagen
2) Ori site
3) Lectin
3) Palindromic sequence
4) Insulin
4) Recognition site
79. Identify the basic amino acid from the 83. Experimental verification of the
following chromosomal theory of inheritance
was done by:
1) Tyrosine
1) Mendel
2) Glutamic Acid
2) Sutton
3) Lysine
3) Boveri
4) Valine
4) Morgan
80. Match the following columns and
select the correct option 84. Match the following columns and
select the correct option.

COLUMN - I COLUMN - II
COLUMN - I COLUMN - II
a) Pituitary gland i) Grave's disease Located between second
a) Floating Ribs i)
and seventh ribs
b) Thyroid gland ii) Diabetes mellitus b) Acromion ii) Head of the Humerus

c) Adrenal gland iii) Diabetes insipidus c) Scapula iii) Clavicle


Do not connect with the
d) Pancrease iv) Addison's disease d) Glenoid cavity iv)
sternum.

a) b) c) d) a) b) c) d)

1) iv iii i ii 1) ii iv i iii

2) iii ii i iv 2) i iii ii iv

3) iii i iv ii 3) iii ii iv i

4) ii i iv iii 4) iv iii i ii
216
Previous Years NEET Questions & Solutions

85. The number of substrate level CHEMISTRY


phosphorylations in one turn of citric
acid cycle is: 91. Identify a molecule which does not
exist.
1) Zero 2) One
1) He2 2) Li2
3) Two 4) Three
3)C2 4) O2
86. Dissolution of the synaptonemal
complex occurs during: 92. Find out the solubility of Ni(OH) 2 in
0.1M NaOH. Given that the ionic
1) Pachytene product of Ni(OH)2 is 2×10–15.
2) Zygotene 1) 2  1013 M 2) 2  108 M
3) Diplotene
3) 1  1013 M 4) 1  10 8 M
4) Leptotene
93. Identify the correct statements from
87. Bilaterally symmetrical and acoelomate the following:
animals are exemplified by :
a) CO2(g) is used as refrigerant for ice-
1) Ctenophora cream and frozen food.
2) Platyhelminthes b) The structure of C60 contains twelve
3) Aschelminthes six carbon rings and twenty five carbon
rings.
4) Annelida
c) ZSM - 5, a type of zeolite, is used to
88. The body of the ovule is fused within convert alcohols into gasoline.
the funicle at:
d) CO is colorless and odourless gas.
1) Hilum 2) Micropyle
1) a, b and c only 2) a and c only
3) Nucellus 4) Chalaza
3) b and c only 4) c and d only
89. Goblet cells of alimentary canal are
modified from: 94. Hydrolysis of sucrose is given by the
following reaction.
1) Squamous epithelial cells
2) Columnar epithelial cells Sucrose  H 2 O  Glu cos e  Fructose

3) Chondrocytes If the equilibrium constant (K c ) is


4) Compound epithelial cells 2×1013 at 300K, the value of  r G at
90. Snow- blindness in Antarctic region is the same temperature will be:
due to:
1) 8.314 J mol K  300K  In  2  10 
1 1 13

1) Freezing of fluids in the eye by low


temperature
2) 8.314 J mol K  300K  In  2  10 
1 1 13

2) Inflammation of cornea due to high


dose of UV- B radiation
3) 8.314 J mol K  300K  In  3  10 
1 1 13

3) High reflection of light from snow


4) Damage to retina caused by infra-
4) 8.314 J mol K  300K  In  4  10 
1 1 13
red rays.
217
Brilliant STUDY CENTRE

95. Identify compound X in the following 98. Which of the following set of molecules
sequence of reactions: will have zero dipole moment?

1) Ammonia, beryllium difluoride,


CH 3 CHO
water, 1,4 - dichlorobenzene.

Cl2/h  2) Boron trifluoride, hydrogen fluoride,


x H2O
carbon dioxide, 1, 3- dichlorobenzene
373 K
3) Nitrogen trifluoride , be ryllium
difluoride, water, 1, 3- dichlorobenzene
Cl CH2Cl
4) Boron trifluoride, beryllium
difluoride, carbon dioxide, 1, 4 -
1) 2) dichlorobenzene.

99. On electrolysis of dil. sulphuric acid


using Platinum (Pt) electrode, the
CHCl2 CCl3 product obtained at anode will be:

1) Hydrogen gas
3) 4) 2) Oxygen gas

3) H2S gas
96. Identify the incorrect match. 4) SO2 gas.

Name IUPAC Official Name


100. Reacti on betwe en acetone and
methylmagnesium chloride followed by
Unnilunium i) Mendelveium hydrolysis will give:

Unniltrium ii) Lawrencium 1) Isopropyl alcohol

Unnilhexium iii) Seaborgium 2) sec. butyl alcohol

Unununnium iv) Darmstadtium 3) Tert. butyl alcohol

4) Isobutyl alcohol
1)  a  ,  i  2)  b  ,  ii 
101. Which of the following oxoacid of
sulphur has –O–O– linkage?
3)  c  ,  iii  4)  d  ,  iv 

97. An element has a body centered cubic 1) H 2SO 3 ,sulphurous acid


(bcc) structure with a cell edge of 288
pm. The atomic radius is : 2) H 2SO 4 ,sulphuric acid

3 2 3) H 2S2 O8 , peroxodisulphuric acid


1)  288pm 2)  288pm
4 4
4) H 2S2 O7 , pyrosulphuric acid
4 4
3)  288pm 4)  288pm
3 2
218
Previous Years NEET Questions & Solutions

102. Which of the following amine will give 106. The number of Faradays(F) required to
the carbylamine test? produce 20 g of calcium from molten
CaCl2 (Atomic mass of Ca = 40 g mol–1)
NH2 NHCH3 is:
1) 1 2) 2

1) 2) 3) 3 4) 4
107. Reaction between benzaldehyde and
acetophenone in presence of dilute
NaOH is known as :
N(CH3)2 NHC2H5
1) Aldol condensation
2) Cannizzaro’s reaction
3) 4)
3) Cross Cannizzaro’s reaction
4) Cross Aldol condensation
103. The calculated spin only magnetic 108. Paper chromatography is an example
moment of Cr2+ ion is: of :
1) 3.87 BM 1) Adsorption chromatography
2) 4.90 BM 2) Partition chromatography
3) 5.92 BM 3) Thin layer chromatography
4) 2.84 BM 4) Column chromatography
104. The correct option for free expansion 109. An increase in the concentration of the
of an ideal gas under adiabatic reactants of a reaction leads to change
conditions is: in :
1) q  0, T  0 and w  0 1) activation energy

2) q  0, T  0 and w  0 2) heat of reaction


3) threshold energy
3) q  0, T  0 and w  0
4) collision frequency
4) q  0, T  0 and w  0
110. A mixture of N 2 and Ar gases in a
105. The freezing point depression constant cylinder contains 7 g of N2 and 8 g of
(Kr)of benzene is 5.12 K kg mol–1. The Ar. If the total pressure of the mixture
fre ezing point depression for the of the gases in the cylinder is 27 bar,
solution of molality 0.078 m containing the partial pressure of N2 is:
a non- electrolyte solute in benzene is [Use atomic masses (in g mol –1 ):
(rounded off up two decimal places) : N = 14, Ar = 40 ]
1) 0.20 K 1) 9 bar
2) 0.80 K 2) 12 bar
3) 0.40 K 3) 15 bar
4) 0.60 K 4) 18 bar

219
Brilliant STUDY CENTRE

111. Identify the correct statement from the 116. Which of the following is the correct
following: order of increasing field strength of
1) Wrought iron is impure iron with 4% ligands to form coordination
carbon. compounds?

2) Blister copper has blistered 1) SCN   F  C2 O 42  CN 


appearance due to evolution of CO2.
3) Vapour phase refining has is carried 2) SCN   F  CN   C2 O42
our for Nickel by Van Arkel method.
4) Pig iron can be moulded into a variety 3) F  SCN   C2 O42   CN
of shapes.
4) CN   C2 O42  SCN   F
112. A tertiary butyl carbocation is more
stable than a secondary butyl 117. Which of the following is a basic amino
carbocation because of which of the acid?
following?
1) Serine 2) Alanine
1) –I effect of –CH3 groups
2) + R effect of – CH3 groups 3) Tyrosine 4) Lysine
3) – R effect of - CH3 groups 118. HCl was passed through a solution of
CaCl2, MgCl2 and NaCl. Which of the
4) Hyperconjugation
following compound(s) crystallise(s)?
113. Which of the following is a cationic
detergent? 1) Both MgCl2 and CaCl2
1) Sodium lauryl sulphate 2) Only NaCl
2) Sodium stearate 3) Only MgCl2
3) Cetyltrimethyl ammonium bromide 4) NaCl, MgCl2 and CaCl2
4) Sodium dodecylbenzene sulphonate
119. Which of the following is a natural
114. Elimination reaction of 2- Bromo - polymer?
pentane to form pent - 2- ene is:
1) cis - 1,4 - polyisoprene
a)   E limination reaction
2) poly (Butadiene - styrene)
b) Follows Zaitsev rule
3) Polybutadiene
c) Dehydrohalogenation reaction
4) poly (Butadiene - acrylonitrile)
d) Dehydration reaction
1) a, b, c 2) a, c, d 120. Which of the following is not correct
about carbon monoxide?
3) b, c, d 4) a, b, d
1) It forms carboxyhaemoglobin
115. The mixture which shows positive
deviation from Raoult’s law is : 2) It reduces oxygen carrying ability of
blood.
1) Ethanol + Acetone
3) The carboxyhaemoglobin
2) Benzene + Toluene (haemoglobin bound to CO) is less
3) Acetone + Chloroform stable than oxyhaemoglobin.

4) Chloroethane + Bromoethane 4) It is produced due to incomplete


combustion.

220
Previous Years NEET Questions & Solutions

121. Sucrose on hydrolysis gives: 126. Identify the incorrect statement.

1)   D  Glu cos e    D  Fructose 1) Cr  d  is a stronger reducing


2 4

2)   D  Glu cos e    D  Glu cos e agent than Fe  d  in water.


2 6

3)   D  Glu cos e    D  Fructose 2) The transition metals and their


compounds are known for the ir
4)   D  Fructose    D  Fructose catalytic activity due to their ability to
122.The following metal ion activates many adopt multiple oxidation states and to
enzymes, participates in the oxidation form complexes.
of glucose to produce ATP and with Na, 3) Interstitial compounds are those
is responsible for the transmission of that are formed when small atoms like
nerve signals. H,C or N are trapped inside the crystal
1) Iron lattices of metals.

2) Copper 4) The oxidation states of chromium in


CrO24 and Cr2 O72  are not the same.
3) Calcium
4) Potassium 127. For the reaction, 2Cl  g   Cl2  g  the
123. Which one o f the following has correct option is :
maximum number of atoms?
1)  r H  0 and  r S  0
1) 1g of Ag(s) [Atomic mass of Ag  108]
2)  r H  0 and  r S  0
2) 1g of Mg(s) [Atomic mass of Mg  24]
3)  r H  0 and  r S  0
3) 1g of O 2 (g) [Atomic mass of O  16]
4)  r H  0 and  r S  0
4) 1g of Li(s) [Atomic mass of Li  7]
128. Measuring Zeta potential is useful in
124. The number of protons, neutrons and determining which property of colloidal
electrons in 175 solution?
71 Lu, respectively, are:
1) Viscosity
1) 71, 104 and 71
2) Solubility
2) 104, 71 and 71
3) Stability of the colloidal particles
3) 71, 71 and 104
4) Size of the colloidal particles.
4) 175, 104 and 71
129. Urea reacts with water to form A which
125. What is the change in oxidation will decompose to form B. B when
number of carbon in the following passed through Cu 2+ (aq), deep blue
reaction? colour solution C is formed. what is
the formula of C from the following?
CH 4  g   4Cl2  g   CCl 4 1  4HCl  g 
2
1) Cu SO 4 2) Cu  NH3 4 
1) +4 to + 4 2) 0 to + 4
3) –4 to + 4 4) 0 to –4 3) Cu  OH  2 4) CuCO 3 .Cu  OH 2

221
Brilliant STUDY CENTRE

130. Match the following and identify the 133. An alkene on ozonolysis gives methanal
correct option as one of the product. Its structure is:

Mg  HCO3 2  CH=CH CH3


a) CO  g   H 2  g  i) Ca HCO
 3 2

1)
b) Temporary ii) An electron
hardness of deficient hydride
water CH2 CH3
CH2
c) B2 H 6 iii) Synthesis gas
2)
Non  planar
d) H2O2 iv)
structure
a) b) c) d)
CH2 CH CH2
1) iii i ii iv
2) iii ii i iv
3) iii iv ii i 3)

4) i iii ii iv
131. Match the following:
CH2CH2CH3
O x id e N a tu re

a) C O i) B as ic
4)

b) B a O ii) N eu tra l
134. Which of the following alkane cannot
c ) A l2 O 3 iii) A c id ic
be made in good yie ld by Wurtz
d) C l 2 O 7 iv) A m pho teric reaction?
1) n - Hexane
2) 2,3 - Dimethylbutane
Which of the following is correct 3) n - Heptane
option? 4) n - Butane
a) b) c) d) 135. Anisole on cleavage with HI gives:
1) i ii iii iv
OH I
2) ii i iv iii
3) iii iv i ii
1) 2)
4) iv iii ii i + CH3I +CH3OH
132. The rate constant for a first order
reaction is 4.606  103 s 1 . The time
required to reduce 2.0 g of the reactant OH I
to 0.2 g is:
1) 100 s 2) 200 s 3) 4)
+C2H5I +C2H5OH
3) 500 s 4) 1000s

222
Previous Years NEET Questions & Solutions

PHYSICS 141. Two cylinders A and B of equal capacity


are connected to each other via a stop
136. For which one of the following, Bohr cock. A contains an ideal gas at
model is not valid? standard temperature and pressure. B
1) Hydrogen atom is completely evacuated. The entire
system is thermally insulated. The
2) Singly ionised helium atom (He+) stop cock is suddenly opened. The
3) Deuteron atom process is :

4) Singly ionised neon atom (Ne+) 1) isothermal

137. The ratio of contributions made by the 2) adiabatic


electri c field and magnetic field 3) isochoric
components to the intensity of an
electromagnetic wave is : (c= speed of 4) isobaric
electromagnetic waves)
142. The energy equivalent of 0.5 g of a
1) c: 1 2) 1:1 substance is :

3) 1: c 4) 1: c2 1) 4.5  1016 J 2) 4.5  1013 J


138. The Brewsters angle ib for an interface 3) 1.5  1013 J 4) 0.5  1013 J
should be:
143. A body weighs 72 N on the surface of
1) 0o  i b  30o 2) 30o  i b  45o the earth. What is the gravitational
force on it, at a height equal to half
3) 45o  i b  90o 4) i b  90o the radius of the earth?

139. A cylinder contains hydrogen gas at 1) 48 N


pressure of 249 kP and temperature 2) 32 N
27oC. Its density is , (R= 8.3Jmol-k–1)
3) 30 N
1) 0.5 kg/m3
4) 24 N
2) 0.2 kg/m3
144. The solids which have the negative
3) 0.1 kg/m3 temperature coefficient of resistance
4) 0.02 kg/m3 are:

140. A ray is incident at an angle of 1) metals


incidence i on one surface of a small 2) insulators only
angle prism (with angle of prism A) and
3) semiconductors only
emerges normally from the opposite
surface. If the refractive index of the 4) insulators and semiconductors
material of the prism is  , then the 145. The phase difference betwe en
angle of incidence is nearly equal to : displacement and acceleration of a
particle in a simple harmonic motion
A 2A is:
1) 2)
2  3
1)  rad 2) rad
2
A
3) A 4) 
2 3) rad 4) zero
2
223
Brilliant STUDY CENTRE

146. A screw gauge has least count of 0.01 151. A spherical conductor of radius 10 cm
mm and there are 50 divisions in its has a charge of 3.2  107 C distributed
circular scale.
uniformly. What is the magnitude of
The pitch of the screw gauge is : electric field at a point 15 cm from the
1) 0.01 mm 2) 0.25 mm centre of the sphere?

3) 0.5 mm 4) 1.0 mm  1 
  9  109 Nm 2 / C2 
147. In a guitar , two strings A and B made  40 
of same material are slightly out of
tune and produce beats of frequency 6 1) 1.28  104 N / C 2) 1.28  105 N / C
Hz. When tension in B is slightly
decreased, the beat fre quency 3) 1.28  106 N / C 4) 1.28  10 7 N / C
increases to 7 Hz. If the frequency of
A is 530 Hz, the original frequency of 152. In a certain region of space with volume
B will be: 0.2 m3, the electric potential is found
1) 523 Hz 2) 524 Hz to be 5 V throughout. The magnitude
of electric field in this region is :
3) 536 Hz 4) 537 Hz
148. Two particles of mass 5 kg and 10 kg 1) Zero 2) 0.5 N/C
respectively are attached to the two 3) 1N/C 4) 5 N/C
ends of a rigid rod of length 1 m with
negligible mass. 153. The increase i n the width of the
The centre of mass of the system from depletion region in a p- n junction
the 5 kg particle is nearly at a distance diode is due to:
of : 1) forward bias only
1) 33 cm 2) 50 cm
2) reverse bias only
3) 67 cm 4) 80 cm
3) both forward bias and reverse bias
149. Find the torque about the origin when
ˆ acts on a particle whose
a force of 3jN 4) increase in forward current.

position vector is 2 kˆ m . 154. A 40F capacitor is connected to a


200V, 50 Hz ac supply. The rms value
ˆ
1) 6iNm 2) 6ˆjNm of the current in the circuit is, nearly:

ˆ
3) 6iNm ˆ
4) 6kNm 1) 1.7 A 2) 2.05 A

150. Light with an average flux of 20 W/cm2 3) 2.5 A 4) 25.1A


falls on a non- reflecting surface at 155. The mean free path for a gas, with
normal incidence having surface area molecular diameter d and number
20 cm2. The energy received by the density n can be expressed as :
surface during time span of 1 minute
is :
1 1
1) 2)
1) 10  10 J
3 2) 12  10 J
3
2nd 2nd 2

3) 24  103 J 4) 48  103 J
1 1
3) 3)
2n d
2 2
2n 2 2 d 2

224
Previous Years NEET Questions & Solutions

156. For transistor action, which of the 160. Two bodies of mass 4 kg and 6 kg are
following statements is correct? tied to the ends of a massless string.
The string passes over a pulley which
1) Base, emitter and collector regions is frictionless (se e fi gure ). The
should have same doping acceleration of the system in terms of
concentrations. acceleration due to gravity(g) is :
2) Base, emitter and collector regions
should have same size

3) Both emitter junction as well as the


collector junction are forward biased.

4) The base region must be very thin


and lightly doped.

157. Light of frequency 1.5 times the


threshold frequency is incident on a 1) g 2) g/2
photosensitive material. What will be
the photoelectric current if the 3) g/5 4) g/10
frequency is halved and intensity is
doubled? 161. A wire of length L, area of cross section
A is hanging from a fixed support. The
1) doubled length of the wire changes to L1 when
mass M is suspended from its free end.
2) four times The expression for Young’s modulus is:
3) one- fourth
MgL1
4) zero 1)
AL
235
158. When a uranium iso tope 92 U is
Mg  L1  L 
bombarded with a neutron, it 2)
89 AL
generates 36 Kr , three neutrons and:
144
1) 56 Ba MgL
3) AL
91 1
2) 40 Zr
101
3) 36 Kr MgL
4) A  L  L 
103 1
4) 36 Kr

159. The energy required to break one bond 162. The average thermal energy for a mono
in DNA is 10–20 J. This value in eV is - atomic gas is : (k B is Boltzmann
nearly: constant and T, absolute temperature)

1) 6 1 3
1) k BT 2) k BT
2) 0.6 2 2

3) 0.06
5 7
3) k BT 4) k BT
4) 0.006 2 2

225
Brilliant STUDY CENTRE

163. Which of the following graph represents 165. In Young’s double slit experiment, if
the separation be twee n cohere nt
the variation of resistivity p with
sources is halved and the distance of
temperature (T) for copper? the screen from the coherent sources
is doubled, the n the fringe width
becomes:
1) 1) double 2) half
3) four times 4) one- fourth
166. The capacitance of a parallel plate
capacitor with air as medium is 6F .
2)
With the introduction of a dielectric
medium, the capacitance becomes
30F . The permittivity of the medium
is :

3)
 0  8.85  1012 C 2 N 1m 2 

1) 0.44  1013 C2 N1m2

2) 1.77 1012 C2 N1m2

3) 0.44  1010 C 2 N 1m 2


4)
4) 5.00C2 N 1m 2

167. Dimensions of stress are:

1)  MLT 
2

164. The color code of a resistance is given


below
2)  ML T 
2 2

3)  ML T 
0 2

4)  ML T 
1 2

The values of resistance and tolerance, 168. Assume that light of wavelength 600
respectively are : nm is coming from a star. The limit of
resolution of telescope whose objective
1) 470k,5% has a diameter of 2m is:
1) 3.66  107 rad
2) 47k,10%
2) 1.83  107 rad
3) 4.7k,5%
3) 7.32  107 rad
4) 470, 5%
4) 6.00  107 rad

226
Previous Years NEET Questions & Solutions

169. A series LCR circuit is connected to 172. A long solenoid of 50 cm length having
an ac voltage source. Whe n L is 100 turns carries a current of 2.5 A.
removed from the circuit, the phase The magnetic field at the centre of the
difference between current and voltage solenoid is :

is
3
. If instead C is removed from the  0  4  107 TmA 1 
circuit, the phase difference is again
1) 6.28  104 T

between current and voltage. The
3 2) 3.14  104 T
power factor of the circuit is :
3) 6.28  105 T
1) zero 2) 0.5
4) 3.14  105 T
3) 1.0 4) –1.0
173. A charged particle having drift velocity
170. A short electric dipole has a dipole of 7.5  104 ms 1 in an electric field of
moment of 16  109 Cm . The electric
3  1010 Vm 1 , has a mobility in m2V–1S-1
potential due to the dipole at a point
of :
at a distance of 0.6 m from the centre
of the dipole, situated on a line making 1) 2.25  1015 2) 2.5  106
an angle of 60o with the dipole axis is:
3) 2.5  106 4) 2.25  1015
 1 
  9  109 Nm 2 / C2  174. The quantities of heat required to raise
 40  the temperature of two solid copper

1) 50 V spheres of radii r1 and r2  r1  1.5 r2 


through 1K are in the ratio:
2) 200 V
27 9
3) 400 V 1) 2)
8 4
4) Zero
3 5
171. An iron rod of susceptibility 599 is 3) 4)
2 3
subjected to a magnetising field of 1200
A m–1. The permeability of the material 175. An electron is accelerated from rest
of the rod is: through a potential difference of V volt.
If the de Broglie wavelength of the
 0  4  107 TmA 1  electron is 1.227×10–2nm, the potential
difference is :

1) 2.4  104 TmA 1 1) 10 V 2) 102V


3) 103V 4) 104V
2) 8.0  10 TmA 5 1

176. Taking into account of the significant


3) 2.4  105 TmA 1 figures, what is the value of 9.99 m-
0.0099 m?
4) 2.4  107 TmA 1 1) 9.9801 m 2) 9.98 m
3) 9.980 m 4) 9.9 m
227
Brilliant STUDY CENTRE

177. A ball is thrown vertically downward 180. For the logic circuit shown, the truth
with a velocity of 20 m/s from the top table is :
o f a t o w e r . I t h i t s t h e gro u nd
afte r som e t ime with a velocity of
80m/s. The height of the tower is :
(g = 10 m / s 2)
1) 360 m 2) 340 m
3) 320 m 4) 300 m
178. A capillary tube of radius r is immersed 1) A B Y 2) A B Y
in water and water rises in it to a 0 0 0 0 0 0
height h. The mass of the water in 0
0 1 0 1 1
the capillary is 5g. Another capillary
tube of radius 2r is immersed in water. 1 0 0 1 0 1
The mass of water that will rise in this 1 1 1 1 1 1
tube is :
1) 2.5 g 2) 5.0 g
3) A B Y 4) A B Y
3) 10.0 g 4) 20.0 g 0 0 1 0 0 1
179. A resistance wire connected in the left 0 1 1 0 1 0
gap of a meter bridge balances a 1 0 1 1 0 0
10 resistance in the right gap at a 1 1 0 1 1 0
point which divides the bridge wire in
the ratio 3:2.If the le ngth of the
resistance wire is 1.5 m, then length
of 1 of the resistance wire is:

1) 1.0  102 m 2) 1.0  101 m

3) 1.5  101 m 4) 1.5  102 m

228
Previous Years NEET Questions & Solutions

KEY WITH HINTS

BIOLOGY
1. 4 16. 2 31. 4 46. 3 61. 3 76. 1
2. 1 17. 3 32. 4 47. 3 62. 3 77. 1
3. 1 18. 2 33. 3 48. 2 63. 1 78. 2
4. 2 19. 3 34. 4 49. 3 64. 2 79. 3
5. 1 20. 1 35. 3 50. 2 65. 1 80. 3
6. 3 21. 1 36. 1 51. 1 66. 1 81. 1
7. 1 22. 4 37. 1 52. 2 67. 4 82. 2
8. 3 23. 4 38. 1 53. 4 68. 2 83. 4
9. 2 24. 1 39. 1 54. 1 69. 2 84. 4
10. 2 25. 3 40. 2 55. 1 70. 4 85. 2
11. 4 26. 4 41. 3 56. 2 71. 4 86. 3
12. 4 27. 2 42. 2 57. 2 72. 2 87. 2
13. 4 28. 3 43. 1 58. 4 73. 2 88. 1
14. 3 29. 3 44. 3 59. 2 74. 3 89. 2
15. 3 30. 1 45. 3 60. 1 75. 3 90. 2

CHEMISTRY 94. 1  r G = -RT lnKc


91. 1 Bon d or der He2 = 0
 8.314  300  n  2  1013 
Ni  OH  2  Ni 2
 20H 
92. 1 s 2s
CH 3 CHCl2
 
NaOH  Na  OH 95. 3
0.1 0.1 0.1
Cl2/h  H 2O
Ionic product 373K

=  Ni 2  OH    2  1015
2
OH
CH CHO
15
2  10 OH
 Ni  2

 2s  0.1
2

H2O
15
2  10
  2  1013 M
102 96. 4 Unununnium  z  III  Rontgenium

93. 4 CO 2s   dry ice  is used as refrigerant. 4R


97. 1 for bcc a 
3
C60 contains 20 6 - membered and
12 5 - membered ring 3a 3
R   288pm
4 4

229
Brilliant STUDY CENTRE

98. 4
103. 2 Cr 2  4So 3d 4 ; n  4
F
spin  n  n  1  45
B F Be F
 20  4.90BM
F F
104. 1 For free expansion of ideal gas,
w=0
Cl
For adeabatic process q = 0
From first law of thermodynamics,

= 0 u  q  w  0  0  0
0  C  0;
105. 3 Tf  ik f .m
Cl
For non- electrolyte solute, 1o  1
99. 2 dil. H2SO4 . On electrolysis gives
 T f  1  5 .1 2  0 .0 7 8
H2 gas librated at cathode
 0.399  0.40k
O2 gas librated at anode
2
106. 1 Ca Cl2  Ca o ;n  2
Number of faradays = 2F for 40 g
O  20 g Ca requers . 1F

100. 3 CH3C  CH3  CH3MgBr 107. 4  dil NaOH


CHO COCH3
+
no H H  3

Electophile nucleophile
OMgBr
H 2O OH
CH3 C CH3 O

CH CH2 C
CH3
  hydroxy ketone
 CH3  C  OH
It is a cross-aldol condensation.
108. 2 Paper chromatography is one of the
O O main partition chromatography be-
O O came in paper chromatography sub-
S S stances are presented between liq-
101. 3 H2S2O8 ; OH OH uid phases.
O O 109. 4 As the concentration increases num-
ber of reactant molecule also in-
o creases which incrases the number
1 amines gives carbylamine test
of collisions.
102. 1

230
Previous Years NEET Questions & Solutions

7
28 Preferred
110. 3 PN 2  PT  N 2  27bar  7 8 1
28 40
2
1 1 114. 1 CH3  CH2  CH2  CH  CH3

 27  4  27  4
1 1 9 Br

4 5 20
alc KOH
CH3 CH2CH  CH  CH3
1 20
 27    15bar
4 9 Saytheff Rule Pent - 2 - ene
  ele min ation

111. 4 Wrought iron is purest iron 115. 1 Ethanol + Acetone  +ve deviation
(Non ideal)
Blestered apperence of Cu due to Benzene + Toluene  Ideal solution
evolution of SO2
Acetone + chloroform  Non ideal
Nickel can be refined by Mond’s pro- (–ve deviation)
cess. Chloroethane + Bromoethane 
Ideal solutions
CH3 116. 1 Spectrochemical series order

112. 4 CH C I  SCN  Cl   S2  F   OH


3 CH3 Number  H  9
 C2O42  H2O  NH3  CN  CO

COOH
CH3 CH Number of  H  6
117. 4 Lysine H 2N H
CH3

As H increases, hyper conjugation CH 2 3 NH 2


increases, stability also increases
Serine  Neutral
113. 3 CTAB - Cetyl trimethyl ammonium
Alamine  Neutral
bromide.
Tyrosine  Neutral
CH 3 118. 2 only NaCl
On passing HCl through CaCl2, MgCl2
CH 3 CH 2 CN CH 3 Br and NaCl crystals of pure NaCl
15
seperate out since Ca and Mg chlo-
CH 3 rides are more soluble than NaCl,
remains in solution.
119. 1 Cis - 1, 4, polyisoprene

231
Brilliant STUDY CENTRE

120. 3 Carboxy haemoglobin is 300 times NH2CONH2  H2O   NH4 2 CO3


more stable than oxyhaemoglobin. 129. 2 A
121. 3 Sucrose contains two monosaccha-
rides ie .,   D glucose and  2NH3  H2O  CO2

  D  Fructose which are obtained 2


4NH 3  Cuaq    Cu  NH 3 4 
on its hydrolysis. B   aq 
deep blue  C 

CH2OH
O
HOH2C
O OH 130. 1 a) CO  H2  synthesis gas  III 
H H
H
OH H
HO O H b) Temporary hardness of water 
CH2OH
H OH OH
Mg  HCO3 2  Ca  HCO3 1
122. 4 Potassium
c) B2H6  An electron deficient hy-
Potassium ions are the most abun- dride (II)
dant cation with in the cell fluids.
d) H2O2  Non planar (IV)
1
123. 4 No. of atoms in 1g of Li   NA H
7
O O
1 H
No. of atoms in 1 g of Ag   NA
108
131. 2 Co - Neutral
1 BaO - Basic
No. of atoms in 1 g of O2   NA
32
Al2O3 - amphoferic
1
No. of atoms in 1 g of Mg  NA Cl2O7 - Acidic
24

124. 1 175
Lu ; no of electrons = no. of 132. 3 For Ist order reaction
71
protones = 71
no of neutrons = 175 - 71 = 104 2.303 R o
t log
125. 3 Oxidation no. of C in CH4 = –4
k R 
Oxidation of C in CCl4 = + 4
2.303 2
126. 4 The oxidation state of  3 1
log  500s
4.606  10 s 0.2
Cr in CrO24 and Cr2O72 ( ie, in chro-
133. 3 For obtain methanol (CH2O), the alk-
mate and in dichromate) is + 6
ene should contain =CH2 group
127. 4 2Cl  g   Cl2  g   Bond formation
134. 3 Alkane with odd number of carbon
 H   ve as the no. of gaseous par- atoms are not obtained in good yield
ticles are decreasing in the reac- by wurtz reaction
tion s is  ve
OCH 3
128. 3 Greater the zeta potential more will OH
HI
be the stability of colloidal particles. 135. 1 + CH 3 I

232
Previous Years NEET Questions & Solutions

PHYSICS
E  mc 2  0.5  103   3  108 
2
142. 2
136. 4 Singly ionised neon atom has more
than one electon in its orbit hence  4.5  1013 J
bohr model is not valid.
137. 2 1:1 . Energy is equally shared by gR 2
143. 2 g1  h  R /2
 R  h
2
electric field and magnetic field.
138. 3 tan p  ;1    
4g
1  tan p   ; g1 
9
tan1 1  p  tan1   
4mg 4
mg1    72  32N
45  p  90 9 9
144. 4 The electric resistance of insulator
249  103  2  10 3 and semiconductor decreases with
139. 2 pv  nRT *  
8.314  300 increase in temperature.

m 145. 1 x  A sin  t   
pv  RT   0.2kg / m 3
M
a  2 x  2 A sin  t   
pm  RT
a  2 A sin  t     
pm

RT 146. 3 Least count

pitch

number of division oncircular scale

pitch
0.01  , pitch  0.5mm
50
140. 3
1 
147. 2 f
2 

90  r1  90  A  180 If tension in B slightly decrease then


frequency of B decreases. If B is
r1  A
536Hz, as the frequency decreases,
beats with A also decreases. If B is
1  sin i   sin r1 524 Hz, as the frequency decreases
A is small beats with A incrases. If tension
decreases F B decreases and be-
So i and r are small angles. comes fB1 .
sin i  i sin r1  r1
f A  fB  6
i  r1 ; i  A 530  fB  6
141. 2 expansion of gas against vacuum is fB  524Hz
adiabatic Q  0
233
Brilliant STUDY CENTRE

148. 3 156. 4

5r1  10 1  r1  ; r1  2  2r1

2 200  width of   width of   width of 


3r1  2; r1  m ; r1  cm  67cm    
3 3  collector   emitter   base 
 
149. 3 F  3ˆj; r  2k
ˆ  Doping   Doping 
     Doping level 
 level of    level of    of base 
   ˆ  emitter   colector   
  r  F  6iNm    
150. 3 E = IAT
f  1.5fo  1
  hfo ; f 1  
2  2  f  fo
157. 4 ;
20
=  20  104  60  24  103 J
104 Incident energy < work function
hence photo ele ctric emission
1 Q 9  109  3.2  10 7
151. 2 E  doesnot occur.
4o r2 225  10 4
158. 1 235
92 U 10 n 89
36 Kr  30  z y
h A

E = 1.28×10 N/C 5

92  0  36  z * z  56
v  5volt  cons tan t
235  1  89  3  A * A  144
152. 1 dv
E 0
dr 1020
159. 3 E  0.06ev
1.6  1019

153. 2 160. 3

In reverse bias, external battery


attracts majority charge carriers. So
width of depletion layer increases.

1 1
154. 3 xc  
c 2fc

v rms  200v, F  50Hz

v rms
Irms    2fc  Vrms   2.5A
xc

155. 2 6g  4g g
a 
10 5
234
Previous Years NEET Questions & Solutions

F L Mg y  L1  L  tan
 xc
 ; x c  3R
161. 4 y ; 
A L A L 3 R

MgL
y
A  L1  L 

f
162. 2 E KB T
2
for monoatomic gas f=3  xL
tan  ; x L  3R
3 R
3
E KB T
2

163. 3   o eT

164. 4 yellow  4
violet  7 xL  xc ; z  R
Brown  1 R
cos   1
Gold  5% z
R  47  101  5% 1  p cos  
R  470  5% 170. 2 v  
4o  r 2 

D   2D 
4 D 9  109  16  109  cos 60
165. 3  * 1
 
 0.6
2
d d /2 d

166. 3 C '  KC v = 200 volt

30  K  6  ; k  5 171. 1 x m  599
r  1  x m  600
  Ko  5  8.85  1012
  o r  4  10 7  600
  0.44  1010 c2 /Nm2   2.4  104 Tm / A

 F   MLT 2 o NI 4  10 7  100  2.5


167. 4 stress    ML1T 2 172. 1 B 
A L2  50  102

 1.22  600  109 B  6.28  104 T


168. 1   1.22 
d 2 vd 7.5  104
173. 2    2.5  106
  3.66  107 rad  3  1010

4 
174. 1 dQ  msdT; dQ   r 3   sdT
3 
169. 3 3 3
dQ1  r1   1.5  27
dQr ; 3
    
dQ2  r2   1  8
235
Brilliant STUDY CENTRE

175. 4   0.1227A o m  r2h  5gram


12.27A o
 2 h
v m1    2r      2m  10gram
2
12.27 P 3 
0.1227   ;P  15; R 
v 179. 2 ; R
10 2 A
v  104 volt 1.5
15 
176. 2 9.99  0.0099  9.9801  9.98 
In substraction the number of deci- 15
mal places in the result should be   1  101 m
equal to the number of decimal 1.5
places of that term in the operation
which contain lesser number of
decimal places.

v 2  u2  80    20 
2 2

177. 4 h 
2g 20 180. 1

h = 300 m y  A  B  A . B; y  A.B
2T cos  AND gate
178. 3 h
rg

2T cos  h
h'  
 2r  g g

236
Previous Years NEET Questions & Solutions

NEET QUESTION PAPER - 2020 [PHASE - II]


PHYSICS
1. The mean free path l for a gas molecule 6. The equivalent resistance between A
de pends upon diameter, d of the and B for the mesh shown in the figure
molecule as is

1 1
1)   2)  
d d2

2)   d 4)   d2
2. An intrinsic semiconductor is
converted into n-type extrinsic
semiconductor by doping it with 1) 4.8 
1) Germanium 2) Phosphorous
2) 7.2 
3) Aluminium 4) Silver
3. The half life of a radioactive sample 3) 16 
undergoing  -decay is 1.4×1017s. If the
number of nuclei in the sample is 4) 30 
2.0×1021, the activity of the sample is
nearly 7. A wheel with 20 metallic spokes each
1m long is rotated with a speed of 120
1) 103 Bq 2) 104 Bq rpm in a plane perpendicular to a
3) 105 Bq 4) 106 Bq magnetic field of 0.4 G. The induced
emf between the axle and rim of the
4. The E.M. wave with shortest
wheel will be (1G=10–4T)
wavelength among the following is
1) Microwaves 1) 2.51 V
2) Ultraviolet rays 2) 2.51×10–4V
3) X-rays
3) 2.51×10–5V
4) Gamma-rays
5. For the circuit shown in the figure, the 4) 4.0×10–5 V
current I will be
8. Out of the following which one is a
forward bisased diode?

1)

2)

1) 0.5 A 2) 0.75 A 3)

3) 1A 4) 1.5 A
4)

237
Brilliant STUDY CENTRE

9. A n-p-n transistor is connected in 13. For the ci rcuit given below, the
common emitter configuration (see Kirchoff’s loop rule for the loop BCDEB
figure) in which collector voltage drop is given by the equation
across load resistance (800  )
connected to the collector circuit is 0.8
V. The collector current is

1) –i2R2+E2+E3+i3R1=0
2) –i2R2+E2–E3+i3R1=0
3) i2R2+E2–E3–i3R1=0
4) i2R2+E2+E3+i3R1=0
1) 0.2 mA 2) 2 mA 14. An ideal gas equation can be written
3) 0.1 mA 4) 1 mA RT
as P  M
10. Two solid conductors are made up of 0

same material, have same length and where  and M0 are respectively
same resistance. One of them has a
circular cross section of area A1 and 1) Number density, mass of the gas
the other one has a square cross 2) Mass density, mass of the gas
section of area A2. The ratio A1/A2 is
3) Number density, molar mass
1) 2 2) 1.5
4) Mass density, molar mass
3) 1 4) 0.8 15. The magnetic flux linked with a coil
11. Two coherent sources of light interfere (in Wb) is given by the equation
and produce fringe pattern on a
screen. For central maximum, the   5t 2  3t  16
phase difference between the two The magnitude of induced emf in the
waves will be coil at the fourth second will be
1)  / 2 2) zero 1) 10 V
2) 33 V
3)  4) 3 / 2
3) 43 V
12. Time intervals measured by a clock give
the following readings 4) 108 V
16. The length of the string of a musical
1.25 s, 1.24 s. 1.27 s, 1.21 s and 1.28 s
instrument is 90 cm and has a
What is the percentage relative error fundamental frequency of 120 Hz.
of the observations? Where should it be pressed to produce
fundamental frequency of 180 Hz?
1) 1.6% 2) 2%
1) 80 cm 2) 75 cm
3) 4% 4) 16%
3) 60 cm 4) 45 cm

238
Previous Years NEET Questions & Solutions

17. The magnetic fi eld in a plane 21. The electric field at a point on the
electromagnetic wave is given by equatorial plane at a distance r from
the centre of a dipole having dipole
 
B y  2  10 7 sin   103 x  3  1011 t T 
moment p is given by, (r>> separation
Calculate the wavelength of two charges forming the dipole, 0 -
1)   10 3 m permittivity of free space)
 
2)   103 m  P  P
1) E   2) E 
4 0 r 3 4 0 r 3
3) 2×10–3 m
 
4) 2×103 m  2P  2P
18. A barometer is constructed using a 3) E  4) E  
4 0 r 3 4 0 r 3
liquid (density = 760 kg/m 3 ). What
would be the height of the liquid- 22. A liquid does not wet the solid surface
column, when a mercury barometer if angle of contact is
reads 76 cm? 1) Zero
(Density of mercury = 13600 kg/m3) 2) equal to 45°
1) 0.76 m 2) 1.36 m 3) equal to 60°
3) 13.6 m 4) 136 m 4) greater than 90°
19. The P.V diagram for an ideal gas in a 23. Three stars A, B, C have surface
piston cylinder assembly undergoing a temperatures TA, TB, TC respectively.
thermodynamic process is shown in Star A appears bluish, star B appears
the figure. The process is reddish and star C yellowish. Hence
1) TA>TC>TB 2) TA>TB>TC
3) TB>TC>TA 4) TC>TB>TA
24. A light bulb and an inductor coil are
connected to an ac source through a
key as shown in the figure below. The
key is closed and after sometime an
iron rod is inserted into the interior of
1) Isothermal the inductor. The glow of the light bulb
2) Adiabatic
3) Isochoric
4) Isobaric
20. The efficiency of a Carnot engine
depends upon
1) The temperature of the source only
2) The temperature of the sink only 1) Increases

3) The temperature of the source and 2) Decreases


sink 3) Remains unchanged
4) The volume of the cylinder of the 4) will fluctuate
engine
239
Brilliant STUDY CENTRE

25. Three identical spheres, each of mass 29. A wire of length L metre carrying a
M, are placed at the corners of a right current of 1 ampere is bent in the form
angle triangle with mutually of circle. Its magnetic moment is
perpendicular sides equal to 2 m (see
figure). Taking the point of intersection 1) 1L2 / 4Am2
of the two mutually perpendicular sides 2) 1L2 / 4Am2
as the origin, find the position vector
of centre of mass. 3) 1L2 / 4Am2

4) 21L2 / Am2
30. What is the depth at which the value
of acceleration due to gravity becomes
1
times the value that at the surface
n
of earth? (radius of earth=R)

R R
1) 2)
n n2

1)
3
 
4 ˆ ˆ
ij  
2) 2 ˆi  ˆj R  n  1 Rn
3) 4)  n  1
n
 
3) ˆi  ˆj 4)
3
 
2 ˆ ˆ
ij 31. An object is placed on the principal axis
of a concave mirror at a distance of
26. The de Broglie wavelength of an 1.5 f( f is the focal length). The image
electron moving with kinetic energy of will be at,
144 eV is nearly
1) 3f 2) –3f
1) 102×10–2 nm 2) 102×10–3 nm
3) 1.5f 4) –1.5f
3) 102×10–4 nm 4) 102×10–5 nm
32. The angular speed of the wheel of a
27. The angle of 1' (minute of arc) in vehicle is increased from 360 rpm to
radian is nearly equal to 1200 rpm in 14 second. Its angular
acceleration is,
1) 1.75×10–2 rad 2) 2.91×10–4 rad
3) 4.85×10–4 rad 4) 4.80×10–6 rad 1) 1 r ad/ s 2 2) 2 rad / s2
28. The total energy of an electron in the
3) 28 rad / s2 4) 120 rad / s2
n th stationary orbit of the hydrogen
atom can be obtained by 33. The acceleration of an electron due to
the mutual attraction between the
1) En  13.6  n2eV electron and a proton when they are
1.6 Ao apart is,
13.6
2) En  eV (me= 9×10–31 kg, e=1.6×10–19C)
n2

13.6 1 2 2
(Take 4  9  10 Nm c )
9
3) En   eV
n2 0

1) 1025 m/s2 2) 1024 m/s2


1.36
4) En  2 eV 3) 1023 m/s2 4) 1022 m/s2
n
240
Previous Years NEET Questions & Solutions

34. What happens to the mass number and 37. A point mass ‘m’ is moved in a vertical
atomic number of an element when it circle of radius ‘r’ with the help of a
emits  -radiation? string. The velocity of the mass is 7gr
1) Mass number increases by four and at the lowest point. The tension in the
atomic number increases by two string at the lowest point is
2) Mass number decreases by four and 1) 1 mg
atomic number decreases by two 2) 6 mg
3) Mass number and atomic number 3) 7 mg
remain unchanged
4) 8 mg
4) Mass number remains unchanged
38. A plano-convex lens of unknown
while atomic number decreases by one
material and unknown focal length is
35. If the critical angle for total internal given. With the help of a spherometer
reflection from a medium to vacuum we can measure the
is 45°, then velocity of light in the 1)Refractive index of the material
medium is,
2) focal length of the lens
1) 3×108 m/s
3) radius of curvature of the curved
2) 1.5 ×108 M/S surface
4) aperture of the lens
3
3)  108 m / s
2 39. A parallel plate capacitor having cross-
sectional area A and separation d has
4) 2  108 m / s air in between the plates. Now an
insulating slab of same area but
36. Calculate the acceleration of the block thickness d/2 is inserted between the
and trolly system shown in the figure. plates as shown in fig ure having
The coefficient of kineti c friction dielectric constant K(=4). The ratio of
between the trolly and the surface is new capacitance to its original
0.05 (g=10 m/s2, mass of the string is capacitance will be,
negligible and no other friction exists).

1) 1.00 m/s2 1) 4:1


2) 1.25 m/s2 2) 2:1

3) 1.50 m/s2 3) 8:5

4) 1.66 m/s2 4) 6:5

241
Brilliant STUDY CENTRE

40. The power of a biconvex lens is 10 42. Which of the following gate is called
dioptre and the radius of curvature of universal gate?
each surface is 10cm. Then the 1) NOT gate
refractive index of the material of the
lens is, 2) OR gate
3 4
1) 2) 3) AND gate
2 3

9 5 4) NAND gate
3) 4)
8 3
43. Identify the function which represents
41. The variation of electrostatic potential a periodic motion
with radial distance r from the centre
of a positively charged metallic thin
1) et
shell of radius R is given by the graph

2) et

3) loge  t 
1)
4) sin t  cos t

44. The wave nature of electrons was


experimentally verified by

1) Davisson and Germer

2) 2) de Broglie

3) Hertz

4) Einstein

45. A person sitting in the ground floor of


a building notices through the window,
of height 1.5 m, a ball dropped from
3)
the roof of the building crosses the
window in 0.1 s. What is the velocity
of the ball when it is at the topmost
point of the window? (g–10 m/s2)

1) 20 m/s

2) 15.5 m/s
4)
3) 14.5 m/s

4) 4.5 m/s

242
Previous Years NEET Questions & Solutions

CHEMISTRY 50. The half-life for a zero order reaction


having 0.02 M initial concentration of
46. Which among the following salt
reactant is 100s. The rate constant (in
solutions is basic in nature?
mol L–1 s–1) for the reaction is
1) Sodium acetate
1) 1.0×10–2 2) 1.0×10–4
2) Ammonium chloride
3) 2.0×10–4 4) 2.0×10–3
3) Ammonium sulphate
51) Match the coordination number and
4) Ammonium nitrate type of hybridisation with distribution
47. If 8g of a non-electrolyte solute is of hybrid orbitals in space based on
dissolved in 114 g of n-octane to reduce Valence bond theory
its vapour pressure to 80%, the molar Coordination Distribution
mass (in g mol–1) of the solute is [Given
that molar mass of n-octane is 114g number and of hybrid
mol–1] type of orbitals
1) 20 2) 40 hybridisation in space
3) 60 4) 80
a) 4, sp3 i) trigonal
48. Identify compound (A) in the following
reaction: bipyramidal
b) 4, dsp2 ii) Octahedral
CHO
c) 5, sp3d iii) Tetrahedral
d) 6, d2sp3 iv) Square planar
A 
H Pd BaSO
2 4

Select the correct option:
1) a-iii, b-i, c-iv, d-ii
1) Benzoic acid
2) a-ii, b-iii, c-iv, d-i
2) Benzoyl chloride
3) a-iii, b-iv, c-i, d-ii
3) Toluene
4) a-iv, b-i, c-ii, d-iii
4) Acetophenone
52. Match the element in column I with
49. Identify the incorrect statement from that in column II
the following
Column I Column II
1) The overall decrease in atomic and
ionic radii from lanthanum to lutetium a) Copper i) Non-metal
is called lanthanoid contraction
b) Fluorine ii) Transition Metal
2) Zirconium and Hafnium have
c) Silicon iii) Lanthanoid
identical radii of 160 pm and 159 pm,
re spectively as a conseque nce of d) Cerium iv) metalloid
lanthanoid contraction
Identify the correct match:
3) Lanthanoids reveal only +3 oxidation
state 1) a-i, b-ii, c-iii d-iv

4) The lanthanoid ions other than the 2) a-ii, b-iv, c-i, d-iii
f 0 type and the f 14 type are all 3) a-ii, b-i, c-iv, d-iii
paramagnetic
4) a-iv, b-iii, c-i, d-ii

243
Brilliant STUDY CENTRE

51. In collision theory of chemical reaction, 57. Match the elements in column I with
ZAB represents methods of purification in column II
1) The fraction of molecules with
energies equal to Ea Column I Column II
a) Boron i) Van Arkel method
2) The fraction of molecules with
energies greater than Ea b) Tin ii) Mond's process
c) Zirconium iii) Liquation
3) The collision frequency of reactants,
d) Nickel iv) Zone refining
A and B
4) Steric factor 1) a-iii, b-iv, c-i, d-ii
54. At standard conditions, if the change 2) a-iv, b-iii, c-i, d-ii
in the enthalpy for the following 3) a-iv, b-iii, c-ii, d-i
reaction is –109kJ mol–1.
4) a-ii, b-i, c-iv, d-iii
H2 g  Br2 g  2HBr g 58. A liquid compound (x) can be purified
by steam distillation only if it is
Given that bond energy of H2 and Br2 1) Not steam volatile, immiscible with
is 435 kJ mol –1 and 192 kJ mol –1 , water
respectively, what is the bond energy
2) Steam volatile, immiscible with
(in kJ mol–1) of HBr?
water
1) 259 2) 368 3) Not steam volatile, miscible with
3) 736 4) 518 water
4) Steam volatile, miscible with water
55. The solubility product for a salt of the
type AB is 4×10–8. What is the molarity 59. What is the role of gypsum, CaSO42H2O
of its standard solution? in setting of cement? Identify the
correct option from the following
1) 4×10–4 mol/L 2) 2×10–4 mol/L 1) to slow down the setting process
3) 16×10–16 mol/L 4) 2×10–16 mol/L 2) to fasten the setting process
56. The potential energy (y) curve for H 2 3) to provide water molecules for
formation as a function of internuclear hydration process
distance (x) of the H atoms is shown 4) to help to remove water molecules
below 60. Which of the following substituted
phenols is the strongest acid?

1) 2)

The bond energy of H2 is


1) (c–a) 2) (b–a)

c  a  b  a  3) 4)
3) 4)
2 2

244
Previous Years NEET Questions & Solutions

61. Deficiency of which vitamin causes Identify the incorrect option


osteomalacia?
66. Which of the following will NOT
1) Vitamin E undergo SN1 reaction with OH ?
2) Vitamin A
CH2Cl
3) Vitamin D
4) Vitamin K 1) 2) CH2=CH-CH2Cl
62. Which one of the following reactions
does not come under hydrolysis type
reaction?
CH2CH2Cl
1) P4O10(s)+6H2O(l)  4H3PO4(aq)
2) SiCl4(l)+2H2O(l)  SiO2(s)+4HCl(aq) 3) (CH3)3CCl 4)
3) Li3N(s)+3H2O(l)  NH3(g)+3LiOH(aq)
4) 2F2(g)+2H2O(l)  4HF(aq)+O2(g) 67. Reaction of propanamide with ethanolic
63. Which one of the following compounds sodium hydroxide and bromine will give
shows both, Frenkel as well as 1) Aniline 2) Ethylamine
Schottky defects?
3) Methylamine 4) Propylamine
1) ZnS
68. In which of the sols, the colloidal
2) AgBr particles are with negative charge?

3) Agl 1) Hydrated Al2O3


2) TiO2
4) NaCl
3) Haemoglobin
64. Which of the following is not true about
chloramphenicol? 4) Starch

1) It is bacteriostatic 69. The minimum pressure required to


compress 600 dm3 of a gas at 1 bar to
2) It inhibits the growth of only gram 150 dm3 at 40°C is
positive bacteria
1) 2.5 bar
3) It is a broad spectrum antibiotic 2) 4.0 bar
4) It is not bactericidal 3) 0.2 bar
65. The oxidation number of the 4) 1.0 bar
underlined atom in the following
species 70. The number of angular nodes and
radial nodes in 3s orbital are
1) HAuCl4 is+3
1) 0 and 1, respectively
2) Cu2O is –1 2) 0 and 2, respectively
3) ClO3 is +5 3) 1 and 0, respectively

4) K2Cr2O7 is +6 4) 3 and 0, respectively

245
Brilliant STUDY CENTRE

71. Which of the following statement is 76. Identify the wrongly match pair
correct about Bakelite?
1) It is a linear polymer M olecule Shape or geometry
of molecule
2) It is a cross linked polymer
1) NH3 Trigonal pyramidal
3) It is an addition polymer 2) PCl5 Trigonal planar
4) It is a branched chain polymer 3) SF6 Octahedral
72. Among the compounds shown below 4) BeCl2 Linear
which one revealed a linear structure?
1) N2O 2) NO2 77. Which of the following statement is
NOT true about acid rain?
3) HOCl 4) O3
1) Its pH is less than 5.6
73. The reaction of concentrated sulphuric 2) It is due to reaction of SO2, NO2 and
acid with carbohydrates (C12H22O11) is CO2 with rain water
an example of
3) Causes no damage to monuments
1) Sulphonation like Taj Mahal
2) Dehydration 4) It is harmful for plants
3) Oxidation 78. Which of the following is a free radical
substitution reaction?
4) Reduction 1) Propene with HBr/(C6H5COO)2
74. Which of the following compound is 2) Benzene with Br2/AlCl3
most reactive in electrophilic aromatic 3) Acetylene with HBr
substitution?
4) Methane with Br2/h 
79. If for a certain reaction  rH is
30 kJ mol–1 at 450 K, the value of r S
(in JK –1 mol –1 ) for which the same
1) 2)
reaction will be spontaneous at the
same temperature is
1) –70 2) 70
3) –33 4) 33
80. Match the compounds of Xe in column
I with the mole cular structure in
3) 4) column II

Column I Column II

75. In a typical fuel cell, the reactant (R) a) XeF2 i) Square planar
and product (P) are b) XeF4 ii) Linear
c) XeO3 iii) Square pyramidal
1) R  H2 g  N2 g : P  NH3aq d) XeOF4 iv) Pyramidal

2) R  H2 g  O2 g : P  H2O2l 1) a-ii, b-i, c-iv, d-iii

3) R  H2 g  O2 g : P  H2Ol 2) a-ii, b-i, c-iii, d-iv


3) a-ii, b-iv, c-iii, d-i
4) R  H2 g  O2 g  Cl2 g : P  HClO4 aq
4) a-ii, b-iii, c-i, d-iv
246
Previous Years NEET Questions & Solutions

81. Which of the following statement is not 85. One mole of carbon atom weighs 12 g,
true about glucose? the number of atoms in it is equal to,
(M ass of car bon -12 is 1.9926×10 –23g)
1) It is an aldopentose
1) 6.022×1023 2) 1.2×1023
2) It is an aldohexose
3) 6.022×1022 4) 12×1022
3) It contains five hydroxyl groups
86. Which of the following oxide is
4) It is a reducing sugar
amphoteric in nature?
82. Identify the correct statement from the 1) CO2 2) SnO2
following
3) SiO2 4) GeO2
1) Lithium chloride is deliquescent and 87. Match the following aspects with the
cr yst alli ses as a h ydr at e, Li Cl.H 2O respective metal
2) The order of hydration enthalpies of
alkaline earth cations
Aspects Metal
Be 2+<Mg2+<Ca2+<Sr2+<Ba2+
The metal which reveals a
3) Lithium and Magnesium show some
maximum number of
similarities in their physical properties
a) oxidation states i) Scandium
as they are diagonally placed in a
periodic table The metal although
placed in 3d block is
4) Lithium is softer among all alkali considered not as a
metals b) transition element ii) Copper
83. Identify the reaction from following The metal which does not
having top position in EMF series exhibit variable oxidation
(Std.red.potential) according to their c) states iii) Manganese
electrode potential at 298 K.
 
1) K  1e  K  s  The metal which in +1
oxidation state in aqueous
2 
solution undergoes
2) Mg  2e  Mg s d) disproportionation iv) Zin

3) Fe 2++2e –  Fe (s)
4) Au3++3e–  Au(s) Select the correct option:
84. Which of the following acid will form 1) a-ii, b-iv, c-i, d-iii
an (a) anhydride on heating and (b) Acid
2) a-i, b-iv, c-ii, d-iii
imide on strong heating with
ammonia? 3) a-iii, b-iv, c-i, d-ii
4) a-iii, b-i, c-iv, d-ii

88. CH3CH2CH  CH2 


B2H6
z . What is Z.
1) 2) H2O,H2O2 ,OH

1)CH3CH2CH2CH3
2) CH3CH2CH2CH2OH
3) CH3CH2CHCH3
3) 4) OH
4) CH3CH2CH2CHO
247
Brilliant STUDY CENTRE

89. Isotonic solutions have same 94. Which of the following statements is
incorrect?
1) Boiling temperature
1) RuBisCo action requires ATP and
2) Vapour pressure
NADPH
3) Freezing temperature
2) RuBisCO is a bifunctional enzyme
4) Osmotic pressure
3) In C 4 plants, the site of RuBisCo
90. How many (i) sp 2
hybridised carbon activity is mesophyll cell
atoms and (ii)  bonds are present in 4) The substrate molecule for RuBisCO
the following compound? activity is a 5-carbon compound
95. Inclusion bodies of blue-green, purple
and green photosynthetic bacteria are
1) Microtubules
1) 8, 5 2) Contractile vacuoles
2) 7, 5 3) Gas vacuoles

3) 8, 6 4) Centrioles

4) 7, 6 96. Which of the following is the correct


floral formula of Liliaceae?
BIOLOGY
1)
91. Chromosomal theory of inheritance
was proposed by 2)
1) Watson and Crick
3)
2) Sutton and Boveri
4)
3) Bateson and Punnet
4) T.H. Morgan 97. Male and female gametophytes do not
have an indepe nde nt free living
92. Which of the following is incorrect existence in:
about cynobacteria?
1) Bryophytes 2) Pteridophytes
1) They have chlorophyll A similar to
3) Algae 4) Angiosperms
green plants
98. In the following in each se t a
2) They are photoautotrophs conservation approach and an example
3) They lack heterocysts of method of conservation are given

4) They often form blooms in polluted a) In situ conservation - Biosphere Reserve


water bodies
b) Ex situ conservation - Sacred groves
93. The impact of immigration on c) In situ conservation - Seed bank
population density is d) Ex situ conservation - Cryopreservation

1) Positive
Select the option with correct match
2) Negative of approach and method
3) Both positive and negative 1) a and b 2) a and c
4) Neutralized by natality 3) a and d 4) b and d
248
Previous Years NEET Questions & Solutions

99. Inhibitory substances in dormant seeds 103. Large, empty colourless cells of the
cannot be removed by subjecting seeds adaxial epidermis along the veins of
to grass leaves are
1) Chilling conditions 1) Bulliform cells
2) Gibberellic acid 2) Lenticels

3) Nitrate 3) Guard cells


4) Bundle sheath cells
4) Ascorbic acid
104. In a mixtu re, DNA fragments are
100. In some plants thalamus contributes separated by
to fruit formation. Such fruits are
termed as 1) Polymerase chain reaction
2) Bioprocess engineering
1) Parthenocarpic fruit
3) Restriction digestion
2) False fruits
4) Electrophoresis
3) Aggregate fruits
105. Which of the following is incorrect for
4) True fruits wind pollinated plants?
101. The biosynthesis of ribosomal RNA 1) Pollen grains are light and non-sticky
occurs in: 2) Well exposed stamens and stigma
1) Nucleolus 3) Many ovules in each ovary
2) Ribosomes 4) Flowers are small and not brightly
coloured
3) Golgi apparatus
106. In a mitotic cycle, the correct sequence
4) Microbodies of phases is
102. Match the following techniques or 1) G1, G2, S, M
instruments with their usage
2) S, G1, G2, M
3) G1, S, G2, M
Separation of DNA
a) Bioreactor i) 4) M, G1, G2, S
fragments

b) Electrophoresis ii)
Production of large 107. Embryological support for evolution was
quantities of products proposed by
Detection of pathogen,
c) PCR iii) based on antigen- 1) Alfred Wallace
antibody reaction 2) Ernst Heckel
Amplification of nucleic
d) ELISA iv)
acids 3) Karl Ernst von Baer
4) Charles Darwin
Select the correct option from following 108. Phycoerythrin is the major pigment in
1) a-ii, b-i, c-iii, d-iv 1) Brown algae

2) a-iii, b-ii, c-iv, d-i 2) Red algae


3) Blue green algae
3) a-ii, b-i, c-iv, d-iii
4) Green algae
4) a-iv, b-iii, c-ii, d-i

249
Brilliant STUDY CENTRE

109. According to Alexander von Homboldt 114. Attachment of spindle fibers to


1) Species richness goes on increasing kinetochores of chromosomes becomes
with increasing area of exploration evident in

2) Species richness decreases with 1) Metaphase 2) Anaphase


increasing area of exploration 3) Telophase 4) Prophase
3) Species richness increases with 115. Correct position of floral parts over
increasing area, but only up to limit thalamus in mustard plant is
4) There is no relationship between 1) Gynoecium is situated in the centre,
species richness and area explored and other parts of the flower are
110. In the polynucleotide chain of DNA, a located at the rim of the thalamus, at
nitrogenous base is linked to the –OH the same level
of 2) Gynoecium occupies the highest

1) 1C pentose sugar position, while the other parts are
situated below it.
2) 2C pentose sugar
3) Margin of the thalamus gro ws
3) 3C pentose sugar
upward, enclosing the ovary
4) 5C pentose sugar completely, and other parts arise below
111. During non-cyclic photophospho- the ovary
rylation, when electrons are lost from
4) Gynoecium is present in the centre
the reaction centre at PS II, what is
and other parts cover it partially
the source whi ch replaces the se
electrons? 116. After about how many ye ars of
1) Light 2) Oxygen formation of earth, life appeared on the
this planet?
3) Water 4) Carbon dioxide
1) 50 billion years
112. In recombinant DNA technology
antibiotics are used 2) 500 billion years

1) As selectable markers 3) 50 million years

2) To keep medium bacteria-free 4) 500 million years

3) to detect alien DNA 117. The term ‘Nuclein’ for the genetic
material was used by
4) To impart disease-resistance to the
host plant 1) Mendel 2) Franklin

113. Which of the following statements is 3) Meischer 4) Chargaff


incorrect? 118. Select the incorrect statement
1) Energy content gradually decreases 1) Elements most easily mobilized in
from first to fourth trophic level plants from one region to another are:
2) Biomass decreases from first to phosphorus, sulphur, nitrogen and
fourth trophic level potassium

3) Energy content gradually increases 2) Transport of molecules in phloem


from first to fourth trophic level can be bidirectional

4) Number of individuals decreases 3) Movement of minerals in xylem is


from first trophic level to fourth trophic unidirectional
level 4) Unloading of sucrose at sink does
not involve the utilization of ATP
250
Previous Years NEET Questions & Solutions

119. The number of contrasting characters 125. Match the items in column I with those
studied by Mendel for his experiments in column II
was
1) 7 2) 14 Colum n I Column II
a) He rbivore s-Plants i) Comme nsalism
3) 4 4) 2 b) Mycorrhiza-Plants ii) Mutualism
120. Vegetative propagule in agave is c) She e p-Cattle iii) Pre dation
termed as d) Orchid-Tre e iv) Compe tition
1) Eye 2) Rhizome
3) Bulbil 4) Offset Select the correct option from following
121. Identify the statement which is 1) a-i, b-iii, c-iv, d-ii
incorrect
2) a-iv, b-ii, c-i, d-iii
1) Tyrosine possesses aromatic ring in
its structure 3) a-iii, b-ii, c-iv, d-i
2) Sulphur is an integral part of
cysteine 4) a-ii, b-i, c-iii, d-iv
3) Glycine is an example of lipids 126. Air (Prevention and control of
4) Lecithin contains phosphorus atom Pollution)Act was amended in 1987 to
in its structure include among pollutants
122. A species which was introduced for 1) Particulates of size 2.5 micrometer
ornamentation but has become a or below
troublesome weed in India
1) Trapa spinosa 2) Vehicular exhaust
2) Parthenium hysterophorus 3) Allergy causing pollen
3) Eichhornia crassipes
4) Noise
4) Prosopis juliflora
123. Pyruvate dehydrogenase activity during 127. In Glycine max, the product of biological
aerobic respiration requires: nitrogen fixation is transported from
the root nodules to other parts as
1) Magnesium 2) Calcium
3) Iron 4) Cobalt 1) Ureides
124. Identify the correct features of Mango 2) Ammonia
and Coconut fruits
i) In both fruit is a drupe 3) Glutamate
ii) Endocarp is edible in both 4) Nitrates
iii) Mesocarp in coconut is fibrous, and
128. Which of the following statements
in mango it is fleshy
about cork cambium is incorrect?
iv) In both, fruit develops from
monocarpellary ovary 1) It is a couple of layers thick
Select the correct option from below 2) It forms secondary cortex on its
1) i and ii only outerside
2) i, iii and iv only 3) It forms a part of periderm
3) i, ii and iii only
4) It is responsible for the formation of
4) i and iv only lenticels

251
Brilliant STUDY CENTRE

129. Match the following 133. Who coined the term ‘Kinetin’?
1) Kurosawa
a) Aquaporin i) Amide 2) Skoog and Miller
b) Asparagine ii) Polysaccharide
c) Abscisic acid iii) Polypeptide 3) Darwin
d) Chitin iv) Carotenoids
4) Went
Select the correct option
1) a-iii, b-i, c-ii, d-iv 134. Which of the following statements is
incorrect regarding the phosphorus
2) a-iii, b-i, c-iv, d-ii cycle?
3) a-ii, b-iii, c-iv, d-i
1) It is sedimentary cycle
4) a-ii, b-i, c-iv, d-iii
130. Which of the following statements is 2) Phosphates are the major form of
incorrect about gymnosperms? phosphorus reservoir
1) Their seeds are not covered
3) Phosphorus solubilising bacteria
2) They are heterosporous facilitate the release of phosphorus
3) Male and female gametophytes are from organic remains
free living
4) There is appreciable respiratory
4) Most of them have narrow leaves with release of phsophorus into atmosphere
thick cuticle
131. Which of the following elements helps 135. First discovered restriction
in maintaining the structure of endonuclease that always cuts DNA
ribosomes? molecule at a particular point by
1) Molybdenum recognising a specific sequence of six
base pairs is
2) Magnesium
3) Zinc 1) Hind II
4) Copper
2) EcoR I
132. Match the following concerning the
activity function and the phytohormone 3) Adenosine deaminase
involved
4) Thermostable DNA polymerase
a) Fruit ripener i) Abscisic acid
b) Herbicide ii) GA3 136. Which of the following is associated
c) Bolting agent iii) 2, 4-D with decrease in cardiac output?
d) Stress hormone iv) Ethephon
1) Adrenal medullary hormones
Select the correct option from following
1) a-iv, b-ii, c-i, d-iii 2) Sympathetic nerves

2) a-ii, b-ii, c-iv, d-i 3) Parasympathetic neural signals


3) a-iii, b-iv, c-ii, d-i
4) Pneumotaxic centre
4) a-iv, b-iii, c-ii, d-i
252
Previous Years NEET Questions & Solutions

137. Match the following group of organisms 140. The total Lung Capacity (TLC) is the
with their respective distinctive total volume of air accomodated in the
characteristics and select the correct lungs at the end of a forced inspiration.
option This includes:
1) RV (Residual Volume)

ERV(Expiratory Reserve Volume)


Organisms Characteristics
a) Platyhelminthes i) Cylindrical body TV ( Tidal Volume); and
with no
segmentation
IRV(Inspiratory Reserve Volume)
b) Echinoderms ii) Warm blooded 2) RV; IC (Inspiratory Capacity);
animals with direct
development EC (Expiratory Capacity); and ERV
c) Hemichordates iii) Bilateral symmetry 3) RV; ERV; IC and EC
with incomplete
digestive system 4) RV; ERV; VC (Vital Capacity) and
d) Aves iv) Radial symmetry FRC (Functional Residual Capacity)
with indirect
development
141. Hormones stored and released from
neurohypophysis are

1) Prolactin and Vasopressin


1) a-i, b-ii, c-iii, d-iv
2) Thyroid stimulating hormone and
2) a-iii, b-iv, c-i, d-ii oxytocin

3) Oxytocin and Vasopressin


3) a-ii, b-iii, c-iv, d-i
4) Follicle stimulating hormone and
4) a-iv, b-i, c-ii, d-iii Leutinizing hormone
138. Which is the basis of genetic mapping 142. According to Central Pollution Control
of human genome as well as DNA finger Board [CPCB] what size (in diameter)
printing? of particulate is responsible for causing
greater harm to human health?
1) Polymorphism in RNA sequence
1) 3.0 micrometers
2) Polymorphism in DNA sequence
2) 3.5 micrometers
3) Single nucleotide polymorphism
3) 2.5 micrometers
4) Polymorphism in hnRNA sequence 4) 4.0 micrometers
139. The best example for pleiotropy is 143. Cyclosporin A, used as immuno
suppression agent, is produced from
1) ABO blood group
1) Trichoderma polysporum
2) Skin colour
2) Monascus purpureus
3) Phenylketoneuria
3) Saccharomyces cerevisiae
4) Colour Blindness
4) Penicillium notatum
253
Brilliant STUDY CENTRE

144. For the commercial and industrial 149. Intrinsic factor that helps in the
production of Citric Acid, which of the absorption of vitamin B12 is secreted
following microbes is used? by
1) Clostridium butylicum 1) Chief cells 2) Goblet cells
2) Aspergillus niger 3) Hepatic cells 4) Oyntic cells
3) Lactobacillus sp 150. Match the following columns with
4) Saccharomyces cerevisiae reference to cockroach and select the
correct option
145. All vertebrates are chordates but all
chordates are not vertebrates, why?
1) All chordates possess notochord C olumn I C olumn II

throughout their life a) Gri ndi ng of the food parti cle i ) He pati c cae cae
b) Se cre te gastri c jui ce i i ) 10th se gm e nt
2) Notochord is replaced by vertebral c) 10 pa i rs i i i ) Prove ntri culus
column in adult of some chordates d) An al C e rci i v) Spi racle s
3) Ventral hollow nerve cord remains v) Alary m uscle s
throughout life in some chordates
4) All chordates possess vertebral 1) a-ii, b-iii, c-i, d-iv
column 2) a-iii, b-i, c-iv, d-ii
146. The phenomenon of e volution of 3) a-iv, b-iii, c-v, d-ii
different species in a given
geographical area starting from a point 4) a-i, b-iv, c-iii, d-ii
and spreading to other habitats is 151. The increase in osmolarity from outer
called to inne r me dulary interstitium is
1) Adaptive radiation maintained due to

2) Saltation i) Close proximity between Henle’s loop


and vasa recta
3) Co-evolution
ii) Counter current mechanism
4) Natural selection
147. E. Coli has only 4.6×106 base pairs and iii) Selective secretion of HCO3 and
completes the process of replication hydrogen ions in PCT
within 18 minutes; then the average iv) Higher blood pressure in glomerular
rate of polymerisation is approximately capillaries
1) 1000 base pairs/second 1) i and ii 2) Only ii
2) 2000 base pairs/second 3) iii and iv 4) i, ii and iii
3) 3000 base pairs/second
152. Select the correct statement:
4) 4000 base pairs/second
1) Reduction in Glomerular Filtration
148. The size o f Pleuropneumonia-like Rate activates JG cells to release renin
organism (PPLO) is
2) Atrial Natriuretic factor increases
1) 0.1m the blood pressure

2) 0.02 m 3) Angiote nsin II is a powerful


vasodilator
3) 1–2 m
4) Counter current pattern of blood flow
4) 10–20 m is not observed in vasa recta

254
Previous Years NEET Questions & Solutions

153. Which of the following STDs are not 157. Match the following columns and
curable? select the correct option
1) Gonorrhoea, Trichomoniasis,
Hepatitis B
Column I Column II
2) Genital herpes, Hepatitis B, HIV a) Dragonflies i) Biocontrol
infection agents of several
plant pathogens
3) Chlamydiasis, Syphilis, Genital b) Bascillus ii) Get rid of Aphids
warts thuringiensis and mosquitoes

4) HIV, Gonorrhoea, Trichomoniasis c) Glomus iii) Narrow spectrum


insecticidal
154. Match the following columns and applications
select the correct option d) Baculoviruses iv) Biocontrol agents of
lepidoteran plant pests
Column I Column II
a) Smooth endoplasmic i) Protein synthesis v) Absorb phosphorus
reticulum from soil
b) Rough endoplasmic ii) Lipid synthesis
reticulum
c) Golgi complex iii) Glycosylation 1) a-ii, b-iv, c-v, d-iii
d) Centriole iv) Spindle formation
2) a-iii, b-v, c-iv, d-i

1) a-i, b-ii, c-iii, d-iv 3) a-ii, b-i, c-iii, d-iv

2) a-ii, b-i, c-iii, d-iv 4) a-ii, b-iii, c-iv, d-v

3) a-iii, b-i, c-ii, d-iv 158. Select the incorrectly matched pair
from following
4) a-iv, b-ii, c-i, d-iii
1) Ostcocytes-Bone cells
155. A hominid fossil discovered in Java in
1891, now extinct, having cranial 2) Chondrocytes-Smooth muscle cells
capacity of about 900 cc was
3) Neurons-Nerve cells
1) Australopithecus
4) Fibroblat-Areolar tissue
2) Homo erectus
159. The ye llowish fluid “colostrum”
3) Neanderthal man secreted by mammary glands of mother
during the initial days of lactation has
4) Homo sapiens abundant antibodies (lgA) to protect the
156. The protcolytic enzyme rennin is found infant. This type of immunity is called
in as

1) Pancreatic juice 1) Autoimmunity

2) Inteatinal juice 2) Passive immunity

3) Bile juice 3) Active immunity

4) Gastric juice 4) Acquired immunity

255
Brilliant STUDY CENTRE

160. Select the correct option of haploid cells 164. Match the following columns and
from the following groups: select the correct option
1) Primary spermatocyte, secondary
spermatocyte, second polar body Column I Column II
a) Ovary i) Human chorionic
2) Primary oocyte, secondary oocyte, Gonadotropin
spermatid
b) Placenta ii) Estrogen and
Progesterone
3) Secondary spermatocyte, first polar
body, ovum c) Corpus luteum iii) Androgens

d) Leydig cells iv) Progesterone only


4) Spermatogonia, primary
spermatocyte, spermatid

161. During Meiosis l, in which stage 1) a-ii, b-i, c-iv, d-iii


synapsis takes place?
2) a-iv, b-iii, c-ii, d-i
1) Leptotene
3) a-i, b-ii, c-iii, d-iv
2) Pachytene
4) a-i, b-iii, c-ii, d-iv
3) Zygotene
165. Match the following columns and
4) Diplotene select the correct option

162. Select the correct statement from the


following Column I Column II
a) Pituitary hormone i) Steroid
1)PCR is used for isolation and b) Epinephrine ii) Neuropeptides
separation of gene of interest
c) Endorphins iii) Peptides, proteins

2) Gel electrophoresis is used for d) Cortisol iv) Biogenic amines


amplification of a DNA segment

3) The polymerase enzyme joins the 1) a-iii, b-iv, c-i, d-ii


gene of interest and the vector DNA
2) a-iv, b-i, c-ii, d-iii
4) Restriction enzyme digestions are
performed by incubating purified DNA 3) a-iii, b-iv, c-ii, d-i
molecules with the restriction enzymes
4) a-iv, b-iii, c-i, d-ii
of optimum conditions
166. The laws and rules to preve nt
163. Spooling is
unauthorise d exploi tation o f bio-
1) Collection of isolated DNA resources are termed as

2) Amplification of DNA 1) Biopiracy

3) Cutting of separated DNA bands from 2) Biopatenting


the agarose gel
3) Bioethics
4) Transfer of separated DNA fragments
to synthetic membranes 4) Bioengineering

256
Previous Years NEET Questions & Solutions

167. Match the following columns and 171. In human beings, at the end of 12
select the correct option weeks (first trimester) of pregnancy,
the following is observed

Colu mn I Column II 1) Movement of the foetus


i) T yphoi d a ) Haemophilus
2) Eyelids and eyelashes are formed
influenzae
i i ) Malari a b) Wuchereria 3) Most of the major organ system are
bancrofti formed
i i i ) Pne um on i a c) Plasmodium vivax 4) The head is covered with fine hair
i v) Fi lari a si s d) Salmonella typhi
172. Progesterons alone or in combination
with estrogen can be used as a
1) i-a, ii-b, iii-d, iv-c contraceptive in the form of
2) i-d, ii-c, iii-a, iv-b 1) Pills only
3) i-c, ii-d, iii-b, iv-a 2) Implants only
4) i-a, ii-c, iii-b, iv-d
3) Injections only
168. RNA interference is used for which of
the following purposes in the field of 4) Pills, injections and implants
biotechnology?
173. Which of the following options does
1) to reduce post harvest losses correctly represent the characteristic
features of phylum Annelida?
2) to develop a plant tolerant to abiotic
stresses 1) Diploblastic, mostly marine and
3) to develop a pest resistant plant radially symmetrical
against infestation by nematode 2) Triploblastic, unsegmented body and
4) to enhance the mineral usage by bilaterally symmetrical
the plant
3) Triploblastic, segmented body and
169. The rate of decomposition is faster in bilaterally symmetrical
the ecosystem due to following factors
EXCEPT: 4) Triplobastic, flattened body and
acoelomate condition
1) Detritus richer in lignin and chitin
174. Inbreeding depression is
2) Detritus rich in sugars
3) Warm and moist environment 1) Reduced fertility and productivity
due to continued close inbreeding
4) Presence of aerobic soil microbes
2) Reduced motility and immunity due
170. Which of the following conditions cause to close inbreeding
erythroblastosis foetalis?
3) Decreased productivity due to
1) Both mother and foetus Rh+ve
mating of superior male and inferior
2) Mother Rh+ve and foetus Rh–ve female
3) Mother Rh–ve and foetus Rh+ve 4) Decrease in body mass of progeny
4) Both mother and foetus Rh–ve due to continued close inbreeding

257
Brilliant STUDY CENTRE

175. Match the following columns and 178. Match the following columns and
select the correct option select the correct option

Column I Column II Column I Column II


a) Rods and cones i) Absence of a) Aptenodytes i) Flying fox
photoreceptor cells
b) Pteropus ii) Angel fish
b) Blind spot ii) Cones are densely c) Pterophyllum iii) Lamprey
packed
d) Petromyzon iv) Penguin
c) Fovea iii) Photoreceptor cells

d) Iris iv) Visible coloured 1) a-ii, b-i, c-iv, d-iii


portion of the eye
2) a-iii, b-iv, c-ii, d-i
1) a-ii, b-iv, c-iii, d-i 3) a-iii, b-iv, c-i, d-ii
2) a-iii, b-i, c-ii, d-iv
3) a-ii, b-iii, c-i, d-iv 4) a-iv, b-i, c-ii, d-iii
4) a-iii, b-iv, c-ii, d-i 179. In cockroach, identify the parts of the
176. Match the following columns and foregut in correct sequence:
select the correct option
1) Mouth  Pharynx  Oe sophagus
Column I Column II  Crop  Gizzard
a) Pneumotaxic centre i) Alveoli 2) Mouth  Oesophagus  Pharynx
b) O2 Dissociation curve ii) Pons region of  Crop  Gizzard
brain
3)Mouth  Crop  Pharynx 
c) Carbonic Anhydrase iii) Haemoglobin Oesophagus  Gizzard
d) Primary site of iv) RBC
4) Mouth  Gizzard  Crop  Pharynx
exchange of gases
 Oesophagus
1) a-iv, b-i, c-iii, d-ii 180. Match the following events that occur
2) a-i, b-iii, c-ii, d-iv in their respective phases of cell cycle
3) a-ii, b-iii, c-iv, d-i and select the correct option
4) a-iii, b-ii, c-iv, d-i
177. Match the following columns and
select the correct option
a) G 1 ph a s e i) C e ll g r ow s a n d
o r g a n e lle
Column I Column II
du pli c a ti o n
a) Gout i) Decreased levels
of estrogen b) S ph a s e ii) D N A r e pli c a ti o n
b) Osteoporosis ii) Low Ca++ ions in and
the blood c h rom os om e
c) G 2 ph a s e i i i ) C y t o pla s m i c
c) Tetany iii) Accumulation of
uric acid crystals g r o w th
d) M e t a ph a s e i v ) A li g n m e n t o f
d) Muscular iv) Genetic disorder i n M -ph a s e c h rom os om e s
dystrophy
1) a-i, b-ii, c-iii, d-iv
1) a-i, b-ii, c-iii, d-iv 2) a-ii, b-iii, c-iv, d-i
2) a-ii, b-i, c-iii, d-iv 3) a-iii, b-iv, c-i, d-ii
3) a-iii, b-i, c-ii, d-iv 4) a-iv, b-i, c-ii, d-iii
4) a-iii, b-ii, c-iv, d-i

258
Previous Years NEET Questions & Solutions

KEY WITH HINTS

PHYSICS

1
1. 2 l
2nd2

2. 2 An instrinsic semi conductor is


converted in to n type extrinsic
semi conductor by doping it with
pentavalent impurity. Phosphorus
is pentavalent

3. 2 T=1.4×1017s

0.693
N   2  1021
1.4  1017

N  0.99  10 4  10 4 Bq

R AB  16
4. 4 Gamma rays

5. 3 7. 2 Induced emf across each spoke is


1
Bl2 all spokes are in parallel
2
combination

1
So Enet= Bl2
2
Enet=2+4=6V
B=0.4×10–4T=4×10–5T
Rnet=1+1+4= 6
2
  120   4 rad / s
Enet 60
I  1A
Rnet l=1

1
Enet=  4  105  4  1  2.51 10 4
2

8. 1
VA=0 VB=–3V VA-VB is +ve
VA>VB so diode is forward biased
6. 3
in all other cases potential of A
with respect to B is –ve. So diode
is reverse biased in all other
cases

259
Brilliant STUDY CENTRE

P
 m / v  RT
9. 4
M

RT
P
m
  density

M  molar mass

15. 3   5t 2  3t  16

d
 10t  3
dt
at t =4
IC[800]=0.8
IC=10–3A=1mA d
 43v
dt
l l
10. 3 
A1 A 2 1 T 180 90
16. 3 f 
2l  120  '
A1=A 2
11. 2 Zero f1  '  60cm

1.25  1.24  1.27  1.21  1.28
12. 1  1.25  t av f' 1
5 '

f' 
 t av  ( t 1  ( t)2  (t)3  ( t)4  ( t)5 
f '
5
17. 3 B  2  10 7 sin    103 x  3   1011 t 
0  0.1  0.2  0.04  0.03
 t av   0.02s
B=B0sin[kx+wt]
5
k    103
 t av 0.02
%error   100   100  1.61
t av 1.25 2
   103

  2  103 m
18. 3 hg  76  10 2  mer  g

h  760  76  102  13600


13. 3
h=13.6m
19. 4 constant pressure process is called
isobaric
consider the loop BCDEB
T2
20. 3   1
 l 2R2  E2  E3  l 3R1  0 T1

14. 4 PV  nRT T2  Temperature of sink

m T1  Temperature of source


PV    RT
M

260
Previous Years NEET Questions & Solutions

the inductor, self inductance


increases due to increase in  r .
So impedence of circuit increases
21. 1 and current decreases


 P
E
4  0r 3

E is antiparallel to P
25. 4

22. 4 m1  m2  m3  M

r1  0

r2  2iˆ

r3  2jˆ
     
 m v  m r  m r r r r
rcm  1 1 2 2 3 3
= 1 2 3
m1  m2  m3 3

 2
 
rcm  ˆi  ˆj
3
23. 1  m T  constant
h h
B   C   A 26. 2  
P 2mE
TA  TC  TB
6.63  10 34
  102  10 12 m
2  9.1 10 31  144  1.6  10 19

  102  103 nm
Another method
24. 2 12.27 12.27
 Ao   1.02A o
E in eV 144

1 1   
  N2 A 27. 2 1minute  60  60  180  rad
L 0 r  

 2.91 10 4 rad
xL=Lw
13.6
z  R2  xL2 28. 3 En  eV
n2
When iron rod is inserted in to

261
Brilliant STUDY CENTRE

29. 1 1 e2
33. 4 F e=1.6×10–19c
4 0 r 2

r=1.6×10–10m
m=9×10–31kg

F 1  e2 
L= 2r a   
m 4 0m  r 2 
L
r 1
2 a= 1022 m/s2  9  109
40
 L2  34. 3  rays are photons. So mass
 
M  I r 2  I  2 
 4 r  number and atomic number
remain unchanged
L2I 1
M=
4 35. 3 sin c= 

 d  n  1 1
30. 3 g'  g 1   d  R 
 R  n  sin 45= 

g  d  2
 g 1 
n  R 
c 3
V   108 m / s
d 1  2
1 
R n

d 1 36. 2
 1
R n
31. 2 u= –1.5 f
1 1 1
 
u v f

1 1 1
 
1.5f v f

1 1 1 1  0.5 
   
V 1.5f f f  1.5 
v= –3f
  0
32. 2    0.05
t
g=10m/s2
2 N=10g = 100 newton
  0  840rpm  840  rad / s
60 fk  N  5newton

  0  28 2g  fk
a
12
t=14s
20  5
  0 28 a  1.25m / s2
   2 rad / s2 12
t 14

262
Previous Years NEET Questions & Solutions

A0 2A0
37. 4 C1    2C
d/2 d

4A0 8A0
C2    8C
d/2 d

C1C2 16
C'   C
C1  C2 10

C' 16 8
 
C 10 5

2    1
40. 1 P
mv 2 R
T-mg=
r
2    1
10 
2 0.1
mv m
T  mg   mg   7gr 
r g 1
 1
2
T  8mg
3
38. 3 Radius of curvature of curved 
surface 2

41. 3

39. 3

r R

1 q
V  
4 0  R  constant
A 0
C
d r=R

1 q
V  
40  R 

r R

263
Brilliant STUDY CENTRE

1 q 
NH4Cl 
HO

 NH4OH+HCl---(2)
2

V
40 r

(NH4) 2SO 4 
HO

 2NH 4OH+H 2SO 4
2

V 1
r ---(3)


NH4NO3 
HO

 NH4OH+HNO3---(4)
2

Since CH3COONa gives strong


base it gives basic solution
47. 2 Weight of solute = 8g
Molecular mass of n butane=
114g
Weight of n butane=114g
42. 4 Any gate can be realized with the P=80% P=0.8Po
suitable combination of NAND
gates Relative lowering of vapour pres-
sure
43. 4 y  sin t  cos t
P  P P  0.8P nB
 B 
sin  t  45 P0 P0 nA  nB
y
2
114
But nA  nB  nAand nA  1
1 114
y sin  t  45
2
P 1  0.8 
  nB
So this function is periodic with Po

time period nB  0.2
2
44. 1 Davisson and Germer mass of solute
nB= molecular massof solute
45. 3 v  u  gt

v-u=10×0.1 8
MB   40g
0.2
v  u  1 ---(1)
48. 2 Rosenmund reduction

 V u
 2 t  s
  O
CHO
v u 
2s C
t Cl
2H
Pd/BaSO
4

v  u  30 ---(2)

(2)–(1)  2u=29 u=14.5m/s 49. 3 Lanthanoid can show oxidation


number from +3 to +7
46. 1

CH 3COONa   50. 2 For zero order reaction
 CH3COOH+NaOH
HO 2

--(1)
264
Previous Years NEET Questions & Solutions

R0 56. 2
t1 
2 2k

R0 0.02
k 
2t 1 2  100
2

0.02
=
2  100 57. 2 Boron = Zone refining
=0.01×10–2 Tin-Lequation
=10 –4
Zr-Van arkel method

CN=4 Ni-Mond’s Process

51. 3 sp3 dsp2 58. 2 For steam distillation, component


tetrahedral square planar must be
1) Steam volatile
CN=5  trigonal bipyramidal-sp3d
2) Immiscible with H2O
CN=6  octahedral  sp3d2
59. 1 Gypsum (CaSO4.2H2O) is used to
52. 3 Copper-group 11 of d-block-transi- increase setting time of cement
tion element
60. 2 EWG excease acidity of phenol
Flurine - group 17 of p-block-Non-
metal 61. 3 Vitamin D deficiency in adult-
osteomalacia
Silicon-group 14 of p-block-metal-
loid Children-Rickets
Cerium-1st lanthanoid 62. 4 2F2+2H2O  4HF+O2  Redox
reaction
53. 3 Collision frequency (ZAB) is de-
fined as the number of collisions 63. 2 AgBr shows both schottky and
between A and B per unit time Frenkel defect
per unit volume 64. 2 Since it is a broad spectrum
54. 2 H2+Br2  2HBr antibiotic, it can be used for any
type of bacteria
Hl   BEr   BEp
65. 2 Cu2O - copper is in +1 oxidation
109  (435  192)  2 BEHBr state
66. 4 Based on stability of carbocation
2BEHBr  627  109  736
CH2CH2+
736
BEHBr   368kJmol1
2
55. 2 AB type
is least stable among them
8
K sp  s  4  10
2
NaOHBr2
67. 2 CH3CH2CONH2 

CH3CH2NH 2
s  2  104 mol / L1 Hoffmann Bromamide
Degradation reaction
265
Brilliant STUDY CENTRE

68. 4 Starch is a –ve colloid


69. 2 By Bole’s law
Xe Pyramidal
P1v1  P2 v 2 XeO3 F F
1×600=P2×150 F
P2= 4 bar
O
70. 2 For 3s orbital n=3 F F
Angular nodes = l=0 XeOF4 Xe Square pyrami-
Radial nodes = n–l–1=3–0–1=2 F F
71. 2 Bakelite (phenol-formaldehyde
dal
resin) is a cross-linked polymer
81. 1 Glucose is an aldehyde with 6
N O carbon atom
72. 1 O and O bent
O O 82. 3 Diagonal realtionship
LiCl is not deliquescent and
O
H Cl-Bent forms LiCl.6H2O

1
73. 2 C12H22O11 
con.H2SO4
 12C+11H2O
 Hydration enthalpy  size of ion
74. 1 +R group increase the electro-
philic substitution of benzene Li is hard
75. 3 83. 4 K++ie–  K(s) - 2.93V
76. 2 PCl5-trigonal bipyramidal Mg2  2e   Mg s  2.36V
77. 3 CaCO3+H 2SO 4  CaSO 4+H 2O+CO 2
78. 4 In presence of light free radial Fe2   2e   Fe  s  0.44V
reaction taken place
Au3++3e –  Au(s)+1.40V
79. 2 For a spontaneous reaction
G   ve O O
 G  H  T S C C
OH O
G  30000J / mol  450  S 84. 2 OH
C O
For ForG   veS should be
70J / kmol O O

F
O
O
C
80. 1 XeF2 Xe linear OH
C
N H
OH + NH3
C O
F
O

F F 85. 1 1 Mole = 6.022×1023 atoms


XeF4 Xe Square planar 86. 2 SiO2, CO2 - acidic GeO2-acidic
F F SnO2 amphoteric

266
Previous Years NEET Questions & Solutions

87. 3 Mn-+7
Zn-Non typical transition element
Sc-Shows +3 oxidation number
Cu  Cu+1  Cu+2 + Cuo
88. 2 Antimarkonikov addition
hydroboration followed by hydroly-
sis
89. 4
90. 4

sp2 O
sp2
   
sp2 C C C O CH3
sp2  sp2
sp2 

BIOLOGY
91. 2 104. 3 117. 3 130. 3 143. 1 156. 4 169. 1
92. 3 105. 3 118. 4 131. 2 144. 2 157. 1 170. 3
93. 1 106. 3 119. 1 132. 4 145. 2 158. 2 171. 3
94. 3 107. 2 120. 3 133. 2 146. 1 159. 2 172. 4
95. 3 108. 2 121. 3 134. 4 147. 2 160. 3 173. 3
96. 4 109. 3 122. 3 135. 1 148. 1 161. 3 174. 1
97. 4 110. 1 123. 1 136. 3 149. 4 162. 4 175. 2
98. 3 111. 3 124. 2 137. 2 150. 2 163. 1 176. 3
99. 4 112. 2 125. 3 138. 2 151. 1 164. 1 177. 3
100. 2 113. 3 126. 4 139. 3 152. 1 165. 3 178. 4
101. 1 114. 1 127. 1 140. 1 153. 2 166. 3 179. 1
102. 3 115. 2 128. 2 141. 3 154. 2 167. 2 180. 1
103. 1 116. 4 129. 2 142. 3 155. 2 168. 3

267

You might also like